Previous Gate by Kanodia Practise Questions All Subjects PDF

You might also like

Download as pdf or txt
Download as pdf or txt
You are on page 1of 250

GATE

ELECTRONICS & COMMUNICATION


Topicwise Solved Paper

Year 2013- 1996

By RK Kanodia & Ashish Murolia

For more GATE Resources, Mock Test and Study material

Join the Community

http://www.facebook.com/gateec2014
GATE Electronics and Communication Topicwise Solved Paper by RK Kanodia & Ashish Murolia Page 1

UNIT 1 (C) x

2012
(D) 1

TWO MARKS
ENGINEERING MATHEMATICS
d 2 y (t) dy (t)
1.8 Consider the differential equation +2 + y (t) = d (t)
dt 2 dt
dy
with y (t) t = 0 =- 2 and =0
-
dt t = 0 -

dy
The numerical value of is
2013 ONE MARK dt t = 0 +
(A) - 2 (B) - 1
1.1 The maximum value of q until which the approximation sin q . q (C) 0 (D) 1
holds to within 10% error is
(A) 10c (B) 18c 1.9 The direction of vector A is radially outward from the origin, with
A = krn . where r2 = x2 + y2 + z2 and k is a constant. The value of
(C) 50c (D) 90c
n for which d:A = 0 is
1.2 The minimum eigen value of the following matrix is (A) - 2 (B) 2
R3 5 2V
S W (C) 1 (D) 0
S5 12 7W
SS2 7 5WW 1.10 A fair coin is tossed till a head appears for the first time. The
T X
(A) 0 (B) 1
SPECIAL EDITION ( STUDY MATERIAL FORM )
(C) 2 (D) 3
At market Book is available in 3 volume i.e. in 3 book binding
1.3 A polynomial f (x) = a 4 x 4 + a 3 x3 + a2 x2 + a1 x - a 0 with all coefficients form. But at NODIA Online Store book is available in 10 book
positive has binding form. Each unit of Book is in separate binding.
(A) no real roots Available Only at NODIA Online Store
(B) no negative real root Click to Buy
(C) odd number of real roots www.nodia.co.in
(D) at least one positive and one negative real root
probability that the number of required tosses is odd, is
(A) 1/3 (B) 1/2
2013 TWO MARKS
(C) 2/3 (D) 3/4
1.4 Let A be an m # n matrix and B an n # m matrix. It is given that
determinant ^Im + AB h = determinant ^In + BAh, where Ik is the 1.11 The maximum value of f (x) = x3 - 9x2 + 24x + 5 in the interval [1, 6]
k # k identity matrix. Using the above property, the determinant of is
the matrix given below is (A) 21 (B) 25
R V
S2 1 1 1W (C) 41 (D) 46
S1 2 1 1W
S1 1 2 1W 1.12 Given that
S W -5 -3 1 0
A=> H and I = >
0 1H
S1 1 1 2W , the value of A3 is
T X 2 0
(A) 2 (B) 5 (A) 15A + 12I (B) 19A + 30I
(C) 8 (D) 16 (C) 17A + 15I (D) 17A + 21I

2012 ONE MARK 2011 ONE MARK


1.5 With initial condition x (1) = 0.5 , the solution of the differential 1.13 Consider a closed surface S surrounding volume V . If rv is the
equation position vector of a point inside S , with nt the unit normal on S ,
t dx + x = t , is the value of the integral ## 5rv $ nt dS is
dt S

(A) x = t - 1 (B) x = t 2 - 1 (A) 3V (B) 5V


2 2
(C) 10V (D) 15V
2
t
(C) x = t (D) x = 2 dy
2 1.14 The solution of the differential equation = ky, y (0) = c is
1 - 2 . dx
1.6 Given f (z) =
z+1 z+3 (A) x = ce-ky (B) x = kecy
If C is a counter clockwise path in the z -plane such that (C) y = cekx (D) y = ce-kx
z + 1 = 1, the value of 1 #
f (z) dz is
2pj C 1.15 The value of the integral # - 3z + 4 dz where c is the circle
(A) - 2 (B) - 1 (z 2 + 4z + 5)
z = 1 is given by c

(C) 1 (D) 2 (A) 0 (B) 1/10


(C) 4/5 (D) 1
1.7 If x = - 1, then the value of xx is
(A) e- p/2 (B) e p/2
GATE Electronics and Communication Topicwise Solved Paper by RK Kanodia & Ashish Murolia Page 2

2011 TWO MARKS 2010 TWO MARKS

1.16 A numerical solution of the equation f (x) + x - 3 = 0 can be 1.22 If ey = x1/x , then y has a
obtained using Newton- Raphson method. If the starting value is (A) maximum at x = e (B) minimum at x = e
x = 2 for the iteration, the value of x that is to be used in the next (C) maximum at x = e-1 (D) minimum at x = e-1
step is
(A) 0.306 (B) 0.739 1.23 A fair coin is tossed independently four times. The probability of the
(C) 1.694 (D) 2.306 event “the number of time heads shown up is more than the number
of times tail shown up”
1.17 The system of equations (A) 1/16 (B) 1/3
x+y+z = 6 (C) 1/4 (D) 5/16
x + 4y + 6y = 20
1.24
v = xyatx + x 2 aty , then
If A # Av $ dlv over the path shown in the figure
x + 4y + l z = m
is C
has NO solution for values of l and μ given by
(A) l = 6, m = 20 (B) l = 6, m =
Y 20
(C) l =
Y 6, m = 20 (D) l =
Y 6, m = 20
1.18 A fair dice is tossed two times. The probability that the second toss
results in a value that is higher than the first toss is
(A) 2/36 (B) 2/6

GATE Electronics & Communication


by RK Kanodia
(A) 0 (B) 2
Now in 3 Volume 3
Purchase Online at maximum discount from online store (C) 1 (D) 2 3
and get POSTAL and Online Test Series Free 1.25 The residues of a complex function
visit www.nodia.co.in x (z) = 1 - 2z
z (z - 1) (z - 2)
(C) 5/12 (D) 1/2 at its poles are
(A) 1 , - 1 and 1 (B) 1 , - 1 and - 1
2 2 2 2
2010 ONE MARKS
(C) 1 , 1 and - 3 (D) 1 , - 1 and 3
1.19 The eigen values of a skew-symmetric matrix are 2 2 2 2
(A) always zero (B) always pure imaginary dy (x)
1.26 Consider differential equation - y (x) = x , with the initial
dx
(C) either zero or pure imaginary (D) always real
condition y (0) = 0 . Using Euler’s first order method with a step
1.20 The trigonometric Fourier series for the waveform f (t) shown below size of 0.1, the value of y (0.3) is
contains (A) 0.01 (B) 0.031
(C) 0.0631 (D) 0.1

Given f (t) = L-1 ; 3 3s + 1


s + 4s2 + (k - 3) s E
1.27 . If lim f (t) = 1, then the value
t"3
of k is
(A) 1 (B) 2
(C) 3 (D) 4
For more GATE Resources, Mock Test and
Study material join the community
http://www.facebook.com/gateec2014
(A) only cosine terms and zero values for the dc components
2009 ONE MARK
(B) only cosine terms and a positive value for the dc components
1.28 The order of the differential equation
(C) only cosine terms and a negative value for the dc components
d2y dy 3
2 + c dt m + y = e
4 -t
(D) only sine terms and a negative value for the dc components dt
is
1.21 A function n (x) satisfied the differential equation (A) 1 (B) 2
d 2 n (x) n (x) (C) 3 (D) 4
- 2 =0
dx 2 L
where L is a constant. The boundary conditions are : n (0) = K 1.29 A fair coin is tossed 10 times. What is the probability that only the
and n (3) = 0 . The solution to this equation is first two tosses will yield heads?
(A) c 1 m (B) 10C2 b 1 l
2 2
(A) n (x) = K exp (x/L) (B) n (x) = K exp (- x/ L )
2 2 2
(C) n (x) = K exp (- x/L) (D) n (x) = K exp (- x/L)
(C) c 1 m (D) 10C 2 b 1 l
10 10

2 2
GATE Electronics and Communication Topicwise Solved Paper by RK Kanodia & Ashish Murolia Page 3

f (x) = exp (x) + exp (- x) is


1.30 If f (z) = c 0 + c1 z-1 , then # 1 +zf (z) dz is given by (A) 2 (B) 1
unit circle (C) 0.5 (D) 0
(A) 2pc1 (B) 2p (1 + c0)
(C) 2pjc1 (D) 2p (1 + c0) 1.38 Which of the following functions would have only odd powers of x
in its Taylor series expansion about the point x = 0 ?
(A) sin (x3) (B) sin (x2)
2009 TWO MARKS (C) cos (x3) (D) cos (x2)
1.31 The Taylor series expansion of sin x at x = p is given by Which of the following is a solution to the differential equation
x-p 1.39

(x - p ) 2
(x - p) 2 dx (t)
(A) 1 + + ... (B) - 1 - + ... + 3x (t) = 0 ?
3! 3! dt
(A) x (t) = 3e - t (B) x (t) = 2e - 3t
(x - p) 2 (x - p) 2
(C) 1 - + ... (D) - 1 + + ... (C) x (t) =- 23 t2 (D) x (t) = 3t2
3! 3!

1.32 Match each differential equation in Group I to its family of solution


2008 TWO MARKS
curves from Group II
Group I Group II 1.40 The recursion relation to solve x = e - x using Newton - Raphson
dy y method is
A. = 1. Circles
dx x
dy y SPECIAL EDITION ( STUDY MATERIAL FORM )
B. =- 2. Straight lines
dx x At market Book is available in 3 volume i.e. in 3 book binding
dy x
C.
dx
=
y
3. Hyperbolas form. But at NODIA Online Store book is available in 10 book
dy binding form. Each unit of Book is in separate binding.
D. =- x Available Only at NODIA Online Store
dx y
(A) A - 2, B - 3, C - 3, D - 1 Click to Buy
(B) A - 1, B - 3, C - 2, D - 1 www.nodia.co.in
(C) A - 2, B - 1, C - 3, D - 3
(A) xn + 1 = e-x n
(B) xn + 1 = xn - e-x n

(D) A - 3, B - 2, C - 1, D - 2 -x x 2 - e-x (1 - xn) - 1


(C) xn + 1 = (1 + xn) e -x
n n

(D) xn + 1 = n
1.33 The Eigen values of following matrix are 1+e n
xn - e-x n

R V
S- 1 3 5 W 1
S- 3 - 1 6 W 1.41 The residue of the function f (z) = at z = 2 is
SS 0 0 3 WW (z + 2) 2 (z - 2) 2
T X (A) - 1 (B) - 1
(A) 3, 3 + 5j, 6 - j (B) - 6 + 5j, 3 + j, 3 - j 32 16
(C) 3 + j, 3 - j, 5 + j (D) 3, - 1 + 3j, - 1 - 3j (C) 1 (D) 1
16 32
0 1
Consider the matrix P = =
- 2 - 3G
2008 ONE MARKS 1.42 . The value of e p is
p11 p12
All the four entries of the 2 # 2 matrix P = =
p21 p22 G
2e-2 - 3e-1 e-1 - e-2 e-1 + e-1 2e-2 - e-1
1.34 are nonzero, (A) > -2 H (B) >2e-1 - 4e2 3e-1 + 2e-2H
2e - 2e-1 5e-2 - e-1
and one of its eigenvalue is zero. Which of the following statements
5e-2 - e-1 3e-1 - e-2 2e-1 - e-2 e-1 - e-2
is true? (C) > -2 H (D) > H
2e - 6e-1 4e-2 + 6-1 - 2e + 2e - e-1 + 2e-2
-1 -2
(A) p11 p12 - p12 p21 = 1 (B) p11 p22 - p12 p21 =- 1
(C) p11 p22 - p12 p21 = 0 (D) p11 p22 + p12 p21 = 0 1.43 In the Taylor series expansion of exp (x) + sin (x) about the point
x = p , the coefficient of (x - p) 2 is
1.35 The system of linear equations (A) exp (p) (B) 0.5 exp (p)
4x + 2y = 7 (C) exp (p) + 1 (D) exp (p) - 1
2x + y = 6 has
(A) a unique solution
1.44 The value of the integral of the function g (x, y) = 4x3 + 10y 4 along
the straight line segment from the point (0, 0) to the point (1, 2) in
(B) no solution
the x - y plane is
(C) an infinite number of solutions (A) 33 (B) 35
(D) exactly two distinct solutions (C) 40 (D) 56
1.36 The equation sin (z) = 10 has 1.45 Consider points P and Q in the x - y plane, with P = (1, 0) and
(A) no real or complex solution Q
(B) exactly two distinct complex solutions Q = (0, 1). The line integral 2
P
#
(xdx + ydy) along the semicircle
(C) a unique solution with the line segment PQ as its diameter
(D) an infinite number of complex solutions (A) is - 1
(B) is 0
1.37 For real values of x , the minimum value of the function
GATE Electronics and Communication Topicwise Solved Paper by RK Kanodia & Ashish Murolia Page 4

(C) is 1 1.53 Three functions f1 (t), f2 (t) and f3 (t) which are zero outside the interval
(D) depends on the direction (clockwise or anit-clockwise) of the [0, T] are shown in the figure. Which of the following statements is
semicircle correct?

2007 ONE MARK

1.46 The following plot shows a function which varies linearly with x .
2
The value of the integral I = #1 ydx is

(A) 1.0 (B) 2.5


(C) 4.0 (D) 5.0

1.47 For x << 1, coth (x) can be approximated as


(A) x (B) x2
GATE Electronics & Communication (A) f1 (t) and f2 (t) are orthogonal
by RK Kanodia (B) f1 (t) and f3 (t) are orthogonal
Now in 3 Volume (C) f2 (t) and f3 (t) are orthogonal
D) f1 (t) and f2 (t) are orthonormal
Purchase Online at maximum discount from online store
and get POSTAL and Online Test Series Free 1.54 If the semi-circular control D of radius 2 is as shown in the figure,
visit www.nodia.co.in then the value of the integral # 2
1 ds is
D
(s - 1)

(C) 1 (D) 12
x x

sin b q l
2
1.48 lim is
q"0 q
(A) 0.5 (B) 1
(C) 2 (D) not defined (A) jp (B) - jp
(C) - p (D) p
1.49 Which one of following functions is strictly bounded?
(A) 1/x2 (B) ex 1.55 It is given that X1, X2 ...XM at M non-zero, orthogonal vectors.
The dimension of the vector space spanned by the 2M vectors
(C) x2 (D) e - x
2

X1, X2,... XM , - X1, - X2,... - XM is


1.50 For the function e - x , the linear approximation around x = 2 is (A) 2M (B) M + 1
(A) (3 - x) e - 2 (B) 1 - x (C) M
(C) 63 + 3 2 - (1 - 2 ) x @e - 2 (D) e - 2 (D) dependent on the choice of X1, X2,... XM

For more GATE Resources, Mock Test and


2007 TWO MARKS
Study material join the community
1.51
d2 y
The solution of the differential equation k2 2 = y - y2 under the http://www.facebook.com/gateec2014
boundary conditions dx
1.56 Consider the function f (x) = x2 - x - 2 . The maximum value of f (x)
(i) y = y1 at x = 0 and in the closed interval [- 4, 4] is
(ii) y = y2 at x = 3 , where k, y1 and y2 are constants, is (A) 18 (B) 10
(C) - 225 (D) indeterminate
(A) y = (y1 - y2) exp a- x2 k + y2 (B) y = (y2 - y1) exp a- x k + y1
k k
1.57 An examination consists of two papers, Paper 1 and Paper 2. The
(C) y = ^y1 - y2h sinh a x k + y1 (D) y = ^y1 - y2h exp a- x k + y2 probability of failing in Paper 1 is 0.3 and that in Paper 2 is 0.2.
k k
Given that a student has failed in Paper 2, the probability of failing
1.52 The equation x3 - x2 + 4x - 4 = 0 is to be solved using the Newton in Paper 1 is 0.6. The probability of a student failing in both the
- Raphson method. If x = 2 is taken as the initial approximation of papers is
the solution, then next approximation using this method will be (A) 0.5 (B) 0.18
(A) 2/3 (B) 4/3 (C) 0.12 (D) 0.06
(C) 1 (D) 3/2
GATE Electronics and Communication Topicwise Solved Paper by RK Kanodia & Ashish Murolia Page 5

2006 ONE MARK


R V
S1 1 1 W 1.67 Three companies X, Y and Z supply computers to a university. The
1.58 The rank of the matrix S1 - 1 0 W is percentage of computers supplied by them and the probability of
SS1 1 1 WW those being defective are tabulated below
(A) 0 T X (B) 1
Company % of Computer Sup- Probability of being
(C) 2 (D) 3
plied supplied defective
1.59 4#4# P , where P is a vector, is equal to X 60% 0.01
(A) P # 4# P - 4 2 P Y 30% 0.02
(B) 4 2 P + 4 (4 # P) Z 10% 0.03
(C) 4 2 P + 4# P
Given that a computer is defective, the probability that was sup-
(D) 4 (4$ P) - 4 2 P
plied by Y is
(A) 0.1 (B) 0.2
1.60 ## (4 # P) $ ds , where P is a vector, is equal to
(A) # P $ dl (C) 0.3 (D) 0.4
4 2
For the matrix =
2 4G
(B) # 4#4# P $ dl 1.68 the eigenvalue corresponding to the eigenvector
101
=101G is
(C) # 4# P $ dl
(D) ### 4$ Pdv
SPECIAL EDITION ( STUDY MATERIAL FORM )
1.61 A probability density function is of the form At market Book is available in 3 volume i.e. in 3 book binding
p (x) = Ke- a x , x ! (- 3, 3) form. But at NODIA Online Store book is available in 10 book
The value of K is binding form. Each unit of Book is in separate binding.
(A) 0.5 (B) 1 Available Only at NODIA Online Store
(C) 0.5a (D) a Click to Buy
1.62 A solution for the differential equation xo (t) + 2x (t) = d (t) with www.nodia.co.in
initial condition x (0-) = 0 is
(A) e - 2t u (t) (B) e2t u (t) (A) 2 (B) 4
-t t
(C) e u (t) (D) e u (t) (C) 6 (D) 8

d2 y
1.69 For the differential equation 2 + k2 y = 0 the boundary conditions
2006 TWO MARKS dx
are
1.63 The eigenvalue and the corresponding eigenvector of 2 # 2 matrix
are given by (i) y = 0 for x = 0 and (ii) y = 0 for x = a
Eigenvalue Eigenvector The form of non-zero solutions of y (where m varies over all inte-
1 gers) are
l1 = 8 v1 = = G
1 (A) y = Am sin mpx
/ (B) y = Am cos mpx
/
m
a m
a
1
l2 = 4 v2 = = G mp
(C) y = / Am x a
mpx
(D) y = / Am e - a
-1 m m
The matrix is
6 2 4 6
(A) = G (B) =
6 4G
1.70 As x increased from - 3 to 3, the function f (x) = ex
2 6 1 + ex
2 4 4 8 (A) monotonically increases
(C) = G (D) =
4 2 8 4G (B) monotonically decreases
(C) increases to a maximum value and then decreases
1.64 For the function of a complex variable W = ln Z (where, W = u + jv
and Z = x + jy , the u = constant lines get mapped in Z -plane as (D) decreases to a minimum value and then increases
(A) set of radial straight lines (B) set of concentric circles
(C) set of confocal hyperbolas (D) set of confocal ellipses 2005 ONE MARK

1 dz is positive sense is The following differential equation has


1.65 The value of the constant integral # z2 + 4
1.71

d2 y dy 3
jp
z-j = 2 3 c 2 m + 4 c m + y2 + 2 = x
(A) (B) - p dt dt
2 2
jp (A) degree = 2 , order = 1 (B) degree = 1, order = 2
(C) - (D) p
2 2 (C) degree = 4 , order = 3 (D) degree = 2 , order = 3
p
1.66 The integral #0 sin3 qdq is given by 1.72 A fair dice is rolled twice. The probability that an odd number will
follow an even number is
(A) 1 (B) 2
2 3 (A) 1/2 (B) 1/6
(C) 4 (D) 8 (C) 1/3 (D) 1/4
3 3
GATE Electronics and Communication Topicwise Solved Paper by RK Kanodia & Ashish Murolia Page 6

1.73 A solution of the following differential equation is given by


d2 y dy 2 - 0.1 1
a
2
-5
dx
+ 6y = 0 1.78 Let, A = = G and A - 1 = = 2 G. Then (a + b) =
dx 0 3 0 b
(A) y = e2x + e-3x (A) 7/20 (B) 3/20
(B) y = e2x + e3x (C) 19/60 (D) 11/20
(C) y = e-2x + 33x 2
1 exp c- x m dx is
3
(D) y = e-2x + e-3x
1.79 The value of the integral I =
2p
#0 8
(A) 1 (B) p
2005 TWO MARKS (C) 2 (D) 2p

1.74 In what range should Re (s) remain so that the Laplace transform of 1.80 Given an orthogonal matrix
R1 1 1 1 V
the function e(a + 2) t + 5 exits. S W
(A) Re (s) > a + 2 S1 1 - 1 - 1 W
A =S
(B) Re (s) > a + 7 1 - 1 0 0W
S W
S0 0 1 1 W
(C) Re (s) < 2
6AA @- 1 is
T T X
(D) Re (s) > a + 5
R1 V R1 V
1.75 The derivative of the symmetric function drawn in given figure will S 4 0 0 0W S2 0 0 0W
S 0 14 0 0 W S0 1
2 0 0
W
look like (A) S 1 W (B) S 1 W
S0 0 2 0W S0 0 2 0W
GATE Electronics & Communication S 0 0 0 12 W S0 0 0 12 W
T X TR XV
by RK Kanodia R1 0 0 0 V
S4
1
0 0 0W
S W
Now in 3 Volume S0 1 0 0 W S0 1
4 0 0
W
(C) S (D) S W
Purchase Online at maximum discount from online store 0 0 1 0W S0 0 14 0 W
S W
and get POSTAL and Online Test Series Free S0 0 0 1 W S0 0 0 14 W
T X T X
visit www.nodia.co.in

1.76 Match the following and choose the correct combination:


Group I Group 2
E. Newton-Raphson method 1. Solving nonlinear equations For more GATE Resources, Mock Test and
F. Runge-kutta method 2. Solving linear simultaneous Study material join the community
equations http://www.facebook.com/gateec2014
G. Simpson’s Rule 3. Solving ordinary differential
equations
H. Gauss elimination 4. Numerical integration
5. Interpolation
6. Calculation of Eigenvalues
(A) E - 6, F - 1, G - 5, H - 3 (B) E - 1, F - 6, G - 4, H - 3
(C) E - 1, F - 3, G - 4, H - 2 (D) E - 5, F - 3, G - 4, H - 1
-4 2
Given the matrix =
4 3G
1.77 , the eigenvector is
3 4
(A) = G (B) = G
2 3
2 -1
(C) = G (D) = G
-1 2
GATE Electronics and Communication Topicwise Solved Paper by RK Kanodia & Ashish Murolia Page 7

SOLUTIONS Consider the given matrix be


R
S2 1 1 1W
V
S1 2 1 1W
Im + AB = S
S1 1 2 1WW
S1 1 1 2W
T X
1.1 Option (B) is correct. where m = 4 so, we obtain
R V R V
Here, as we know S2 1 1 1W S1 0 0 0W
Lim sin q . 0
S1 2 1 1W S0 1 0 0W
AB = S -
q"0
S1 1 2 1WW SS0 0 1 0WW
but for 10% error, we can check option (B) first, S1 1 1 2W S0 0 0 1W
T X T X
q = 18c = 18c # p = 0.314 R V R V
180c
S1 1 1 1W S1W 61 1 1 1@
sin q = sin 18c = 0.309 S1 1 1 1W S1W
=S =
S1 1 1 1WW SS1WW
% error = 0.314 - 0.309 # 100% = 0.49%
0.309 S1 1 1 1W S1W
Now, we check it for q = 50c Hence, we get T X T X

q = 50c = 50c # p = 0.873


180c
sin q = sin 50c = 0.77 SPECIAL EDITION ( STUDY MATERIAL FORM )
% error = 0.77 - 0.873 =- 12.25% At market Book is available in 3 volume i.e. in 3 book binding
0.873
form. But at NODIA Online Store book is available in 10 book
so, the error is more than 10% . Hence, for error less than 10%,
binding form. Each unit of Book is in separate binding.
q = 18c can have the approximation Available Only at NODIA Online Store
sin q . q
Click to Buy
Option (A) is correct.
1.2
www.nodia.co.in
For, a given matrix 6A@ the eigen value is calculated as
A - lI = 0 R V
S1W
where l gives the eigen values of matrix. Here, the minimum eigen S1W
A = S W, B = 81 1 1 1B
value among the given options is S1W
l =0 S1W
T X
We check the characteristic equation of matrix for this eigen value R V
Therefore, BA = 81 1 1 1B S1W = 4
A - lI = A (for l = 0 )
S1W
S1W
3 5 2 S W
= 5 12 7 S1W
From the given property T X
2 7 5
Det ^Im + AB h = Det ^Im + BAh
= 3 ^60 - 49h - 5 ^25 - 14h + 2 ^35 - 24h R V ZR V _
S2 1 1 1W ]S1 0 0 0W b
= 33 - 55 + 22 S1 2 1 1W ]S0 1 0 0W b
& Det S W = Det [S W + 4`
=0 S1 1 2 1W 0 0 1 0W
]S b
Hence, it satisfied the characteristic equation and so, the minimum S1 1 1 2W ]S0 0 0 1W b
T X \T X a
eigen value is
= 1+4
l =0
=5
1.3 Option (D) is correct.
Given, the polynomial Note : Determinant of identity matrix is always 1.
f ^x h = a 4 x 4 + a 3 x3 + a2 x2 + a1 x - a 0 1.5 Option (D) is correct.
Since, all the coefficients are positive so, the roots of equation is t dx + x = t
dt
given by dx + x = 1
f ^x h = 0 dt t
It will have at least one pole in right hand plane as there will be dx + Px = Q (General form)
dt
least one sign change from ^a1h to ^a 0h in the Routh matrix 1 st col-
IF = e # = e = e lnt = t
1

Integrating factor, Pdt # dt


t

umn. Also, there will be a corresponding pole in left hand plane


i.e.; at least one positive root (in R.H.P) Solution has the form,
and at least one negative root (in L.H.P) x # IF = # ^Q # IF hdt + C
Rest of the roots will be either on imaginary axis or in L.H.P
1.4 Option (B) is correct. x # t = # (1) (t) dt + C
GATE Electronics and Communication Topicwise Solved Paper by RK Kanodia & Ashish Murolia Page 8

2
dy (t)
xt = t + C = e-t u (t) + te-t u (t)
2 dt
Taking the initial condition, dy
At t = 0+ , = e0 + 0 = 1
dt t = 0
x (1) = 0.5
+

0.5 = 1 + C & C = 0
2 1.9 Option (A) is correct.
2 Divergence of A in spherical coordinates is given as
So, xt = t & x = t
2 2
d:A = 12 2 (r 2 Ar ) = 12 2 (krn + 2)
1.6 Option (C) is correct. r 2r r 2r

f (z) = 1 - 2 = k2 (n + 2) rn + 1
z+1 z+3 r
1 = k (n + 2) rn - 1 = 0 (given)
2 pj # f (z) dz
C
= sum of the residues of the poles which lie
n+2 = 0 & n =- 2
inside the given closed region.
1.10 Option (C) is correct.
C & z+1 = 1
Probability of appearing a head is 1/2. If the number of required
Only pole z =- 1 inside the circle, so residue at z =- 1 is.
tosses is odd, we have following sequence of events.
f (z) = -z + 1
(z + 1) (z + 3) H, TTH, TTTTH, ...........
(z + 1) (- z + 1) 2
P = 1 + b 1 l + b 1 l + .....
3 5
= lim = =1 Probability
z "- 1 (z + 1) (z + 3) 2 2 2 2
1
GATE Electronics & Communication P = 2
=2
3
1 - 14
by RK Kanodia
Now in 3 Volume 1.11 Option (B) is correct.
Purchase Online at maximum discount from online store f (x) = x3 - 9x2 + 24x + 5
and get POSTAL and Online Test Series Free df (x)
= 3x2 - 18x + 24 = 0
visit www.nodia.co.in dx
df (x)
1 & = x2 - 6x + 8 = 0 x = 4, x = 2
So
2 pj # f (z) dz
C
=1 dx
d 2 f (x)
Option (A) is correct. = 6x - 18
1.7
dx 2
x= - 1 = i = cos p + i sin p d 2 f (x)
2 2 For x = 2, = 12 - 18 =- 6 < 0
p
dx2
So, x = ei 2 So at x = 2, f (x) will be maximum
xx = ^ei 2 h & ^ei 2 h = e- 2
p x p i p
f (x) max
= (2) 3 - 9 (2) 2 + 24 (2) + 5
1.8 Option (D) is correct. = 8 - 36 + 48 + 5 = 25
d 2 y (t) 2dy (t) Option (B) is correct.
+ + y (t) = d (t) 1.12
dt 2 dt
Characteristic equation.
By taking Laplace transform with initial conditions
A - lI = 0
dy
;s Y (s) - sy (0) - dt E + 2 [sy (s) - y (0)] + Y (s) = 1
2
-5 - l -3
t=0 =0
2 -l
6s Y (s) + 2s - 0@ + 2 6sY (s) + 2@ + Y (s) = 1
2
&
Y (s) [s2 + 2s + 1] = 1 - 2s - 4 5l + l2 + 6 = 0
For more GATE Resources, Mock Test and
Y (s) = 2- 2s - 3
s + 2s + 1 Study material join the community
http://www.facebook.com/gateec2014
L
We know that, If, y (t) Y (s)
dy (t) L
then, sY (s) - y (0) l2 + 5l + 6 = 0
dt
(- 2s - 3) s Since characteristic equation satisfies its own matrix, so
So, sY (s) - y (0) = +2
(s2 + 2s + 1) A2 + 5A + 6 = 0 & A2 =- 5A - 6I
Multiplying with A
2 2
A3 + 5A2 + 6A = 0
= - 2s - 32 s + 2s + 4s + 2
(s + 2s + 1) A3 + 5 (- 5A - 6I) + 6A = 0
sY (s) - y (0) = s + 2 2 = s + 1 2 + 1 A3 = 19A + 30I
(s + 1) (s + 1) (s + 1) 2
1.13 Option (D) is correct.
= 1 + 1
s + 1 (s + 1) 2 From Divergence theorem, we have
Taking inverse Laplace transform v = #A
v Adv
### 4$ v $ nt ds
s
GATE Electronics and Communication Topicwise Solved Paper by RK Kanodia & Ashish Murolia Page 9

The position vector Thus P (E) = No. of favourable outcomes = 15 = 5


No. of total outcomes 36 12
rv = ^utx x + uty y + utz z h
v = 5rv, thus
Here, A 1.19 Option (C) is correct.
4$ A v Eigen value of a Skew-symmetric matrix are either zero or pure
imaginary in conjugate pairs.
= cutx 2 + uty 2 + utz 2 m : ^utx x + uty y + utz z h
2x 2y 2z 1.20 Option (C) is correct.
dy dz For a function x (t) trigonometric fourier series is
= c dx +
dx dy dz m
+ 5 = 3 # 5 = 15
3
x (t) = Ao + / [An cos nwt + Bn sin nwt]
So, ## 5rv $ nt ds = ### 15 dv = 15V
s
n=1

1.14 Option (C) is correct. Where, Ao = 1 # x (t) dt T0 " fundamental period


T0 T
dy 0

We have = ky
dx
An = 2 # x (t) cos nwt dt
dy T0 T
Integrating # y
= # k dx + A 0

or ln y = kx + A Bn = 2 # x (t) sin nwt dt


T0 T
0

Since y (0) = c thus ln c = A For an even function x (t), Bn = 0


So, we get, ln y = kx + ln c Since given function is even function so coefficient Bn = 0 , only cosine
or ln y = ln ekx + ln c
SPECIAL EDITION ( STUDY MATERIAL FORM )
or y = cekx
At market Book is available in 3 volume i.e. in 3 book binding
1.15 Option (A) is correct. form. But at NODIA Online Store book is available in 10 book
C R Integrals is # - 2
3z + 4 dz where C is circle z = 1
binding form. Each unit of Book is in separate binding.
C
z + 4z + 5
Available Only at NODIA Online Store
# f (z) dz = 0 if poles are outside C.
C Click to Buy
www.nodia.co.in
2
Now z + 4z + 5 = 0
(z + 2) 2 + 1 = 0
and constant terms are present in its fourier series representation.
Thus z1, 2 =- 2 ! j & z1, 2 > 1
Constant term :
So poles are outside the unit circle.
A0 = 1 # x (t) dt = 1 : # Adt + # - 2AdtD
3T/4 T/4 3T/4

1.16 Option (C) is correct. T -T/4 T -T/4 T/4

We have f (x) = x + x - 3 = 0 = 1 :TA - 2AT D =- A


T 2 2 2
f l (x) = 1 + 1 Constant term is negative.
2 x
Substituting x 0 = 2 we get 1.21 Option (D) is correct.
f l (x 0) = 1.35355 and f (x 0) = 2 + 2 - 3 = 0.414 Given differential equation
Newton Raphson Method d 2 n (x) n (x)
- 2 =0
f (x 0) dx 2 L
x1 = x 0 - lx
f l (x 0) Let n (x) = Ae
lx
Substituting all values we have So, Al2 elx - Ae2 = 0
L
x 1 = 2 - 0.414 = 1.694
1.3535
l2 - 12 = 0 & l = ! 1
L L
1.17 Option (B) is correct.
Writing A: B we have Boundary condition, n (3) = 0 so take l =- 1
L
R V
S1 1 1 : 6 W n (x) = Ae- L
x

S1 4 6 : 20W
S W n (0) = Ae0 = K & A = K
S1 4 l : m W
T X So, n (x) = Ke- (x/L)
Apply R 3 " R 3 - R2
R V
1.22 Option (A) is correct.
S1 1 1 : 6 W
Given that
1
ey = x x
S1 4 6 : 20 W
1
S W or ln ey = ln x x
S0 0 l - 6 : m - 20W
T X
For equation to have solution, rank of A and A: B must be same. or y = 1 ln x
x
Thus for no solution; l = 6, m ! 20
= 1 1 + ln x ^- x- x h = 12 - ln2
dy 1
Now 2

1.18 Option (C) is correct. dx xx x x


For maxima and minima :
Total outcome are 36 out of which favorable outcomes are :
dy
(1, 2), (1, 3), (1, 4), (1, 5), (1, 6), (2, 3), (2, 4), (2, 5), (2, 6); = 12 (1 - ln x) = 0
dx x
(3, 4), (3, 5), (3, 6), (4, 5), (4, 6), (5, 6) which are 15.
ln x = 1 " x = e 1
GATE Electronics and Communication Topicwise Solved Paper by RK Kanodia & Ashish Murolia Page 10

d 2y From table, at x = 0.3, y (x = 0.3) = 0.031


Now =- 23 - ln x b- 23 l - 12 b 1 l
dx 2 x x x x 1.27 Option (D) is correct.
=- 22 + 2 ln3 x - 13 Given that
x x x
f (t) = L - 1 ; 3 3s + 1
s + 4s 2 + (K - 3) s E
2
d x = -22 + 23 - 13 < 0
dy 2 at x = e e e e
lim f (t) = 1
1

1
So, y has a maximum at x = e t"3

1.23 Option (D) is correct. By final value theorem


According to given condition head should comes 3 times or 4 times lim f (t) = lim sF (s) = 1
t"3 s"0

P (Heads comes 3 times or 4 times) = 4C 4 b 1 l + 4C 3 b 1 l b 1 l


4 3
s : (3s + 1)
2 2 2 or lim =1
s"0 s3 + 4s2 + (K - 3) s
s (3s + 1)
or lim 2 =1
= 1: 1 +4:1 :1 = 5 s " 0 s [s + 4s + (K - 3)]
16 8 2 16
1 =1
1.24 Option (C) is correct. K-3
v = xyatx + x 2 aty
A or K =4
v = dxatx + dyaty
dl 1.28 Option (B) is correct.
# Av : dl
v = # (xyatx + x 2 aty) : (dxatx + dyaty) The highest derivative terms present in DE is of 2nd order.
C C
1.29 Option (C) is correct.
GATE Electronics & Communication Number of elements in sample space is 210 . Only one element
by RK Kanodia "H, H, T, T, T, T, T, T, T, T , is event. Thus probability is 10
1
2
Now in 3 Volume 1.30 Option (C) is correct.
Purchase Online at maximum discount from online store We have
and get POSTAL and Online Test Series Free f (z) = c0 + c1 z - 1
visit www.nodia.co.in f1 (z) =
1 + f (z) 1 + c0 + c1 z - 1
= =
z (1 + c0) + c1
z z z2
= # (xydx + x 2 dy) Since f1 (z) has double pole at z = 0 , the residue at z = 0 is
z (1 + c0) + c1
C
Res f1 (z) z = 0 = lim z2 .f1 (z) = lim z2 . c m = c1
= #1/
2/ 3
xdx +
1/ 3
#2/ 3xdx + #1
3 4 dy + #3
1 1 dy z"0 z"0 z2
3 3 3 3
Hence
= 1 : 4 - 1 D + 3 :1 - 4 D + 4 [3 - 1] + 1 [1 - 3] [1 + f (z)]
2 3 3 2 3 3 3 3 #
f1 (z) dz = # z
dz = 2pj [Residue at z = 0 ]
=1 unit circle unit circle

1.25 Option (C) is correct. = 2pjc1


Given function 1.31 Option (D) is correct.
X (z ) =1 - 2z We have f (x) = sin x
z (z - 1) (z - 2) x-p
Poles are located at z = 0, z = 1, and z = 2 Substituting x - p = y ,we get
At Z = 0 residues is sin (y + p) sin y
f (y + p) = =- = - 1 (sin y)
y y y
R 0 = z : X (z) Z = 0 = 1 - 2 # 0 = 1
(0 - 1) (0 - 2) 2 3 5
= - 1 cy - + - ...m
y y
at z = 1, R1 = (Z - 1) : X (Z ) Z = 1 y 3! 5!
For more GATE Resources, Mock Test and
= 1-2#1 = 1
1 (1 - 2) Study material join the community
At z = 2 , R2 = (z - 2) : X (z) z = 2 http://www.facebook.com/gateec2014
= 1 - 2 # 2 =- 3 y2 y 4
2 (2 - 1) 2 or f (y + p) =- 1 +- + ...
3! 5!
1.26 Option (B) is correct.
Substituting x - p = y we get
Taking step size h = 0.1, y (0) = 0
(x - p) 2 (x - p) 4
x y dy dy f (x) =- 1 + - + ...
= x+y yi + 1 = yi + h 3! 5!
dx dx
1.32 Option (A) is correct.
0 0 0 y1 = 0 + 0.1 (0) = 0
dy y
(A) =
0.1 0 0.1 y2 = 0 + 0.1 (0.1) = 0.01 dx x
dy
0.2 0.01 0.21 y 3 = 0.01 + 0.21 # 0.1 = 0.031 or # = dx #
y x
0.3 0.031 or log y = log x + log c
GATE Electronics and Communication Topicwise Solved Paper by RK Kanodia & Ashish Murolia Page 11

or y = cx Straight Line The Newton-Raphson iterative formula is


Thus option (A) and (C) may be correct. f (xn)
xn + 1 = xn -
dy y f'( xn)
(B) =-
dx x Now f (xn) = xn - e - x n

dy f'( xn) = 1 + e - x
=- dx
# # n
or
y x - xn
(1 + xn) e - x
xn + 1 = xn - xn - e- x =
n

or log y =- log x + log c Thus


1+e n
1 + e-x n

or log y = log 1 + log c 1.41 Option (A) is correct.


x
1 dn - 1 6(z - a) n f (z)@
or y =c Hyperbola Res f (z) z = a =
(n - 1)! dzn - 1 z=a
x
1.33 Option (D) is correct. Here we have n = 2 and a = 2
Sum of the principal diagonal element of matrix is equal to the sum 1 d (z - 2) 2 1
(2 - 1)! dz ; (z - 2) (z + 2) 2 Ez = a
Thus Res f (z) z = 2 =
of Eigen values. Sum of the diagonal element is - 1 - 1 + 3 = 1.In 2

only option (D), the sum of Eigen values is 1.


= d ; 1 -2
dz (z + 2) 2 Ez = a ; (z + 2) 3 Ez = a
=
1.34 Option (C) is correct.
The product of Eigen value is equal to the determinant of the matrix.
Since one of the Eigen value is zero, the product of Eigen value is =- 2 =- 1
64 32
zero, thus determinant of the matrix is zero. SPECIAL EDITION ( STUDY MATERIAL FORM )
Thus p11 p22 - p12 p21 = 0 At market Book is available in 3 volume i.e. in 3 book binding
form. But at NODIA Online Store book is available in 10 book
1.35 Option (B) is correct. binding form. Each unit of Book is in separate binding.
The given system is Available Only at NODIA Online Store
4 2 x 7
=2 1G=y G = = 6 G
Click to Buy
www.nodia.co.in
4 2
A ==
2 1G
We have
1.42 Option (D) is correct.
4 2
and A =
2 1
=0 Rank of matrix r (A) < 2 eP = L- 1 6(sI - A) - 1@
s 0 0 1 -1
= L e=
0 s G =- 2 - 3Go
4 2 7
C == G
-1
Now Rank of matrix r (C) = 2 -
2 1 6
s - 1 -1
= L e= o
2 s + 3G
Since r (A) ! r (C) there is no solution. -1

1.36 Option (A) is correct. s+3 1


sin z can have value between - 1 to + 1. Thus no solution. = L f> Hp
-1 (s + 1)( s + 2) (s + 1)( s + 2)
-2 s
(s + 1)( s + 2) (s + 1)( s + 2)
1.37 Option (A) is correct.
2e - 1 - e - 2 e-1 - e-2
== G
We have f (x) = ex + e-x
For x > 0 , ex > 1 and 0 < e-x < 1 - 2e - 1 + 2e - 2 - e - 1 + 2e - 2
For x < 0 , 0 < ex < 1 and e-x > 1 1.43 Option (B) is correct.
Thus f (x) have minimum values at x = 0 and that is e0 + e-0 = 2 . Taylor series is given as
(x - a) 2
1.38 Option (A) is correct. f (x) = f (a) + x - a f'( a) + f"( a) + ...
3 5
1! 2!
sin x = x + x + x + ... For x = p we have
3! 5!
2
(x - p)
f (x) = f (p) + x - p f'( p) +
2 4
cos x = 1 + x + x + ... Thus
1! 2!
f"( x)...
2! 4!
Thus only sin (x3) will have odd power of x . Now = ex + sin x
f (x)
1.39 Option (B) is correct. = ex + cos x
f'( x)
dx (t) = ex - sin x
f"( x)
We have + 3x (t) = 0
dt = e p - sin p = e p
f"( p)
or (D + 3) x (t) = 0 f"( p)
Thus the coefficient of (x - p) 2 is
Since m =- 3 , x (t) = Ce - 3t Thus only (B) may be 2!
solution. 1.44 Option (A) is correct.
The equation of straight line from (0, 0) to (1, 2) is y = 2x .
1.40 Option (C) is correct.
Now g (x, y) = 4x3 + 10y 4
We have x = e-x
or, g (x, 2x) = 4x3 + 160x 4
or f (x) = x - e - x 1 1
f'( x) = 1 + e -x Now #0 g (x, 2x) = #0 (4x3 + 160x4) dx
GATE Electronics and Communication Topicwise Solved Paper by RK Kanodia & Ashish Murolia Page 12

- y22
c 2 m = y2
= [x 4 + 32x5] 10 = 33 P.I. = 1
1.45 Option (B) is correct. D2 - 12 k
Thus solution is k
Q Q Q
I =2 #P (xdx + ydy) = 2 #P xdx + 2 #P ydy x x

y = C1 e - + C2 e + y2
k k

0 1 From y (0) = y1 we get


=2 #1 xdx + 2 #0 ydy = 0
C1 + C2 = y1 - y2
1.46 Option (B) is correct. From y (3) = y2 we get that C1 must be zero.
The given plot is straight line whose equation is Thus C2 = y1 - y2
x +y =1 x

-1 1 y = (y1 - y2) e - + y2
k

or y = x+1 1.52 Option (B) is correct.


2 2 We have
Now I = #1 ydx = #1 (x + 1) dx f (x) = x3 - x2 + 4x - 4
(x + 1) 2 2
=; E = - = 2.5
9 4 f'( x) = 3x2 - 2x + 4
2 2 2 Taking x0 = 2 in Newton-Raphosn method
1.47 Option (C) is correct. f (x0) 23 - 22 + 4 (2) - 4
x1 = x0 - = 2- =4
coth x = cosh x f'( x0) 3 (2) 2 - 2 (2) + 4 3
sinh x
1.53 Option (C) is correct.
as x << 1, cosh x . 1 and sinh x . x
For two orthogonal signal f (x) and g (x)
GATE Electronics & Communication +3

by RK Kanodia
#- 3 f (x) g (x) dx = 0

Now in 3 Volume i.e. common area between f (x) and g (x) is zero.
Purchase Online at maximum discount from online store 1.54 Option (A) is correct.
and get POSTAL and Online Test Series Free We know that
1 ds = 2pj
visit www.nodia.co.in # 2
[sum of residues]
D
s - 1
Singular points are at s = ! 1 but only s =+ 1 lies inside the given
Thus coth x . 1
x contour, Thus Residue at s =+ 1 is
1.48 Option (A) is correct. lim (s - 1) f (s) = lim (s - 1) 2 1 = 1
sin ^ q2 h sin ^ q2 h s -1 2
1 lim sin ^ 2 h = 1 = 0.5
q s"1 s"1

= lim =
`2j
lim 1 ds = 2pj 1 = pj
q"0 q q " 0 2^ q h 2 q " 0 ^ q2 h 2 # 2
2
D
s -1
1.49 Option (D) is correct.
Option (C) is correct.
lim 12 = 3
1.55
We have,
x"0 x For two orthogonal vectors, we require two dimensions to define
lim x2 = 3 them and similarly for three orthogonal vector we require three
x"3
dimensions to define them. 2M vectors are basically M orthogonal
lim e - x = 3 vector and we require M dimensions to define them.
x"3

lim e - x = 0
2
1.56 Option (A) is correct.
x"3
We have
lim e - x = 1 Thus e - x is strictly bounded.
2 2

x"0 f (x) = x2 - x + 2
1.50 Option (A) is correct.
We have f (x) = e - x = e - (x - 2) - 2 = e - (x - 2) e - 2 For more GATE Resources, Mock Test and
(x - 2) 2 Study material join the community
= ;1 - (x - 2) + ...E e - 2
2!
http://www.facebook.com/gateec2014
= 61 - (x - 2)@ e - 2 Neglecting higher powers
-2
= (3 - x) e f'( x) = 2x - 1 = 0 " x = 1
2
1.51 Option (D) is correct.
f"( x) = 2
d2 y
We have k2 = y - y2
dx2 Since f"( x) = 2 > 0 , thus x = 1 is minimum point. The maximum
2
d2 y y y value in closed interval 6- 4, 4@ will be at x =- 4 or x = 4
or 2
- 2 =- 22
dx k k Now maximum value
= max [f (- 4), f (4)]
A.E. D2 - 12 = 0
k = max (18, 10) = 18
or D =! 1 1.57 Option (C) is correct.
k
x x Probability of failing in paper 1 is P (A) = 0.3
C.F. = C1 e - + C2 e k k

Possibility of failing in Paper 2 is P (B) = 0.2


GATE Electronics and Communication Topicwise Solved Paper by RK Kanodia & Ashish Murolia Page 13

Probability of failing in paper 1, when Now x = eu cos v and y = eu sin v


student has failed in paper 2 is P ^ BA h = 0.6 Thus x 2 + y 2 = e 2u Equation of circle
We know that 1.65 Option (D) is correct.
(P + B)
Pb A l = We have
B P (B)
1 dz = 1
or P (A + B) = P (B) P b A l = 0.6 # 0.2 = 0.12
# z 2
+ 4
# (z + 2i) (z - 2i)
dz
B z-j = 2 z-j = 2

1.58 Option (C) is correct. P (0, 2) lies inside the circle z - j = 2 and P (0, - 2) does not lie.
We have Thus By cauchy’s integral formula
R V R V 1 2pi = p
S1 1 1 W S1 1 1 W I = 2pi lim (z - 2i) = #
A = S1 - 1 0 W + S1 - 1 0 W R3 - R1 z " 2i (z + 2i)( z - 2i) 2i + 2i 2
C
SS1 1 1 WW SS0 0 0 WW 1.66 Option (C) is correct.
T X T X p
Since one full row is zero, r (A) < 3 I = #0 sin3 qdq
1 1
` j
3 sin q - sin 3q dq
p
sin 3q = 3 sin q - 4 sin3 q
Now
1 -1
=- 2 ! 0 , thus r (A) = 2 = #0 4
= :- 3 cos qD = : ws3q D = 8 3 + 3 B - 8 1 + 1 B = 4
p p
1.59 Option (D) is correct.
4 12 0 4 4 12 12 3
The vector Triple Product is 0

A # (B # C) = B (A $ C) - C (A $ B) SPECIAL EDITION ( STUDY MATERIAL FORM )


Thus 4#4# P = 4 (4$ P) - P (4$4) At market Book is available in 3 volume i.e. in 3 book binding
= 4 (4$ P) - 4 2 P form. But at NODIA Online Store book is available in 10 book
1.60 Option (A) is correct. binding form. Each unit of Book is in separate binding.
Available Only at NODIA Online Store
The Stokes theorem is
Click to Buy
## (4 # F) $ ds = # A $ dl www.nodia.co.in
1.61 Option (C) is correct.
Option (D) is correct.
# p (x) dx
3 1.67
We know =1
-3 Let d " defective and y " supply by Y
#
3
Thus Ke- a x dx = 1 y P (y + d)
pa k =
-3 d P (d)
0
# # Ke
3
or Keax dx + - ax
dx = 1 P (y + d) = 0.3 # 0.02 = 0.006
-3 0
P (d) = 0.6 # 0.1 + 0.3 # 0.02 + 0.1 # 0.03 = 0.015
a 6 @- 3 (- a) 6 @0
or K eax 0 + k e- ax 3 = 1
y
P a k = 0.006 = 0.4
K +K =1 d 0.015
or
a a 1.68 Option (C) is correct.
K =a 4 2
A ==
2 4G
or
2 We have
1.62 Option (A) is correct.
We have xo (t) + 2x (t) = s (t) Now 6A - lI @ [X] = 0
Taking Laplace transform both sides 4 - l 2 101 0
or = 2 4 - l G=101G = = 0 G
sX (s) - x (0) + 2X (s) = 1
or sX (s) + 2X (s) = 1 Since x (0 -) = 0 or (101)( 4 - l) + 2 (101) = 0
X (s) = 1 or l =6
s+2
1.69 Option (A) is correct.
Now taking inverse Laplace transform we have
d2 y
x (t) = e u (t) - 2t We have 2
+ k2 y = 0
dx
1.63 Option (A) is correct. or D2 y + k2 y = 0
Sum of the Eigen values must be equal to the sum of element of The AE is m2 + k2 = 0
principal diagonal of matrix.
The solution of AE is m = ! ik
6 2
Only matrix =
2 6G
satisfy this condition. Thus y = A sin kx + B cos kx
From x = 0 , y = 0 we get B = 0 and x = a, y = 0 we get
1.64 Option (B) is correct.
A sin ka = 0
We have W = ln z
or sin ka = 0
u + jv = ln (x + jy)
or eu + jv = x + jy k = mpx
a
or eu e jv = x + jy Thus y = Am sin ` mpx j
/a
eu (cos v + j sin v) = x + jy m
GATE Electronics and Communication Topicwise Solved Paper by RK Kanodia & Ashish Murolia Page 14

1.70 Option (A) is correct. (A - lI) Xi = 0


x 1 - (- 5) 2 x1 0
= G= G == G
We have f (x) =e
1 + ex 4 8 - 4 x2 0
For x " 3, the value of f (x) monotonically increases.
1 2 x1 0
1.71 Option (B) is correct. =0 0G=x G = = 0 G R2 - 4R1
2
Order is the highest derivative term present in the equation and
x1 + 2x2 = 0
degree is the power of highest derivative term.
Let - x1 = 2 & x2 =- 1,
Order = 2 , degree = 1
2
X ==
-1G
1.72 Option (D) is correct. Thus Eigen vector
Probability of coming odd number is 12 and the probability of
coming even number is 12 . Both the events are independent to each
1.78 Option (A) is correct.
other, thus probability of coming odd number after an even number We have
2 - 0.1 1
a
A ==
0 3 G
is 12 # 12 = 14 . and A = = 2 G
-1
0 b
1.73 Option (B) is correct.
Now AA - 1 = I
d2 y dy
We have -5 + 6y = 0
dx 2 dx 2 - 0.1 1 a 1 0
or =0 3 G= 2 G = =0 1G
The A.E. is m2 - 5m + 6 = 0 0 b
m = 3, 2 1 2a - 0.1b 1 0
or =0 G ==
0 1G
GATE Electronics & Communication 3b
by RK Kanodia or 2a - 0.1 = 0 and 3b = 1
Now in 3 Volume Thus solving above we have b = 1 and a = 1
3 60
Purchase Online at maximum discount from online store
Therefore a+b = 1 + 1 = 7
and get POSTAL and Online Test Series Free 3 60 20
visit www.nodia.co.in 1.79 Option (A) is correct.
Gaussian PDF is
The CF is yc = C1 e3x + C2 e2x
f (x) = 1 #
3 - (x - m)2
e 2s2 dx for - 3 # x # 3
Since Q = 0 , thus y = C1 e3x + C2 e2x 2p s -3

Thus only (B) may be correct.


# f (x) dx
3
and =1
1.74 Option (A) is correct. -3

We have f (t) = e(a + 2) t + 5 = e5 .e(a + 2) t Substituting m = 0 and s = 2 in above we get


1
#
3 - x2
Taking Laplace transform we get e dx = 18

2p 2 - 3
F (s) = e5 ;
s - (a + 2) E
1 Thus Re (s) > (a + 2)
1 2
#
3 - x2
or e 8
dx = 1
1.75 Option (C) is correct. 2p 2 0

For x > 0 the slope of given curve is negative. Only (C) satisfy this
1
#
3 - x2
condition. or e 8
dx = 1
2p 0
1.76 Option (C) is correct. 1.80 Option (C) is correct.
Newton - Raphson " Method-Solving nonlinear eq. From orthogonal matrix
Runge - kutta Method " Solving ordinary differential eq. [AAT ] = I
Simpson’s Rule Numerical Integration
"
For more GATE Resources, Mock Test and
Gauss elimination Solving linear simultaneous eq.
Study material join the community
"
Option (C) is correct.
1.77
http://www.facebook.com/gateec2014
-4 2
A ==
4 3G
We have
Since the inverse of I is I , thus
Characteristic equation is [AAT ] -1 = I-1 = I
A - lI = 0
4-l 2
or =0
4 3-l
or (- 4 - l)(3 - l) - 8 = 0
or - 12 + l + l2 - 8 = 0
or l2 + l - 20 = 0
or l =- 5, 4 Eigen values
Eigen vector for l =- 5
GATE Electronics and Communication Topicwise Solved Paper by RK Kanodia & Ashish Murolia Page 15

UNIT 2
NETWORKS

(A) 125/100 and 80/100 (B) 100/100 and 80/100


(C) 100/100 and 100/100 (D) 80/100 and 80/100

2013 ONE MARK 2.6 Three capacitors C1 , C2 and C 3 whose values are 10 mF , 5 mF , and
2 mF respectively, have breakdown voltages of 10 V, 5 V and 2 V
2.1 Consider a delta connection of resistors and its equivalent star
respectively. For the interconnection shown below, the maximum
connection as shown below. If all elements of the delta connection
safe voltage in Volts that can be applied across the combination,
are scaled by a factor k , k > 0 , the elements of the corresponding
and the corresponding total charge in mC stored in the effective
star equivalent will be scaled by a factor of
capacitance across the terminals are respectively,

(A) k2 (B) k
(C) 1/k (D) k SPECIAL EDITION ( STUDY MATERIAL FORM )
At market Book is available in 3 volume i.e. in 3 book binding
V2 ^s h
The transfer function of the circuit shown below is form. But at NODIA Online Store book is available in 10 book
V1 ^s h
2.2

binding form. Each unit of Book is in separate binding.


Available Only at NODIA Online Store

Click to Buy
www.nodia.co.in

(A) 0.5s + 1 (B) 3s + 6


s+1 s+2
(C) s + 2 (D) s + 1
s+1 s+2
2.3 A source vs ^ t h = V cos 100pt has an internal impedance of ^4 + j3h W
. If a purely resistive load connected to this source has to extract the
maximum power out of the source, its value in W should be
(A) 3 (B) 4
(C) 5 (D) 7 (A) 2.8 and 36 (B) 7 and 119
(C) 2.8 and 32 (D) 7 and 80

2013 TWO MARKS

2.4 In the circuit shown below, if the source voltage VS = 100+53.13c V


Common Data For Q. 8 and 9:
then the Thevenin’s equivalent voltage in Volts as seen by the load Consider the following figure
resistance RL is

(A) 100+90c (B) 800+0c 2.7 The current IS in Amps in the voltage source, and voltage VS in
Volts across the current source respectively, are
(C) 800+90c (D) 100+60c
(A) 13, - 20 (B) 8, - 10
2.5 The following arrangement consists of an ideal transformer and (C) - 8, 20 (D) - 13, 20
an attenuator which attenuates by a factor of 0.8. An ac voltage
VWX1 = 100 V is applied across WX to get an open circuit voltage 2.8 The current in the 1W resistor in Amps is
VYZ1 across YZ. Next, an ac voltage VYZ2 = 100 V is applied across (A) 2 (B) 3.33
YZ to get an open circuit voltage VWX2 across WX. Then, VYZ1 /VWX1 (C) 10 (D) 12
, VWX2 /VYZ2 are respectively,
2.9 Two magnetically uncoupled inductive coils have Q factors q1 and q2
GATE Electronics and Communication Topicwise Solved Paper by RK Kanodia & Ashish Murolia Page 16

at the chosen operating frequency. Their respective resistances are


R1 and R2 . When connected in series, their effective Q factor at the
same operating frequency is
(A) q1 + q2 (B) ^1/q1h + ^1/q2h
(C) ^q1 R1 + q2 R2h / ^R1 + R2h (D) ^q1 R2 + q2 R1h / ^R1 + R2h

(A) 0.8 W (B) 1.4 W


2012 ONE MARK (C) 2 W (D) 2.8 W
2.10 In the following figure, C1 and C2 are ideal capacitors. C1 has been 2.14 If VA - VB = 6 V then VC - VD is
charged to 12 V before the ideal switch S is closed at t = 0. The
current i (t) for all t is

(A) zero (B) a step function (A) - 5 V (B) 2 V


(C) 3 V (D) 6 V
GATE Electronics & Communication Common Data For Q. 48 and 49 :
by RK Kanodia With 10 V dc connected at port A in the linear nonreciprocal two-
Now in 3 Volume port network shown below, the following were observed :
Purchase Online at maximum discount from online store (i) 1 W connected at port B draws a current of 3 A
(ii) 2.5 W connected at port B draws a current of 2 A
and get POSTAL and Online Test Series Free
visit www.nodia.co.in
(C) an exponentially decaying function (D) an impulse function

2.11 The average power delivered to an impedance (4 - j3) W by a current


5 cos (100pt + 100) A is
(A) 44.2 W (B) 50 W 2.15 With 10 V dc connected at port A, the current drawn by 7 W
connected at port B is
(C) 62.5 W (D) 125 W
(A) 3/7 A (B) 5/7 A
2.12 In the circuit shown below, the current through the inductor is (C) 1 A (D) 9/7 A

2.16 For the same network, with 6 V dc connected at port A, 1 W


connected at port B draws 7/3 A. If 8 V dc is connected to port A
, the open circuit voltage at port B is
(A) 6 V (B) 7 V
(C) 8 V (D) 9 V

2011 ONE MARK

For more GATE Resources, Mock Test and


Study material join the community
http://www.facebook.com/gateec2014
(A) 2 A (B) - 1 A 2.17 In the circuit shown below, the Norton equivalent current in amperes
1+j 1+j
with respect to the terminals P and Q is
(C) 1 A (D) 0 A
1+j

2012 TWO MARKS

2.13 Assuming both the voltage sources are in phase, the value of R for
which maximum power is transferred from circuit A to circuit B is

(A) 6.4 - j 4.8 (B) 6.56 - j 7.87


(C) 10 + j 0 (D) 16 + j 0

2.18 In the circuit shown below, the value of RL such that the power
GATE Electronics and Communication Topicwise Solved Paper by RK Kanodia & Ashish Murolia Page 17

transferred to RL is maximum is (A) i (t) = 15 exp (- 2 # 103 t) A


(B) i (t) = 5 exp (- 2 # 103 t) A
(C) i (t) = 10 exp (- 2 # 103 t) A
(D) i (t) =- 5 exp (- 2 # 103 t) A

2010 ONE MARK

2.23 For the two-port network shown below, the short-circuit admittance
(A) 5 W (B) 10 W
parameter matrix is
(C) 15 W (D) 20 W

2.19 The circuit shown below is driven by a sinusoidal input


vi = Vp cos (t/RC ). The steady state output vo is

4 -2 1 - 0.5
(A) >
4H
(B) >
1H
S S
-2 - 0.5

(A) (Vp /3) cos (t/RC ) (B) (Vp /3) sin (t/RC ) SPECIAL EDITION ( STUDY MATERIAL FORM )
(C) (Vp /2) cos (t/RC ) (D) (Vp /2) sin (t/RC ) At market Book is available in 3 volume i.e. in 3 book binding
form. But at NODIA Online Store book is available in 10 book
2011 TWO MARKS
binding form. Each unit of Book is in separate binding.
Available Only at NODIA Online Store
2.20 In the circuit shown below, the current I is equal to Click to Buy
www.nodia.co.in

1 0.5 4 2
(C) >
1H
(D) >
4H
S S
0.5 2
2.24 For parallel RLC circuit, which one of the following statements is
NOT correct ?
(A) The bandwidth of the circuit decreases if R is increased
(A) 1.4+0c A (B) 2.0+0c A (B) The bandwidth of the circuit remains same if L is increased
(C) 2.8+0c A (D) 3.2+0c A (C) At resonance, input impedance is a real quantity
2.21 In the circuit shown below, the network N is described by the (D) At resonance, the magnitude of input impedance attains its
following Y matrix: minimum value.
0.1 S - 0.01 S
Y => . the voltage gain V2 is
0.01 S 0.1 SH V1 2010 TWO MARKS

2.25 In the circuit shown, the switch S is open for a long time and is
closed at t = 0 . The current i (t) for t $ 0+ is

(A) 1/90 (B) –1/90


(C) –1/99 (D) –1/11
(A) i (t) = 0.5 - 0.125e-1000t A (B) i (t) = 1.5 - 0.125e-1000t A
2.22 In the circuit shown below, the initial charge on the capacitor is 2.5
mC, with the voltage polarity as indicated. The switch is closed at (C) i (t) = 0.5 - 0.5e-1000t A (D) i (t) = 0.375e-1000t A
time t = 0 . The current i (t) at a time t after the switch is closed is 2.26 The current I in the circuit shown is

(A) - j1 A (B) j1 A
GATE Electronics and Communication Topicwise Solved Paper by RK Kanodia & Ashish Murolia Page 18

(C) 0 A (D) 20 A battery deliver during this talk-time?

2.27 In the circuit shown, the power supplied by the voltage source is

(A) 220 J (B) 12 kJ


(C) 13.2 kJ (D) 14.4 J

GATE 2009 TWO MARK


(A) 0 W (B) 5 W 2.31 An AC source of RMS voltage 20 V with internal impedance
(C) 10 W (D) 100 W Zs = (1 + 2j) W feeds a load of impedance ZL = (7 + 4j) W in the
figure below. The reactive power consumed by the load is

GATE 2009 ONE MARK

2.28 In the interconnection of ideal sources shown in the figure, it is


known that the 60 V source is absorbing power.

GATE Electronics & Communication


by RK Kanodia
Now in 3 Volume (A) 8 VAR (B) 16 VAR
Purchase Online at maximum discount from online store (C) 28 VAR (D) 32 VAR
and get POSTAL and Online Test Series Free 2.32 The switch in the circuit shown was on position a for a long time,
visit www.nodia.co.in and is move to position b at time t = 0 . The current i (t) for t > 0
is given by

(A) 0.2e-125t u (t) mA (B) 20e-1250t u (t) mA


(C) 0.2e-1250t u (t) mA (D) 20e-1000t u (t) mA

2.33 In the circuit shown, what value of RL maximizes the power delivered
to RL ?
Which of the following can be the value of the current source I ?
(A) 10 A (B) 13 A
(C) 15 A (D) 18 A

2.29 If the transfer function of the following network is


Vo (s) For more GATE Resources, Mock Test and
= 1
Vi (s) 2 + sCR Study material join the community
http://www.facebook.com/gateec2014

The value of the load resistance RL is


(A) R (B) R
4 2
(C) R (D) 2R

2.30 A fully charged mobile phone with a 12 V battery is good for a 10


minute talk-time. Assume that, during the talk-time the battery
delivers a constant current of 2 A and its voltage drops linearly from
12 V to 10 V as shown in the figure. How much energy does the
GATE Electronics and Communication Topicwise Solved Paper by RK Kanodia & Ashish Murolia Page 19

(A) 2.4 W (B) 8 W The component values are


3 (A) L = 5 H, R = 0.5 W, C = 0.1 F
(C) 4 W (D) 6 W (B) L = 0.1 H, R = 0.5 W, C = 5 F
2.34 The time domain behavior of an RL circuit is represented by (C) L = 5 H, R = 2 W, C = 0.1 F
(D) L = 0.1 H, R = 2 W, C = 5 F
L di + Ri = V0 (1 + Be-Rt/L sin t) u (t).
dt
2.39 The circuit shown in the figure is used to charge the capacitor C
For an initial current of i (0) = V0 , the steady state value of the alternately from two current sources as indicated. The switches S1
R
current is given by and S2 are mechanically coupled and connected as follows:
(A) i (t) " V0 (B) i (t) " 2V0 For 2nT # t # (2n + 1) T , (n = 0, 1, 2,..) S1 to P1 and S2 to P2
R R For (2n + 1) T # t # (2n + 2) T, (n = 0, 1, 2,...) S1 to Q1 and S2 to
(C) i (t) " V0 (1 + B) (D) i (t) " 2V0 (1 + B) Q2
R R

GATE 2008 ONE MARK

2.35 In the following graph, the number of trees (P) and the number of
cut-set (Q) are

SPECIAL EDITION ( STUDY MATERIAL FORM )


At market Book is available in 3 volume i.e. in 3 book binding
form. But at NODIA Online Store book is available in 10 book
binding form. Each unit of Book is in separate binding.
(A) P = 2, Q = 2 (B) P = 2, Q = 6 Available Only at NODIA Online Store

(C) P = 4, Q = 6 (D) P = 4, Q = 10 Click to Buy


2.36 In the following circuit, the switch S is closed at t = 0 . The rate of
www.nodia.co.in
change of current di (0+) is given by
dt

(A) 0 (B) Rs Is
L
(R + Rs) Is
(C) (D) 3
L

GATE 2008 TWO MARKS

2.37 The Thevenin equivalent impedance Zth between the nodes P and Assume that the capacitor has zero initial charge. Given that u (t)
Q in the following circuit is is a unit step function , the voltage vc (t) across the capacitor is
given by
3
(A) / (- 1) n tu (t - nT)
n=1
3
(B) u (t) + 2 / (- 1) n u (t - nT)
n=1
3
(C) tu (t) + 2 / (- 1) n u (t - nT) (t - nT)
n=1

(A) 1 (B) 1 + s + 1
(D) / 60.5 - e- (t - 2nT) + 0.5e- (t - 2nT) - T @
3
s
n=1
2
(C) 2 + s + 1 (D) s2 + s + 1
s s + 2s + 1
2.38 The driving point impedance of the following network is given by
Common Data For Q. 2.23 & 2.24 :
The following series RLC circuit with zero conditions is excited by
Z (s) = 2 0.2s a unit impulse functions d (t).
s + 0.1s + 2
GATE Electronics and Communication Topicwise Solved Paper by RK Kanodia & Ashish Murolia Page 20

2.40 For t > 0 , the output voltage vC ^ t h is

(A) 2 ^e t - e t h (B) 2 te 2 t
-1 3 -1
2 2

3 3
(C) 2 e 2 t cos c 3 t m (D) 2 e 2 t sin c 3 t m
-1 -1

3 2 3 2
2.41 For t > 0 , the voltage across the resistor is
(A) a low-pass filter (B) a high-pass filter
(A) 1 _e i
3t -1t
2 -e 2 (C) a band-pass filter (D) a band-reject filter
3
3 1 sin 3 t
=cos c 2 t m - c 2 mG
-1 t
(B) e 2
3 GATE 2007 TWO MARKS
2 e -21 t sin 3 t
(C) c 2 m 2.46 Two series resonant filters are as shown in the figure. Let the 3-dB
3 bandwidth of Filter 1 be B1 and that of Filter 2 be B2 . the value
(D) 2 e 2 t cos c 3 t m
-1
B1 is
3 2 B2

Statement for linked Answers Questions 2.25 & 2.26:


A two-port network shown below is excited by external DC source.
The voltage and the current are measured with voltmeters V1, V2
GATE Electronics & Communication
(A) 4 (B) 1
by RK Kanodia
(C) 1/2 (D) 1/4
Now in 3 Volume
Purchase Online at maximum discount from online store 2.47 For the circuit shown in the figure, the Thevenin voltage and
resistance looking into X - Y are
and get POSTAL and Online Test Series Free
visit www.nodia.co.in

and ammeters. A1, A2 (all assumed to be ideal), as indicated

(A) 4
3 V, 2 W (B) 4 V, 23 W
(C) 4
3 V, 23 W (D) 4 V, 2 W

Under following conditions, the readings obtained are: 2.48 In the circuit shown, vC is 0 volts at t = 0 sec. For t > 0 , the capacitor
(1) S1 -open, S2 - closed A1 = 0,V1 = 4.5 V,V2 = 1.5 V, A2 = 1 A current iC (t), where t is in seconds is given by
(2) S1 -open, S2 - closed A1 = 4 A,V1 = 6 V,V2 = 6 V, A2 = 0

2.42 The z -parameter matrix for this network is


1.5 1.5 1.5 4.5
(A) =
1.5G
(B) =
4.5 1.5 4.5G
1.5 4.5 4.5 1.5
(C) =
1.5 G
(D) =
1.5 1.5 4.5G (A) 0.50 exp (- 25t) mA (B) 0.25 exp (- 25t) mA
2.43 The h -parameter matrix for this network is For more GATE Resources, Mock Test and
(A) =
-3 3 -3 -1 Study material join the community
- 1 0.67 G
(B) =
3 0.67 G
http://www.facebook.com/gateec2014
3 3 3 1
(C) =
1 0.67 G
(D) =
- 3 - 0.67 G (C) 0.50 exp (- 12.5t) mA (D) 0.25 exp (- 6.25t) mA

GATE 2007 ONE MARK 2.49 In the ac network shown in the figure, the phasor voltage VAB (in
Volts) is
2.44 An independent voltage source in series with an impedance
Zs = Rs + jXs delivers a maximum average power to a load impedance
ZL when
(A) ZL = Rs + jXs (B) ZL = Rs
(C) ZL = jXs (D) ZL = Rs - jXs

2.45 The RC circuit shown in the figure is


(A) 0 (B) 5+30c
(C) 12.5+30c (D) 17+30c
GATE Electronics and Communication Topicwise Solved Paper by RK Kanodia & Ashish Murolia Page 21

GATE 2006 TWO MARKS

2.50 A two-port network is represented by ABCD parameters given by


V1 A B V2
= I G = =C D G=- I G
1 2

If port-2 is terminated by RL , the input impedance seen at port-1


is given by
(A) A + BRL (B) ARL + C (A) Rneg # Re Z1 (jw), 6w (B) Rneg # Z1 (jw) , 6w
C + DRL BRL + D (C) Rneg # Im Z1 (jw), 6w (D) Rneg # +Z1 (jw), 6w
(C) DRL + A (D) B + ARL
BRL + C D + CRL

2.51 In the two port network shown in the figure below, Z12 and Z21 and GATE 2005 ONE MARK
respectively 2.56 The condition on R, L and C such that the step response y (t) in the
figure has no oscillations, is

(A) re and br0 (B) 0 and - br0 SPECIAL EDITION ( STUDY MATERIAL FORM )
(C) 0 and bro (D) re and - br0 At market Book is available in 3 volume i.e. in 3 book binding
form. But at NODIA Online Store book is available in 10 book
2.52 The first and the last critical frequencies (singularities) of a driving
binding form. Each unit of Book is in separate binding.
point impedance function of a passive network having two kinds of Available Only at NODIA Online Store
elements, are a pole and a zero respectively. The above property
will be satisfied by Click to Buy
(A) RL network only (B) RC network only www.nodia.co.in
(C) LC network only (D) RC as well as RL networks

2.53 A 2 mH inductor with some initial current can be represented as


shown below, where s is the Laplace Transform variable. The value
of initial current is

(A) R $ 1 L L
(B) R $
2 C C
(C) R $ 2 L (D) R = 1
C LC
(A) 0.5 A (B) 2.0 A 2.57 The ABCD parameters of an ideal n: 1 transformer shown in the
(C) 1.0 A (D) 0.0 A figure are
n 0
2.54 In the figure shown below, assume that all the capacitors are initially >0 x H
uncharged. If vi (t) = 10u (t) Volts, vo (t) is given by

The value of x will be


(A) n (B) 1
(A) 8e -t/0.004
Volts (B) 8 (1 - e -t/0.004
) Volts n
(C) 8u (t) Volts (D) 8 Volts (C) n2 (D) 12
n
2.55 A negative resistance Rneg is connected to a passive network N 2.58 In a series RLC circuit, R = 2 kW , L = 1 H, and C = 1 mF The
400
having driving point impedance as shown below. For Z2 (s) to be resonant frequency is
positive real, (A) 2 # 10 4 Hz (B) 1 # 10 4 Hz
p
(C) 10 4 Hz (D) 2p # 10 4 Hz

2.59 The maximum power that can be transferred to the load resistor RL
from the voltage source in the figure is
GATE Electronics and Communication Topicwise Solved Paper by RK Kanodia & Ashish Murolia Page 22

the figure, then the reading in the ideal voltmeter connected between
a and b is

(A) 1 W (B) 10 W
(C) 0.25 W (D) 0.5 W

2.60 The first and the last critical frequency of an RC -driving point
impedance function must respectively be (A) 0.238 V (B) 0.138 V
(A) a zero and a pole (B) a zero and a zero (C) - 0.238 V (D) 1 V
(C) a pole and a pole (D) a pole and a zero 2.65 The h parameters of the circuit shown in the figure are

GATE 2005 TWO MARKS

2.61 For the circuit shown in the figure, the instantaneous current i1 (t) is

0.1 0.1 10 - 1
(A) =
- 0.1 0.3G
(B) =
GATE Electronics & Communication 1 0.05G
by RK Kanodia 30 20 10 1
(C) =
20 20G
(D) =
Now in 3 Volume - 1 0.05G
Purchase Online at maximum discount from online store 2.66 A square pulse of 3 volts amplitude is applied to C - R circuit
and get POSTAL and Online Test Series Free shown in the figure. The capacitor is initially uncharged. The output
voltage V2 at time t = 2 sec is
visit www.nodia.co.in

(A) 3 V (B) - 3 V
(C) 4 V (D) - 4 V
(A) 10 3 90c A (B) 10 3 - 90c A
2 2
(C) 5 60c A (D) 5 - 60c A GATE 2004 ONE MARK

2.67 Consider the network graph shown in the figure. Which one of the
2.62 Impedance Z as shown in the given figure is
following is NOT a ‘tree’ of this graph ?

For more GATE Resources, Mock Test and


(A) j29 W (B) j9 W Study material join the community
(C) j19 W (D) j39 W http://www.facebook.com/gateec2014
2.63 For the circuit shown in the figure, Thevenin’s voltage and Thevenin’s
equivalent resistance at terminals a - b is

(A) 5 V and 2 W (B) 7.5 V and 2.5 W


(C) 4 V and 2 W (D) 3 V and 2.5 W

2.64 If R1 = R2 = R4 = R and R3 = 1.1R in the bridge circuit shown in


GATE Electronics and Communication Topicwise Solved Paper by RK Kanodia & Ashish Murolia Page 23

(A) a (B) b
(C) c (D) d

2.68 The equivalent inductance measured between the terminals 1 and 2


for the circuit shown in the figure is
GATE 2004 TWO MARKS

2.72 For the lattice shown in the figure, Za = j2 W and Zb = 2 W . The


z11 z12
values of the open circuit impedance parameters 6 z @ = =
z21 z22 G
are

(A) L1 + L2 + M (B) L1 + L2 - M
SPECIAL EDITION ( STUDY MATERIAL FORM )
(C) L1 + L2 + 2M (D)L1 + L2 - 2M
At market Book is available in 3 volume i.e. in 3 book binding
2.69 The circuit shown in the figure, with R = 1 W, L = 1 H and C = 3 F form. But at NODIA Online Store book is available in 10 book
3 4
has input voltage v (t) = sin 2t . The resulting current i (t) is binding form. Each unit of Book is in separate binding.
Available Only at NODIA Online Store

Click to Buy
www.nodia.co.in

(A) 5 sin (2t + 53.1c)


(B) 5 sin (2t - 53.1c)
(C) 25 sin (2t + 53.1c)
(D) 25 sin (2t - 53.1c)

2.70 For the circuit shown in the figure, the time constant RC = 1 ms.
The input voltage is vi (t) = 2 sin 103 t . The output voltage vo (t) is 1-j 1+j 1-j 1+j
(A) =
1 + jG
(B) =
equal to 1+j -1 + j 1 - j G
1+j 1+j 1 + j -1 + j
(C) =
1 - jG
(D) =
1-j -1 + j 1 + j G

2.73 The circuit shown in the figure has initial current iL (0-) = 1 A
through the inductor and an initial voltage vC (0-) =- 1 V across
the capacitor. For input v (t) = u (t), the Laplace transform of the
(A) sin (103 t - 45c) (B) sin (103 t + 45c) current i (t) for t $ 0 is
(C) sin (103 t - 53c) (D) sin (103 t + 53c)

2.71 For the R - L circuit shown in the figure, the input voltage
vi (t) = u (t). The current i (t) is

(A) s (B)s+2
s2 + s + 1 s2 + s + 1
(C) 2 s - 2 (D) 2 1
s +s+1 s +s+1
V (s)
2.74 The transfer function H (s) = o of an RLC circuit is given by
Vi (s)
H (s) = 106
s + 20s + 106
2

The Quality factor (Q-factor) of this circuit is


(A) 25 (B) 50
GATE Electronics and Communication Topicwise Solved Paper by RK Kanodia & Ashish Murolia Page 24

(C) 100 (D) 5000 (C) 100 (D) 200

2.75 For the circuit shown in the figure, the initial conditions are zero. Its 2.80 The differential equation for the current i (t) in the circuit of the
V (s) figure is
transfer function H (s) = c is
Vi (s)

2 2
(A) 2 1 (B) 2 106 (A) 2 d 2i + 2 di + i (t) = sin t (B) d 2i + 2 di + 2i (t) = cos t
dt dt dt dt
s + 106 s + 106 s + 103 s + 106
2 2
(C) 2 103 (D) 2 106 (C) 2 d 2i + 2 di + i (t) = cos t (D) d 2i + 2 di + 2i (t) = sin t
dt dt dt dt
s + 103 s + 106 s + 106 s + 106
2.76 Consider the following statements S1 and S2 GATE 2003 TWO MARKS
S1 : At the resonant frequency the impedance of a series RLC
circuit is zero.
2.81 Twelve 1 W resistance are used as edges to form a cube. The resistance
S2 : In a parallel GLC circuit, increasing the conductance G re- between two diagonally opposite corners of the cube is
sults in increase in its Q factor. (A) 5 W (B) 1 W
6

GATE Electronics & Communication (C) 6 W (D) 3 W


5 2
by RK Kanodia 2.82 The current flowing through the resistance R in the circuit in the
Now in 3 Volume figure has the form P cos 4t where P is
Purchase Online at maximum discount from online store
and get POSTAL and Online Test Series Free
visit www.nodia.co.in
Which one of the following is correct?
(A) S1 is FALSE and S2 is TRUE
(A) (0.18 + j0.72) (B) (0.46 + j1.90)
(B) Both S1 and S2 are TRUE
(C) - (0.18 + j1.90) (D) - (0.192 + j0.144)
(C) S1 is TRUE and S2 is FALSE
(D) Both S1 and S2 are FALSE
The circuit for Q. 2.66 & 2.67 is given below.
Assume that the switch S is in position 1 for a long time and
GATE 2003 ONE MARK
thrown to position 2 at t = 0 .
2.77 The minimum number of equations required to analyze the circuit
shown in the figure is

For more GATE Resources, Mock Test and


(A) 3 (B) 4
Study material join the community
(C) 6 (D) 7 http://www.facebook.com/gateec2014
2.83 At t = 0+ , the current i1 is
2.78 A source of angular frequency 1 rad/sec has a source impedance
consisting of 1 W resistance in series with 1 H inductance. The load (A) - V (B) - V
2R R
that will obtain the maximum power transfer is
(C) - V (D) zero
(A) 1 W resistance 4R
(B) 1 W resistance in parallel with 1 H inductance 2.84 I1 (s) and I2 (s) are the Laplace transforms of i1 (t) and i2 (t) respectively.
(C) 1 W resistance in series with 1 F capacitor The equations for the loop currents I1 (s) and I2 (s) for the circuit
shown in the figure, after the switch is brought from position 1 to
(D) 1 W resistance in parallel with 1 F capacitor
position 2 at t = 0 , are

(A) > 1 H=
R + Ls + Cs1 - Ls I1 (s) V
G == G
2.79 A series RLC circuit has a resonance frequency of 1 kHz and a s
quality factor Q = 100 . If each of R, L and C is doubled from its - Ls R + Cs I2 (s) 0
original value, the new Q of the circuit is
(B) >
R + Cs1 H=I2 (s)G = 0 G
R + Ls + Cs1 - Ls I1 (s) - Vs
(A) 25 (B) 50 =
- Ls
GATE Electronics and Communication Topicwise Solved Paper by RK Kanodia & Ashish Murolia Page 25

(A) 25 V (B) 50 V
(C) > H
R + Ls + Cs1 - Ls I1 (s) - Vs
- Ls =
R + Ls + Cs1 I2 (s) G = = G (C) - 50 V (D) 0 V
0

(D) > 1 H=
R + Ls + Cs1 - Cs I1 (s) V
G == G
s
- Ls R + Ls + Cs I2 (s) 0 GATE 2002 TWO MARKS

2.85 The driving point impedance Z (s) of a network has the pole-zero 2.90 In the network of the fig, the maximum power is delivered to RL if
locations as shown in the figure. If Z (0) = 3 , then Z (s) is its value is

3 (s + 3) 2 (s + 3)
(A) 2
(B) 2 (A) 16 W (B) 40 W
s + 2s + 3 s + 2s + 2 3
3 (s + 3) 2 (s - 3) (C) 60 W (D) 20 W
(C) 2 (D) 2
s + 2s + 2 s - 2s - 3
2.86 An input voltage v (t) = 10 2 cos (t + 10c) + 10 5 cos (2t + 10c) 2.91 If the 3-phase balanced source in the figure delivers 1500 W at
V is applied to a series combination of resistance R = 1 W and
an inductance L = 1 H. The resulting steady-state current i (t) in SPECIAL EDITION ( STUDY MATERIAL FORM )
ampere is At market Book is available in 3 volume i.e. in 3 book binding
(A) 10 cos (t + 55c) + 10 cos (2t + 10c + tan-1 2) form. But at NODIA Online Store book is available in 10 book
(B) 10 cos (t + 55c) + 10 23 cos (2t + 55c) binding form. Each unit of Book is in separate binding.
(C) 10 cos (t - 35c) + 10 cos (2t + 10c - tan-1 2) Available Only at NODIA Online Store

(D) 10 cos (t - 35c) + 3


2 cos (2t - 35c) Click to Buy
www.nodia.co.in
2.87 The impedance parameters z11 and z12 of the two-port network in
the figure are
a leading power factor 0.844 then the value of ZL (in ohm) is
approximately

(A) z11 = 2.75 W and z12 = 0.25 W


(B) z11 = 3 W and z12 = 0.5 W (A) 90+32.44c (B) 80+32.44c
(C) z11 = 3 W and z12 = 0.25 W (C) 80+ - 32.44c (D) 90+ - 32.44c
(D) z11 = 2.25 W and z12 = 0.5 W
GATE 2001 ONE MARK
GATE 2002 ONE MARK 2.92 The Voltage e0 in the figure is
2.88 The dependent current source shown in the figure

(A) 2 V (B) 4/3 V


(A) delivers 80 W (B) absorbs 80 W
(C) 4 V (D) 8 V
(C) delivers 40 W (D) absorbs 40 W
2.93 If each branch of Delta circuit has impedance 3 Z , then each
2.89 In the figure, the switch was closed for a long time before opening
branch of the equivalent Wye circuit has impedance
at t = 0 . The voltage vx at t = 0+ is
(A) Z (B) 3Z
3
(C) 3 3 Z (D) Z
3
2.94 The admittance parameter Y12 in the 2-port network in Figure is
GATE Electronics and Communication Topicwise Solved Paper by RK Kanodia & Ashish Murolia Page 26

2.98 The z parameters z11 and z21 for the 2-port network in the figure are

(A) - 0.02 mho (B) 0.1 mho


(C) - 0.05 mho (D) 0.05 mho
(A) z11 = 6 W; z21 = 16 W (B) z11 = 6 W; z21 = 4 W
11 11 11 11
GATE 2001 TWO MARKS (C) z11 = 6 W; z21 =- 16 W (D) z11 = 4 W; z21 = 4 W
11 11 11 11
2.95 The voltage e0 in the figure is
GATE 2000 ONE MARK

2.99 The circuit of the figure represents a

(A) 48 V (B) 24 V
(C) 36 V (D) 28 V

GATE Electronics & Communication (A) Low pass filter (B) High pass filter
by RK Kanodia (C) band pass filter (D) band reject filter
Now in 3 Volume
2.100 In the circuit of the figure, the voltage v (t) is
Purchase Online at maximum discount from online store
and get POSTAL and Online Test Series Free
visit www.nodia.co.in
2.96 When the angular frequency w in the figure is varied 0 to 3, the
locus of the current phasor I2 is given by
(A) eat - ebt (B) eat + ebt
(C) aeat - bebt (D) aeat + bebt

2.101 In the circuit of the figure, the value of the voltage source E is

(A) - 16 V (B) 4 V

For more GATE Resources, Mock Test and


Study material join the community
http://www.facebook.com/gateec2014
(C) - 6 V (D) 16 V

2.97 In the figure, the value of the load resistor RL which maximizes the GATE 2000 TWO MARKS
power delivered to it is 2.102 Use the data of the figure (a). The current i in the circuit of the
figure (b)

(A) 14.14 W (B) 10 W


(C) 200 W (D) 28.28 W
GATE Electronics and Communication Topicwise Solved Paper by RK Kanodia & Ashish Murolia Page 27

2.107 A Delta-connected network with its Wye-equivalent is shown in the


given figure. The resistance R1, R2 and R3 (in ohms) are respectively

(A) 1.5, 3 and 9 (B) 3, 9 and 1.5


(A) - 2 A (B) 2 A (C) 9, 3 and 1.5 (D) 3, 1.5 and 9
(C) - 4 A (D) 4 A
GATE 1998 ONE MARK
GATE 1999 ONE MARK 2.108 A network has 7 nodes and 5 independent loops. The number of
2.103 Identify which of the following is NOT a tree of the graph shown in branches in the network is
the given figure is (A) 13 (B) 12
(C) 11 (D) 10

2.109 The nodal method of circuit analysis is based on


SPECIAL EDITION ( STUDY MATERIAL FORM )
At market Book is available in 3 volume i.e. in 3 book binding
form. But at NODIA Online Store book is available in 10 book
(A) begh (B) defg binding form. Each unit of Book is in separate binding.
(C) abfg (D) aegh Available Only at NODIA Online Store

Click to Buy
2.104 A 2-port network is shown in the given figure. The parameter h21 for
this network can be given by www.nodia.co.in
(A) KVL and Ohm’s law (B) KCL and Ohm’s law
(C) KCL and KVL (D) KCL, KVL and Ohm’s law

2.110 Superposition theorem is NOT applicable to networks containing


(A) nonlinear elements (B) dependent voltage sources
(C) dependent current sources (D) transformers
(A) - 1/2 (B) + 1/2
(C) - 3/2 (D) + 3/2
2.111 The parallel RLC circuit shown in the figure is in resonance. In this
circuit

GATE 1999 TWO MARK

2.105 The Thevenin equivalent voltage VTH appearing between the


terminals A and B of the network shown in the given figure is given
by
(A) IR < 1 mA (B) IR + IL > 1 mA
(C) IR + IC < 1 mA (D) IR + IC > 1 mA
0 - 1/2
The short-circuit admittance matrix a two-port network is >
1/2 0 H
2.112

The two-port network is


(A) non-reciprocal and passive (B) non-reciprocal and active
(A) j16 (3 - j4) (B) j16 (3 + j4) (C) reciprocal and passive (D) reciprocal and active
(C) 16 (3 + j4) (D) 16 (3 - j4) 2.113 The voltage across the terminals a and b in the figure is
2.106 The value of R (in ohms) required for maximum power transfer in
the network shown in the given figure is

(A) 0.5 V (B) 3.0 V


(C) 3.5 V (D) 4.0 V
(A) 2 (B) 4
(C) 8 (D) 16 2.114 A high-Q quartz crystal exhibits series resonance at the frequency
GATE Electronics and Communication Topicwise Solved Paper by RK Kanodia & Ashish Murolia Page 28

ws and parallel resonance at the frequency wp . Then (A) 10 V (B) 15 V


(A) ws is very close to, but less than wp (C) 5 V (D) None of the above
(B) ws << wp
(C) ws is very close to, but greater than wp 2.119 In the circuit of the figure is the energy absorbed by the 4 W resistor
(D) ws >> wp in the time interval (0, 3) is

GATE 1997 ONE MARK

2.115 The current i4 in the circuit of the figure is equal to

(A) 36 Joules (B) 16 Joules


(C) 256 Joules (D) None of the above

2.120 In the circuit of the figure the equivalent impedance seen across
terminals a, b, is

(A) 12 A (B) - 12 A
(C) 4 A (D) None or these

2.116 The voltage V in the figure equal to


GATE Electronics & Communication
by RK Kanodia
Now in 3 Volume
Purchase Online at maximum discount from online store
and get POSTAL and Online Test Series Free
visit www.nodia.co.in (A) b 16 l W (B) b 8 l W
3 3
(C) b 8 + 12j l W (D) None of the above
3

GATE 1996 ONE MARK

2.121 In the given figure, A1, A2 and A3 are ideal ammeters. If A2 and A3
read 3 A and 4 A respectively, then A1 should read

(A) 3 V (B) - 3 V
(C) 5 V (D) None of these

2.117 The voltage V in the figure is always equal to (A) 1 A (B) 5 A

For more GATE Resources, Mock Test and


Study material join the community
http://www.facebook.com/gateec2014

(C) 7 A (D) None of these


(A) 9 V (B) 5 V 2.122 The number of independent loops for a network with n nodes and
(C) 1 V (D) None of the above b branches is
(A) n - 1
2.118 The voltage V in the figure is
(B) b - n
(C) b - n + 1
(D) independent of the number of nodes

GATE 1996 TWO MARKS

2.123 The voltages VC1, VC2, and VC3 across the capacitors in the circuit in
GATE Electronics and Communication Topicwise Solved Paper by RK Kanodia & Ashish Murolia Page 29

the given figure, under steady state, are respectively.

(A) 80 V, 32 V, 48 V (B) 80 V, 48 V, 32 V
(C) 20 V, 8 V, 12 V (D) 20 V, 12 V, 8 V

SPECIAL EDITION ( STUDY MATERIAL FORM )


At market Book is available in 3 volume i.e. in 3 book binding
form. But at NODIA Online Store book is available in 10 book
binding form. Each unit of Book is in separate binding.
Available Only at NODIA Online Store

Click to Buy
www.nodia.co.in
GATE Electronics and Communication Topicwise Solved Paper by RK Kanodia & Ashish Murolia Page 30

SOLUTIONS As the circuit open across RL so


I2 = 0
or, j40I2 = 0
i.e., the dependent source in loop 1 is short circuited. Therefore,
^ j4h Vs
VL1 =
j4 + 3
2.1 Option (B) is correct.
In the equivalent star connection, the resistance can be given as
Rb Ra j40
RC = VTh = 10 VL1 = 100 53.13c
Ra + Rb + Rc j4 + 3
40 90c
RB = Ra Rc = 100 53.13c
Ra + Rb + Rc 5 53.13c
= 800 90c
RA = Rb Rc
Ra + Rb + Rc 2.5 Option (C) is correct.
So, if the delta connection components Ra , Rb and Rc are scaled For the given transformer, we have
V = 1.25
by a factor k then VWX 1
^k Rb h^k Rc h
RAl =
kRa + kRb + kRc
2
=k Rb Rc
k Ra + Rb + Rc
GATE Electronics & Communication
by RK Kanodia
Now in 3 Volume Since, VYZ = 0.8 (attenuation factor)
V
Purchase Online at maximum discount from online store
and get POSTAL and Online Test Series Free So,
VWX ^ h^ h
VYZ = 0.8 1.25 = 1

visit www.nodia.co.in or, VYZ = VWX


V
at VWX = 100 V ; YZ = 100
1
1
VWX 100
= k RA 1

VWX
hence, it is also scaled by a factor k at VWZ = 100 V ; 2
= 100
2
VYZ2
100
2.2 Option (D) is correct.
For the given capacitance, C = 100mF in the circuit, we have the
2.6 Option (C) is correct.
reactance. The quality factor of the inductances are given by

XC = 1 = 1 = 10 4 q1 = wL1
sc s # 100 # 10-6 s R1
So, and q2 = wL2
10 4 + 10 4 R2
V2 ^s h
= 4 s So, in series circuit, the effective quality factor is given by
V1 ^s h 10 + 10 4 + 10 4 XLeq
s s Q = = wL 1 + wL 2
Req R1 + R 2
= s+1
s+2 wL 1 + wL 2 q1 q
+ 2 q R + q2 R2
R
= 1 2 R R R
1 2
= 2 R R 2
= 1 1
2.3 Option (C) is correct. 1 + 1 1 + 1 R1 + R 2
For the purely resistive load, maximum average power is transferred R 2 R1 R 2 R1
when For more GATE Resources, Mock Test and
2 2
RL = RTh + XTh Study material join the community
where RTh + jXTh is the equivalent thevinin (input) impedance of
the circuit. Hence, we obtain
http://www.facebook.com/gateec2014
RL = 42 + 32 2.7 Option (C) is correct.
5W
2.4 Option (C) is correct.
For evaluating the equivalent thevenin voltage seen by the load RL
, we open the circuit across it (also if it consist dependent source).
The equivalent circuit is shown below

Consider that the voltage across the three capacitors C1 , C2 and C 3


are V1 , V2 and V3 respectively. So, we can write
V2 = C 3
V3 C 2
....(1)
GATE Electronics and Communication Topicwise Solved Paper by RK Kanodia & Ashish Murolia Page 31

Since, Voltage is inversely proportional to capacitance vc (0) /s v (0)


I (s) = = c
Now, given that 1 + 1 1 + 1
C1 s C 2 s C1 C 2
C1 = 10 mF ; ^V1hmax = 10V
C2 = 5 mF ; ^V2hmax = 5 V I (s) = b C1 C2 l (12 V) = 12Ceq vC (0) = 12 V
C1 + C 2
C 3 = 2 mF ; ^V3hmax = 2V Taking inverse Laplace transform for the current in time domain,
So, from Eq (1) we have i (t) = 12Ceq d (t) (Impulse)
V2 = 2
V3 5
for ^V3hmax = 2 2.11 Option (B) is correct.
We obtain, In phasor form, Z = 4 - j 3 = 5 - 36.86cW
V2 = 2 # 2 = 0.8 volt < 5 I = 5 100c A
5
i.e., V2 < ^V2hmax Average power delivered.
Hence, this is the voltage at C2 . Therefore, Pavg. = 1 I 2 Z cos q = 1 # 25 # 5 cos 36.86c = 50 W
2 2
V3 = 2 volt
V2 = 0.8 volt
and V1 = V2 + V3 = 2.8 volt Alternate method:
Now, equivalent capacitance across the terminal is Z = (4 - j3) W , I = 5 cos (100pt + 100) A
Ceq = C 2 C 3 + C1 = 5 # 2 + 10 = 80 mF
C2 + C3 5+2 7 SPECIAL EDITION ( STUDY MATERIAL FORM )
Equivalent voltage is (max. value) At market Book is available in 3 volume i.e. in 3 book binding
Vmax = V1 = 2.8
form. But at NODIA Online Store book is available in 10 book
So, charge stored in the effective capacitance is
binding form. Each unit of Book is in separate binding.
Q = Ceq Vmax = b 80 l # ^2.8h = 32 mC Available Only at NODIA Online Store
7
2.8 Option (D) is correct. Click to Buy
www.nodia.co.in

Pavg = 1 Re $ I Z . = 1 # Re "(5) 2 # (4 - j3),


2
2 2
= 1 # 100 = 50 W
2
2.12 Option (C) is correct

At the node 1, voltage is given as


V1 = 10 volt
Applying KCL at node 1
IS + V1 + V1 - 2 = 0
2 1
10 10
IS + + - 2 = 0
2 1
IS =- 13 A
Also, from the circuit,
VS - 5 # 2 = V1
VS = 10 + V1
= 20 volt
Applying nodal analysis at top node.
2.9 Option (C) is correct.
V1 + 1 0c V1 + 1 0c
Again from the shown circuit, the current in 1 W resistor is + = 1 0c
1 j1
I = V1 = 10 = 10 A
1 1 V1 (j 1 + 1) + j 1 + 1 0c = j1
Option (D) is correct.
V1 = - 1
2.10

The s -domain equivalent circuit is shown as below. 1 + j1


1
V1 + 1 0c - 1 + j + 1
Current I1 = =
j1 j1
j
= = 1 A
(1 + j) j 1 + j
2.13 Option (A) is correct.
We obtain Thevenin equivalent of circuit B .
GATE Electronics and Communication Topicwise Solved Paper by RK Kanodia & Ashish Murolia Page 32

So current in the branch will be


IAB = 6 = 3 A
2
We can see, that the circuit is a one port circuit looking from
terminal BD as shown below

Thevenin Impedance :

ZTh = R

Thevenin Voltage : For a one port network current entering one terminal, equals the
VTh = 3 0c V current leaving the second terminal. Thus the outgoing current from
Now, circuit becomes as A to B will be equal to the incoming current from D to C as shown
i.e. IDC = IAB = 3 A

GATE Electronics & Communication


by RK Kanodia
Now in 3 Volume
Purchase Online at maximum discount from online store
and get POSTAL and Online Test Series Free The total current in the resistor 1 W will be
I1 = 2 + IDC (By writing KCL at node D )
visit www.nodia.co.in
= 2+3 = 5A
So, VCD = 1 # (- I1) =- 5 V
2.15 Option (C) is correct.
When 10 V is connected at port A the network is

Current in the circuit, I1 = 10 - 3


2+R
Now, we obtain Thevenin equivalent for the circuit seen at load
Power transfer from circuit A to B
terminal, let Thevenin voltage is VTh, 10 V with 10 V applied at port A
P = (I 12) 2 R + 3I1 and Thevenin resistance is RTh .
P = :10 - 3D R + 3 :10 - 3D
2

2+R 2+R
For more GATE Resources, Mock Test and
P = 49R 2 + 21
(2 + R) (2 + R) Study material join the community
49R + 21 (2 + R)
P = http://www.facebook.com/gateec2014
(2 + R) 2
P = 42 + 70R2
(2 + R)
2
dP = (2 + R) 70 - (42 + 70R) 2 (2 + R) = 0
dR (2 + R) 4
(2 + R) [(2 + R) 70 - (42 + 70R) 2] = 0
140 + 70R - 84 - 140R = 0
VTh,10 V
56 = 70R IL =
RTh + RL
R = 0.8 W For RL = 1 W , IL = 3 A
V
2.14 Option (A) is correct. 3 = Th,10 V ...(i)
RTh + 1
In the given circuit For RL = 2.5 W , IL = 2 A
V
VA - VB = 6 V = Th,10 V ...(ii)
RTh + 2.5
GATE Electronics and Communication Topicwise Solved Paper by RK Kanodia & Ashish Murolia Page 33

ISC = 25 (16 0 ) = (6.4 - j4.8) A


Dividing above two 25 + 15 + j30
3 = RTh + 2.5 2.18 Option (C) is correct.
2 RTh + 1
Power transferred to RL will be maximum when RL is equal to the
3RTh + 3 = 2RTh + 5
Thevenin resistance. We determine Thevenin resistance by killing
RTh = 2 W all source as follows :
Substituting RTh into equation (i)
VTh,10 V = 3 (2 + 1) = 9 V
Note that it is a non reciprocal two port network. Thevenin voltage
seen at port B depends on the voltage connected at port A. Therefore
we took subscript VTh,10 V . This is Thevenin voltage only when 10 V
source is connected at input port A. If the voltage connected to port
A is different, then Thevenin voltage will be different. However,
Thevenin’s resistance remains same.
RTH = 10 # 10 + 10 = 15 W
Now, the circuit is as shown below : 10 + 10
2.19 Option (A) is correct.
The given circuit is shown below

SPECIAL EDITION ( STUDY MATERIAL FORM )


At market Book is available in 3 volume i.e. in 3 book binding
VTh,10 V form. But at NODIA Online Store book is available in 10 book
For RL = 7 W , IL = = 9 = 1A
2 + RL 2 + 7 binding form. Each unit of Book is in separate binding.
Available Only at NODIA Online Store
2.16 Option (B) is correct.
Now, when 6 V connected at port A let Thevenin voltage seen at Click to Buy
port B is VTh,6 V . Here RL = 1 W and IL = 7 A
3
www.nodia.co.in

VTh, 6 V = RTh # 7 + 1 # 7 = 2 # 7 + 7 = 7 V For parallel combination of R and C equivalent impedance is


3 3 3 3
This is a linear network, so VTh at port B can be written as R$ 1
jwC R
VTh = V1 a + b Zp = =
R+ 1 1 + j wRC
where V1 is the input applied at port A. jwC
We have V1 = 10 V , VTh,10 V = 9 V Transfer function can be written as
` 9 = 10a + b ...(i) R
When V1 = 6 V , VTh, 6 V = 9 V Vout = Z p 1 + jwRC
=
Vin Zs + Zp R+ 1 + R
` 7 = 6a + b ...(ii) jwC 1 + jwRC
Solving (i) and (ii)
jwRC
a = 0.5 , b = 4 =
jwRC + (1 + jwRC) 2
Thus, with any voltage V1 applied at port A, Thevenin voltage or j
open circuit voltage at port B will be = Here w = 1
j + (1 + j) 2
RC
So, VTh, V = 0.5V1 + 4 Vout j
= =1
1

For V1 = 8 V Vin (1 + j) 2 + j 3
V
VTh,8 V = 0.5 # 8 + 4 = 8 = Voc (open circuit voltage) Thus v out = b p l cos (t/RC)
3
2.17 Option (A) is correct.
Replacing P - Q by short circuit as shown below we have 2.20 Option (B) is correct.
From star delta conversion we have

Using current divider rule the current Isc is


GATE Electronics and Communication Topicwise Solved Paper by RK Kanodia & Ashish Murolia Page 34

Thus R1 = Ra Rb = 6.6 = 2W Given circuit is as shown below


Ra + Rb + Rc 6 + 6 + 6
Here R1 = R 2 = R 3 = 2 W
Replacing in circuit we have the circuit shown below :

By writing node equation at input port


I1 = V1 + V1 - V2 = 4V1 - 2V2 ...(1)
0.5 0.5
By writing node equation at output port
I2 = V2 + V2 - V1 =- 2V1 + 4V2 ...(2)
0.5 0.5
Now the total impedance of circuit is From (1) and (2), we have admittance matrix
(2 + j4) (2 - j4) 4 -2
Y =>
Z =
(2 + j4) (2 - j4)
+2 = 7W - 2 4H

Current I = 14+0c = 2+0c 2.24 Option (D) is correct.


7
A parallel RLC circuit is shown below :
2.21 Option (D) is correct.

GATE Electronics & Communication


by RK Kanodia
Now in 3 Volume
Purchase Online at maximum discount from online store
and get POSTAL and Online Test Series Free Input impedance Z in = 1
1 + 1 + jw C
visit www.nodia.co.in R jwL

At resonance 1 = wC
From given admittance matrix we get wL
I1 = 0.1V1 - 0.01V2 and ...(1) So, Z in = 1 = R (maximum at resonance)
1/R
I2 = 0.01V1 + 0.1V2 ...(2)
Thus (D) is not true.
Now, applying KVL in outer loop;
Furthermore bandwidth is wB i.e wB \ 1 and is independent of L
V2 =- 100I2 R
,
or I2 =- 0.01V2 ...(3)
Hence statements A, B, C, are true.
From eq (2) and eq (3) we have
- 0.01V2 = 0.01V1 + 0.1V2 2.25 Option (A) is correct.
- 0.11V2 = 0.01V1 Let the current i (t) = A + Be-t/t t " Time constant
V2 = - 1 When the switch S is open for a long time before t < 0 , the circuit is
V1 11
2.22 Option (A) is correct.
Here we take the current flow direction as positive.
At t = 0- voltage across capacitor is
Q -3 For more GATE Resources, Mock Test and
VC (0-) =- =- 2.5 # 10-6 =- 50 V
C 50 # 10 Study material join the community
Thus +
VC (0 ) =- 50 V http://www.facebook.com/gateec2014
In steady state capacitor behave as open circuit thus
V (3) = 100 V
Now, VC (t) = VC (3) + (VC (0+) - VC (3)) e-t/RC
-t
= 100 + (- 50 - 100) e 10 # 50 # 10-6

3
= 100 - 150e- (2 # 10 t)
Now ic (t) = C dV
dt
3
At t = 0 , inductor current does not change simultaneously, So the
= 50 # 10-6 # 150 # 2 # 103 e-2 # 10 t A circuit is
3
= 15e-2 # 10 t
ic (t) = 15 exp (- 2 # 103 t) A
2.23 Option (A) is correct.
GATE Electronics and Communication Topicwise Solved Paper by RK Kanodia & Ashish Murolia Page 35

Applying nodal analysis


VA - 10 + 1 + VA - 0 = 0
2 2
2VA - 10 + 2 = 0 = V4 = 4 V
Current, I1 = 10 - 4 = 3 A
2
Current is resistor (AB)
Current from voltage source is
i (0) = 0.75 = 0.375 A
2 I 2 = I1 - 3 = 0
Similarly for steady state the circuit is as shown below Since current through voltage source is zero, therefore power
delivered is zero.
2.28 Option (A) is correct.
Circuit is as shown below

i (3) = 15 = 0.5 A
3
-3
t = L = 15 # 10 = 10-3 sec
Req 10 + (10 || 10)
t
i (t) = A + Be- 1 # 10 = A + Be-100t
-3
SPECIAL EDITION ( STUDY MATERIAL FORM )
Now i (0) = A + B = 0.375 At market Book is available in 3 volume i.e. in 3 book binding
and i (3) = A = 0.5 form. But at NODIA Online Store book is available in 10 book
So, B = 0.375 - 0.5 =- 0.125 binding form. Each unit of Book is in separate binding.
Available Only at NODIA Online Store
Hence i (t) = 0.5 - 0.125e-1000 t A
2.26 Option (A) is correct. Click to Buy
Circuit is redrawn as shown below www.nodia.co.in

Where, Z1 = jwL = j # 103 # 20 # 10-3 = 20j


Z2 = R || XC
XC = 1 = 1 =- 20j
jwC j # 103 # 50 # 10-6
1 (- 20j)
Z2 = R = 1W Since 60 V source is absorbing power. So, in 60 V source current
1 - 20j
flows from + to - ve direction
Voltage across Z2
- 20j So, I + I1 = 12
c 1 - 20j m I = 12 - I1
VZ = Z2 : 20 0 = : 20
Z1 + Z 2 20j
c 20j - 1 - 20j m I is always less then 12 A So, only option (A) satisfies this condi-
2

tions.
(- 20j)
=c
20j + 400 - 20j m
: 20 =- j 2.29 Option (C) is correct.
For given network we have
Current in resistor R is
(RL XC ) Vi
V j V0 =
I = Z =- =- j A2
R + (RL XC )
R 1
RL
Option (A) is correct. V0 (s)
= 1 + sRL C RL
2.27
=
The circuit can be redrawn as Vi (s) R+ R L R + RRL sC + RL
1 + sRL C
= RL = 1
R + RRL sC + RL 1+ R + RsC
RL
But we have been given
V (s) 1
T . F. = 0 =
Vi (s) 2 + sCR
Comparing, we get
1 + R = 2 & RL = R
RL
2.30 Option (C) is correct.
GATE Electronics and Communication Topicwise Solved Paper by RK Kanodia & Ashish Murolia Page 36

The energy delivered in 10 minutes is


t t
E = #0 VIdt = I #0Vdt = I # Area

= 2 # 1 (10 + 12) # 600 = 13.2 kJ


2
2.31 Option (B) is correct.
From given circuit the load current is
IL = V = 20+0c = 20+0c
Zs + ZL (1 + 2j) + (7 + 4j) 8 + 6j The open circuit voltage is
= 1 (8 - 6j) = 20+0c = 2+ - f where f = tan - 1 3 Voc = 100 V
5 10+f 4
From fig I1 = 100 = 12.5 A
The voltage across load is 8
VL = IL ZL Vx =- 4 # 12.5 =- 50 V
The reactive power consumed by load is I2 = 100 + Vx = 100 - 50 = 12.5 A
4 4
Pr = VL IL* = IL ZL # IL* = ZL IL 2
2 Isc = I1 + I2 = 25 A
= (7 # 4j) 20+0c = (7 + 4j) = 28 + 16j
8 + 6j Rth = Voc = 100 = 4 W
Isc 25
Thus average power is 28 and reactive power is 16.
Thus for maximum power transfer RL = Req = 4 W
2.32 Option (B) is correct.
2.34 Option (A) is correct.
GATE Electronics & Communication Steady state all transient effect die out and inductor act as short
by RK Kanodia circuits and forced response acts only. It doesn’t depend on initial
Now in 3 Volume current state. From the given time domain behavior we get that
Purchase Online at maximum discount from online store circuit has only R and L in series with V0 . Thus at steady state
and get POSTAL and Online Test Series Free i (t) " i (3) = V0
R
visit www.nodia.co.in
2.35 Option (C) is correct.
-
At t = 0 , the circuit is as shown in fig below : The given graph is

There can be four possible tree of this graph which are as follows:

V (0-) = 100 V
Thus V (0+) = 100 V
At t = 0+ , the circuit is as shown below
There can be 6 different possible cut-set.

For more GATE Resources, Mock Test and


Study material join the community
I (0+) = 100 = 20 mA http://www.facebook.com/gateec2014
5k
At steady state i.e. at t = 3 is I (3)= 0
2.36 Option (B) is correct.
i (t) = I (0+) e-
t
Now RCeq
u (t)
Initially i (0-) = 0 therefore due to inductor i (0+) = 0 . Thus all
(0.5m + 0.3m) 0.2m current Is will flow in resistor R and voltage across resistor will be
Ceq = = 0.16 m F
0.5m + 0.3m + 0.2m Is Rs . The voltage across inductor will be equal to voltage across Rs
1 = 1 = 1250 as no current flow through R.
RCeq 5 # 103 # 0.16 # 10-6
i (t) = 20e-1250t u (t) mA
2.33 Option (C) is correct.
For Pmax the load resistance RL must be equal to thevenin resistance
Req i.e. RL = Req . The open circuit and short circuit is as shown
below
GATE Electronics and Communication Topicwise Solved Paper by RK Kanodia & Ashish Murolia Page 37

Thus vL (0+) = Is Rs For 4T < t < 5T , capacitor will be charged from 0 V


di (0+) Vc =
t
#4Tdt = t - 4T
but vL (0+) = L
dt
di (0+) vL (0+) Is Rs At t = 5T, Vc = T Volts
Thus = =
dt L L Thus the output waveform is
2.37 Option (A) is correct.
Killing all current source and voltage sources we have,

Only option C satisfy this waveform.


2.40 Option (D) is correct.
Zth = (1 + s) ( s1 + 1) Writing in transform domain we have
(1 + s)( s1 + 1) [ s1 + 1 + 1 + s] Vc (s) 1
= = = 1 s = 2 1
(1 + s) + ( s1 + 1) s + s1 + 1 + 1 Vs (s) ^ s + s + 1h (s + s + 1)
or Zth = 1 Since Vs (t) = d (t) " Vs (s) = 1 and
Alternative :
SPECIAL EDITION ( STUDY MATERIAL FORM )
Here at DC source capacitor act as open circuit and inductor act
At market Book is available in 3 volume i.e. in 3 book binding
as short circuit. Thus we can directly calculate thevenin Imped-
form. But at NODIA Online Store book is available in 10 book
ance as 1 W
binding form. Each unit of Book is in separate binding.
2.38 Option (D) is correct. Available Only at NODIA Online Store
s
Z (s) = R 1 sL = 2 C Click to Buy
sC s + + www.nodia.co.in
s 1
RC LC
We have been given
Z (s) = 2 0.2s Vc (s) = 1
s + 0.1s + 2 2
(s + s + 1)
Comparing with given we get 3
1 = 0.2 or C = 5 F or Vc (s) = 2 = 2
G
C 3 (s + 12 ) 2 + 43
1 = 0.1 or R = 2 W Taking inverse Laplace transform we have
RC
Vt = 2 e- sin c 3 t m
t
2

1 = 2 or L = 0.1 H 3 2
LC
2.41 Option (B) is correct.
2.39 Option (C) is correct. Let voltage across resistor be vR
Voltage across capacitor is VR (s)
t = 1 1 = s
Vc = 1 idt # VS (s) ( s + s + 1) (s2 + s + 1)
C 0
Since vs = d (t) " Vs (s) = 1 we get
Here C = 1 F and i = 1 A. Therefore
t VR (s) = 2 s = s
1 2
Vc = #0 dt (s + s + 1) (s + 2 ) + 43

For 0 < t < T , capacitor will be charged from 0 V (s + 12 ) 1


= - 2
t (s + 12 ) 2 + 3
4 (s + 1 2
2) + 4
3
Vc = #0 dt = t
or vR (t) = e- cos
1
2
3 t-1 2 e- sin 3 t 1
2

At t = T, Vc = T Volts 2 2# 3 2
For T < t < 2T , capacitor will be discharged from T volts as t
= e- 2 =cos 3 t - 1 sin 3 t
2 G
t
Vc = T - #T dt = 2T - t 2 3
At t = 2T, Vc = 0 volts
2.42 Option (C) is correct.
From the problem statement we have
For 2T < t < 3T , capacitor will be charged from 0 V
t z11 = v1 = 6 = 1.5W
Vc = #2Tdt = t - 2T i1 i = 0 42

v
z12 = 1 = 4.5 = 4.5W
At t = 3T, Vc = T Volts i2 i = 0 1
1
For 3T < t < 4T , capacitor will be discharged from T Volts z21 = v2 = 6 = 1.5W
t i1 i = 0 4
#3Tdt = 4T - t
2
Vc = T -
z22 = v2 = 1.5 = 1.5W
i2 i = 0 1
At t = 4T, Vc = 0 Volts 2

Thus z -parameter matrix is


GATE Electronics and Communication Topicwise Solved Paper by RK Kanodia & Ashish Murolia Page 38

z11 z12 1.5 4.5


=z z G = =1.5 1.5 G
2.46 Option (D) is correct.
21 22 We know that bandwidth of series RLC circuit is R . Therefore
L
2.43 Option (A) is correct. Bandwidth of filter 1 is B1 = R
L1
From the problem statement we have
Bandwidth of filter 2 is B2 = R = R = 4R
h12 = v1 = 4.5 = 3 L2 L1 /4 L1
v2 i = 0 1.5 B 1
Dividing above equation 1 =
1

h22 = i2 = 1 = 0.67 B2 4
v2 i = 0 1.5
Option (D) is correct.
1
2.47
From z matrix, we have
Here Vth is voltage across node also. Applying nodal analysis we get
v1 = z11 i1 + z12 i2
v2 = z21 i1 + z22 i2
If v2 = 0
Then i2 = - z21 = - 1.5 =- 1 = h
21
i1 z22 1.5
or i2 =- i1
Putting in equation for v1, we get Vth + Vth + Vth - 2i = 2
2 1 1
v1 = (z11 - z12) i1
v1 = h11 = z11 - z12 = 1.5 - 4.5 =- 3 But from circuit i = Vth = Vth
i1 v2 = 0
1
Therefore
GATE Electronics & Communication
Vth + Vth + Vth - 2Vth = 2
by RK Kanodia 2 1 1
Now in 3 Volume or Vth = 4 volt
Purchase Online at maximum discount from online store From the figure shown below it may be easily seen that the short
and get POSTAL and Online Test Series Free circuit current at terminal XY is isc = 2 A because i = 0 due to
visit www.nodia.co.in short circuit of 1 W resistor and all current will pass through short
circuit.
Hence h -parameter will be
h11 h12 -3 3
=h h G = =- 1 0.67 G
21 22

2.44 Option (D) is correct.


According to maximum Power Transform Theorem
ZL = Zs* = (Rs - jXs)
2.45 Option (C) is correct. Therefore Rth = Vth = 4 = 2 W
isc 2
At w " 3 , capacitor acts as short circuited and circuit acts as shown
2.48 Option (A) is correct.
in fig below
The voltage across capacitor is
At t = 0+ , Vc (0+) = 0
At t = 3 , VC (3) = 5 V
The equivalent resistance seen by capacitor as shown in fig is
Req = 20 20 = 10kW
Here we get V0 = 0
Vi
For more GATE Resources, Mock Test and
At w " 0 , capacitor acts as open circuited and circuit look like as
shown in fig below Study material join the community
http://www.facebook.com/gateec2014

Here we get also V0 = 0


Vi
So frequency response of the circuit is as shown in fig and circuit is
a Band pass filter. Time constant of the circuit is
t = Req C = 10k # 4m = 0.04 s
Using direct formula
Vc (t) = VC (3) - [Vc (3) - Vc (0)] e-t/t
= VC (3) (1 - e-t/t) + VC (0) e-t/t = 5 (1 - e-t/0.04)
or Vc (t) = 5 (1 - e-25t)
GATE Electronics and Communication Topicwise Solved Paper by RK Kanodia & Ashish Murolia Page 39

dVC (t)
Now IC (t) = C
dt Taking Laplace transform we get
= 4 # 10-6 # (- 5 # 25e-25t) = 0.5e-25t mA V (s) = sLI (s) - Li (0+)
2.49 Option (D) is correct. As per given in question
(5 - 3j) # (5 + 3j) - Li (0+) =- 1 mV
Impedance = (5 - 3j) (5 + 3j) =
5 - 3j + 5 + 3 j Thus i (0+) = 1 mV = 0.5 A
(5) 2 - (3j) 2 2 mH
= = 25 + 9 = 3.4
10 10 2.54 Option (B) is correct.
VAB = Current # Impedance At initial all voltage are zero. So output is also zero.
= 5+30c # 34 = 17+30c Thus v0 (0+) = 0
2.50 Option (D) is correct. At steady state capacitor act as open circuit.
The network is shown in figure below.

Now V1 = AV2 - BI2 ...(1) SPECIAL EDITION ( STUDY MATERIAL FORM )


and I1 = CV2 - DI2 ...(2) At market Book is available in 3 volume i.e. in 3 book binding
also V2 =- I2 RL ...(3) form. But at NODIA Online Store book is available in 10 book
From (1) and (2) we get binding form. Each unit of Book is in separate binding.
V1 = AV2 - BI2 Available Only at NODIA Online Store
Thus
I1 CV2 - DI2 Click to Buy
Substituting value of V2 from (3) we get www.nodia.co.in
Input Impedance Zin = - A # I2 RL - BI2
- C # I2 RL - DI2
Thus, v0 (3) = 4 # vi = 4 # 10 = 8
or Zin = ARL + B 5 5
CRL + D The equivalent resistance and capacitance can be calculate after
2.51 Option (B) is correct. killing all source
The circuit is as shown below.

At input port V1 = re I1 Req = 1 4 = 0.8 kW


At output port V2 = r0 (I2 - bI1) =- r0 bI1 + r0 I2 Ceq = 4 1 = 5 mF
Comparing standard equation t = Req Ceq = 0.8kW # 5mF = 4 ms
V1 = z11 I1 + z12 I2 v0 (t) = v 0 (3) - [v 0 (3) - v 0 (0+)] e-t/t
V2 = z21 I1 + z22 I2 = 8 - (8 - 0) e-t/0.004
z12 = 0 and z21 =- r0 b v0 (t) = 8 (1 - e-t/0.004) Volts
2.52 Option (B) is correct. 2.55 Option (A) is correct.
For series RC network input impedance is
Here Z2 (s) = Rneg + Z1 (s)
Zins = 1 + R = 1 + sRC or Z2 (s) = Rneg + Re Z1 (s) + j Im Z1 (s)
sC sC
Thus pole is at origin and zero is at - 1 For Z2 (s) to be positive real, Re Z2 (s) $ 0
RC Thus Rneg + Re Z1 (s) $ 0
For parallel RC network input impedance is or Re Z1 (s) $- Rneg
1 R
sC sC But Rneg is negative quantity and - Rneg is positive quantity.
Zin = =
1 +R 1 + sRC Therefore
sC
Re Z1 (s) $ Rneg
Thus pole is at - 1 and zero is at infinity. or Rneg # Re Z1 (jw) For all w.
RC
2.56 Option (C) is correct.
2.53 Option (A) is correct.
Transfer function is
We know v = Ldi
dt
GATE Electronics and Communication Topicwise Solved Paper by RK Kanodia & Ashish Murolia Page 40

1 For RC parallel network the driving point impedance is


Y (s) sC 1
= = 2 R 1
U (s) R + sL + 1 s LC + scR + 1 Cs = R
1 sC Zinp =
LC R+ 1 1 + sRC
= Cs
s2 + R s + 1
L LC Here pole is s =- 1/RC and zero is at 3, therefore first critical
2
wn2
Comparing with s + 2xwn s + = 0 we have frequency is a pole and last critical frequency is a zero.
Here 2xwn = R , 2.61 Option (A) is correct.
L
Applying KCL we get
and wn = 1
LC i1 (t) + 5+0c = 10+60c
Thus x = R LC = R C or i1 (t) = 10+60c - 5+0c = 5 + 5 3j - 5
2L 2 L
or i1 (t) = 5 3 +90c = 10 3 +90c
For no oscillations, x $ 1 2

Thus R C $1 2.62 Option (B) is correct.


2 L If L1 = j5W and L3 = j2W the mutual induction is subtractive because
or R $2 L current enters from dotted terminal of j2W coil and exit from dotted
C
terminal of j5W. If L2 = j2W and L3 = j2W the mutual induction is
2.57 Option (B) is correct. additive because current enters from dotted terminal of both coil.
For given transformer Thus Z = L1 - M13 + L2 + M23 + L3 - M31 + M32
GATE Electronics & Communication = j5 + j10 + j2 + j10 + j2 - j10 + j10 = j9
by RK Kanodia 2.63 Option (B) is correct.
Now in 3 Volume Open circuit at terminal ab is shown below
Purchase Online at maximum discount from online store
and get POSTAL and Online Test Series Free
visit www.nodia.co.in
I2 = V1 = n
I1 V2 1
Applying KCL at node we get
or I1 = I2 and V1 = nV2 Vab + Vab - 10 = 1
n
Comparing with standard equation 5 5
V1 = AV2 + BI2 or Vab = 7.5 = Vth
I1 = CV2 + DI2 Short circuit at terminal ab is shown below
A B n 0
=C D G = = 0 1 G
n

Thus x = 1
n
2.58 Option (B) is correct.
We have L = 1H and C = 1 # 10-6
400
Short circuit current from terminal ab is
Resonant frequency
Isc = 1 + 10 = 3 A
f0 = 1 == 1 5
2p LC 2p 1 # 1 # 10 - 6 For more GATE Resources, Mock Test and
400
3
= 10 # 20 = 10 Hz
4 Study material join the community
2p p http://www.facebook.com/gateec2014
2.59 Option (C) is correct.
Maximum power will be transferred when RL = Rs = 100W Thus Rth = Vth = 7.5 = 2.5 W
Isc 3
In this case voltage across RL is 5 V, therefore
2 Here current source being in series with dependent voltage source
Pmax = V = 5 # 5 = 0.25 W
R 100 make it ineffective.
2.60 Option (C) is correct. 2.64 Option (C) is correct.
For stability poles and zero interlace on real axis. In RC series Here Va = 5 V because R1 = R2 and total voltage drop is 10 V.
network the driving point impedance is Now Vb = R3 # 10 = 1.1 # 10 = 5.238 V
R3 + R4 2.1
Zins = R + 1 = 1 + sRC
Cs sC V = Va - Vb = 5 - 5.238 =- 0.238 V
Here pole is at origin and zero is at s =- 1/RC , therefore first 2.65 Option (D) is correct.
critical frequency is a pole and last critical frequency is a zero. For h parameters we have to write V1 and I2 in terms of I1 and V2 .
GATE Electronics and Communication Topicwise Solved Paper by RK Kanodia & Ashish Murolia Page 41

V1 = h11 I1 + h12 V2 Impedance Z (s) = s + 2


I2 = h21 I1 + h22 V2 Applying KVL at input port V (s) 1
I (s) = i =
V1 = 10I1 + V2 s + 2 s (s + 2)
Applying KCL at output port or I (s) = 1 ; 1 - 1 E
2 s s+2
V2 = I + I
20 1 2 Taking inverse Laplace transform
i (t) = 1 (1 - e-2t) u (t)
or I2 =- I1 + V2 2
20
At t = 0 , i (t) = 0
Thus from above equation we get
h11 h12 10 1 At t = 12 , i (t) = 0.31
=h h G = =- 1 0.05G At t = 3 , i (t) = 0.5
12 22

2.66 Option (B) is correct. Graph (C) satisfies all these conditions.
Time constant RC = 0.1 # 10 - 6 # 103 = 10 - 4 sec 2.72 Option (D) is correct.
Since time constant RC is very small, so steady state will be We know that
reached in 2 sec. At t = 2 sec the circuit is as shown in fig. V1 = z11 I1 + z12 I2
V2 = z11 I1 + z22 I2
where z11 = V1
I1 I = 0
SPECIAL EDITION ( STUDY MATERIAL FORM )
2

At market Book is available in 3 volume i.e. in 3 book binding


form. But at NODIA Online Store book is available in 10 book
binding form. Each unit of Book is in separate binding.
Vc = 3 V Available Only at NODIA Online Store
V2 =- Vc =- 3 V
Click to Buy
2.67 Option (B) is correct.
www.nodia.co.in
For a tree there must not be any loop. So a, c, and d don’t have any
loop. Only b has loop.
Option (D) is correct. z21 = V2
2.68 I1 I1 = 0
The sign of M is as per sign of L If current enters or exit the dotted Consider the given lattice network, when I2 = 0 . There is two simi-
terminals of both coil. The sign of M is opposite of L If current
lar path in the circuit for the current I1. So I = 1 I1
enters in dotted terminal of a coil and exit from the dotted terminal 2
of other coil.
Thus Leq = L1 + L2 - 2M
2.69 Option (A) is correct.
Here w = 2 and V = 1+0c
Y = 1 + jwC + 1
R jwL
= 3 + j2 # 3 + 1 1 = 3 + j4 For z11 applying KVL at input port we get
j2 # 4
V1 = I (Za + Zb)
= 5+ tan - 1 4 = 5+53.11c
3 Thus V1 = 1 I1 (Za + Zb)
2
I = V * Y = (1+0c)( 5+53.1c) = 5+53.1c
Thus i (t) = 5 sin (2t + 53.1c) z11 = 1 (Za + Zb)
2
2.70 Option (A) is correct. For Z21 applying KVL at output port we get
3
vi (t) = 2 sin 10 t V2 = Za I1 - Zb I1
3
Here w = 10 rad and Vi = 2 +0c 2 2
1 Thus V2 = 1 I1 (Za - Zb)
jwC 2
Now V0 = .Vt = 1 V
R+ 1 1 + jwCR i z21 = 1 (Za - Zb)
jwC 2
1 For this circuit z11 = z22 and z12 = z21. Thus
= 2 + 0c R V
1 + j # 103 # 10 - 3 S Za + Zb Za - Zb W
z11 z12
=z z G = SS Za - Zb Za + Zb WW
= 1 - 45c 2 2
21 22
v0 (t) = sin (103 t - 45c) S 2 2 W
T X
2.71 Option (C) is correct. Here Za = 2j and Zb = 2W
Input voltage vi (t) = u (t) z11 z12 1+j j-1
Thus =z z G = = j - 1 1 + j G
Taking Laplace transform Vi (s) = 1 21 22
s
GATE Electronics and Communication Topicwise Solved Paper by RK Kanodia & Ashish Murolia Page 42

2.73 Option (B) is correct. = minimum number of equation


Applying KVL, Number of branches = b = 8
Ldi (t) 1 3
Number of nodes = n = 5
v (t) = Ri (t) +
dt
+
C
#0 i (t) dt
Minimum number of equation
Taking L.T. on both sides,
= 8-5+1 = 4
+ I (s) vc (0+)
V (s) = RI (s) + LsI (s) - Li (0 ) + + 2.78 Option (C) is correct.
sC sC
For maximum power transfer
v (t) = u (t) thus V (s) = 1
s ZL = ZS* = Rs - jXs
1 I (s) 1 Thus ZL = 1 - 1j
Hence = I (s) + sI (s) - 1 + -
s s s
2.79 Option (B) is correct.
6s + s + 1@
2 +1 I (s) 2
= L
s s Q = 1
R C
or I (s) = 2s+2 When R, L and C are doubled,
s +s+1
Q' = 1 2L = 1 L =Q
2.74 Option (B) is correct. 2R 2C 2R C 2
Characteristics equation is
Thus Q' = 100 = 50
s2 + 20s + 106 = 0 2
Comparing with s2 + 2xwn s + wn2 = 0 we have 2.80 Option (C) is correct.
Applying KVL we get,
GATE Electronics & Communication
di (t) 1
by RK Kanodia sin t = Ri (t) + L
dt
+
C
#i (t) dt
Now in 3 Volume di (t)
Purchase Online at maximum discount from online store
or sin t = 2i (t) + 2
dt
#
+ i (t) dt

and get POSTAL and Online Test Series Free Differentiating with respect to t , we get
visit www.nodia.co.in cos t =
2di (t) 2d2 i (t)
+ + i (t)
dt dt2
wn = 106 = 103 2.81 Option (A) is correct.
2xw = 20 For current i there is 3 similar path. So current will be divide in
three path
Thus 2x = 203 = 0.02
10
Now Q = 1 = 1 = 50
2x 0.02
2.75 Option (D) is correct.
V0 (s)
H (s) =
Vi (s)
1
= sC = 2 1
R + sL + 1 s LC + sCR + 1
sC
= 2 -2 1
s (10 # 10 ) + s (10-4 # 10 4) + 1
-4 so, we get

= -6 2 1 = 2 106 Vab - b i # 1l - b i # 1l - b 1 # 1l = 0
10 s + s + 1 s + 106 s + 106 3 6 3
For more GATE Resources, Mock Test and
Option (D) is correct.
Study material join the community
2.76

Impedance of series RLC circuit at resonant frequency is minimum,


not zero. Actually imaginary part is zero. http://www.facebook.com/gateec2014
Z = R + j ` wL - 1 j
wC Vab = R = 1 + 1 + 1 = 5 W
eq
At resonance wL - 1 = 0 and Z = R that is purely resistive. i 3 6 3 6
wC
Thus S1 is false 2.82 Option ( ) is correct.
Data are missing in question as L1 &L2 are not given.
Now quality factor Q =R C
L 2.83 Option (A) is correct.
Since G = 1 , Q = 1 C At t = 0 - circuit is in steady state. So inductor act as short circuit
R G L and capacitor act as open circuit.
If G - then Q . provided C and L are constant. Thus S2 is also
false.
2.77 Option (B) is correct.
Number of loops = b - n + 1
GATE Electronics and Communication Topicwise Solved Paper by RK Kanodia & Ashish Murolia Page 43

Now Z1 = R + jw1 L = 1 + j1
Z2 = R + jw2 L = 1 + j2
v (t) v (t)
i (t) = 1 + 2
Z1 Z2
10 2 cos (t + 10c) 10 5 cos (2t + 10c)
= +
1+j 1 + j2
10 2 cos (t + 10c) 10 5 cos (2t + 10c)
= +
At t = 0 - , i1 (0 -) = i2 (0 -) = 0 12 + 22 + tan-1 1 12 + 22 tan-1 2
vc (0 -) = V 10 2 cos (t + 10c) 10 5 cos (2t + 10c)
= +
At t = 0+ the circuit is as shown in fig. The voltage across capaci- 2 + tan-1 45c 5 tan-1 2
tor and current in inductor can’t be changed instantaneously. Thus i (t) = 10 cos (t - 35c) + 10 cos (2t + 10c - tan-1 2)
2.87 Option (A) is correct.
Using 3- Y conversion

SPECIAL EDITION ( STUDY MATERIAL FORM )


At market Book is available in 3 volume i.e. in 3 book binding
At t = 0+ , i1 = i2 =- V
2R form. But at NODIA Online Store book is available in 10 book
2.84 Option (C) is correct. binding form. Each unit of Book is in separate binding.
When switch is in position 2, as shown in fig in question, applying Available Only at NODIA Online Store
KVL in loop (1), Click to Buy
RI1 (s) + V + 1 I1 (s) + sL [I1 (s) - I2 (s)] = 0
s sC
www.nodia.co.in
or I1 (s) 8R + 1 + sL B - I2 (s) sL = - V
sc s
z11 I1 + z12 I2 = V1
Applying KVL in loop 2,
sL [I2 (s) - I1 (s)] + RI2 (s) + 1 I2 (s) = 0
sC
Z12 I1 + Z22 I2 = V2
or - sLI1 (s) + 8R + sL + 1 BI2 (s) = 0
sc
Now comparing with R1 = 2 # 1 = 2 = 0.5
Z11 Z12 I1 V1 2+1+1 4
=Z Z G=I G = =V G
21 22 2 2 R2 = 1 # 1 = 1 = 0.25
2+1+1 4
we get
R V R3 = 2 # 1 = 0.5
SR + sL + 1 - sL W I1 (s) -V
2+1+1
= G > sH
sC
S W =
SS - sL R + sL + 1 WW I2 (s) 0
Now the circuit is as shown in figure below.
sC
T X
2.85 Option (B) is correct.
Zeros =- 3
Pole1 =- 1 + j
Pole 2 =- 1 - j
K (s + 3)
Z (s) =
(s + 1 + j)( s + 1 - j) Now z11 = V1 = 2 + 0.5 + 0.25 = 2.75
I1 I2 = 0
K (s + 3) K (s + 3) z12 = R3 = 0.25
= 2 2
=
(s + 1) - j (s + 1) 2 + 1
From problem statement Z (0) w = 0 = 3 2.88 Option (A) is correct.
Applying KCL at for node 2,
Thus 3K = 3 and we get K = 2
2
2 (s + 3)
Z (s) = 2
s + 2s + 2
2.86 Option (C) is correct.
v (t) = 10 2 cos (t + 10c) + 10 5 cos (2t + 10c)
1 4444 2 4444 3 1 4444 4 2 4444 43
v1 v2
Thus we get w1 = 1 and w2 = 2 V2 + V2 - V1 = V1
5 5 5
GATE Electronics and Communication Topicwise Solved Paper by RK Kanodia & Ashish Murolia Page 44

or V2 = V1 = 20 V 2.91 Option (D) is correct.


Voltage across dependent current source is 20 thus power deliv- IP , VP " Phase current and Phase voltage
ered by it is IL, VL " Line current and line voltage

PV2 # V1 = 20 # 20 = 80 W Now VP = c VL m and IP = IL


5 5 3
It deliver power because current flows from its +ive terminals. So, Power = 3VP IL cos q
2.89 Option (C) is correct. 1500 = 3 c VL m (IL) cos q
3
When switch was closed, in steady state, iL (0 -) = 2.5 A
also IL = c VL m
3 ZL

1500 = 3 c VL mc VL m cos q
3 3 ZL
(400) 2 (.844)
ZL = = 90 W
1500
As power factor is leading
So, cos q = 0.844 " q = 32.44
At t = 0+ , iL (0+) = iL (0 -) = 2.5 A and all this current of will pass As phase current leads phase voltage
through 2 W resistor. Thus ZL = 90+ - q = 90+ - 32.44c
GATE Electronics & Communication
2.92 Option (C) is correct.
by RK Kanodia
Applying KCL, we get
Now in 3 Volume e0 - 12 + e0 + e0 = 0
Purchase Online at maximum discount from online store 4 4 2+2
and get POSTAL and Online Test Series Free or e0 = 4 V
visit www.nodia.co.in 2.93 Option (A) is correct.
The star delta circuit is shown as below
Vx =- 2.5 # 20 =- 50 V
2.90 Option (A) is correct.
For maximum power delivered, RL must be equal to Rth across same
terminal.

Here ZAB = ZBC = ZCA = 3 Z


Applying KCL at Node, we get and ZA = ZAB ZCA
ZAB + ZBC + ZCA
0.5I1 = Vth + I1 ZAB ZBC
20 ZB =
ZAB + ZBC + ZCA
or Vth + 10I1 = 0
ZC = ZBC ZCA
but I1 = Vth - 50 ZAB + ZBC + ZCA
40 For more GATE Resources, Mock Test and
Thus Vth + Vth - 50 = 0 Study material join the community
4
http://www.facebook.com/gateec2014
or Vth = 10 V
For Isc the circuit is shown in figure below. 3Z 3Z
Now ZA = ZB = ZC = = Z
3Z+ 3Z+ 3Z 3

2.94 Option (C) is correct.


y11 y12 y1 + y3 - y3
=y y G = = - y y + y G
21 22 3 2 3

y12 =- y3
Isc = 0.5I1 - I1 =- 0.5I1
y12 =- 1 =- 0.05 mho
but I1 =- 50 =- 1.25 A 20
40
Isc =- 0.5 # - 12.5 = 0.625 A 2.95 Option (D) is correct.
We apply source conversion the circuit as shown in fig below.
Rth = Vth = 10 = 16 W
Isc 0.625
GATE Electronics and Communication Topicwise Solved Paper by RK Kanodia & Ashish Murolia Page 45

Now applying nodal analysis we have


e0 - 80 + e0 + e0 - 16 = 0 V0 = RL
10 + 2 12 6 (finite value)
Vs RL + Rs
or 4e0 = 112 1 circuit acts as shown in fig and
At resonant frequency w =
e0 = 112 = 28 V LC
4 V0 = 0 .
2.96 Option (A) is correct.
jw C
I2 = Em +01c = Em +0c
R2 + jwC 1 + jwCR2

+I2 = +90c
+ tan-1 wCR2
I2 = E m wC + (90c - tan-1 wCR2)
2 2
1 + w C R2 2 Thus it is a band reject filter.
At w = 0 I2 = 0 SPECIAL EDITION ( STUDY MATERIAL FORM )
and at w = 3, I2 = Em
At market Book is available in 3 volume i.e. in 3 book binding
R2 form. But at NODIA Online Store book is available in 10 book
Only fig. given in option (A) satisfies both conditions. binding form. Each unit of Book is in separate binding.
2.97 Option (A) is correct. Available Only at NODIA Online Store

Xs = wL = 10 W Click to Buy
For maximum power transfer www.nodia.co.in
RL = Rs2 + Xs2 = 102 + 102 = 14.14 W 2.100 Option (D) is correct.
2.98 Option (C) is correct. Applying KCL we get
Applying KVL in LHS loop iL = eat + ebt
E1 = 2I1 + 4 (I1 + I2) - 10E1
Now V (t) = vL = L diL = L d [eat + ebt] = aeat + bebt
dt dt
or E1 = 6I1 + 4I2
11 11 2.101 Option (A) is correct.
Thus z11 = 6 Going from 10 V to 0 V
11
Applying KVL in RHS loop
E2 = 4 (I1 + I2) - 10E1
= 4 (I1 + I2) - 10 c 6I1 + 4I2 m
11 11
=- 16I1 + 4I2
11 11
Thus z21 =- 16
11
2.99 Option (D) is correct. 10 + 5 + E + 1 = 0
At w = 0 , circuit act as shown in figure below. or E =- 16 V
2.102 Option (C) is correct.
This is a reciprocal and linear network. So we can apply reciprocity
theorem which states “Two loops A & B of a network N and if an
ideal voltage source E in loop A produces a current I in loop B ,
then interchanging positions an identical source in loop B produces
the same current in loop A. Since network is linear, principle of
homogeneity may be applied and when volt source is doubled,
V0 = RL (finite value) current also doubles.
Vs RL + Rs
Now applying reciprocity theorem
At w = 3 , circuit act as shown in figure below: i = 2 A for 10 V
V = 10 V, i = 2 A
V =- 20 V, i =- 4 A
2.103 Option (C) is correct.
Tree is the set of those branch which does not make any loop and
GATE Electronics and Communication Topicwise Solved Paper by RK Kanodia & Ashish Murolia Page 46

connects all the nodes. Req = 5W 20W + 4W


abfg is not a tree because it contains a loop l node (4) is not connected
Req = 5.20 + 4 = 4 + 4 = 8 W
5 + 20
2.107 Option (D) is correct.
Delta to star conversion
R1 = Rab Rac = 5 # 30 = 150 = 3 W
Rab + Rac + Rbc 5 + 30 + 15 50
R2 = Rab Rbc = 5 # 15 = 1.5 W
2.104 Option (A) is correct. Rab + Rac + Rbc 5 + 30 + 15
For a 2-port network the parameter h21 is defined as Rac Rbc
R3 = = 15 # 30 = 9 W
h21 = I2 Rab + Rac + Rbc 5 + 30 + 15
I1 V = 0 (short circuit)
2
2.108 Option (C) is correct.
No. of branches = n + l - 1 = 7 + 5 - 1 = 11
2.109 Option (B) is correct.
In nodal method we sum up all the currents coming & going at
the node So it is based on KCL. Furthermore we use ohms law to
determine current in individual branch. Thus it is also based on
Applying node equation at node a we get ohms law.
2.110 Option (A) is correct.
GATE Electronics & Communication Superposition theorem is applicable to only linear circuits.
by RK Kanodia 2.111 Option (B) is correct.
Now in 3 Volume 2.112 Option (B) is correct.
Purchase Online at maximum discount from online store For reciprocal network y12 = y21 but here y12 =- 12 ! y21 = 12 . Thus
and get POSTAL and Online Test Series Free circuit is non reciprocal. Furthermore only reciprocal circuit are
visit www.nodia.co.in passive circuit.
2.113 Option (C) is correct.
Va - V1 + Va - 0 + Va - 0 = 0
R R R Taking b as reference node and applying KCL at a we get
Vab - 1 + Vab = 3
3Va = V1 & Va = V1 2 2
3
or Vab - 1 + Vab = 6
V1 - V1
V1 - Va 3 = 2V1
Now I1 =
R
=
R 3R or Vab = 6 + 1 = 3.5 V
2

0 - V 0 - V1 - V 2.114 Option (A) is correct.


and I2 = a
= 3 = 1
R R 3R 2.115 Option (B) is correct.
I2 - V1 /3R - 1 The given figure is shown below.
Thus = h21 = =
I1 V = 0
2
2V1 /3R 2
2.105 Option (A) is correct.
Applying node equation at node A
Vth - 100 (1 + j0) Vth - 0
+ =0
3 4j
or 4jVth - 4j100 + 3Vth = 0
or Vth (3 + 4j) = 4j100
For more GATE Resources, Mock Test and
4j100 Study material join the community
Vth =
By simplifying
3 + 4j http://www.facebook.com/gateec2014
4j100 3 - 4j
Vth =
3 + 4j # 3 - 4j
Vth = 16j (3 - j4)
2.106 Option (C) is correct.
For maximum power transfer RL should be equal to RTh at same
terminal.
so, equivalent Resistor of the circuit is

Applying KCL at node a we have


I = i 0 + i1 = 7 + 5 = 12 A
GATE Electronics and Communication Topicwise Solved Paper by RK Kanodia & Ashish Murolia Page 47

Applying KCL at node f In the steady state condition all capacitors behaves as open circuit
I =- i 4 & Inductors behaves as short circuits as shown below :
so i 4 =- 12 amp
2.116 Option (A) is correct.

Thus voltage across capacitor C1 is


VC = 100 # 40 = 80 V
1
10 + 40
Now the circuit faced by capacitor C2 and C 3 can be drawn as
so V = 3 - 0 = 3 volt below :
2.117 Option (D) is correct.
Can not determined V without knowing the elements in box.
2.118 Option (A) is correct.
The voltage V is the voltage across voltage source and that is 10 V.
2.119 Option (B) is correct.
Voltage across capacitor SPECIAL EDITION ( STUDY MATERIAL FORM )
-t At market Book is available in 3 volume i.e. in 3 book binding
VC (t) = VC (3) + (VC (0) - VC (3)) e RC
form. But at NODIA Online Store book is available in 10 book
Here VC (3) = 10 V and (VC (0) = 6 V. Thus
-t -t -t
binding form. Each unit of Book is in separate binding.
VC (t) = 10 + (6 - 10) e RC = 10 - 4e RC = 10 - 4e 8 Available Only at NODIA Online Store
Now VR (t) = 10 - VC (t) Click to Buy
www.nodia.co.in
-t -t
= 10 - 10 + 4e RC = 4e RC

Energy absorbed by resistor


2 -t

#0 3V RR(t) = #0 3 164e 4
-t
E = #0 3 4e 4 = 16 J

2.120 Option (B) is correct.


It is a balanced whetstone bridge
R1 R 3
b R2 = R 4 l
so equivalent circuit is
Voltage across capacitor C2 and C 3 are
VC = 80 C 3 = 80 # 3 = 48 volt
2
C2 + C3 5
VC = 80 C2 = 80 # 2 = 32 volt
3
C2 + C3 5

Zeq = (4W 8W) = 4 # 8 = 8


4+8 3
2.121 Option (B) is correct.
Current in A2 , I2 = 3 amp
Inductor current can be defined as I2 =- 3j
Current in A 3 , I3 = 4
Total current I1 = I 2 + I 3
I1 = 4 - 3j
I = (4) 2 + (3) 2 = 5 amp
2.122 Option (C) is correct.
For a tree we have (n - 1) branches. Links are the branches which
from a loop, when connect two nodes of tree.
so if total no. of branches = b
No. of links = b - (n - 1) = b - n + 1
Total no. of links in equal to total no. of independent loops.
2.123 Option (B) is correct.
GATE Electronics and Communication Topicwise Solved Paper by RK Kanodia & Ashish Murolia Page 47

UNIT 3 In the CMOS circuit shown, electron and hole mobilities are equal,
3.6

and M1 and M2 are equally sized. The device M1 is in the linear


region if
ELECTRONICS DEVICES

2013 ONE MARK

3.1 In a forward biased pn junction diode, the sequence of events that


(A) Vin < 1.875 V (B) 1.875 V < Vin < 3.125 V
best describes the mechanism of current flow is
(A) injection, and subsequent diffusion and recombination of mi- (C) Vin > 3.125 V (D) 0 < Vin < 5 V
nority carriers
(B) injection, and subsequent drift and generation of minority car- Common Data For Q. 2 and 3 :
riers
In the three dimensional view of a silicon n -channel MOS transis-
(C) extraction, and subsequent diffusion and generation of minor- tor shown below, d = 20 nm . The transistor is of width 1 mm . The
ity carriers depletion width formed at every p-n junction is 10 nm. The rela-
(D) extraction, and subsequent drift and recombination of minority
carriers SPECIAL EDITION ( STUDY MATERIAL FORM )
At market Book is available in 3 volume i.e. in 3 book binding
3.2 In IC technology, dry oxidation (using dry oxygen) as compared to
wet oxidation (using steam or water vapor) produces
form. But at NODIA Online Store book is available in 10 book
(A) superior quality oxide with a higher growth rate binding form. Each unit of Book is in separate binding.
Available Only at NODIA Online Store
(B) inferior quality oxide with a higher growth rate
(C) inferior quality oxide with a lower growth rate
Click to Buy
(D) superior quality oxide with a lower growth rate www.nodia.co.in
3.3 In a MOSFET operating in the saturation region, the channel length tive permittivity of Si and SiO 2 , respectively, are 11.7 and 3.9, and
modulation effect causes e0 = 8.9 # 10-12 F/m .
(A) an increase in the gate-source capacitance
(B) a decrease in the transconductance
(C) a decrease in the unity-gain cutoff frequency
(D) a decrease in the output resistance

2013 TWO MARKS

3.4 The small-signal resistance (i.e., dVB /dID ) in kW offered by the


n-channel MOSFET M shown in the figure below, at a bias point of
VB = 2 V is (device data for M: device transconductance parameter
kN = mn C 0' x ^W/L h = 40 mA/V2 , threshold voltage VTN = 1 V , and
neglect body effect and channel length modulation effects)

3.7 The gate source overlap capacitance is approximately


(A) 0.7 fF (B) 0.7 pF
(C) 0.35 fF (D) 0.24 pF

3.8 The source-body junction capacitance is approximately


(A) 2 fF (B) 7 fF
(A) 12.5 (B) 25
(C) 2 pF (D) 7 pF
(C) 50 (D) 100

2011 ONE MARK


2012 TWO MARKS
3.9 Drift current in the semiconductors depends upon
3.5 The source of a silicon (ni = 1010 per cm3) n -channel MOS transistor (A) only the electric field
has an area of 1 sq mm and a depth of 1 mm . If the dopant density in
(B) only the carrier concentration gradient
the source is 1019 /cm3 , the number of holes in the source region with
the above volume is approximately (C) both the electric field and the carrier concentration
(A) 107 (B) 100 (D) both the electric field and the carrier concentration gradient
(C) 10 (D) 0 3.10 A Zener diode, when used in voltage stabilization circuits, is biased
GATE Electronics and Communication Topicwise Solved Paper by RK Kanodia & Ashish Murolia Page 48

in (C) epitaxial oxidation (D) ion implantation


(A) reverse bias region below the breakdown voltage
(B) reverse breakdown region
2010 TWO MARKS
(C) forward bias region
3.16 In a uniformly doped BJT, assume that NE , NB and NC are the
(D) forward bias constant current mode
emitter, base and collector doping in atoms/cm3 , respectively. If the
3.11 A silicon PN junction is forward biased with a constant current at emitter injection efficiency of the BJT is close unity, which one of
room temperature. When the temperature is increased by 10ºC, the the following condition is TRUE
forward bias voltage across the PN junction (A) NE = NB = NC (B) NE >> NB and NB > NC
(A) increases by 60 mV (C) NE = NB and NB < NC (D) NE < NB < NC
(B) decreases by 60 mV
3.17 Compared to a p-n junction with NA = ND = 1014 /cm3 , which
(C) increases by 25 mV one of the following statements is TRUE for a p-n junction with
(D) decreases by 25 mV NA = ND = 1020 /cm3 ?
(A) Reverse breakdown voltage is lower and depletion capacitance
is lower
2011 TWO MARKS
(B) Reverse breakdown voltage is higher and depletion capacitance
is lower
Common Data For Q. 3.12 & 3.13 : (C) Reverse breakdown voltage is lower and depletion capacitance
The channel resistance of an N-channel JFET shown in the fig- is higher
GATE Electronics & Communication (D) Reverse breakdown voltage is higher and depletion capacitance
is higher
by RK Kanodia
Now in 3 Volume
Purchase Online at maximum discount from online store Statements for Linked Answer Question : 3.10 & 3.11 :
and get POSTAL and Online Test Series Free The silicon sample with unit cross-sectional area shown below is in
visit www.nodia.co.in thermal equilibrium. The following information is given: T = 300 K
electronic charge = 1.6 # 10-19 C , thermal voltage = 26 mV and
ure below is 600 W when the full channel thickness (tch ) of 10 μm electron mobility = 1350 cm2 / V-s
is available for conduction. The built-in voltage of the gate P+ N
junction (Vbi ) is - 1 V . When the gate to source voltage (VGS ) is 0
V, the channel is depleted by 1 μm on each side due to the built in
voltage and hence the thickness available for conduction is only 8
μm

3.18 The magnitude of the electric field at x = 0.5 mm is


(A) 1 kV/cm (B) 5 kV/cm
(C) 10 kV/cm (D) 26 kV/cm

3.19 The magnitude of the electron of the electron drift current density
at x = 0.5 mm is
(A) 2.16 # 10 4 A/cm2 (B) 1.08 # 10 4 A/m2
The channel resistance when VGS =- 3 V is
3.12
For more GATE Resources, Mock Test and
(A) 360 W (B) 917 W
Study material join the community
(C) 1000 W (D) 3000 W
http://www.facebook.com/gateec2014
3.13 The channel resistance when VGS = 0 V is
(A) 480 W (B) 600 W (C) 4.32 # 103 A/cm2 (D) 6.48 # 102 A/cm2
(C) 750 W (D) 1000 W
2009 ONE MARK
2010 ONE MARK 3.20 In an n-type silicon crystal at room temperature, which of the
3.14 At room temperature, a possible value for the mobility of electrons following can have a concentration of 4 # 1019 cm - 3 ?
in the inversion layer of a silicon n -channel MOSFET is (A) Silicon atoms (B) Holes
(A) 450 cm2 / V-s (B) 1350 cm2 / V-s (C) Dopant atoms (D) Valence electrons
(C) 1800 cm2 / V-s (D) 3600 cm2 / V-s 3.21 The ratio of the mobility to the diffusion coefficient in a semiconductor
3.15 Thin gate oxide in a CMOS process in preferably grown using has the units
(A) wet oxidation (B) dry oxidation (A) V - 1 (B) cm.V1
GATE Electronics and Communication Topicwise Solved Paper by RK Kanodia & Ashish Murolia Page 49

(C) V.cm - 1 (D) V.s 3.28 The drain current of MOSFET in saturation is given by
ID = K (VGS - VT ) 2 where K is a constant.
The magnitude of the transconductance gm is
2009 TWO MARKS
K (VGS - VT ) 2
3.22 Consider the following two statements about the internal conditions (A) (B) 2K (VGS - VT )
VDS
in a n -channel MOSFET operating in the active region.
Id K (VGS - VT ) 2
S1 : The inversion charge decreases from source to drain (C) (D)
VGS - VDS VGS
S2 : The channel potential increases from source to drain.
Which of the following is correct?
(A) Only S2 is true 2008 TWO MARKS

(B) Both S1 and S2 are false 3.29 The measured trans conductance gm of an NMOS transistor
(C) Both S1 and S2 are true, but S2 is not a reason for S1 operating in the linear region is plotted against the gate voltage
VG at a constant drain voltage VD . Which of the following figures
(D) Both S1 and S2 are true, and S2 is a reason for S1
represents the expected dependence of gm on VG ?

Common Data For Q. 3.13 and 3.14


Consider a silicon p - n junction at room temperature having the
following parameters:
Doping on the n -side = 1 # 1017 cm - 3
SPECIAL EDITION ( STUDY MATERIAL FORM )
Depletion width on the n -side = 0.1mm
At market Book is available in 3 volume i.e. in 3 book binding
Depletion width on the p -side = 1.0mm
form. But at NODIA Online Store book is available in 10 book
Intrinsic carrier concentration = 1.4 # 1010 cm - 3
binding form. Each unit of Book is in separate binding.
Thermal voltage = 26 mV Available Only at NODIA Online Store
- 14
Permittivity of free space = 8.85 # 10 F.cm - 1 Click to Buy
Dielectric constant of silicon = 12 www.nodia.co.in
3.23 The built-in potential of the junction
(A) is 0.70 V
(B) is 0.76 V
(C) is 0.82 V
(D) Cannot be estimated from the data given

3.24 The peak electric field in the device is


(A) 0.15 MV . cm - 1, directed from p -region to n -region
(B) 0.15 MV . cm - 1, directed from n -region to p -region
(C) 1.80 MV . cm - 1, directed from p-region to n -region
(D) 1.80 MV . cm - 1, directed from n -region to p -region

2008 ONE MARK

3.25 Which of the following is NOT associated with a p - n junction ?


(A) Junction Capacitance (B) Charge Storage Capacitance
(C) Depletion Capacitance (D) Channel Length Modula-
tions
3.30 Silicon is doped with boron to a concentration of 4 # 1017 atoms cm3
3.26 Which of the following is true? . Assume the intrinsic carrier concentration of silicon to be 1.5 # 1010
(A) A silicon wafer heavily doped with boron is a p+ substrate / cm 3 and the value of kT/q to be 25 mV at 300 K. Compared to
(B) A silicon wafer lightly doped with boron is a p+ substrate undopped silicon, the fermi level of doped silicon
(C) A silicon wafer heavily doped with arsenic is a p+ substrate (A) goes down by 0.31 eV (B) goes up by 0.13 eV
(D) A silicon wafer lightly doped with arsenic is a p+ substrate (C) goes down by 0.427 eV (D) goes up by 0.427 eV

3.27 A silicon wafer has 100 nm of oxide on it and is furnace at a 3.31 The cross section of a JFET is shown in the following figure. Let
temperature above 1000c C for further oxidation in dry oxygen. The Vc be - 2 V and let VP be the initial pinch -off voltage. If the width
oxidation rate W is doubled (with other geometrical parameters and doping levels
(A) is independent of current oxide thickness and temperature remaining the same), then the ratio between the mutual trans
(B) is independent of current oxide thickness but depends on tem- conductances of the initial and the modified JFET is
perature
(C) slows down as the oxide grows
(D) is zero as the existing oxide prevents further oxidation
GATE Electronics and Communication Topicwise Solved Paper by RK Kanodia & Ashish Murolia Page 50

(C) P - 2, Q - 2, R - 1, S- -2
(D) P - 2, Q - 1, R - 2, S - 2

3.36 Group I lists four different semiconductor devices. match each device
in Group I with its charactecteristic property in Group II
Group-I Group-II
(P) BJT (1) Population iniversion
1 - 2/Vp (Q) MOS capacitor (2) Pinch-off voltage
(B) 1 e
2 1 - 1/2Vp o
(A) 4 (R) LASER diode (3) Early effect
(S) JFET (4) Flat-band voltage
1 - 2/Vp 1 - (2 - Vp )
(C) e o (D)
1 - 1/2Vp 1 - [1 (2 Vp )] (A) P - 3, Q - 1, R - 4, S - 2
(B) P - 1, Q - 4, R - 3, S - 2
3.32 Consider the following assertions.
S1 : For Zener effect to occur, a very abrupt junction is required. (C) P - 3, Q - 4, R - 1, S - 2
S2 : For quantum tunneling to occur, a very narrow energy barrier (D) P - 3, Q - 2, R - 1, S - 4
is required. 3.37 A p+ n junction has a built-in potential of 0.8 V. The depletion layer
Which of the following is correct ? width a reverse bias of 1.2 V is 2 mm. For a reverse bias of 7.2 V, the
(A) Only S2 is true depletion layer width will be
(B) S1 and S2 are both true but S2 is not a reason for S1 (A) 4 mm (B) 4.9 mm
(C) 8 mm (D) 12 mm
GATE Electronics & Communication
The DC current gain (b) of a BJT is 50. Assuming that the emitter
by RK Kanodia
3.38

injection efficiency is 0.995, the base transport factor is


Now in 3 Volume (A) 0.980 (B) 0.985
Purchase Online at maximum discount from online store (C) 0.990 (D) 0.995
and get POSTAL and Online Test Series Free
visit www.nodia.co.in
Common Data For Q. 2.29, 2.30 and 2.31 :
(C) S1 and S2 and are both true but S2 is not a reason for S1 The figure shows the high-frequency capacitance - voltage charac-
(D) Both S1 and S2 are false teristics of Metal/Sio 2 /silicon (MOS) capacitor having an area of
1 # 10 - 4 cm 2 . Assume that the permittivities (e0 er ) of silicon and
Sio2 are 1 # 10 - 12 F/cm and 3.5 # 10 - 13 F/cm respectively.
2007 ONE MARK

3.33 The electron and hole concentrations in an intrinsic semiconductor


are ni per cm3 at 300 K. Now, if acceptor impurities are introduced
with a concentration of NA per cm3 (where NA >> ni , the electron
concentration per cm3 at 300 K will be)
(A) ni (B) ni + NA
2
(C) NA - ni (D) ni
NA
+ 3.39 The gate oxide thickness in the MOS capacitor is
3.34 In a p n junction diode under reverse biased the magnitude of
(A) 50 nm (B) 143 nm
electric field is maximum at
(A) the edge of the depletion region on the p-side (C) 350 nm (D) 1 mm
(B) the edge of the depletion region on the n -side
(C) the p+ n junction
For more GATE Resources, Mock Test and
(D) the centre of the depletion region on the n -side Study material join the community
http://www.facebook.com/gateec2014
2007 TWO MARKS 3.40 The maximum depletion layer width in silicon is
3.35 Group I lists four types of p - n junction diodes. Match each device (A) 0.143 mm (B) 0.857 mm
in Group I with one of the option in Group II to indicate the bias (C) 1 mm (D) 1.143 mm
condition of the device in its normal mode of operation.
Group - I Group-II
3.41 Consider the following statements about the C - V characteristics
(P) Zener Diode (1) Forward bias plot :
(Q) Solar cell (2) Reverse bias S1 : The MOS capacitor has as n -type substrate
(R) LASER diode S2 : If positive charges are introduced in the oxide, the C - V polt
(S) Avalanche Photodiode will shift to the left.
Then which of the following is true?
(A) P - 1, Q - 2, R - 1, S - 2
(A) Both S1 and S2 are true
(B) P - 2, Q - 1, R - 1, S - 2
(B) S1 is true and S2 is false
GATE Electronics and Communication Topicwise Solved Paper by RK Kanodia & Ashish Murolia Page 51

(C) S1 is false and S2 is true (B) E - 3, F - 4, G - 1, H - 3


(D) Both S1 and S2 are false (C) E - 2, F - 4, G - 1, H - 2
(D) E - 1, F - 3, G - 2, H - 4
2006 ONE MARK 3.49 A heavily doped n - type semiconductor has the following data:
3.42 The values of voltage (VD) across a tunnel-diode corresponding to Hole-electron ratio : 0.4
peak and valley currents are Vp, VD respectively. The range of tunnel- Doping concentration : 4.2 # 108 atoms/m3
diode voltage for VD which the slope of its I - VD characteristics is Intrinsic concentration : 1.5 # 10 4 atoms/m 3
negative would be The ratio of conductance of the n -type semiconductor to that of
(A) VD < 0 (B) 0 # VD < Vp the intrinsic semiconductor of same material and ate same tem-
perature is given by
(C) Vp # VD < Vv (D) VD $ Vv
(A) 0.00005 (B) 2000
3.43 The concentration of minority carriers in an extrinsic semiconductor (C) 10000 (D) 20000
under equilibrium is
(A) Directly proportional to doping concentration
2005 ONE MARK
(B) Inversely proportional to the doping concentration
(C) Directly proportional to the intrinsic concentration 3.50 The bandgap of Silicon at room temperature is
(A) 1.3 eV (B) 0.7 eV
(D) Inversely proportional to the intrinsic concentration

3.44 Under low level injection assumption, the injected minority carrier SPECIAL EDITION ( STUDY MATERIAL FORM )
current for an extrinsic semiconductor is essentially the At market Book is available in 3 volume i.e. in 3 book binding
(A) Diffusion current (B) Drift current form. But at NODIA Online Store book is available in 10 book
(C) Recombination current (D) Induced current binding form. Each unit of Book is in separate binding.
Available Only at NODIA Online Store
3.45 The phenomenon known as “Early Effect” in a bipolar transistor
refers to a reduction of the effective base-width caused by Click to Buy
(A) Electron - hole recombination at the base www.nodia.co.in
(B) The reverse biasing of the base - collector junction
(C) The forward biasing of emitter-base junction (C) 1.1 eV (D) 1.4 eV
(D) The early removal of stored base charge during saturation-to- 3.51 A Silicon PN junction at a temperature of 20c C has a reverse
cut off switching saturation current of 10 pico - Ameres (pA). The reserve saturation
current at 40cC for the same bias is approximately
2006 TWO MARKS (A) 30 pA (B) 40 pA
(C) 50 pA (D) 60 pA
3.46 In the circuit shown below, the switch was connected to position 1
at t < 0 and at t = 0 , it is changed to position 2. Assume that the 3.52 The primary reason for the widespread use of Silicon in semiconductor
diode has zero voltage drop and a storage time ts . For 0 < t # ts, vR device technology is
is given by (all in Volts) (A) abundance of Silicon on the surface of the Earth.
(B) larger bandgap of Silicon in comparison to Germanium.
(C) favorable properties of Silicon - dioxide (SiO2)
(D) lower melting point

2005 TWO MARKS


(A) vR =- 5 (B) vR =+ 5
3.53 A Silicon sample A is doped with 1018 atoms/cm 3 of boron. Another
(C) 0 # vR < 5 (D) - 5 # vR < 0 sample B of identical dimension is doped with 1018 atoms/cm 3
3.47 The majority carriers in an n-type semiconductor have an average phosphorus. The ratio of electron to hole mobility is 3. The ratio of
drift velocity v in a direction perpendicular to a uniform magnetic conductivity of the sample A to B is
field B . The electric field E induced due to Hall effect acts in the (A) 3 (B) 1
3
direction
(A) v # B (B) B # v (C) 2 (D) 3
3 2
(C) along v (D) opposite to v 3.54 A Silicon PN junction diode under reverse bias has depletion region
3.48 Find the correct match between Group 1 and Group 2 of width 10 mm. The relative permittivity of Silicon, er = 11.7 and
Group 1 Group 2 the permittivity of free space e0 = 8.85 # 10 - 12 F/m. The depletion
E - Varactor diode 1. Voltage reference capacitance of the diode per square meter is
F - PIN diode 2. High frequency switch (A) 100 mF (B) 10 mF
G - Zener diode 3. Tuned circuits (C) 1 mF (D) 20 mF
H - Schottky diode 4. Current controlled attenuator
3.55 A MOS capacitor made using p type substrate is in the accumulation
(A) E - 4, F - 2, G - 1, H - 3 mode. The dominant charge in the channel is due to the presence of
GATE Electronics and Communication Topicwise Solved Paper by RK Kanodia & Ashish Murolia Page 52

(A) holes (B) electrons (C) a CMOS inverter


(C) positively charged icons (D) negatively charged ions (D) a BJT inverter

3.56 For an n -channel MOSFET and its transfer curve shown in the 3.61 Assuming VCEsat = 0.2 V and b = 50 , the minimum base current (IB)
figure, the threshold voltage is required to drive the transistor in the figure to saturation is

(A) 1 V and the device is in active region


(A) 56 mA (B) 140 mA
(B) - 1 V and the device is in saturation region
(C) 60 mA (D) 3 mA
(C) 1 V and the device is in saturation region
(D) - 1 V and the device is an active region
2004 TWO MARKS

2004 ONE MARK


3.62 In an abrupt p - n junction, the doping concentrations on the p -
side and n -side are NA = 9 # 1016 /cm 3 respectively. The p - n
junction is reverse biased and the total depletion width is 3 mm.
GATE Electronics & Communication The depletion width on the p -side is
by RK Kanodia (A) 2.7 mm (B) 0.3 mm
Now in 3 Volume (C) 2.25 mm (D) 0.75 mm
Purchase Online at maximum discount from online store 3.63 The resistivity of a uniformly doped n -type silicon sample is 0.5W -
and get POSTAL and Online Test Series Free mc. If the electron mobility (mn) is 1250 cm 2 /V-sec and the charge of
visit www.nodia.co.in an electron is 1.6 # 10 - 19 Coulomb, the donor impurity concentration
(ND) in the sample is
3.57 The impurity commonly used for realizing the base region of a silicon (A) 2 # 1016 /cm 3 (B) 1 # 1016 /cm 3
n - p - n transistor is (C) 2.5 # 1015 /cm 3 (D) 5 # 1015 /cm 3
(A) Gallium (B) Indium
(C) Boron (D) Phosphorus 3.64 Consider an abrupt p - n junction. Let Vbi be the built-in potential
of this junction and VR be the applied reverse bias. If the junction
3.58 If for a silicon npn transistor, the base-to-emitter voltage (VBE ) is capacitance (Cj ) is 1 pF for Vbi + VR = 1 V, then for Vbi + VR = 4 V,
0.7 V and the collector-to-base voltage (VCB) is 0.2 V, then the Cj will be
transistor is operating in the (A) 4 pF (B) 2 pF
(A) normal active mode (B) saturation mode (C) 0.25 pF (D) 0.5 pF
(C) inverse active mode (D) cutoff mode
3.65 Consider the following statements Sq and S2.
3.59 Consider the following statements S1 and S2. S1 : The threshold voltage (VT ) of MOS capacitor decreases with
S1 : The b of a bipolar transistor reduces if the base width is in- increase in gate oxide thickness.
creased. S2 : The threshold voltage (VT ) of a MOS capacitor decreases with
S2 : The b of a bipolar transistor increases if the dopoing concen- increase in substrate doping concentration.
tration in the base is increased. Which Marks of the following is correct ?
Which remarks of the following is correct ?
(A) S1 is FALSE and S2 is TRUE
For more GATE Resources, Mock Test and
(B) Both S1 and S2 are TRUE
Study material join the community
(C) Both S1 and S2 are FALSE http://www.facebook.com/gateec2014
(D) S1 is TRUE and S2 is FALSE
(A) S1 is FALSE and S2 is TRUE
3.60 Given figure is the voltage transfer characteristic of (B) Both S1 and S2 are TRUE
(C) Both S1 and S2 are FALSE
(D) S1 is TRUE and S2 is FALSE

3.66 The drain of an n-channel MOSFET is shorted to the gate so that


VGS = VDS . The threshold voltage (VT ) of the MOSFET is 1 V. If the
drain current (ID) is 1 mA for VGS = 2 V, then for VGS = 3 V, ID is
(A) 2 mA (B) 3 mA
(A) an NOMS inverter with enhancement mode transistor as load (C) 9 mA (D) 4 mA
(B) an NMOS inverter with depletion mode transistor as load
3.67 The longest wavelength that can be absorbed by silicon, which has
GATE Electronics and Communication Topicwise Solved Paper by RK Kanodia & Ashish Murolia Page 53

the bandgap of 1.12 eV, is 1.1 mm. If the longest wavelength that Group 1 Group 2
can be absorbed by another material is 0.87 mm, then bandgap of P. LED 1. Heavy doping
this material is Q. Avalanche photo diode 2. Coherent radiation
(A) 1.416 A/cm 2 (B) 0.886 eV R. Tunnel diode 3. Spontaneous emission
(C) 0.854 eV (D) 0.706 eV S. LASER 4. Current gain

3.68 The neutral base width of a bipolar transistor, biased in the active (A) P - 1, Q - 2, R - 4, S - 3
region, is 0.5 mm. The maximum electron concentration and the (B) P - 2, Q - 3, R - 1, S - 4
diffusion constant in the base are 1014 / cm 3 and Dn = 25 cm 2 / (C) P - 3 Q - 4, R - 1, S - 2
sec respectively. Assuming negligible recombination in the base, (D) P - 2, Q - 1, R - 4, S - 3
the collector current density is (the electron charge is 1.6 # 10 - 19 3.77 At 300 K, for a diode current of 1 mA, a certain germanium diode
Coulomb) requires a forward bias of 0.1435 V, whereas a certain silicon diode
(A) 800 A/cm 2 (B) 8 A/cm 2 requires a forward bias of 0.718 V. Under the conditions state above,
(C) 200 A/cm 2 (D) 2 A/cm 2 the closest approximation of the ratio of reverse saturation current
in germanium diode to that in silicon diode is
(A) 1 (B) 5
2003 ONE MARK
(C) 4 # 103 (D) 8 # 103
3.69 n -type silicon is obtained by doping silicon with
(A) Germanium (B) Aluminium
3.78 A particular green LED emits light of wavelength 5490 Ac. The
(C) Boron (D) Phosphorus SPECIAL EDITION ( STUDY MATERIAL FORM )
At market Book is available in 3 volume i.e. in 3 book binding
3.70 The Bandgap of silicon at 300 K is
(A) 1.36 eV (B) 1.10 eV form. But at NODIA Online Store book is available in 10 book
(C) 0.80 eV (D) 0.67 eV binding form. Each unit of Book is in separate binding.
Available Only at NODIA Online Store
3.71 The intrinsic carrier concentration of silicon sample at 300 K is Click to Buy
1.5 # 1016 /m 3 . If after doping, the number of majority carriers is
5 # 1020 /m 3 , the minority carrier density is
www.nodia.co.in
(A) 4.50 # 1011/m 3 (B) 3.333 # 10 4 /m 3 energy bandgap of the semiconductor material used there is
20
(C) 5.00 # 10 /m 3 -5
(D) 3.00 # 10 /m 3 (Plank’s constant = 6.626 # 10 - 34 J - s )
(A) 2.26 eV (B) 1.98 eV
3.72 Choose proper substitutes for X and Y to make the following (C) 1.17 eV (D) 0.74 eV
statement correct Tunnel diode and Avalanche photo diode are
operated in X bias ad Y bias respectively 3.79 When the gate-to-source voltage (VGs) of a MOSFET with threshold
(A) X: reverse, Y: reverse (B) X: reverse, Y: forward voltage of 400 mV, working in saturation is 900 mV, the drain current
(C) X: forward, Y: reverse (D) X: forward, Y: forward is observed to be 1 mA. Neglecting the channel width modulation
effect and assuming that the MOSFET is operating at saturation,
3.73 For an n - channel enhancement type MOSFET, if the source is the drain current for an applied VGS of 1400 mV is
connected at a higher potential than that of the bulk (i.e. VSB > 0 ), (A) 0.5 mA (B) 2.0 mA
the threshold voltage VT of the MOSFET will (C) 3.5 mA (D) 4.0 mA
(A) remain unchanged (B) decrease
(C) change polarity (D) increase 3.80 If P is Passivation, Q is n -well implant, R is metallization and S is
source/drain diffusion, then the order in which they are carried out
in a standard n -well CMOS fabrication process, is
2003 TWO MARKS (A) P - Q - R - S (B) Q - S - R - P
3.74 An n -type silicon bar 0.1 cm long and 100 mm2 i cross-sectional (C) R - P - S - Q (D) S - R - Q - P
area has a majority carrier concentration of 5 # 1020 /m 2 and the 3.81 The action of JFET in its equivalent circuit can best be represented
carrier mobility is 0.13 m2 /V-s at 300 K. If the charge of an electron
as a
is 1.5 # 10 - 19 coulomb, then the resistance of the bar is
(A) Current controlled current source
(A) 106 Ohm (B) 10 4 Ohm
(B) Current controlled voltage source
(C) 10 - 1 Ohm (D) 10 - 4 Ohm
(C) Voltage controlled voltage source
3.75 The electron concentration in a sample of uniformly doped n -type (D) Voltage controlled current source
silicon at 300 K varies linearly from 1017 /cm 3 at x = 0 to 6 # 1016 /
cm 3 at x = 2mm . Assume a situation that electrons are supplied to
keep this concentration gradient constant with time. If electronic 2002 ONE MARK
charge is 1.6 # 10 - 19 coulomb and the diffusion constant Dn = 35 cm 3.82 In the figure, silicon diode is carrying a constant current of 1 mA.
2 /s, the current density in the silicon, if no electric field is present, is
When the temperature of the diode is 20cC, VD is found to be 700
(A) zero (B) -112 A/cm 2 mV. If the temperature rises to 40cC, VD becomes approximately
(C) +1120 A/cm 2 (D) -1120 A/cm 2 equal to
3.76 Match items in Group 1 with items in Group 2, most suitably.
GATE Electronics and Communication Topicwise Solved Paper by RK Kanodia & Ashish Murolia Page 54

1998 ONE MARK

3.89 The electron and hole concentrations in a intrinsic semiconductor


are ni and pi respectively. When doped with a p-type material,
these change to n and p, respectively, Then
(A) n + p = ni + pi (B) n + ni = p + pi
(A) 740 mV (B) 660 mV
(C) npi = ni p (D) np = ni pi
(C) 680 mV (D) 700 mV
3.90 The fT of a BJT is related to its gm, C p and C m as follows
3.83 If the transistor in the figure is in saturation, then
Cp + Cm 2p (C p + C m)
(A) fT = (B) fT =
gm gm
gm gm
(C) fT = (D) fT =
Cp + Cm 2p (C p + C m)
3.91 The static characteristic of an adequately forward biased p-n
junction is a straight line, if the plot is of
(A) log I vs log V (B) log I vs V
(A) IC is always equal to bdc IB (C) I vs log V (D) I vs V
(B) IC is always equal to - bde IB 3.92 A long specimen of p-type semiconductor material
(C) IC is greater than or equal to bdc IB (A) is positively charged
(D) IC is less than or equal to bdc IB (B) is electrically neutral
GATE Electronics & Communication (C) has an electric field directed along its length
by RK Kanodia (D) acts as a dipole
Now in 3 Volume
3.93 Two identical FETs, each characterized by the parameters gm and rd
Purchase Online at maximum discount from online store are connected in parallel. The composite FET is then characterized
and get POSTAL and Online Test Series Free by the parameters
visit www.nodia.co.in g
(A) m and 2rd
g
(B) m and rd
2 2 2
2001 ONE MARK
(C) 2gm and rd (D) 2gm and 2rd
3.84 MOSFET can be used as a 2
(A) current controlled capacitor (B) voltage controlled capacitor q
3.94 The units of are
(C) current controlled inductor (D) voltage controlled inductor kT
(A) V (B) V-1
3.85 The effective channel length of MOSFET in saturation decreases (C) J (D) J/K
with increase in
(A) gate voltage (B) drain voltage
1997 ONE MARK
(C) source voltage (D) body voltage
3.95 For a MOS capacitor fabricated on a p-type semiconductor, strong
inversion occurs when
1999 ONE MARK (A) surface potential is equal to Fermi potential
3.86 The early effect in a bipolar junction transistor is caused by (B) surface potential is zero
(A) fast turn-on (C) surface potential is negative and
(B) fast turn-off equal to Fermi potential in magnitude
(C) large collector-base reverse bias (D) surface potential is positive and equal to twice the Fermi po-
(D) large emitter-base forward bias For more GATE Resources, Mock Test and
Study material join the community
1999 TWO MARKS http://www.facebook.com/gateec2014
3.87 An n -channel JEFT has IDSS = 2 mA and Vp =- 4 V . Its
tential
transconductance gm (in milliohm) for an applied gate-to-source
voltage VGS of - 2 V is 3.96 The intrinsic carrier density at 300 K is 1.5 # 1010 /cm3 , in silicon.
(A) 0.25 (B) 0.5 For n -type silicon doped to 2.25 # 1015 atoms/cm3 , the equilibrium
(C) 0.75 (D) 1.0 electron and hole densities are
(A) n = 1.5 # 1015 /cm3, p = 1.5 # 1010 /cm3
3.88 An npn transistor (with C = 0.3 pF ) has a unity-gain cutoff
(B) n = 1.5 # 1010 /cm3, p = 2.25 # 1015 /cm3
frequency fT of 400 MHz at a dc bias current Ic = 1 mA . The value
of its Cm (in pF) is approximately (VT = 26 mV) (C) n = 2.25 # 1015 /cm3, p = 1.0 # 1015 /cm3
(A) 15 (B) 30 (D) n = 1.5 # 1010 /cm3, p = 1.5 # 1010 /cm3
(C) 50 (D) 96
GATE Electronics and Communication Topicwise Solved Paper by RK Kanodia & Ashish Murolia Page 55

1996 ONE MARK

3.97 The p-type substrate in a conventional pn -junction isolated


integrated circuit should be connected to
(A) nowhere, i.e. left floating
(B) a DC ground potential
(C) the most positive potential available in the circuit
(D) the most negative potential available in the circuit

3.98 If a transistor is operating with both of its junctions forward biased,


but with the collector base forward bias greater than the emitter
base forward bias, then it is operating in the
(A) forward active mode (B) reverse saturation mode
(C) reverse active mode (D) forward saturation mode

3.99 The common-emitter short-circuit current gain b of a transistor


(A) is a monotonically increasing function of the collector current
IC
(B) is a monotonically decreasing function of IC
(C) increase with IC , for low IC , reaches a maximum and then de-
creases with further increase in IC SPECIAL EDITION ( STUDY MATERIAL FORM )
(D) is not a function of IC At market Book is available in 3 volume i.e. in 3 book binding
form. But at NODIA Online Store book is available in 10 book
3.100 A n -channel silicon (Eg = 1.1 eV) MOSFET was fabricated using binding form. Each unit of Book is in separate binding.
n +poly-silicon gate and the threshold voltage was found to be 1 V. Available Only at NODIA Online Store
Now, if the gate is changed to p+ poly-silicon, other things remaining
Click to Buy
the same, the new threshold voltage should be
(A) - 0.1 V (B) 0 V www.nodia.co.in
(C) 1.0 V (D) 2.1 V

1996 TWO MARKS

3.101 In a bipolar transistor at room temperature, if the emitter current


is doubled the voltage across its base-emitter junction
(A) doubles (B) halves
(C) increases by about 20 mV (D) decreases by about 20 mV

3.102 An npn transistor has a beta cut-off frequency fb of 1 MHz and


common emitter short circuit low-frequency current gain bo of
200 it unity gain frequency fT and the alpha cut-off frequency fa
respectively are
(A) 200 MHz, 201 MHz (B) 200 MHz, 199 MHz
(C) 199 MHz, 200 MHz (D) 201 MHz, 200 MHz

3.103 A silicon n MOSFET has a threshold voltage of 1 V and oxide


thickness of Ao.
[er (SiO 2) = 3.9, e0 = 8.854 # 10-14 F/cm, q = 1.6 # 10-19 C]
The region under the gate is ion implanted for threshold voltage
tailoring. The dose and type of the implant (assumed to be a sheet
charge at the interface) required to shift the threshold voltage to
- 1 V are
(A) 1.08 # 1012 /cm2 , p-type (B) 1.08 # 1012 /cm2 , n-type
(C) 5.4 # 1011 /cm2 , p-type (D) 5.4 # 1011 /cm2 , n-type
GATE Electronics and Communication Topicwise Solved Paper by RK Kanodia & Ashish Murolia Page 56

SOLUTIONS = 10-12 cm3


= 10-8 cm2 # 10-4 cm

So total no. of holes is,


p = p 0 # V = 10 # 10-12 = 10-11
Which is approximately equal to zero.
3.1 Option (A) is correct. 3.6 Option (A) is correct.
The potential barrier of the pn junction is lowered when a forward Given the circuit as below :
bias voltage is applied, allowing electrons and holes to flow across
the space charge region (Injection) when holes flow from the p
region across the space charge region into the n region, they become
excess minority carrier holes and are subject to diffuse, drift and
recombination processes.
3.2 Option (D) is correct.
In IC technology, dry oxidation as compared to wet oxidation
produces superior quality oxide with a lower growth rate
3.3 Option (D) is correct. Since all the parameters of PMOS and NMOS are equal.
In a MOSFET operating in the saturation region, the channel length
So, mn = mp
modulation effect causes a decrease in output resistance.
COX bW l = COX bW l = COX bW l
GATE Electronics & Communication L M1 L M2 L
Given that M1 is in linear region. So, we assume that M2 is either in
by RK Kanodia
cutoff or saturation.
Now in 3 Volume Case 1 : M2 is in cut off
Purchase Online at maximum discount from online store So, I 2 = I1 = 0
and get POSTAL and Online Test Series Free Where I1 is drain current in M1 and I2 is drain current in M2 .
visit www.nodia.co.in m C
Since, I1 = p OX bW l82VSD ^VSG - VTp h - V SD
2
B
2 L
3.4 Option (A) is correct. m C
Given, & 0 = p OX bW l [2VSD ^VSG - VTp h - V SD
2
]
2 L
VB = 2V Solving it we get,
VTN = 1V 2 ^VSG - VTp h = VSD
So, we have & 2 ^5 - Vin - 1h = 5 - VD
Drain voltage VD = 2 volt
VG = 2 volt & Vin = VD + 3
2
VS = 0 (Ground) For I1 = 0 , VD = 5 V
Therefore, VGS = 2 > VTN
and VDS = 2 > VGS - VTN So, Vin = 5 + 3 = 4 V
2
So, the MOSFET is in the saturation region. Therefore, drain cur- So for the NMOS
rent is VGS = Vin - 0 = 4 - 0 = 4 V and VGS > VTn
ID = kN ^VGS - VTN h2 So it can’t be in cutoff region.
ID = kN ^VB - 1h Case 2 : M2 must be in saturation region.
2
or,
Differentiating both side with respect to ID So, I1 = I 2
1 = kN 2 ^VB - 1hdVB For more GATE Resources, Mock Test and
dID
Study material join the community
Since, VBQ = 2 volt (at D.C. Voltage)
Hence, we obtain http://www.facebook.com/gateec2014
dVB = 1
dID 2kN ^VB - 1h mp COX W mn COX W
2 L6 @ = 2 L (VGS - VTn)
2 2
1 2 (VSG - VTp) VSD - V SD
=
2 # 40 # 10-6 # ^2 - 1h 2
& 2 (VSG - VTp) VSD - V SD = (VGS - VTn) 2
= 12.5 # 103 W
= 12.5 kW & 2 (5 - Vin - 1) (5 - VD) - (5 - VD) 2 = (Vin - 0 - 1) 2
& 2 (4 - Vin) (5 - VD) - (5 - VD) 2 = (Vin - 1) 2
3.5 Option (D) is correct.
Substituting VD = VDS = VGS - VTn and for N -MOS & VD = Vin - 1
For the semiconductor, n 0 p 0 = n i2
2 20 & 2 (4 - Vin) (6 - Vin) - (6 - Vin) 2 = (Vin - 1) 2
p 0 = n i = 1019 = 10 per cm3 48 - 36 - 8Vin =- 2Vin + 1
n 0 10 &
Volume of given device, V = Area # depth & 6Vin = 11
= 1 mm 2 # 1 mm & Vin = 11 = 1.833 V
6
GATE Electronics and Communication Topicwise Solved Paper by RK Kanodia & Ashish Murolia Page 57

So for M2 to be in saturation Vin < 1.833 V or Vin < 1.875 V rL rL a = 600 5


rl = = # 3.26 = 917 W
W # b Wa # b
3.7 Option (B) is correct.
Gate source overlap capacitance. 3.13 Option (C) is correct.
Co = dWeox e0 (medium Sio 2 ) At VGS = 0 V , b = 4 mm since 2b = 8 mm
tox rL a = 600 5 = 750 W
Thus rl =
-9 -6 -12 Wa # b #4
= 20 # 10 # 1 # 10 #-93.9 # 8.9 # 10
1 # 10 Option (A) is correct.
= 0.69 # 10-15 F 3.14

At room temperature mobility of electrons for Si sample is given


3.8 Option (B) is correct. mn = 1350 cm2 /Vs . For an n -channel MOSFET to create an inversion
Source body junction capacitance. layer of electrons, a large positive gate voltage is to be applied.
Therefore, induced electric field increases and mobility decreases.
Cs = Aer e0
d So, Mobility mn < 1350 cm2 /Vs for n -channel MOSFET
A 3.15 Option (B) is correct.
= (0.2 mm + 0.2 mm + 0.2 mm) # 1 mm + 2 (0.2 mm # 0.2 mm) Dry oxidation is used to achieve high quality oxide growth.
= 0.68 mm2 3.16 Option (B) is correct.
d = 10 nm (depletion width of all junction) Emitter injection efficiency is given as
-12
Cs = 0.68 # 10 # 11.7 -# 8.9 # 10-12 g = 1
1 + NB
9
10 # 10
= 7 # 10-15 F SPECIAL EDITION ( STUDY
NE MATERIAL FORM )
At market Book is available in 3 volume i.e. in 3 book binding
3.9 Option (C) is correct.
form. But at NODIA Online Store book is available in 10 book
Drift current Id = qnmn E
binding form. Each unit of Book is in separate binding.
It depends upon Electric field E and carrier concentration n Available Only at NODIA Online Store
3.10 Option (B) is correct. Click to Buy
Zener diode operates in reverse breakdown region.
www.nodia.co.in

To achieve g = 1, NE >> NB
3.17 Option (C) is correct.
Reverse bias breakdown or Zener effect occurs in highly doped
PN junction through tunneling mechanism. In a highly doped PN
junction, the conduction and valence bands on opposite sides of
the junction are sufficiently close during reverse bias that electron
3.11 Option (D) is correct.
may tunnel directly from the valence band on the p-side into the
For every 1c C increase in temperature, forward bias voltage across
conduction band on n -side.
diode decreases by 2.5 mV. Thus for 10c C increase, there us 25 mV
decreases. Breakdown voltage VB \ 1
NA ND
3.12 Option (B) is correct. So, breakdown voltage decreases as concentration increases
Full channel resistance is Depletion capacitance
r L C =' ees NA ND 1/2
r # = 600 W
W#a
...(1) 2 (Vbi + VR) (NA + ND) 1
If VGS is applied, Channel resistance is Thus C \ NA ND
r L VGS Depletion capacitance increases as concentration increases
where b = a c1 -
Vp m
rl = #
W#b 3.18 Option (C) is correct.
Pinch off voltage, Sample is in thermal equilibrium so, electric field
qN
Vp = D a2 ...(2)
2e 1 = 10 kV/cm
E =
If depletion on each side is d = 1 μm at VGS = 0 . 1 mm
3.19 Option (A) is correct.
qN
Vj = D d2 Electron drift current density
2e
qN qND Jd = ND mn eE = 1016 # 1350 # 1.6 # 10-19 # 10 # 1013
or 1 = D (1 # 10-6) 2 & = 1012
2e 2e = 2.16 # 10 4 A/cm2
Now from equation (2), we have 3.20 Option (C) is correct.
-6 2
12
Vp = 10 # (5 # 10 ) Only dopant atoms can have concentration of 4 # 1019 cm - 3 in n -
type silicon at room temperature.
or Vp =- 25 V
3.21 Option (A) is correct.
At VGS =- 3 V ; 2
- 3 mm = 3.26 mm Unit of mobility mn is = cm
b = 5 b1 -
- 25 l V. sec
GATE Electronics and Communication Topicwise Solved Paper by RK Kanodia & Ashish Murolia Page 58

2 17
Unit of diffusion current Dn is = cm E2 - E1 = 25 # 10-3 e ln 4 # 10 10 = 0.427 eV
sec 1.5 # 10
m 2 2
Thus unit of n is = cm / cm = 1 = V-1 Hence fermi level goes down by 0.427 eV as silicon is doped with
Dn V $ sec sec V
boron.
3.22 Option (D) is correct. 3.31 Option (C) is correct.
Both S1 and S2 are true and S2 is a reason for S1. 2
Pinch off voltage VP = eW ND
3.23 Option (B) is correct. es
We know that Let VP = VP1
NA WP = ND WN VP1 = W12 = W2
-6 Now
17
VP2 W22 (2W) 2
or NA = ND WN = 1 # 10 # 0.1-6# 10 = 1 # 1016
WP 1 # 10 or 4VP1 = VP2
The built-in potential is Initial transconductance
Vbi = VT 1n c NA N
n i2 m gm = Kn ;1 - Vbi - VGS E
D

Vp
17 16
= 26 # 10-3 ln e 1 # 10 # 1 10
# 10 = 0.760
o For first condition gm1
(1.4 # 10 ) 2
0 - (- 2) 2
= Kn =1 -
VP1 G
= Kn ;1 -
3.24 Option (B) is correct. VP1 E
The peak electric field in device is directed from p to n and is
For second condition
0 - (- 2) 2
gm2 = Kn =1 -
VP2 G
= K2 ;1 -
GATE Electronics & Communication 4VP1 E
by RK Kanodia gm1 1 - 2/VP1
Dividing =f p
Now in 3 Volume gm2 1 - 1/ (2VP1)
Purchase Online at maximum discount from online store Hence VP = VP1
and get POSTAL and Online Test Series Free 3.32 Option (A) is correct.
visit www.nodia.co.in 3.33 Option (D) is correct.
As per mass action law
np = ni2
E =- eND xn from p to n
es If acceptor impurities are introduces
= eND xn from n to p p = NA
es
-19 17 -5
Thus nNA = ni2
= 1.6 # 10 # 1 #-10 # 1 # 10 = 0.15 2
14
8.85 # 10 # 12 or n = ni
MV/cm NA
3.34 Option (C) is correct.
3.25 Option (D) is correct. The electric field has the maximum value at the junction of p+ n .
Channel length modulation is not associated with a p - n junction.
3.35 Option (B) is correct.
It is being associated with MOSFET in which effective channel
Zener diode and Avalanche diode works in the reverse bias and laser
length decreases, producing the phenomenon called channel length
diode works in forward bias.
modulation.
In solar cell diode works in forward bias but photo current is in
3.26 Option (A) is correct. reverse direction. Thus
Trivalent impurities are used for making p - type semiconductors.
Zener diode : Reverse Bias
So, Silicon wafer heavily doped with boron is a p+ substrate.
For more GATE Resources, Mock Test and
Option (D) is correct.
Study material join the community
3.27

Oxidation rate is zero because the existing oxide prevent the further
oxidation. http://www.facebook.com/gateec2014
3.28 Option (B) is correct.
Solar Cell : Forward Bias
gm = 2ID = 2 K (VGS - VT ) 2 = 2K (VGS - VT )
2VGS 2VGS Laser Diode : Forward Bias
3.29 Option (C) is correct. Avalanche Photo diode : Reverse Bias
As VD = constant 3.36 Option (C) is correct.
Thus gm \ (VGS - VT ) Which is straight line. In BJT as the B-C reverse bias voltage increases, the B-C space
charge region width increases which xB (i.e. neutral base width) > A
3.30 Option (C) is correct.
change in neutral base width will change the collector current. A
E2 - E1 = kT ln NA reduction in base width will causes the gradient in minority carrier
ni
concentration to increase, which in turn causes an increased in the
NA = 4 # 1017
diffusion current. This effect si known as base modulation as early
ni = 1.5 # 1010 effect.
GATE Electronics and Communication Topicwise Solved Paper by RK Kanodia & Ashish Murolia Page 59

In JFET the gate to source voltage that must be applied to achieve C2 = 7 pF


pinch off voltage is described as pinch off voltage and is also called 6
- 12 -4
as turn voltage or threshold voltage. Now D2 = e0 er2 A = 1 # 710 #- 1210 = 6 # 10 - 4 cm
In LASER population inversion occurs on the condition when C2 6 # 10
7
= 0.857 mm
concentration of electrons in one energy state is greater than that in
lower energy state, i.e. a non equilibrium condition. 3.41 Option (C) is correct.
In MOS capacitor, flat band voltage is the gate voltage that must be Depletion region will not be formed if the MOS capacitor has n type
applied to create flat ban condition in which there is no space charge substrate but from C-V characteristics, C reduces if V is increased.
region in semiconductor under oxide. Thus depletion region must be formed. Hence S1 is false
Therefore If positive charges is introduced in the oxide layer, then to equalize
BJT : Early effect the effect the applied voltage V must be reduced. Thus the C - V
MOS capacitor : Flat-band voltage plot moves to the left. Hence S2 is true.
LASER diode : Population inversion 3.42 Option (C) is correct.
JFET : Pinch-off voltage For the case of negative slope it is the negative resistance region
3.37 Option (A) is correct.
W = K V + VR
Now 2m = K 0.8 + 1.2
From above two equation we get SPECIAL EDITION ( STUDY MATERIAL FORM )
W = 0.8 + 7.2 = 8 =2 At market Book is available in 3 volume i.e. in 3 book binding
2m 0.8 + 1.2 2
form. But at NODIA Online Store book is available in 10 book
or W2 = 4 m m
binding form. Each unit of Book is in separate binding.
3.38 Option (B) is correct. Available Only at NODIA Online Store
b Click to Buy
a= = 50 = 50
b + 1 50 + 1 51
Current Gain = Base Transport Factor # Emitter injection Effi-
www.nodia.co.in
ciency
a = b1 # b2
or b1 = a = 50 = 0.985
b2 51 # 0.995
3.39 Option (A) is correct.
At low voltage when there is no depletion region and capacitance is
decide by SiO2 thickness only,
C = e0 er1 A
D
-13
or D = e0 er1 A = 3.5 # 10 -# 10-4 = 50 nm
C 7 # 10 12 3.43 Option (A) is correct.
For n -type p is minority carrier concentration
3.40 Option (B) is correct.
The construction of given capacitor is shown in fig below np = ni2
np = Constant Since ni is constant
p \ 1
n
Thus p is inversely proportional to n .
3.44 Option (A) is correct.
Diffusion current, since the drift current is negligible for minority
carrier.
3.45 Option (B) is correct.
When applied voltage is 0 volts, there will be no depletion region In BJT as the B-C reverse bias voltage increases, the B-C space
and we get charge region width increases which xB (i.e. neutral base width) > A
C1 = 7 pF change in neutral base width will change the collector current. A
When applied voltage is V , a depletion region will be formed as reduction in base width will causes the gradient in minority carrier
shown in fig an total capacitance is 1 pF. Thus concentration to increases, which in turn causes an increases in the
diffusion current. This effect si known as base modulation as early
CT = 1 pF
effect.
or CT = C1 C2 = 1 pF
C1 + C2 3.46 Option (A) is correct.
or 1 = 1 + 1 For t < 0 diode forward biased and VR = 5 . At t = 0 diode abruptly
CT C1 C2 changes to reverse biased and current across resistor must be 0.
Substituting values of CT and C1 we get But in storage time 0 < t < ts diode retain its resistance of forward
GATE Electronics and Communication Topicwise Solved Paper by RK Kanodia & Ashish Murolia Page 60

biased. Thus for 0 < t < ts it will be ON and sp = pqmp (n = p)


VR =- 5 V sp m
= p =1
sn mn 3
3.47 Option (B) is correct.
According to Hall effect the direction of electric field is same as that 3.54 Option (B) is correct.
of direction of force exerted. C = e0 er A
d
E =- v # B
or C = e0 er = 8.85 # 10-12 # 11.7 = 10.35 m F
or E = B#v d
A 10 # 10-6
3.48 Option (B) is correct. 3.55 Option (B) is correct.
The varacter diode is used in tuned circuit as it can provide frequently In accumulation mode for NMOS having p -substrate, when positive
stability. voltage is applied at the gate, this will induce negative charge near
PIN diode is used as a current controlled attenuator. p - type surface beneath the gate. When VGS is made sufficiently
Zener diode is used in regulated voltage supply or fixed voltage large, an inversion of electrons is formed and this in effect forms and
reference. n - channel.
Schottkey diode has metal-semiconductor function so it has fast
switching action so it is used as high frequency switch
3.56 Option (C) is correct.
From the graph it can be easily seen that Vth = 1 V
Varactor diode : Tuned circuits
Now VGS = 3 - 1 = 2 V
PIN Diode : Current controlled attenuator
and VDS = 5 - 1 = 4 V
Zener diode : Voltage reference
Since VDS > VGS $ VDS > VGS - Vth
GATE Electronics & Communication Thus MOSFET is in saturation region.
by RK Kanodia 3.57 Option (C) is correct.
Now in 3 Volume Trivalent impurities are used for making p type semiconductor.
Purchase Online at maximum discount from online store Boron is trivalent.
and get POSTAL and Online Test Series Free 3.58 Option (A) is correct.
visit www.nodia.co.in Here emitter base junction is forward biased and base collector
junction is reversed biased. Thus transistor is operating in normal
Schottky diode : High frequency switch active region.
3.49 Option (D) is correct. 3.59 Option (D) is correct.
mP a
We have = 0.4 We have b =
mn 1-a
Conductance of n type semiconductor Thus a -" b -
sn = nqmn a ." b .
Conductance of intrinsic semiconductor If the base width increases, recombination of carrier in base region
si = ni q (mn + mp) increases and a decreases & hence b decreases. If doping in base
sn = nmn n region increases, recombination of carrier in base increases and a
Ratio is =
i^ + mn h
mp
s i ni (mn + mp ) n 1
4 . 2 # 10 8
4
decreases thereby decreasing b . Thus S1 is true and S2 is false.
= = 2 # 10
1.5 # 10 4 (1 + 0.4) 3.60 Option (C) is correct.
3.50 Option (C) is correct. 3.61 Option (A) is correct.
For silicon at 0 K, Applying KVL we get
Eg0 = 1.21 eV VCC - IC RC - VCE = 0
At any temperature For more GATE Resources, Mock Test and
EgT = Eg0 - 3.6 # 10 T -4
Study material join the community
At T = 300 K, http://www.facebook.com/gateec2014
Eg300 = 1.21 - 3.6 # 10 - 4 # 300 = 1.1 eV
This is standard value, that must be remembered. or IC = VCC - VCE = 3 - 0.2 = 2.8 mA
RC 1k
3.51 Option (B) is correct.
Now IB = IC = 2.8m = 56 mA
The reverse saturation current doubles for every 10cC rise in b 50
temperature as follows : 3.62 Option (B) is correct.
(T - T1)/10
I0 (T) = I 01 # 2 We know that
Thus at 40c C, I0 = 40 pA Wp NA = Wn ND
Option (A) is correct. 3 m # 1016
Wp = Wn # ND =
3.52
or = 0.3 m m
Silicon is abundant on the surface of earth in the from of SiO2 . NA 9 # 1016
3.53 Option (B) is correct. 3.63 Option (B) is correct.
sn = nqmn Conductivity s = nqun
GATE Electronics and Communication Topicwise Solved Paper by RK Kanodia & Ashish Murolia Page 61

or resistivity r = 1 = 1 At T = 300 K,
s nqmn
Eg300 = 1.21 - 3.6 # 10 - 4 # 300 = 1.1 eV
Thus n = 1 This is standard value, that must be remembered.
qrmn
= 1 16
= 10 /cm 3
- 19 3.71 Option (A) is correct.
1.6 # 10 # 0.5 # 1250
By Mass action law
For n type semiconductor n = ND
np = ni2
3.64 Option (D) is correct. 2 16
We know that p = ni = 1.5 # 10 # 120.5 # 1016 = 4.5 # 1011
n 5 # 10
Cj = ; eeS NA ND
2 (Vbi + VR)( NA + ND) E
1
2

3.72 Option (C) is correct.


1 Tunnel diode shows the negative characteristics in forward bias. It
Thus Cj \ is used in forward bias.
(Vbi + VR)
C j2 (Vbi + VR) 1 1 =1 Avalanche photo diode is used in reverse bias.
Now = =
C j1 (Vbi + VR) 2 4 2 3.73 Option (D) is correct.
C
or Cj2 = j1 = 1 = 0.5 pF 3.74 Option (A) is correct.
2 2
rl
We that R = , r = 1 and a = nqun
3.65 Option (C) is correct. A s
Increase in gate oxide thickness makes difficult to induce charges
in channel. Thus VT increases if we increases gate oxide thickness. SPECIAL EDITION ( STUDY MATERIAL FORM )
Hence S1 is false. At market Book is available in 3 volume i.e. in 3 book binding
Increase in substrate doping concentration require more gate voltage
form. But at NODIA Online Store book is available in 10 book
because initially induce charges will get combine in substrate. Thus
VT increases if we increase substrate doping concentration. Hence S2
binding form. Each unit of Book is in separate binding.
Available Only at NODIA Online Store
is false.
Click to Buy
3.66 Option (D) is correct.
We know that www.nodia.co.in
ID = K (VGS - VT ) 2 From above relation we have
I (VGS2 - VT ) 2 R = 1 = 0.1 # 10 - 2
Thus DS
= nqmn A
IDI (VGS1 - VT ) 2 5 # 10 # 1.6 # 10 - 19 # 0.13 # 100 # 10 - 12
20
= 106 W
Substituting the values we have
2
ID2 = (3 - 1) = 4 3.75 Option (D) is correct.
ID1 (2 - 1) 2 dn = 6 # 1016 - 1017 =- 2 # 1020
or ID2 = 4IDI = 4 mA dx 2 # 10 - 4 - 0
3.67 Option (A) is correct. Now Jn = nqme E + Dn q dn
dx
Eg \ 1 Since no electric field is present, E = 0 and we get
l
Thus
Eg2
= l1 = 1.1
So, Jn = qDn dn
Eg1 l2 0.87 dx
= 1.6 # 10 - 19 # 35 # (- 2 # 1020) =- 1120 A/cm 2
or Eg2 = 1.1 # 1.12 = 1.416 eV
0.87 3.76 Option (C) is correct.
3.68 Option (B) is correct. LED works on the principal of spontaneous emission.
Concentration gradient In the avalanche photo diode due to the avalanche effect there is
dn = 1014 large current gain.
= 2 # 1018
dx 0.5 # 10 -4 Tunnel diode has very large doping.
q = 1.6 # 10 - 19 C LASER diode are used for coherent radiation.
Dn = 25 3.77 Option (C) is correct.
I = Io `e h V - 1j
VD1
dn = 1014 We know that si T

dx 0.5 # 10 - 4 where h = 1 for germanium and h = 2 silicon. As per question


JC = qDn dn = 1.6 # 10 - 19 # 25 # 2 # 1018 = 8 A/cm 2
Io `e e - 1j = Io `e hV - 1j
VDsi VDGe
dx n
hVT
Ge T

3.69 Option (D) is correct. VDsi 0.718


Io
= eVhV - 1 = e 2 #026.1435
# 10 - 1
= 4 # 103
-3

Pentavalent make n -type semiconductor and phosphorous is or si T

Io DGe
e 26 # 10 - 1
pentavalent.
si
e hV - 1T
-3

3.70 Option (C) is correct. 3.78 Option (A) is correct.


-34
For silicon at 0 K Eg0 = 1.21 eV 8
Eg = hc = 6.626 # 10 # -310# 10 = 3.62 J
At any temperature l 54900 # 10
E (J) -19
EgT = Eg0 - 3.6 # 10 - 4 T In eV Eg (eV) = g = 3.62 # 10-19 = 2.26 eV
e 1.6 # 10
GATE Electronics and Communication Topicwise Solved Paper by RK Kanodia & Ashish Murolia Page 62

Alternatively At a given value of vBE , increasing the reverse-bias voltage on the


1.24 collector-base junction and thus increases the width of the depletion
Eg = 1.24 eV = = 2.26 eV
l (mm) 5490 # 10-4 mm region of this junction. This in turn results in a decrease in the
3.79 Option (D) is correct. effective base width W . Since IS is inversely proportional to W , IS
We know that increases and that iC increases proportionally. This is early effect.
ID = K (VGS - VT ) 2 3.87 Option (B) is correct.
ID2 = (VGS2 - VT )
2 For an n -channel JEFT trans-conductance is
Thus -3 (- 2)
ID1 (VGS1 - VT ) 2 gm = - 2IDSS b1 - VGS l = - 2 # 2 # 10 =1 -
Substituting the values we have VP VP -4 (- 4)G
2
ID2 = (1.4 - 0.4) = 4 = 10-3 # 1 = 0.5 mho
2
ID1 (0.9 - 0.4) 2
3.88 Option (A) is correct.
or ID2 = 4IDI = 4 mA
Option (B) is correct. We have gm = IC = 1
3.80
VT 26
In n -well CMOS fabrication following are the steps : gm
(A) n - well implant Now fT =
2p (C p + C m)
(B) Source drain diffusion 1/26
(C) Metalization or 400 =
2p (0.3 # 10-12 + C m)
(D) Passivation 1
or (0.3 # 10-12 + C m) = = 15.3 # 10-12
2p # 26 # 400
GATE Electronics & Communication or C m 15.3 # 10-12 - 0.3 # 10-12 = 15 # 10-12 15 pF
by RK Kanodia 3.89 Option (D) is correct.
Now in 3 Volume For any semiconductor (Intrinsic or extrinsic) the product n p
Purchase Online at maximum discount from online store remains constant at a given temperature so here
and get POSTAL and Online Test Series Free np = ni pi
visit www.nodia.co.in 3.90 Option (D) is correct.
Option (D) is correct. gm
3.81 fT =
2p (C p + C m)
For a JFET in active region we have
Option (B) is correct.
IDS = IDSS c1 - VGS m
2 3.91

VP For a Forward Bias p-n junction, current equation


From above equation it is clear that the action of a JFET is volt- I = I 0 (eV/kT - 1)
age controlled current source. or I + 1 = eV/kT
I0
3.82 Option (B) is correct.
At constant current the rate of change of voltage with respect to or kT log b I + 1l = V
I0
temperature is
So if we plot log I vs V we get a straight line.
dV =- 2.5 mV per degree centigrade
dT 3.92 Option (B) is correct.
Here 3 T = T2 - T1 = 40 - 20 = 20cC A specimen of p - type or n - type is always electrical neutral.
Thus 3 VD =- 2.5 # 20 = 50 mV 3.93 Option (C) is correct.
Therefore, VD = 700 - 50 = 650 mV 3.94 Option (B) is correct.
3.83 Option (D) is correct. The unit of q is e and unit of kT is eV. Thus unit of e/kT is
Condition for saturation is IC < bIB For more GATE Resources, Mock Test and
3.84 Option (B) is correct. Study material join the community
The metal area of the gate in conjunction with the insulating
http://www.facebook.com/gateec2014
dielectric oxide layer and semiconductor channel, form a parallel
plate capacitor. It is voltage controlled capacitor because in active
e/eV = V-1 .
region the current voltage relationship is given by
3.95 Option (D) is correct.
IDS = K (VGS - VT ) 2
3.96 Option (C) is correct.
3.85 Option (D) is correct.
In MOSFET the body (substrate) is connected to power supply in We have ni = 1.5 # 1010 /cm3
such a way to maintain the body (substrate) to channel junction in Nd = 2.25 # 1015 atoms/cm3
cutoff condition. The resulting reverse bias voltage between source For n type doping we have electron concentration
and body will have an effect on device function. The reverse bias n - Nd = 2.25 # 1015 atom/cm3
will widen the depletion region resulting the reduction in channel For a given temperature
length. np = n i2
Option (C) is correct. 2
(1.5 # 1010) 2
p = ni =
3.86
Hole concentration = 1.0 # 105 /cm3
n 2.25 # 1015
GATE Electronics and Communication Topicwise Solved Paper by RK Kanodia & Ashish Murolia Page 63

3.97 Option (D) is correct.


In p n -junction isolated circuit we should have high impedance, so
that p n junction should be kept in reverse bias. (So connect p to
negative potential in the circuit)
3.98 Option (B) is correct.

If both junction are forward biased and collector base junction is


more forward biased then IC will be flowing out wards (opposite
direction to normal mode) the collector and it will be in reverse
saturation mode.
3.99 Option (C) is correct.
For normal active mode we have
b = IC
IB
SPECIAL EDITION ( STUDY MATERIAL FORM )
For small values of IC , if we increases IC , b also increases until we
At market Book is available in 3 volume i.e. in 3 book binding
reach (IC ) saturation. Further increases in IC (since transistor is in
form. But at NODIA Online Store book is available in 10 book
saturation mode know) will increases IB and b decreases.
binding form. Each unit of Book is in separate binding.
3.100 Option (C) is correct. Available Only at NODIA Online Store
For a n -channel mosfet thresholds voltage is given by Click to Buy
VTN = VGS - VDS (sat) www.nodia.co.in
for p-channel [p+ polysilicon used in gate]
VTP = VSD (sat) - VGS
so VTP =- VDS (sat) + VGS
so threshold voltage will be same.
3.101 Option (C) is correct.
Emitter current is given by
IE = I 0 (eV /kT - 1)
BE

or IE = I 0 eV /kT
BE
eV
BE /kT
>> 1
or VBE = kT ln b IE l
I0
Now (VBE ) 1 = kT ln b IE 1 l
I0
(VBE ) 2 = kT ln b IE 2 l
I0
or (VBE ) 2 - (VBE ) 1 = kT ;ln b IE 2 lE = kT ln b 2IE 1 l
IE 1 IE 1
Now if emitter current is double i.e. IE 2 = 2IE1
(VBE ) 2 = (VBE ) 1 + (25 # 0.60) m volt
= (VBE ) 1 + 15 m volt
Thus if emitter current is doubled the base emitter junction volt-
age is increased by 15 mV.
3.102 Option (A) is correct.
Unity gain frequency is given by
fT = fB # b = 106 # 200 = 200 MHz
a-cutoff frequency is given by
f fb
fa = b = = fb (b + 1)
1-a b
1-
b+1
= 106 # (200 + 1) = 201 MHz
3.103 Option (A) is correct.
GATE Electronics and Communication Topicwise Solved Paper by RK Kanodia & Ashish Murolia Page 64

UNIT 4
ANALOG CIRCUITS

2013 ONE MARK

4.1 In the circuit shown below what is the output voltage ^Vouth if a (A) 125 and 125 (B) 125 and 250
silicon transistor Q and an ideal op-amp are used? (C) 250 and 125 (D) 250 and 250

4.4 The ac schematic of an NMOS common-source state is shown in the


figure below, where part of the biasing circuits has been omitted for
simplicity. For the n -channel MOSFET M, the transconductance
gm = 1 mA/V , and body effect and channel length modulation effect
are to be neglected. The lower cutoff frequency in HZ of the circuit
is approximately at

GATE Electronics & Communication


by RK Kanodia
Now in 3 Volume
Purchase Online at maximum discount from online store
and get POSTAL and Online Test Series Free
visit www.nodia.co.in
(A) 8 (B) 32
(A) - 15 V (B) - 0.7 V (C) 50 (D) 200
(C) + 0.7 V (D) + 15 V 4.5 In the circuit shown below the op-amps are ideal. Then, Vout in Volts
4.2 In a voltage-voltage feedback as shown below, which one of the is
following statements is TRUE if the gain k is increased?

(A) The input impedance increases and output impedance decreas-


es For more GATE Resources, Mock Test and
(B) The input impedance increases and output impedance also Study material join the community
increases
http://www.facebook.com/gateec2014
(C) The input impedance decreases and output impedance also
decreases (A) 4 (B) 6
(D) The input impedance decreases and output impedance increas- (C) 8 (D) 10
es
4.6 In the circuit shown below, Q1 has negligible collector-to-emitter
saturation voltage and the diode drops negligible voltage across it
2013 TWO MARKS
under forward bias. If Vcc is + 5 V , X and Y are digital signals with
4.3 In the circuit shown below, the knee current of the ideal Zener 0 V as logic 0 and Vcc as logic 1, then the Boolean expression for Z is
dioide is 10 mA. To maintain 5 V across RL , the minimum value of
RL in W and the minimum power rating of the Zener diode in mW
, respectively, are
GATE Electronics and Communication Topicwise Solved Paper by RK Kanodia & Ashish Murolia Page 65

The current in the circuit is


(A) 10 mA (B) 9.3 mA
(C) 6.67 mA (D) 6.2 mA

(A) XY (B) XY 4.11 The diodes and capacitors in the circuit shown are ideal. The voltage
(C) XY (D) XY v (t) across the diode D1 is

4.7 A voltage 1000 sin wt Volts is applied across YZ . Assuming ideal


diodes, the voltage measured across WX in Volts, is

(A) cos (wt) - 1 (B) sin (wt)


SPECIAL EDITION ( STUDY MATERIAL FORM )
At market Book is available in 3 volume i.e. in 3 book binding
form. But at NODIA Online Store book is available in 10 book
binding form. Each unit of Book is in separate binding.
(A) sin wt (B) _sin wt + sin wt i /2 Available Only at NODIA Online Store

(C) ^sin wt - sin wt h /2 (D) 0 for all t Click to Buy


www.nodia.co.in
4.8 In the circuit shown below, the silicon npn transistor Q has a
very high value of b . The required value of R2 in kW to produce
(C) 1 - cos (wt) (D) 1 - sin (wt)
IC = 1 mA is
4.12 The impedance looking into nodes 1 and 2 in the given circuit is

(A) 50 W (B) 100 W


(A) 20 (B) 30
(C) 5 kW (D) 10.1 kW
(C) 40 (D) 50

2012 TWO MARKS


2012 ONE MARK
4.13 The circuit shown is a
4.9 The current ib through the base of a silicon npn transistor is
1 + 0.1 cos (10000pt) mA At 300 K, the rp in the small signal model
of the transistor is

(A) low pass filter with f3dB = 1 rad/s


(R1 + R2) C

(A) 250 W (B) 27.5 W (B) high pass filter with f3dB = 1 rad/s
R1 C
(C) 25 W (D) 22.5 W
(C) low pass filter with f3dB = 1 rad/s
R1 C
4.10 The i -v characteristics of the diode in the circuit given below are 1
v - 0.7 A, v $ 0.7 V (D) high pass filter with f3dB = rad/s
(R1 + R2) C
i = * 500
4.14 The voltage gain Av of the circuit shown below is
0A v < 0. 7 V
GATE Electronics and Communication Topicwise Solved Paper by RK Kanodia & Ashish Murolia Page 66

and the threshold voltage VT = 1 V . The voltage Vx at the source of


the upper transistor is

(A) Av . 200 (B) Av . 100


(A) 1 V (B) 2 V
(C) Av . 20 (D) Av . 10
(C) 3 V (D) 3.67 V

4.18 For the BJT, Q1 in the circuit shown below,


2011 ONE MARK
b = 3, VBEon = 0.7 V, VCEsat = 0.7 V . The switch is initially closed.
4.15 In the circuit shown below, capacitors C1 and C2 are very large and At time t = 0 , the switch is opened. The time t at which Q1 leaves
are shorts at the input frequency. vi is a small signal input. The gain the active region is
magnitude vo at 10 M rad/s is
vi

GATE Electronics & Communication


by RK Kanodia
Now in 3 Volume
Purchase Online at maximum discount from online store
and get POSTAL and Online Test Series Free
visit www.nodia.co.in
(A) 10 ms (B) 25 ms
(C) 50 ms (D) 100 ms

4.19 For a BJT, the common base current gain a = 0.98 and the collector
base junction reverse bias saturation current ICO = 0.6 mA . This
BJT is connected in the common emitter mode and operated in the
active region with a base drive current IB = 20 mA . The collector
current IC for this mode of operation is
(A) 0.98 mA (B) 0.99 mA
(C) 1.0 mA (D) 1.01 mA

Statement for Linked Answer Questions: 4.6 & 4.7


(A) maximum (B) minimum In the circuit shown below, assume that the voltage drop
(C) unity (D) zero across a forward biased diode is 0.7 V. The thermal voltage
Vt = kT/q = 25 mV . The small signal input vi = Vp cos ^wt h where
4.16 The circuit below implements a filter between the input current ii Vp = 100 mV.
and the output voltage vo . Assume that the op-amp is ideal. The For more GATE Resources, Mock Test and
filter implemented is a
Study material join the community
http://www.facebook.com/gateec2014

(A) low pass filter (B) band pass filter


(C) band stop filter (D) high pass filter

2011 TWO MARKS

4.17 In the circuit shown below, for the MOS transistors, mn Cox = 100 mA/V 2
GATE Electronics and Communication Topicwise Solved Paper by RK Kanodia & Ashish Murolia Page 67

4.20 The bias current IDC through the diodes is 2010 TWO MARKS
(A) 1 mA (B) 1.28 mA
(C) 1.5 mA (D) 2 mA Common Data For Q. 4.11 & 4.12 :
4.21 The ac output voltage vac is Consider the common emitter amplifier shown below with the fol-
(A) 0.25 cos ^wt h mV (B) 1 cos (wt) mV lowing circuit parameters:
(C) 2 cos (wt) mV (D) 22 cos (wt) mV b = 100, gm = 0.3861 A/V, r0 = 259 W, RS = 1 kW, RB = 93 kW,
RC = 250 kW, RL = 1 kW, C1 = 3 and C2 = 4.7 mF

2010 ONE MARK

4.22 The amplifier circuit shown below uses a silicon transistor. The
capacitors CC and CE can be assumed to be short at signal frequency
and effect of output resistance r0 can be ignored. If CE is disconnected
from the circuit, which one of the following statements is true

SPECIAL EDITION ( STUDY MATERIAL FORM )


At market Book is available in 3 volume i.e. in 3 book binding
form. But at NODIA Online Store book is available in 10 book
binding form. Each unit of Book is in separate binding.
Available Only at NODIA Online Store

Click to Buy
(A) The input resistance Ri increases and magnitude of voltage
gainAV decreases www.nodia.co.in
(B) The input resistance Ri decreases and magnitude of voltage 4.25 The resistance seen by the source vS is
gain AV increases (A) 258 W (B) 1258 W
(C) Both input resistance Ri and magnitude of voltage gain AV (C) 93 kW (D) 3
decreases
4.26 The lower cut-off frequency due to C2 is
(D) Both input resistance Ri and the magnitude of voltage gain (A) 33.9 Hz (B) 27.1 Hz
AV increases
(C) 13.6 Hz (D) 16.9 Hz
4.23 In the silicon BJT circuit shown below, assume that the emitter
4.27 The transfer characteristic for the precision rectifier circuit shown
area of transistor Q1 is half that of transistor Q2
below is (assume ideal OP-AMP and practical diodes)

The value of current Io is approximately


(A) 0.5 mA (B) 2 mA
(C) 9.3 mA (D) 15 mA

4.24 Assuming the OP-AMP to be ideal, the voltage gain of the amplifier
shown below is

(A) - R2 (B) - R 3
R1 R1
R || R 3
(C) - 2 (D) -b R2 + R 3 l
R1 R1
GATE Electronics and Communication Topicwise Solved Paper by RK Kanodia & Ashish Murolia Page 68

2009 TWO MARKS 4.30 For small increase in VG beyond 1V, which of the following gives the
4.28 In the circuit below, the diode is ideal. The voltage V is given by correct description of the region of operation of each MOSFET
(A) Both the MOSFETs are in saturation region
(B) Both the MOSFETs are in triode region
(C) n-MOSFETs is in triode and p -MOSFET is in saturation
region
(D) n- MOSFET is in saturation and p -MOSFET is in triode
region

(A) min (Vi, 1) (B) max (Vi, 1) 4.31 Estimate the output voltage V0 for VG = 1.5 V. [Hints : Use the
(C) min (- Vi, 1) (D) max (- Vi, 1) appropriate current-voltage equation for each MOSFET, based on
the answer to Q.4.16]
4.29 In the following a stable multivibrator circuit, which properties of (A) 4 - 1 (B) 4 + 1
v0 (t) depend on R2 ? 2 2
(C) 4 - 3 (D) 4 + 3
2 2

4.32 In the circuit shown below, the op-amp is ideal, the transistor has
VBE = 0.6 V and b = 150 . Decide whether the feedback in the circuit
is positive or negative and determine the voltage V at the output of
the op-amp.
GATE Electronics & Communication
by RK Kanodia
Now in 3 Volume
Purchase Online at maximum discount from online store
and get POSTAL and Online Test Series Free
visit www.nodia.co.in

(A) Positive feedback, V = 10 V


(B) Positive feedback, V = 0 V
(C) Negative feedback, V = 5 V
(D) Negative feedback, V = 2 V
4.33 A small signal source Vi (t) = A cos 20t + B sin 106 t is applied to a
transistor amplifier as shown below. The transistor has b = 150 and
hie = 3W . Which expression best approximate V0 (t)

(A) Only the frequency


(B) Only the amplitude
(C) Both the amplitude and the frequency
(D) Neither the amplitude nor the frequency
For more GATE Resources, Mock Test and
Study material join the community
Statement for Linked Answer Question 4.16 and 4.17
http://www.facebook.com/gateec2014
Consider for CMOS circuit shown, where the gate voltage v0 of
the n-MOSFET is increased from zero, while the gate voltage of
the p -MOSFET is kept constant at 3 V. Assume, that, for both
transistors, the magnitude of the threshold voltage is 1 V and the
product of the trans-conductance parameter is 1mA. V - 2
GATE Electronics and Communication Topicwise Solved Paper by RK Kanodia & Ashish Murolia Page 69

(A) 0 V (B) 0.1 V


(C) 0.7 V (D) 1.1 V

4.37 The OPAMP circuit shown above represents a

(A) V0 (t) =- 1500 (A cos 20t + B sin 106 t)


(B) V0 (t) = - 1500( A cos 20t + B sin 106 t)
(C) V0 (t) =- 1500B sin 106 t
(D) V0 (t) =- 150B sin 106 t

2008 ONE MARK

4.34 In the following limiter circuit, an input voltage Vi = 10 sin 100pt


is applied. Assume that the diode drop is 0.7 V when it is forward SPECIAL EDITION ( STUDY MATERIAL FORM )
biased. When it is forward biased. The zener breakdown voltage is At market Book is available in 3 volume i.e. in 3 book binding
6.8 V
form. But at NODIA Online Store book is available in 10 book
The maximum and minimum values of the output voltage respec-
tively are binding form. Each unit of Book is in separate binding.
Available Only at NODIA Online Store

Click to Buy
www.nodia.co.in

(A) high pass filter (B) low pass filter


(C) band pass filter (D) band reject filter

(A) 6.1 V, - 0.7 V (B) 0.7 V, - 7.5 V 4.38 Two identical NMOS transistors M1 and M2 are connected as shown
(C) 7.5 V, - 0.7 V (D) 7.5 V, - 7.5 V below. Vbias is chosen so that both transistors are in saturation. The
equivalent gm of the pair is defied to be 2Iout at constant Vout
2Vi
The equivalent gm of the pair is
2008 TWO MARSK

4.35 For the circuit shown in the following figure, transistor M1 and M2
are identical NMOS transistors. Assume the M2 is in saturation and
the output is unloaded.

(A) the sum of individual gm ' s of the transistors


(B) the product of individual gm ’s of the transistors
(C) nearly equal to the gm of M1
g
(D) nearly equal to m of M2
g0
4.39 Consider the Schmidt trigger circuit shown below
The current Ix is related to Ibias as A triangular wave which goes from -12 to 12 V is applied to the
(A) Ix = Ibias + Is (B) Ix = Ibias inverting input of OPMAP. Assume that the output of the OPA-
MP swings from +15 V to -15 V. The voltage at the non-inverting
(C) Ix = Ibias - cVDD - Vout m (D) Ix = Ibias - Is
RE input switches between
4.36 Consider the following circuit using an ideal OPAMP. The I-V
characteristic of the diode is described by the relation I = I 0 _eV - 1i
V
t

where VT = 25 mV, I0 = 1m A and V is the voltage across the diode


(taken as positive for forward bias). For an input voltage Vi =- 1 V
, the output voltage V0 is
GATE Electronics and Communication Topicwise Solved Paper by RK Kanodia & Ashish Murolia Page 70

(A) - 12V to +12 V (B) -7.5 V to 7.5 V


(C) -5 V to +5 V (D) 0 V and 5 V

Statement for Linked Answer Question 3.26 and 3.27:


In the following transistor circuit, VBE = 0.7 V, r3 = 25 mV/IE , and
b and all the capacitances are very large
(A) -2 V (B) -1 V
(C) -0.5 V (D) 0.5 V

4.45 For the BJT circuit shown, assume that the b of the transistor is
very large and VBE = 0.7 V. The mode of operation of the BJT is

4.40 The value of DC current IE is


(A) 1 mA (B) 2 mA
(C) 5 mA (D) 10 mA
(A) cut-off (B) saturation
(C) normal active (D) reverse active
GATE Electronics & Communication
by RK Kanodia 4.46 In the Op-Amp circuit shown, assume that the diode current
Now in 3 Volume follows the equation I = Is exp (V/VT ). For Vi = 2V, V0 = V01, and for
Vi = 4V, V0 = V02 .
Purchase Online at maximum discount from online store The relationship between V01 and V02 is
and get POSTAL and Online Test Series Free
visit www.nodia.co.in

4.41 The mid-band voltage gain of the amplifier is approximately


(A) -180 (B) -120
(C) -90 (D) -60
(A) V02 = 2 Vo1 (B) Vo2 = e2 Vo1
2007 ONE MARK (C) Vo2 = Vo1 1n2 (D) Vo1 - Vo2 = VT 1n2

4.42 The correct full wave rectifier circuit is 4.47 In the CMOS inverter circuit shown, if the trans conductance
parameters of the NMOS and PMOS transistors are
W
kn = kp = mn Cox Wn = mCox p = 40mA/V2
Ln Lp
and their threshold voltages ae VTHn = VTHp = 1 V the current I is

For more GATE Resources, Mock Test and


Study material join the community
http://www.facebook.com/gateec2014
4.43 In a transconductance amplifier, it is desirable to have
(A) a large input resistance and a large output resistance
(B) a large input resistance and a small output resistance
(C) a small input resistance and a large output resistance
(D) a small input resistance and a small output resistance

2007 TWO MARKS

4.44 For the Op-Amp circuit shown in the figure, V0 is (A) 0 A (B) 25 mA
(C) 45 mA (D) 90 mA

4.48 For the Zener diode shown in the figure, the Zener voltage at knee is
7 V, the knee current is negligible and the Zener dynamic resistance
GATE Electronics and Communication Topicwise Solved Paper by RK Kanodia & Ashish Murolia Page 71

is 10 W. If the input voltage (Vi) range is from 10 to 16 V, the output (A) 0 Volt (B) 6.3 Volt
voltage (V0) ranges from (C) 9.45 Volts (D) 10 Volts

4.54 For the circuit shown below, assume that the zener diode is ideal
with a breakdown voltage of 6 volts. The waveform observed across
R is

(A) 7.00 to 7.29 V (B) 7.14 to 7.29 V


(C) 7.14 to 7.43 V (D) 7.29 to 7.43 V

Statement for Linked Answer Questions 4.35 & 4.36:


Consider the Op-Amp circuit shown in the figure.

SPECIAL EDITION ( STUDY MATERIAL FORM )


4.49 The transfer function V0 (s)/ Vi (s) is At market Book is available in 3 volume i.e. in 3 book binding
(A) 1 - sRC (B) 1 + sRC
form. But at NODIA Online Store book is available in 10 book
1 + sRC 1 - sRC binding form. Each unit of Book is in separate binding.
(C) 1 (D) 1 Available Only at NODIA Online Store
1 - sRC 1 + sRC
Click to Buy
4.50 If Vi = V1 sin (wt) and V0 = V2 sin (wt + f), then the minimum and www.nodia.co.in
maximum values of f (in radians) are respectively
(A) - p and p (B) 0 and p
2 2 2
(C) - p and 0 (D) - p and 0
2

2006 ONE MARK

4.51 The input impedance (Zi) and the output impedance (Z0) of an ideal
trans-conductance (voltage controlled current source) amplifier are
(A) Zi = 0, Z0 = 0 (B) Zi = 0, Z0 = 3
(C) Zi = 3, Z0 = 0 (D) Zi = 3, Z0 = 3

4.52 An n-channel depletion MOSFET has following two points on its


ID - VGs curve:
(i) VGS = 0 at ID = 12 mA and
(ii) VGS =- 6 Volts at ID = 0 mA
Which of the following Q point will given the highest trans con-
ductance gain for small signals?
(A) VGS =- 6 Volts (B) VGS =- 3 Volts
(C) VGS = 0 Volts (D) VGS = 3 Volts

2006 TWO MARKS

4.53 For the circuit shown in the following figure, the capacitor C is Common Data For Q. 4.41, 4.42 and 4.43 :
initially uncharged. At t = 0 the switch S is closed. The Vc across In the transistor amplifier circuit shown in the figure below, the
the capacitor at t = 1 millisecond is transistor has the following parameters:
In the figure shown above, the OP-AMP is supplied with ! 15V .
bDC = 60 , VBE = 0.7V, hie " 3
The capacitance CC can be assumed to be infinite.
In the figure above, the ground has been shown by the symbol 4
GATE Electronics and Communication Topicwise Solved Paper by RK Kanodia & Ashish Murolia Page 72

(A) 30 kW (B) 10 kW
4
4.55 Under the DC conditions, the collector-or-emitter voltage drop is
(C) 40 kW (D) infinite
(A) 4.8 Volts (B) 5.3 Volts
(C) 6.0 Volts (D) 6.6 Volts 4.61 The effect of current shunt feedback in an amplifier is to
(A) increase the input resistance and decrease the output resist-
4.56 If bDC is increased by 10%, the collector-to-emitter voltage drop ance
(A) increases by less than or equal to 10%
(B) increases both input and output resistance
(B) decreases by less than or equal to 10%
(C) decrease both input and output resistance
(C) increase by more than 10%
(D) decrease the input resistance and increase the output resist-
(D) decreases by more than 10% ance

4.57 The small-signal gain of the amplifier vc is 4.62 The cascade amplifier is a multistage configuration of
vs
(A) CC - CB (B) CE - CB
GATE Electronics & Communication (C) CB - CC (D) CE - CC
by RK Kanodia
Now in 3 Volume 2005 TWO MARKS
Purchase Online at maximum discount from online store
4.63 In an ideal differential amplifier shown in the figure, a large value
and get POSTAL and Online Test Series Free of (RE ).
visit www.nodia.co.in (A) increase both the differential and common - mode gains.
(B) increases the common mode gain only.
(A) -10 (B) -5.3
(C) decreases the differential mode gain only.
(C) 5.3 (D) 10
(D) decreases the common mode gain only.

4.64 For an npn transistor connected as shown in figure VBE = 0.7 volts.
Common Data For Q. 4.44 & 4.45:
Given that reverse saturation current of the junction at room
A regulated power supply, shown in figure below, has an unregu- temperature 300 K is 10 - 13 A, the emitter current is
lated input (UR) of 15 Volts and generates a regulated output Vout .
Use the component values shown in the figure.

(A) 30 mA (B) 39 mA
(C) 49 mA (D) 20 mA

4.65 The voltage e0 is indicated in the figure has been measured by an


For more GATE Resources, Mock Test and
Study material join the community
4.58 The power dissipation across the transistor Q1 shown in the figure is
(A) 4.8 Watts (B) 5.0 Watts http://www.facebook.com/gateec2014
(C) 5.4 Watts (D) 6.0 Watts
ideal voltmeter. Which of the following can be calculated ?
4.59 If the unregulated voltage increases by 20%, the power dissipation
across the transistor Q1
(A) increases by 20% (B) increases by 50%
(C) remains unchanged (D) decreases by 20%

2005 ONE MARK

4.60 The input resistance Ri of the amplifier shown in the figure is


(A) Bias current of the inverting input only
(B) Bias current of the inverting and non-inverting inputs only
GATE Electronics and Communication Topicwise Solved Paper by RK Kanodia & Ashish Murolia Page 73

(C) Input offset current only


(D) Both the bias currents and the input offset current

4.66 The Op-amp circuit shown in the figure is filter. The type of filter
and its cut. Off frequency are respectively

4.70 Zi and Z0 of the circuit are respectively


(A) 2 MW and 2 kW (B) 2 MW and 20 kW
(A) high pass, 1000 rad/sec. (B) Low pass, 1000 rad/sec 11
(C) high pass, 1000 rad/sec (D) low pass, 10000 rad/sec (C) infinity and 2 MW (D) infinity and 20 kW
11
4.67 The circuit using a BJT with b = 50 and VBE = 0.7V is shown in 4.71 ID and VDS under DC conditions are respectively
the figure. The base current IB and collector voltage by VC and (A) 5.625 mA and 8.75 V (B) 1.875 mA and 5.00 V
respectively (C) 4.500 mA and 11.00 V (D) 6.250 mA and 7.50 V

SPECIAL EDITION ( STUDY MATERIAL FORM )


At market Book is available in 3 volume i.e. in 3 book binding
form. But at NODIA Online Store book is available in 10 book
binding form. Each unit of Book is in separate binding.
Available Only at NODIA Online Store

Click to Buy
www.nodia.co.in
(A) 43 mA and 11.4 Volts (B) 40 mA and 16 Volts
4.72 Transconductance in milli-Siemens (mS) and voltage gain of the
(C) 45 mA and 11 Volts (D) 50 mA and 10 Volts
amplifier are respectively
4.68 The Zener diode in the regulator circuit shown in the figure has a (A) 1.875 mS and 3.41 (B) 1.875 ms and -3.41
Zener voltage of 5.8 volts and a zener knee current of 0.5 mA. The (C) 3.3 mS and -6 (D) 3.3 mS and 6
maximum load current drawn from this current ensuring proper
functioning over the input voltage range between 20 and 30 volts, is 4.73 Given the ideal operational amplifier circuit shown in the figure
indicate the correct transfer characteristics assuming ideal diodes
with zero cut-in voltage.

(A) 23.7 mA (B) 14.2 mA


(C) 13.7 mA (D) 24.2 mA

4.69 Both transistors T1 and T2 show in the figure, have a b = 100 ,


threshold voltage of 1 Volts. The device parameters K1 and K2 of
T1 and T2 are, respectively, 36 mA/V2 and 9 mA/V 2 . The output
voltage Vo i s

(A) 1 V (B) 2 V
(C) 3 V (D) 4 V

Common Data For Q. 4.58, 4.59 and 4.60 :


Given, rd = 20kW , IDSS = 10 mA, Vp =- 8 V
GATE Electronics and Communication Topicwise Solved Paper by RK Kanodia & Ashish Murolia Page 74

2004 ONE MARK

4.74 An ideal op-amp is an ideal


(A) voltage controlled current source
(B) voltage controlled voltage source
(C) current controlled current source
(D) current controlled voltage source

4.75 Voltage series feedback (also called series-shunt feedback) results in


(A) increase in both input and output impedances
(B) decrease in both input and output impedances (A) 1 mF (B) 2p mF
2p
(C) increase in input impedance and decrease in output impedance 1
(C) mF (D) 2p 6 mF
(D) decrease in input impedance and increase in output impedance 2p 6
4.76 The circuit in the figure is a 4.79 In the op-amp circuit given in the figure, the load current iL is

GATE Electronics & Communication


by RK Kanodia
Now in 3 Volume
Purchase Online at maximum discount from online store (A) - Vs (B) Vs
R2 R2
and get POSTAL and Online Test Series Free
(C) - Vs (D) Vs
visit www.nodia.co.in RL R1

4.80 In the voltage regulator shown in the figure, the load current can
vary from 100 mA to 500 mA. Assuming that the Zener diode is ideal
(i.e., the Zener knee current is negligibly small and Zener resistance
is zero in the breakdown region), the value of R is

(A) 7 W (B) 70 W
(C) 70 W (D) 14 W
3
(A) low-pass filter (B) high-pass filter
4.81 In a full-wave rectifier using two ideal diodes, Vdc and Vm are the dc
(C) band-pass filter (D) band-reject filter and peak values of the voltage respectively across a resistive load. If
PIV is the peak inverse voltage of the diode, then the appropriate
2004 TWO MARKS
For more GATE Resources, Mock Test and
4.77 A bipolar transistor is operating in the active region with a collector Study material join the community
current of 1 mA. Assuming that the b of the transistor is 100 and
http://www.facebook.com/gateec2014
the thermal voltage (VT ) is 25 mV, the transconductance (gm) and
the input resistance (rp) of the transistor in the common emitter relationships for this rectifier are
configuration, are
(A) Vdc = Vm , PIV = 2Vm (B) Idc = 2 Vm , PIV = 2Vm
(A) gm = 25 mA/V and rp = 15.625 kW p p
(B) gm = 40 mA/V and rp = 4.0 kW (C) Vdc = 2 Vm , PIV = Vm (D) Vdc Vm , PIV = Vm
p p
(C) gm = 25 mA/V and rp = 2.5 k W
4.82 Assume that the b of transistor is extremely large and VBE = 0.7V, IC
(D) gm = 40 mA/V and rp = 2.5 kW
and VCE in the circuit shown in the figure
4.78 The value of C required for sinusoidal oscillations of frequency 1
kHz in the circuit of the figure is
GATE Electronics and Communication Topicwise Solved Paper by RK Kanodia & Ashish Murolia Page 75

resistance of the current-shunt negative feedback amplifier using the


above amplifier with a feedback factor of 0.2, is
(A) 1 kW (B) 1 kW
11 5
(C) 5 kW (D) 11 kW

4.89 In the amplifier circuit shown in the figure, the values of R1 and R2
are such that the transistor is operating at VCE = 3 V and IC = 1.5
mA when its b is 150. For a transistor with b of 200, the operating
point (VCE , IC ) is
(A) IC = 1 mA, VCE = 4.7 V (B) IC = 0.5 mA, VCE = 3.75 V
(C) IC = 1 mA, VCE = 2.5 V (D) IC = 0.5 mA, VCE = 3.9 V

2003 ONE MARK

4.83 Choose the correct match for input resistance of various amplifier
configurations shown below :
Configuration Input resistance (A) (2 V, 2 mA) (B) (3 V, 2 mA)
CB : Common Base LO : Low
(C) (4 V, 2 mA) (D) (4 V, 1 mA)
CC : Common Collector MO : Moderate
CE : Common Emitter HI : High SPECIAL EDITION ( STUDY MATERIAL FORM )
(A) CB - LO, CC - MO, CE - HI At market Book is available in 3 volume i.e. in 3 book binding
(B) CB - LO, CC - HI, CE - MO form. But at NODIA Online Store book is available in 10 book
(C) CB - MO, CC - HI, CE - LO binding form. Each unit of Book is in separate binding.
(D) CB - HI, CC - LO, CE - MO Available Only at NODIA Online Store

Click to Buy
4.84 The circuit shown in the figure is best described as a
www.nodia.co.in

4.90 The oscillator circuit shown in the figure has an ideal inverting
amplifier. Its frequency of oscillation (in Hz) is

(A) bridge rectifier (B) ring modulator


(C) frequency discriminator (D) voltage double

4.85 If the input to the ideal comparators shown in the figure is a


sinusoidal signal of 8 V (peak to peak) without any DC component,
then the output of the comparators has a duty cycle of

(A) 1 (B) 1
(2p 6 RC) (2pRC)
(C) 1 (D) 6
(A) 1/2 (B) 1/3 ( 6 RC) (2pRC)
(C) 1/6 (D) 1/2 4.91 The output voltage of the regulated power supply shown in the
4.86 If the differential voltage gain and the common mode voltage gain figure is
of a differential amplifier are 48 dB and 2 dB respectively, then
common mode rejection ratio is
(A) 23 dB (B) 25 dB
(C) 46 dB (D) 50 dB

4.87 Generally, the gain of a transistor amplifier falls at high frequencies


due to the
(A) internal capacitances of the device
(B) coupling capacitor at the input
(C) skin effect (A) 3 V (B) 6 V
(D) coupling capacitor at the output (C) 9 V (D) 12 V

4.92 If the op-amp in the figure is ideal, the output voltage Vout will be
2003 TWO MARKS equal to

4.88 An amplifier without feedback has a voltage gain of 50, input


resistance of 1 k W and output resistance of 2.5 kW. The input
GATE Electronics and Communication Topicwise Solved Paper by RK Kanodia & Ashish Murolia Page 76

(A) 1 V (B) 6 V
(C) 14 V (D) 17 V

4.93 Three identical amplifiers with each one having a voltage gain of 50,
input resistance of 1 kW and output resistance of 250 W are cascaded.
The opened circuit voltages gain of the combined amplifier is
(A) 49 dB (B) 51 dB
(C) 98 dB (D) 102 dB

4.94 An ideal sawtooth voltages waveform of frequency of 500 Hz and


amplitude 3 V is generated by charging a capacitor of 2 mF in every
2002 TWO MARKS
cycle. The charging requires
(A) Constant voltage source of 3 V for 1 ms 4.98 The circuit in the figure employs positive feedback and is
GATE Electronics & Communication intended to generate sinusoidal oscillation. If at a frequency
V (f) 1
by RK Kanodia f0, B (f) = 3 f = +0c, then to sustain oscillation at this frequency
V0 (f) 6
Now in 3 Volume
Purchase Online at maximum discount from online store
and get POSTAL and Online Test Series Free
visit www.nodia.co.in
(B) Constant voltage source of 3 V for 2 ms
(C) Constant voltage source of 1 mA for 1 ms
(D) Constant voltage source of 3 mA for 2 ms
(A) R2 = 5R1 (B) R2 = 6R1
(C) R2 = R1 (D) R2 = R1
2002 ONE MARK 6 5

4.95 In a negative feedback amplifier using voltage-series (i.e. voltage- 4.99 An amplifier using an opamp with a slew-rate SR = 1 V/m sec has
sampling, series mixing) feedback. a gain of 40 dB. If this amplifier has to faithfully amplify sinusoidal
(A) Ri decreases and R0 decreases signals from dc to 20 kHz without introducing any slew-rate induced
(B) Ri decreases and R0 increases distortion, then the input signal level must not exceed.
(A) 795 mV (B) 395 mV
(C) Ri increases and R0 decreases
(C) 79.5 mV (D) 39.5 mV
(D) Ri increases and R0 increases
(Ri and R0 denote the input and output resistance respectively) 4.100 A zener diode regulator in the figure is to be designed to meet the
specifications: IL = 10 mA V0 = 10 V and Vin varies from 30 V to 50
4.96 A 741-type opamp has a gain-bandwidth product of 1 MHz. A non- V. The zener diode has Vz = 10 V and Izk (knee current) =1 mA. For
inverting amplifier suing this opamp and having a voltage gain of 20
dB will exhibit a -3 dB bandwidth of For more GATE Resources, Mock Test and
(A) 50 kHz (B) 100 kHz Study material join the community
(C) 1000 kHz (D) 1000 kHz http://www.facebook.com/gateec2014
17 7.07
4.97 Three identical RC-coupled transistor amplifiers are cascaded. If satisfactory operation
each of the amplifiers has a frequency response as shown in the
figure, the overall frequency response is as given in

(A) R # 1800W (B) 2000W # R # 2200W


(C) 3700W # R # 4000W (D) R $ 4000W

4.101 The voltage gain Av = v0 of the JFET amplifier shown in the figure
vt
is IDSS = 10 mA Vp =- 5 V(Assume C1, C2 and Cs to be very large
GATE Electronics and Communication Topicwise Solved Paper by RK Kanodia & Ashish Murolia Page 77

(D) Pz = 115 mW, PT = 11.9 W

4.107 The oscillator circuit shown in the figure is

(A) +16 (B) -16


(C) +8 (D) -6 4

(A) Hartely oscillator with foscillation = 79.6 MHz


2001 ONE MARK (B) Colpitts oscillator with foscillation = 50.3 MHz
4.102 The current gain of a BJT is (C) Hartley oscillator with foscillation = 159.2 MHz
gm (D) Colpitts oscillator with foscillation = 159.3 MHz
(A) gm r0 (B)
r
g 4.108 The inverting OP-AMP shown in the figure has an open-loop gain
(C) gm rp (D) m
rp SPECIAL EDITION ( STUDY MATERIAL FORM )
At market Book is available in 3 volume i.e. in 3 book binding
4.103 Thee ideal OP-AMP has the following characteristics.
(A) Ri = 3, A = 3, R0 = 0 (B) Ri = 0, A = 3, R0 = 0 form. But at NODIA Online Store book is available in 10 book
(C) Ri = 3, A = 3, R0 = 3 (D) Ri = 0, A = 3, R0 = 3 binding form. Each unit of Book is in separate binding.
Available Only at NODIA Online Store
4.104 Consider the following two statements : Click to Buy
Statement 1 :
www.nodia.co.in
A stable multi vibrator can be used for generating square wave.
Statement 2: of 100.
Bistable multi vibrator can be used for storing binary information.
(A) Only statement 1 is correct
(B) Only statement 2 is correct
(C) Both the statements 1 and 2 are correct
(D) Both the statements 1 and 2 are incorrect

2001 TWO MARKS


The closed-loop gain V0 is
An npn BJT has gm = 38 mA/V, C m = 10 -14
F, C p = 4 # 10 -13
F, Vs
4.105
(A) - 8 (B) - 9
and DC current gain b0 = 90 . For this transistor fT and fb are
(C) - 10 (D) - 11
(A) fT = 1.64 # 108 Hz and fb = 1.47 # 1010 Hz
(B) fT = 1.47 # 1010 Hz and fb = 1.64 # 108 Hz 4.109 In the figure assume the OP-AMPs to be ideal. The output v0 of
(C) fT = 1.33 # 1012 Hz and fb = 1.47 # 1010 Hz the circuit is
(D) fT = 1.47 # 1010 Hz and fb = 1.33 # 1012 Hz

4.106 The transistor shunt regulator shown in the figure has a regulated
output voltage of 10 V, when the input varies from 20 V to 30 V.
The relevant parameters for the zener diode and the transistor are
: Vz = 9.5 , VBE = 0.3 V, b = 99 , Neglect the current through RB .
Then the maximum power dissipated in the zener diode (Pz ) and the
transistor (PT ) are t
(A) 10 cos (100t) (B) 10 #0 cos (100t) dt
t
(C) 10 - 4 #0 cos (100t) dt (D) 10 - 4 d cos (100t)
dt

2000 ONE MARK

4.110 In the differential amplifier of the figure, if the source resistance of


the current source IEE is infinite, then the common-mode gain is
(A) Pz = 75 mW, PT = 7.9 W
(B) Pz = 85 mW, PT = 8.9 W
(C) Pz = 95 mW, PT = 9.9 W
GATE Electronics and Communication Topicwise Solved Paper by RK Kanodia & Ashish Murolia Page 78

(A) zero (B) infinite (A) precision integrator (B) Hartely oscillator
(C) Butterworth high pass filter (D) Wien-bridge oscillator
(C) indeterminate (D) Vin1 + Vin2
2VT
4.116 Assume that the op-amp of the figure is ideal. If vi is a triangular
4.111 In the circuit of the figure, V0 is
wave, then v0 will be

GATE Electronics & Communication


by RK Kanodia (A) square wave (B) triangular wave
Now in 3 Volume (C) parabolic wave (D) sine wave
Purchase Online at maximum discount from online store 4.117 The most commonly used amplifier is sample and hold circuits is
and get POSTAL and Online Test Series Free (A) a unity gain inverting amplifier
visit www.nodia.co.in (B) a unity gain non-inverting amplifier
(A) -1 V (B) 2 V (C) an inverting amplifier with a gain of 10
(C) +1 V (D) +15 V (D) an inverting amplifier with a gain of 100

4.112 Introducing a resistor in the emitter of a common amplifier stabilizes


2000 TWO MARKS
the dc operating point against variations in
(A) only the temperature (B) only the b of the transistor 4.118 In the circuit of figure, assume that the transistor is in the active
(C) both temperature and b (D) none of the above region. It has a large b and its base-emitter voltage is 0.7 V. The
value of Ic is
4.113 The current gain of a bipolar transistor drops at high frequencies
because of
(A) transistor capacitances
(B) high current effects in the base
(C) parasitic inductive elements
(D) the Early effect

4.114 If the op-amp in the figure, is ideal, then v0 is

For more GATE Resources, Mock Test and


Study material join the community
http://www.facebook.com/gateec2014

(A) zero (B) (V1 - V2) sin wt


(C) - (V1 + V2) sin wt (D) (V1 + V2) sin wt

4.115 The configuration of the figure is a


GATE Electronics and Communication Topicwise Solved Paper by RK Kanodia & Ashish Murolia Page 79

4.125 A dc power supply has a no-load voltage of 30 V, and a full-load


voltage of 25 V at a full-load current of 1 A. Its output resistance
and load regulation, respectively, are
(A) 5 W and 20% (B) 25 W and 20%
(C) 5 W and 16.7% (D) 25 W and 16.7%

1998 ONE MARK

4.126 The circuit of the figure is an example of feedback of the following


(A) Indeterminate since Rc is not given (B) 1 mA type
(C) 5 mA (D) 10 mA

4.119 If the op-amp in the figure has an input offset voltage of 5 mV and
an open-loop voltage gain of 10000, then v0 will be

(A) current series (B) current shunt


(C) voltage series (D) voltage shunt

SPECIAL EDITION ( STUDY MATERIAL FORM )


(A) 0 V (B) 5 mV At market Book is available in 3 volume i.e. in 3 book binding
(C) + 15 V or -15 V (D) +50 V or -50 V form. But at NODIA Online Store book is available in 10 book
binding form. Each unit of Book is in separate binding.
1999 ONE MARK Available Only at NODIA Online Store

4.120 The first dominant pole encountered in the frequency response of a


Click to Buy
compensated op-amp is approximately at www.nodia.co.in
(A) 5 Hz (B) 10 kHz
4.127 In a differential amplifier, CMRR can be improved by using an
(C) 1 MHz (D) 100 MHz
increased
4.121 Negative feedback in an amplifier (A) emitter resistance (B) collector resistance
(A) reduces gain (C) power supply voltages (D) source resistance
(B) increases frequency and phase distortions
4.128 From a measurement of the rise time of the output pulse of an
(C) reduces bandwidth amplifier whose is a small amplitude square wave, one can estimate
(D) increases noise the following parameter of the amplifier
(A) gain-bandwidth product (B) slow rate
4.122 In the cascade amplifier shown in the given figure, if the common-
emitter stage (Q1) has a transconductance gm1 , and the common (C) upper 3–dB frequency (D) lower 3–dB frequency
base stage (Q2) has a transconductance gm2 , then the overall 4.129 The emitter coupled pair of BJT’s given a linear transfer relation
transconductance g (= i 0 /vi) of the cascade amplifier is between the differential output voltage and the differential output
voltage and the differential input voltage Vid is less a times the
thermal voltage, where a is
(A) 4 (B) 3
(C) 2 (D) 1

(A) gm1 (B) gm2 4.130 In a shunt-shunt negative feedback amplifier, as compared to the
g g basic amplifier
(C) m1 (D) m2
2 2 (A) both, input and output impedances,decrease
4.123 Crossover distortion behavior is characteristic of (B) input impedance decreases but output impedance increases
(A) Class A output stage (B) Class B output stage (C) input impedance increase but output
(C) Class AB output stage (D) Common-base output stage (D) both input and output impedances increases.

1999 TWO MARK 1998 TWO MARKS

4.124 An amplifier has an open-loop gain of 100, an input impedance of 4.131 A multistage amplifier has a low-pass response with three real poles
1 kW,and an output impedance of 100 W. A feedback network with at s =- w1 - w2 and w3 . The approximate overall bandwidth B of the
a feedback factor of 0.99 is connected to the amplifier in a voltage amplifier will be given by
series feedback mode. The new input and output impedances, (A) B = w1 + w2 + w3 (B) 1 = 1 + 1 + 1
respectively, are B w1 w2 w3
(A) 10 W and 1W (B) 10 W and 10 kW (C) B = (w1 + w2 + w3) 1/3 (D) B = w12 + w22 + w23
(C) 100 kW and 1 W (D) 100 kW and 1 kW
GATE Electronics and Communication Topicwise Solved Paper by RK Kanodia & Ashish Murolia Page 80

4.132 One input terminal of high gain comparator circuit is connected to


ground and a sinusoidal voltage is applied to the other input. The
output of comparator will be
(A) a sinusoid (B) a full rectified sinusoid
(C) a half rectified sinusoid (D) a square wave

4.133 In a series regulated power supply circuit, the voltage gain Av of the
‘pass’ transistor satisfies the condition
(A) Av " 3 (B) 1 << Av < 3
(C) Av . 1 (D) Av << 1

4.134 For full wave rectification, a four diode bridge rectifier is claimed to (A) decrease the voltage gain and decrease the input impedance
have the following advantages over a two diode circuit : (B) increase the voltage gain and decrease the input impedance
(A) less expensive transformer, (C) decrease the voltage gain and increase the input impedance
(B) smaller size transformer, and (D) increase the voltage gain and increase the input impedance
(C) suitability for higher voltage application.
4.138 A cascade amplifier stags is equivalent to
Of these,
(A) a common emitter stage followed by a common base stage
(A) only (1) and (2) are true
(B) a common base stage followed by an emitter follower
(B) only (1) and (3) are true
(C) an emitter follower stage followed by a common base stage
GATE Electronics & Communication (D) a common base stage followed by a common emitter stage
by RK Kanodia 4.139 In a common emitter BJT amplifier, the maximum usable supply
Now in 3 Volume voltage is limited by
Purchase Online at maximum discount from online store (A) Avalanche breakdown of Base-Emitter junction
and get POSTAL and Online Test Series Free (B) Collector-Base breakdown voltage with emitter open (BVCBO)
visit www.nodia.co.in (C) Collector-Emitter breakdown voltage with base open (BVCBO)
(D) Zener breakdown voltage of the Emitter-Base junction
(C) only (2) and (3) are true
(D) (1), (2) as well as (3) are true 1997 TWO MARKS
4.135 In the MOSFET amplifier of the figure is the signal output V1 and 4.140 In the circuit of in the figure is the current iD through the ideal
V2 obey the relationship diode (zero cut in voltage and forward resistance) equals

(A) 0 A (B) 4 A
(C) 1 A (D) None of the above
(A) V1 = V2 (B) V1 =-V2
2 2 4.141 The output voltage V0 of the circuit shown in the figure is
(C) V1 = 2V2 (D) V1 =- 2V2
For more GATE Resources, Mock Test and
4.136 For small signal ac operation, a practical forward biased diode can
be modelled as Study material join the community
(A) a resistance and a capacitance in series http://www.facebook.com/gateec2014
(B) an ideal diode and resistance in parallel
(C) a resistance and an ideal diode in series
(D) a resistance

1997 ONE MARK

4.137 In the BJT amplifier shown in the figure is the transistor is based in
the forward active region. Putting a capacitor across RE will
GATE Electronics and Communication Topicwise Solved Paper by RK Kanodia & Ashish Murolia Page 81

(A) - 4 V (B) 6 V
(C) 5 V (D) - 5.5 V

4.142 A half wave rectifier uses a diode with a forward resistance Rf . The
voltage is Vm sin wt and the load resistance is RL . The DC current
is given by
(A) Vm (B) Vm
2 RL p (R f + RL)
(A) gm1 (B) 0.5 gm1
(C) 2 V m
(D) Vm
p RL (C) gm2 (D) 0.5 gm2

4.147 Value of R in the oscillator circuit shown in the given figure, so


1996 ONE MARK
chosen that it just oscillates at an angular frequency of w. The value
4.143 In the circuit of the given figure, assume that the diodes are ideal of w and the required value of R will respectively be
and the meter is an average indicating ammeter. The ammeter will
read

SPECIAL EDITION ( STUDY MATERIAL FORM )


At market Book is available in 3 volume i.e. in 3 book binding
form. But at NODIA Online Store book is available in 10 book
(A) 0.4 2 A (B) 0.4 A binding form. Each unit of Book is in separate binding.
(C) 0.8 A (D) 0.4 mamp Available Only at NODIA Online Store
p p
Click to Buy
4.144 The circuit shown in the figure is that of
www.nodia.co.in

(A) a non-inverting amplifier (B) an inverting amplifier


(C) an oscillator (D) a Schmitt trigger

1996 TWO MARKS

4.145 In the circuit shown in the given figure N is a finite gain amplifier
with a gain of k , a very large input impedance, and a very low
output impedance. The input impedance of the feedback amplifier
with the feedback impedance Z connected as shown will be

(A) 105 rad/ sec, 2 # 10 4 W (B) 2 # 10 4 rad/ sec, 2 # 10 4 W


(C) 2 # 10 4 rad/ sec, 105 W (D) 105 rad/ sec, 105 W

4.148 A zener diode in the circuit shown in the figure is has a knee current
of 5 mA, and a maximum allowed power dissipation of 300 mW
(A) Z b1 - 1 l (B) Z (1 - k) . What are the minimum and maximum load currents that can
k be drawn safely from the circuit, keeping the output voltage V0
(C) Z (D) Z constant at 6 V?
(k - 1) (1 - k)
4.146 A Darlington stage is shown in the figure. If the transconductance of
c
Q1 is gm1 and Q2 is gm2 , then the overall transconductance gmc ;T i cc E
vbe
is given by

(A) 0 mA, 180 mA (B) 5 mA, 110 mA


(C) 10 mA, 55 mA (D) 60 mA, 180 mA
GATE Electronics and Communication Topicwise Solved Paper by RK Kanodia & Ashish Murolia Page 82

***********

GATE Electronics & Communication


by RK Kanodia
Now in 3 Volume
Purchase Online at maximum discount from online store
and get POSTAL and Online Test Series Free
visit www.nodia.co.in

For more GATE Resources, Mock Test and


Study material join the community
http://www.facebook.com/gateec2014
GATE Electronics and Communication Topicwise Solved Paper by RK Kanodia & Ashish Murolia Page 83

SOLUTIONS (1)
Since, voltage across zener diode is 5 V so, current through 100 W
resistor is obtained as
Is = 10 - 5 = 0.05 A
100
Therefore, the load current is given by
4.1 Option (B) is correct. IL = 5
RL
For the given ideal op-amp, negative terminal will be also ground Since, for proper operation, we must
(at zero voltage) and so, the collector terminal of the BJT will be have
at zero voltage. IZ $ Iknes
i.e., VC = 0 volt So, from Eq. (1), we write
The current in 1 kW resistor is given by 0.05 A - 5 $ 10 mA
RL
I = 5 - 0 = 5 mA
1 kW 50 mA - 5 $ 10 mA
RL
This current will flow completely through the BJT since, no cur-
40 mA $ 5
rent will flow into the ideal op-amp ( I/P resistance of ideal op- RL
amp is infinity). So, for BJT we have 40 # 10 $ 5
-3
RL
VC = 0
VB = 0 SPECIAL EDITION ( STUDY MATERIAL FORM )
IC = 5 mA At market Book is available in 3 volume i.e. in 3 book binding
i.e.,the base collector junction is reverse biased (zero voltage) form. But at NODIA Online Store book is available in 10 book
therefore, the collector current (IC ) can have a value only if base- binding form. Each unit of Book is in separate binding.
Available Only at NODIA Online Store
emitter is forward biased. Hence,
VBE = 0.7 volts Click to Buy
& VB - VE = 0.7 www.nodia.co.in
& 0 - Vout = 0.7
or, Vout =- 0.7 volt 1 RL
-3 # 5
40 # 10
4.2 Option (A) is correct.
5 # RL
The i/p voltage of the system is given as 40 # 10-3
Vin = V1 + Vf or, 125 W # RL
= V1 + k Vout Therefore, minimum value of RL = 125 W
= V1 + k A 0 V1 ^Vout = A 0 V1h Now, we know that power rating of Zener diode is given by
PR = VZ IZ^maxh
= V1 ^1 + k A 0h
IZ^maxh is maximum current through zener diode in reverse bias.
Therefore, if k is increased then input voltage is also increased so, Maximum currrent through zener diode flows when load current is
the input impedance increases. Now, we have zero. i.e.,
Vout = A 0 V1 IZ^maxh = Is = 10 - 5 = 0.05
Vin 100
= A0
^1 + k A 0h Therefore, PR = 5 # 0.05 W
= A 0 Vin = 250 mW
^1 + k A 0h
4.4 Option (A) is correct.
Since, Vin is independent of k when seen from output mode, the
For the given circuit, we obtain the small signal model as shown in
output voltage decreases with increase in k that leads to the decrease
figure below :
of output impedance. Thus, input impedance increases and output
impedance decreases.
4.3 Option (B) is correct.

We obtain the node voltage at V1 as


V1 + V1 + gm Vi = 0
RD R + 1
L
sC
- gm Vi
& V1 =
1 + 1
From the circuit, we have RD R + 1
L
Is = IZ + I L sC
or, IZ = Is - I L Therefore, the output voltage V0 is obtained as
GATE Electronics and Communication Topicwise Solved Paper by RK Kanodia & Ashish Murolia Page 84

V0 = V1 RL Option (B) is correct.


4.6

RL + 1 For the given circuit, we can make the truth table as below
sC X Y Z
RL J - gm Vi N 0 0 0
= K 1 O
RL + 1 K RD +
1
O 0 1 1
sC K RL + 1 O 1 0 0
L sC P
so, the transfer function is 1 1 0
V0 = - RD RL sCgm Logic 0 means voltage is v = 0 volt and logic 1 means voltage is
Vi 1 + sC ^RD + RL h
5 volt
Then, we have the pole at w = 1
C ^RD + RL h For x = 0 , y = 0 , Transistor is at cut off mode and diode is forward
It gives the lower cutoff frequency of transfer function. biased. Since, there is no drop across forward biased diode.
i.e., w0 = 1 So, Z =Y=0
C ^RD + RL h For x = 0 , y = 1, Again Transistor is in cutoff mode, and diode is
or, f0 = 1
forward biased. with no current flowing through resistor.
2pC ^RD + RL h
1 So, Z =Y=1
=
2p # 10-6 # 20 # 103 For x = 1, y = 0 , Transistor is in saturation mode and so, z directly
= 7.97 connected to ground irrespective of any value of Y .
. 8 Hz i.e., Z = 0 (ground)
4.5 Option (C) is correct. Similarly for X = Y = 1
Z = 0 (ground)
GATE Electronics & Communication Hence, from the obtained truth table, we get
by RK Kanodia Z =XY
Now in 3 Volume 4.7 Option (D) is correct.
Purchase Online at maximum discount from online store Given, the input voltage
VYZ = 100 sin wt
and get POSTAL and Online Test Series Free
visit www.nodia.co.in

For + ve half cycle


VYZ > 0
i.e., VY is a higher voltage than VZ
So, the diode will be in cutoff region. Therefore, there will no volt-
age difference between X and W node.
i.e., VWX = 0
Now, for - ve half cycle all the four diodes will active and so, X
and W terminal is short circuited

For the given ideal op-Amps we can assume For more GATE Resources, Mock Test and
V 2- = V 2+ = V2 (ideal) Study material join the community
V 1+ = V 1- = V1 (ideal) http://www.facebook.com/gateec2014
So, by voltage division
V1 = Vout # 1 i.e., VWX = 0
2 Hence, VWX = 0 for all t
Vout = 2V1
4.8 Option (C) is correct.
and, as the I/P current in Op-amp is always zero therefore, there
The equivalent circuit can be shown as
will be no voltage drop across 1 KW in II op-amp
i.e., V2 = 1 V
Therefore,
V1 - V2 = V2 - ^- 2h
1 1
& V1 - 1 = 1 + 2
or, V1 = 4
Hence,
Vout = 2V1 = 8 volt VTh = VCC R2
R1 + R 2
GATE Electronics and Communication Topicwise Solved Paper by RK Kanodia & Ashish Murolia Page 85

3R2
=
R1 + R 2
and RTh = R2 R1
R 2 + R1
Since, IC = bIB has b . 3 (very high) so, IB is negative in
comparison to IC . Therefore, we can write the base voltage
VB = VTh
The peak rectifier adds + 1 V to peak voltage, so overall peak voltage
So, VTh - 0.7 - IC RE = 0
lowers down by - 1 volt.
or,
R1 + R 2 ^ h^ h
3R2 - 0.7 - 10-3 500 = 0 So, vo = cos wt - 1

or, 3R2 = 0.7 + 0.5


4.12 Option (A) is correct.
60 kW + R2 We put a test source between terminal 1, 2 to obtain equivalent
or, 3R2 = ^60 kWh^1.2h + 1.2R2 impedance

or, 1.8R2 = ^60 kWh # ^1.2h

Hence, R2 = 60 # 1.2 = 40 kW
1.8
4.9 Option (C) is correct.
Given ib = 1 + 0.1 cos (1000pt) mA SPECIAL EDITION ( STUDY MATERIAL FORM )
So, IB = DC component of ib At market Book is available in 3 volume i.e. in 3 book binding
= 1 mA form. But at NODIA Online Store book is available in 10 book
In small signal model of the transistor binding form. Each unit of Book is in separate binding.
bVT Available Only at NODIA Online Store
rp = VT " Thermal voltage
IC Click to Buy
= VT = VT IC = I
IC /b I B b B www.nodia.co.in
= VT
IB
So, rp = 25 mV = 25 W VT = 25 mV, IB = 1 mA
1 mA
4.10 Option (D) is correct.
Let v > 0.7 V and diode is forward biased. By applying Kirchoff’s
voltage law
10 - i # 1k - v = 0
10 - :v - 0.7 D (1000) - v = 0
500
10 - (v - 0.7) # 2 - v = 0 ZTh = Vtest
Itest
10 - 3v + 1.4 = 0 Applying KCL at top right node
v = 11.4 = 3.8 V > 0.7 (Assumption is true) Vtest + Vtest - 99I = I
3 9 k + 1k 100 b test

So, i = v - 0.7 = 3.8 - 0.7 = 6.2 mA Vtest + Vtest - 99I = I


500 500 10 k 100 b test

4.11 Option (A) is correct. ...(i)


The circuit composed of a clamper and a peak rectifier as shown.
But Ib =- Vtest =-Vtest
9k + 1k 10k
Substituting Ib into equation (i), we have
Vtest + Vtest + 99Vtest = I
test
10 k 100 10 k
100Vtest + Vtest = I
test
10 # 103 100
2Vtest = I
test
100
Clamper clamps the voltage to zero voltage, as shown ZTh = Vtest = 50 W
Itest
4.13 Option (B) is correct.
First we obtain the transfer function.
GATE Electronics and Communication Topicwise Solved Paper by RK Kanodia & Ashish Murolia Page 86

IC - IE = 13.7 - VC = (b + 1) IB
12k
13.7 - VC = 100I ...(ii)
B
12 # 103
Solving equation (i) and (ii),
IB = 0.01 mA
Small Signal Analysis :
0 - Vi (jw) 0 - Vo (jw) Transforming given input voltage source into equivalent current
+ =0 source.
1 +R R2
1
jwC
Vo (jw) - Vi (jw)
=
R2 1 +R
1
j wC
Vi (jw) R2
Vo (jw) =-
R1 - j 1
wC
At w " 0 (Low frequencies), 1 " 3, so V = 0
o
wC This is a shunt-shunt feedback amplifier.
At w " 3 (higher frequencies) Given parameters,
1 " 0, so V (jw) =- R2 V (jw)
wC o
R1 i rp = VT = 25 mV = 2.5 kW
IB 0.01 mA
GATE Electronics & Communication b 100
gm = = = 0.04 s
by RK Kanodia rp 2.5 # 1000
Now in 3 Volume Writing KCL at output node
v0 + g v + v0 - vp = 0
Purchase Online at maximum discount from online store RC m p
RF
and get POSTAL and Online Test Series Free
v 0 : 1 + 1 D + v p :gm - 1 D = 0
visit www.nodia.co.in RC RF RF
Substituting RC = 12 kW, RF = 100 kW, gm = 0.04 s
The filter passes high frequencies so it is a high pass filter. v 0 (9.33 # 10-5) + v p (0.04) = 0
H (jw) = Vo = - R2 v 0 =- 428.72Vp
Vi R1 - j 1 ...(i)
wC
H (3) = - R 2
= R 2 Writing KCL at input node
R1 R1 vi = v p + v p + v p - vo
At 3 dB frequency, gain will be 2 times of maximum gain Rs Rs rp RF
6H (3)@ vi = v 1 + 1 + 1 - v 0
p:
Rs Rs rp RF D RF
H ^ jw0h = 1 H (3) vi = v (5.1 10-4) - v 0
2 p #
Rs RF
R2 1 R2
So, = b l Substituting Vp from equation (i)
R 12 + 21 2 2 R1
w0 C
vi = - 5.1 # 10-4 v - v 0
0
2R 12 = R 12 + 1 Rs 428.72 RF
w C2
2
0 vi =- 1.16 # 10-6 v 0 - 1 # 10-5 v 0 Rs = 10 kW
R = 21 2
2
1
10 # 103
w C
For more GATE Resources, Mock Test and
w0 = 1
R1 C Study material join the community
Option (D) is correct.
http://www.facebook.com/gateec2014
4.14

DC Analysis :

(source resistance)
vi =- 1.116 # 10-5
10 # 103
Av = v 0 = 1 - 8.96
vi 10 # 10 # 1.116 # 10-5
3

4.15 Option (A) is correct.


For the parallel RLC circuit resonance frequency is,
Using KVL in input loop,
wr = 1 = 1 = 10 M rad/s
VC - 100IB - 0.7 = 0 LC 10 # 10 # 1 # 10-9
-6

VC = 100IB + 0.7 ...(i) Thus given frequency is resonance frequency and parallel RLC
circuit has maximum impedance at resonance frequency
GATE Electronics and Communication Topicwise Solved Paper by RK Kanodia & Ashish Murolia Page 87

Gain of the amplifier is gm # (ZC RL) where ZC is impedance of In active region, for common emitter amplifier,
parallel RLC circuit. IC = bIB + (1 + b) ICO ...(1)
At w = wr , ZC = R = 2 kW = ZC max . Substituting ICO = 0.6 mA and IB = 20 mA in above eq we have,
Hence at this frequency (wr ), gain is IC = 1.01 mA
Gain w = w = gm (ZC RL) = gm (2k 2k) = gm # 103 which is 4.19 Option (C) is correct.
r

maximum. Therefore gain is maximum at wr = 10 M rad/ sec . In active region VBEon = 0.7 V
Emitter voltage VE = VB - VBEon =- 5.7 V
4.16 Option (D) is correct.
V - (- 10) - 5.7 - (- 10)
The given circuit is shown below : Emitter Current IE = E = = 1 mA
4.3k 4.3k
Now IC . IE = 1 mA
Applying KCL at collector
i1 = 0.5 mA
Since i1 = C dVC
dt
or VC = 1 # i1 dt = i1 t ...(1)
C C

From diagram we can write SPECIAL EDITION ( STUDY MATERIAL FORM )


Ii = Vo + Vo At market Book is available in 3 volume i.e. in 3 book binding
R1 sL1
Transfer function
form. But at NODIA Online Store book is available in 10 book
binding form. Each unit of Book is in separate binding.
H (s) = Vo = sR1 L1 Available Only at NODIA Online Store
I1 R1 + sL1
jwR1 L1 Click to Buy
or H (jw) =
R 1 + j wL 1 www.nodia.co.in
At w = 0 H (jw) = 0
At w = 3 H (jw) = R1 = constant . Hence HPF.
4.17 Option (C) is correct.
Given circuit is shown below.

with time, the capacitor charges and voltage across collector changes
from 0 towards negative.
For transistor M2 ,
When saturation starts, VCE = 0.7 & VC =+ 5 V (across
VGS = VG - VS = Vx - 0 = Vx
capacitor)
VDS = VD - VS = Vx - 0 = Vx
Thus from (1) we get, + 5 = 0.5 mA T
5 mA
Since VGS - VT = Vx - 1 < VDS , thus M2 is in saturation. -6
By assuming M1 to be in saturation we have or T = 5 # 5 # 10-3 = 50 m sec
0.5 # 10
IDS (M ) = IDS (M )
1 2 4.20 Option (A) is correct.
mn C 0x m C The current flows in the circuit if all the diodes are forward biased.
(4) (5 - Vx - 1) 2 = n 0x 1 (Vx - 1) 2
2 2 In forward biased there will be 0.7 V drop across each diode.
4 (4 - Vx ) 2 = (Vx - 1) 2 12.7 - 4 (0.7)
Thus IDC = = 1 mA
or 2 (4 - Vx ) = ! (Vx - 1) 9900
Taking positive root, 4.21 Option (B) is correct.
8 - 2Vx = Vx - 1 The forward resistance of each diode is
Vx = 3 V r = VT = 25 mV = 25 W
IC 1 mA
At Vx = 3 V for M1,VGS = 5 - 3 = 2 V < VDS . Thus our assumption
4 (r)
Vac = Vi # e
4 (r) + 9900 o
is true and Vx = 3 V . Thus
4.18 Option (D) is correct.
= 100 mV cos (wt) 0.01
We have a = 0.98
= 1 cos (wt) mV
Now b = a = 4.9
1-a 4.22 Option (A) is correct.
GATE Electronics and Communication Topicwise Solved Paper by RK Kanodia & Ashish Murolia Page 88

The equivalent circuit of given amplifier circuit (when CE is 4.24 Option (A) is correct.
connected, RE is short-circuited) The circuit is as shown below :

Input impedance Ri = RB || r p
Voltage gain AV = gm RC
Now, if CE is disconnected, resistance RE appears in the circuit
So, 0 - Vi + 0 - Vo = 0
R1 R2
or Vo =- R2
Vi R1
4.25 Option (B) is correct.
By small signal equivalent circuit analysis

Input impedance R in = RB || [rp + (b + 1)] RE


GATE Electronics & Communication
by RK Kanodia
Now in 3 Volume Input resistance seen by source vs
Purchase Online at maximum discount from online store R in = vs = Rs + Rs || rs
and get POSTAL and Online Test Series Free is

visit www.nodia.co.in = (1000 W) + (93 kW || 259 W) = 1258 W


4.26 Option (B) is correct.
Input impedance increases Cut-off frequency due to C2
gm RC 1
Voltage gain AV = Voltage gain decreases. fo =
1 + gm R E 2p (RC + RL) C2
4.23 Option (B) is correct. fo
Since, emitter area of transistor Q1 is half of transistor Q2 , so current = 1 = 271 Hz
2 # 3.14 # 1250 # 4.7 # 10-6
IE = 1 IE and IB = 1 IB Lower cut-off frequency
1
2 2
21 2

f
The circuit is as shown below : fL . o = 271 = 27.1 Hz
10 10
4.27 Option (B) is correct.
The circuit is as shown below

For more GATE Resources, Mock Test and


Study material join the community
VB =- 10 - (- 0.7) =- 9.3 V http://www.facebook.com/gateec2014
Collector current
0 - (- 9.3)
I1 = = 1 mA
(9.3 kW)
b 1 = 700 (high), So IC . IE 1

Applying KCL at base we have


1 - IE = IB + IB
1 2

1 - (b 1 + 1) IB = IB + IB
1 1 2

IB
1 = (700 + 1 + 1) 2
+ IB
2 2

IB . 2
2
702
I 0 = IC = b 2 : IB = 715 # 2 . 2 mA
2 2
702
GATE Electronics and Communication Topicwise Solved Paper by RK Kanodia & Ashish Murolia Page 89

Current I = 20 - 0 + Vi - 0 = 5 + Vi Thus IC = 10 - 5 = 1 mA
4R R R 5k
If I > 0, diode D2 conducts VE = IE RE = 1m # 1.4k = 1.4V IE = IC
So, for 5 + VI > 0 & VI > - 5, D2 conducts = 0.6 + 1.4 = 2V
2
Thus the feedback is negative and output voltage is V = 2V .
Equivalent circuit is shown below
4.33 Option (D) is correct.
The output voltage is
hfe RC
V0 = Ar Vi .- Vi
hie
Here RC = 3 W and hie = 3 kW
Thus V0 . - 150 # 3k Vi
3k
Output is Vo = 0 . If I < 0 , diode D2 will be off
.- 150 (A cos 20t + B sin 106 t)
5 + VI < 0 & V < - 5, D is off
R I 2 Since coupling capacitor is large so low frequency signal will be
The circuit is shown below filtered out, and best approximation is
V0 .- 150B sin 106 t
4.34 Option (C) is correct.
SPECIAL EDITION ( STUDY MATERIAL FORM )
At market Book is available in 3 volume i.e. in 3 book binding
form. But at NODIA Online Store book is available in 10 book
0 - Vi + 0 - 20 + 0 - Vo = 0 binding form. Each unit of Book is in separate binding.
R 4R R Available Only at NODIA Online Store

or Vo =- Vi - 5 Click to Buy
www.nodia.co.in
At Vi =- 5 V, Vo = 0
At Vi =- 10 V, Vo = 5 V For the positive half of Vi , the diode D1 is forward bias, D2 is reverse
4.28 Option (A) is correct. bias and the zener diode is in breakdown state because Vi > 6.8 .
Let diode be OFF. In this case 1 A current will flow in resistor and Thus output voltage is
voltage across resistor will be V = 1.V V0 = 0.7 + 6.8 = 7.5 V
Diode is off, it must be in reverse biased, therefore For the negative half of Vi, D2 is forward bias thus
Vi - 1 > 0 " Vi > 1 Then V0 =- 0.7 V
Thus for Vi > 1 diode is off and V = 1V
Option (B) and (C) doesn’t satisfy this condition.
4.35 Option (B) is correct.
Let Vi < 1. In this case diode will be on and voltage across diode will By Current mirror,
^Lh
W
be zero and V = Vi Ix = W 2 Ibias
Thus V = min (Vi, 1) ^ L h1
Since MOSFETs are identical,
4.29 Option (A) is correct.
W W
The R2 decide only the frequency. Thus b L l =b L l
2 2
4.30 Option (D) is correct. Hence Ix = Ibias
For small increase in VG beyond 1 V the n - channel MOSFET goes
4.36 Option (B) is correct.
into saturation as VGS "+ ive and p - MOSFET is always in active
The circuit is using ideal OPAMP. The non inverting terminal of
region or triode region.
OPAMP is at ground, thus inverting terminal is also at virtual
4.31 Option (C) is correct. ground.
4.32 Option (D) is correct.
The circuit is shown in fig below

Thus current will flow from -ive terminal (0 Volt) to -1 Volt source.
Thus the current I is
0 - (- 1)
I = = 1
100k 100k
The voltage at non inverting terminal is 5 V because OP AMP is The current through diode is
I = I 0 _eV - 1i
ideal and inverting terminal is at 5 V. V
t
GATE Electronics and Communication Topicwise Solved Paper by RK Kanodia & Ashish Murolia Page 90

Now VT = 25 mV and I0 = 1 mA
I = 10-6 8e 25 # 10 - 1B = 1 5
V
Thus -3

10
or V = 0.06 V
Now V0 = I # 4k + V = 1 # 4k + 0.06 = 0.1
100k
V

4.37 Option (B) is correct. The Thevenin resistance and voltage are
The circuit is using ideal OPAMP. The non inverting terminal of VTH = 10 # 9 = 3 V
OPAMP is at ground, thus inverting terminal is also at virtual 10 + 20
ground. and total RTH = 10k # 20k = 6.67 kW
10k + 20k
Since b is very large, therefore IB is small and can be ignored
Thus IE = VTH - VBE = 3 - 0.7 = 1 mA
RE 2.3k
4.41 Option (D) is correct.
The small signal model is shown in fig below

GATE Electronics & Communication


by RK Kanodia
Now in 3 Volume
Purchase Online at maximum discount from online store gm =
IC
= 1m = 1 A/V IC . IE
and get POSTAL and Online Test Series Free VT 25m 25
visit www.nodia.co.in Vo =- gm Vp # (3k 3k )
=- 1 Vin (1.5k) Vp = Vin
Thus we can write 25
vi = -v =- 60Vin
R1 + sL R2
sR2 C2 + 1 or Am = Vo =- 60
Vin
or v0 =- R2
vi (R1 + sL)( sR2 C2 + 1) 4.42 Option (C) is correct.
and from this equation it may be easily seen that this is the stand- The circuit shown in (C) is correct full wave rectifier circuit.
ard form of T.F. of low pass filter
H (s) = K
(R1 + sL)( sR2 C2 + 1)
and form this equation it may be easily seen that this is the stand-
ard form of T.F. of low pass filter
H (s) = 2 K
as + bs + b 4.43 Option (A) is correct.
4.38 Option ( ) is correct. In the transconductance amplifier it is desirable to have large input
The current in both transistor are equal. Thus gm is decide by M1. resistance and large output resistance.
Hence (C) is correct option. For more GATE Resources, Mock Test and
4.39 Option (C) is correct. Study material join the community
Let the voltage at non inverting terminal be V1, then after applying
http://www.facebook.com/gateec2014
KCL at non inverting terminal side we have
15 - V1 + V0 - V1 = V1 - (- 15) 4.44 Option (C) is correct.
10 10 10 We redraw the circuit as shown in fig.
or V1 = V0
3
If V0 swings from -15 to +15 V then V1 swings between -5 V to +5
V.
4.40 Option (A) is correct.
For the given DC values the Thevenin equivalent circuit is as follows

Applying voltage division rule


v+ = 0.5 V
GATE Electronics and Communication Topicwise Solved Paper by RK Kanodia & Ashish Murolia Page 91

We know that v+ = v- The range of current through 200 kW is


Thus v- = 0.5 V 3 = 15 mA to 9 = 45 mA
200k 200k
Now i = 1 - 0.5 = 0.5 mA
1k The range of variation in output voltage
and i = 0.5 - v0 = 0.5 mA 15m # RZ = 0.15 V to 45m # RZ = 0.45
2k
Thus the range of output voltage is 7.15 Volt to 7.45 Volt
or v0 = 0.5 - 1 =- 0.5 V
4.49 Option (A) is correct.
4.45 Option (B) is correct. The voltage at non-inverting terminal is
If we assume b very large, then IB = 0 and IE = IC ; VBE = 0.7 V. We 1
V+ = sC
Vi = 1 V
assume that BJT is in active, so applying KVL in Base-emitter loop 1 + sCR i
R + sC1
IE = 2 - VBE = 2 - 0.7 = 1.3 mA Now V- = V+ = 1 V
RE 1k 1 + sCR i
Since b is very large, we have IE = IC , thus Applying voltage division rule
IC = 1.3 mA (V + Vi)
V+ = R1 (V0 + Vi) = o
Now applying KVL in collector-emitter loop R1 + R1 2
1 (Vo + Vi)
10 - 10IC - VCE - IC = 0 or V =
1 + sCR i 2
or VCE =- 4.3 V Vo =- 1 + 2
or
Now VBC = VBE - VCE Vi 1 + sRC
SPECIAL EDITION ( STUDY MATERIAL FORM )
= 0.7 - (- 4.3) = 5 V
At market Book is available in 3 volume i.e. in 3 book binding
Since VBC > 0.7 V, thus transistor in saturation.
form. But at NODIA Online Store book is available in 10 book
4.46 Option (D) is correct. binding form. Each unit of Book is in separate binding.
Here the inverting terminal is at virtual ground and the current in Available Only at NODIA Online Store
resistor and diode current is equal i.e.
Click to Buy
IR = ID
Vi = I eV /V
www.nodia.co.in
or s
D T

R
V0 = 1 - sRC
or VD = VT 1n Vi Vi 1 + sRC
Is R
For the first condition 4.50 Option (C) is correct.
VD = 0 - Vo1 = VT 1n 2 V0 = H (s) = 1 - sRC
Is R Vi 1 + sRC
For the first condition 1 - jwRC
H (jw) =
1 + jwRC
VD = 0 - Vo1 = VT 1n 4
Is R +H (jw) = f =- tan - 1 wRC - tan - 1 wRC
Subtracting above equation =- 2 tan - 2 wRC
Vo1 - Vo2 = VT 1n 4 - VT 1n 2 Minimum value, fmin = - p (at w " 3)
Is R Is R
Maximum value, fmax = 0( at w = 0)
or Vo1 - Vo2 = VT 1n 4 = VT 1n2
2 4.51 Option (D) is correct.
4.47 Option (D) is correct. In the transconductance amplifier it is desirable to have large input
We have Vthp = Vthp = 1 V impedance and large output impedance.
WP W 4.52 Option (C) is correct.
and = N = 40mA/V2
LP LN
4.53 Option (D) is correct.
From figure it may be easily seen that Vas for each NMOS and The voltage at inverting terminal is
PMOS is 2.5 V V- = V+ = 10 V
mA
Thus ID = K (Vas - VT ) 2 = 40 2 (2.5 - 1) 2 = 90 m A Here note that current through the capacitor is constant and that
V
is
4.48 Option (C) is correct.
We have VZ = 7 volt, VK = 0, RZ = 10W I = V- = 10 = 10 mA
1k 1k
Circuit can be modeled as shown in fig below
Thus the voltage across capacitor at t = 1 msec is
1m 1m
VC = 1 Idt = 1 10mdt
# #
Im
C 0 1m 0
4
#0
= 10 dt = 10 V

4.54 Option (A) is correct.


Since Vi is lies between 10 to 16 V, the range of voltage across 200 In forward bias Zener diode works as normal diode.
kW Thus for negative cycle of input Zener diode is forward biased and
V200 = Vi - VZ = 3 to 9 volt it conducts giving VR = Vin .
GATE Electronics and Communication Topicwise Solved Paper by RK Kanodia & Ashish Murolia Page 92

For positive cycle of input Zener diode is reversed biased Now VCE = 15 - 9 = 6 V
when 0 < Vin < 6 , Diode is OFF and VR = 0 The power dissipated in transistor is
when Vin > 6 Diode conducts and voltage across diode is 6 V. Thus P = VCE IC = 6 # 0.9 = 5.4 W
voltage across is resistor is
4.59 Option (B) is correct.
VR = Vin - 6
If the unregulated voltage increase by 20%, them the unregulated
Only option (B) satisfy this condition. voltage is 18 V, but the VZ = Vin = 6 remain same and hence Vout
4.55 Option (C) is correct. and IC remain same. There will be change in VCE
The circuit under DC condition is shown in fig below Thus, VCE - 18 - 9 = 9 V
IC = 0.9 A
Power dissipation P = VCE IC = 9 # 0.9 = 8.1 W
Thus % increase in power is
8.1 - 5.4 # 100 = 50%
5.4
4.60 Option (B) is correct.
Since the inverting terminal is at virtual ground, the current flowing
through the voltage source is
Applying KVL we have
VCC - RC (IC + IB) - VCE = 0 ...(1) Is = Vs
10k
Vs = 10 kW = R
GATE Electronics & Communication or
Is in

by RK Kanodia 4.61 Option (D) is correct.


Now in 3 Volume The effect of current shunt feedback in an amplifier is to decrease
Purchase Online at maximum discount from online store the input resistance and increase the output resistance as :
and get POSTAL and Online Test Series Free Rif = Ri
1 + Ab
visit www.nodia.co.in Rof = R0 (1 + Ab)
where Ri " Input resistance without feedback
and VCC - RB IB - VBE = 0 ...(2) Rif " Input resistance with feedback.
Substituting IC = bIB in (1) we have
4.62 Option (B) is correct.
VCC - RC (bIB + IB) - VCE = 0 ...(3)
The CE configuration has high voltage gain as well as high
Solving (2) and (3) we get current gain. It performs basic function of amplifications. The CB
VCE = VCC - VCC - VBE ...(4) configuration has lowest Ri and highest Ro . It is used as last step to
1+ RB match a very low impedance source and to drain a high impedance
RC (1 + b)
load
Now substituting values we get
Thus cascade amplifier is a multistage configuration of CE-CB
VCE = 12 - 12 - 0.7 = 5.95 V
53 4.63 Option (D) is correct.
1+
1 + (1 + 60) Common mode gain
4.56 Option (B) is correct. ACM =- RC
2RE
We have b' = 110 # 60 = 66
100 And differential mode gain
Substituting b' = 66 with other values in (iv) in previous solutions ADM =- gm RC
VCE = 12 - 12 - 0.7 = 5.29 V For more GATE Resources, Mock Test and
1+ 53
1 + (1 + 66) Study material join the community
Thus change is = 5.29 - 59.5 # 100 =- 4.3%
http://www.facebook.com/gateec2014
5.95
4.57 Option (A) is correct. Thus only common mode gain depends on RE and for large value
4.58 Option (C) is correct. of RE it decreases.
The Zener diode is in breakdown region, thus 4.64 Option (C) is correct.
IE = Is `e nV - 1j
VBE
V+ = VZ = 6 V = Vin T
R
Vo = Vin c1 + f m 0.7
c e1 # 26 # 10 - 1m = 49 mA
We know that = 10 - 13
R1 -3

or Vout = Vo = 6`1 + 12k j = 9 V Option (C) is correct.


24k 4.65

The current in 12 kW branch is negligible as comparison to 10 W. The circuit is as shown below


Thus Current
IC . IE . = Vout = 9 = 0.9 A
RL 10
GATE Electronics and Communication Topicwise Solved Paper by RK Kanodia & Ashish Murolia Page 93

or IL = 24.2 - 0.5 = 23.7 mA


4.69 Option (D) is correct.
4.70 Option (B) is correct.
The small signal model is as shown below

Writing equation for I- have


e 0 - V- = I
-
1M
or e0 = I- (1M) + V- ...(1)
Writing equation for I+ we have From the figure we have
0 - V+ Zin = 2 MW
= I+
1M
and Z0 = rd RD = 20k 2k = 20 kW
or V+ = - I+ (1M) ...(2) 11
Since for ideal OPAMP V+ = V- , from (1) and (2) we have 4.71 Option (A) is correct.
e0 = I- (1M) - I + (1M) The circuit in DC condition is shown below
= (I- - I+) (1M) = IOS (1M) SPECIAL EDITION ( STUDY MATERIAL FORM )
Thus if e0 has been measured, we can calculate input offset current At market Book is available in 3 volume i.e. in 3 book binding
IOS only. form. But at NODIA Online Store book is available in 10 book
4.66 Option (C) is correct. binding form. Each unit of Book is in separate binding.
Available Only at NODIA Online Store
At low frequency capacitor is open circuit and voltage acr s non-
inverting terminal is zero. At high frequency capacitor act as short Click to Buy
circuit and all input voltage appear at non-inverting terminal. Thus, www.nodia.co.in
this is high pass circuit.
The frequency is given by
w = 1 = 1 = 1000
RC 1 # 10 # 1 # 10 - 6
3
rad/sec

4.67 Option (B) is correct.


The circuit under DC condition is shown in fig below

Since the FET has high input resistance, gate current can be neglect
and we get VGS =- 2 V
Since VP < VGS < 0 , FET is operating in active region
(- 2) 2
ID = IDSS c1 - VGS m = 10 c1 -
(- 8) m
2
Now
VP
= 5.625 mA

Now VDS = VDD - ID RD = 20 - 5.625 m # 2 k


Applying KVL we have
= 8.75 V
VCC - RB IB - VBE - RE IE = 0
4.72 Option (B) is correct.
or VCC - RB IB - VBE - RE (b + 1) IB = 0
The transconductance is
Since IE = IB + bIB 2
gm =
or IB = VCC - VBE VP ID IDSS
RB + (b + 1) RE
= 20 - 0.7 = 40m A or, = 2 5.625mA # 10mA = 1.875 mS
430k + (50 + 1)1 k 8
Now IC = bIB = 50 # 40m = 2 mA The gain is A =- gm (rd RD)
VC = VCC - RC IC = 20 - 2m # 2k = 16 V So, = 1.875ms # 20 K =- 3.41
11
4.68 Option (A) is correct. 4.73 Option (B) is correct.
The maximum load current will be at maximum input voltage i.e. Only one diode will be in ON conditions
Vmax = 30 V i.e. When lower diode is in ON condition, then
Vmax - VZ = I + I
1k L Z Vu = 2k Vsat = 2 10 = 8 V
2.5k 2.5
or 30 - 5.8 = I = 0.5 m when upper diode is in ON condition
L
1k
GATE Electronics and Communication Topicwise Solved Paper by RK Kanodia & Ashish Murolia Page 94

Vu = 2k Vsat = 2 (- 10) =- 5 V or 2V+ - Vo + IL R2 = 0 ...(2)


2.5k 4
Since V- = V+ , from (1) and (2) we have
4.74 Option (B) is correct. Vs + IL R2 = 0
An ideal OPAMP is an ideal voltage controlled voltage source.
or IL =- Vs
4.75 Option (C) is correct. R2
In voltage series feed back amplifier, input impedance increases 4.80 Option (D) is correct.
by factor (1 + Ab) and output impedance decreases by the factor If IZ is negligible the load current is
(1 + Ab). 12 - Vz = I
L
Rif = Ri (1 + Ab) R
Rof = Ro as per given condition
(1 + Ab)
100 mA # 12 - VZ # 500 mA
4.76 Option (A) is correct. R
This is a Low pass filter, because At IL = 100 mA 12 - 5 = 100 mA VZ = 5 V
R
At w = 3 V0 = 0
Vin or R = 70W

and at w = 0 V0 = 1 At IL = 500 mA 12 - 5 = 500 mA VZ = 5 V


Vin R
4.77 Option (D) is correct. or R = 14 W
When IC >> ICO Thus taking minimum we get
GATE Electronics & Communication R = 14 W
by RK Kanodia 4.81 Option (B) is correct.
Now in 3 Volume 4.82 Option (C) is correct.
Purchase Online at maximum discount from online store The Thevenin equivalent is shown below

and get POSTAL and Online Test Series Free


visit www.nodia.co.in
IC
gm = = 1mA = 0.04 = 40 mA/V
VT 25mV
b
rp = = 100 - 3 = 2.5 kW
gm 40 # 10
4.78 Option (A) is correct.
VT = R1 V = 1
The given circuit is wein bridge oscillator. The frequency of oscillation #5 = 1 V
R1 + R2 C 4+1
is Since b is large is large, IC . IE , IB . 0 and
2pf = 1
RC IE = VT - VBE = 1 - 0.7 = 3 mA
RE 300
or C = 1 = 1 = 1 m Now VCE = 5 - 2.2kIC - 300IE
2pRf 3
2p # 10 # 10 3 2p
4.79 Option (A) is correct. = 5 - 2.2k # 1m - 300 # 1m
The circuit is as shown below = 2.5 V
4.83 Option (B) is correct.
For the different combinations the table is as follows

For more GATE Resources, Mock Test and


Study material join the community
http://www.facebook.com/gateec2014

CE CE CC CB
We know that for ideal OPAMP Ai High High Unity
V- = V+ Av High Unity High
Applying KCL at inverting terminal Ri Medium High Low
V- - Vs + V- - V0 = 0 Ro Medium Low High
R1 R1
or 2V- - Vo = Vs ...(1) 4.84 Option (D) is correct.
Applying KCL at non-inverting terminal This circuit having two diode and capacitor pair in parallel, works
V+ V - Vo as voltage doubler.
+ IL + + =0
R2 R2 4.85 Option (B) is correct.
GATE Electronics and Communication Topicwise Solved Paper by RK Kanodia & Ashish Murolia Page 95

If the input is sinusoidal signal of 8 V (peak to peak) then or R2 = 2kW


Vi = 4 sin wt IB1 = IC1 = 1.5m = 0.01 mA
b1 150
The output of comparator will be high when input is higher than
In second case IB2 will we equal to IB1 as there is no in R1.
Vref = 2 V and will be low when input is lower than Vref = 2 V.
Thus IC2 = b2 IB2 = 200 # 0.01 = 2 mA
Thus the waveform for input is shown below
VCE2 = VCC - IC2 R2 = 6 - 2m # 2 kW = 2 V
4.90 Option (A) is correct.
The given circuit is a R - C phase shift oscillator and frequency of
its oscillation is
f = 1
2p 6 RC
4.91 Option (C) is correct.
If we see th figure we find that the voltage at non-inverting terminal
is 3 V by the zener diode and voltage at inverting terminal will be 3
V. Thus Vo can be get by applying voltage division rule, i.e.
20 V = 3
From fig, first crossover is at wt1 and second crossover is at wt2 20 + 40 o
where or V0 = 9 V
4 sin wt1 = 2V SPECIAL EDITION ( STUDY MATERIAL FORM )
Thus wt1 = sin - 1 1 = p At market Book is available in 3 volume i.e. in 3 book binding
2 6 form. But at NODIA Online Store book is available in 10 book
wt2 = p - p = 5p binding form. Each unit of Book is in separate binding.
6 6
Available Only at NODIA Online Store
5p p
-6
Duty Cycle = 6 =1 Click to Buy
2p 3
Thus the output of comparators has a duty cycle of 1 .
www.nodia.co.in
3
4.86 Option (C) is correct.
4.92 Option (B) is correct.
The circuit is as shown below
CMMR = Ad
Ac
or 20 log CMMR = 20 log Ad - 20 log Ac
= 48 - 2 = 46 dB
Where Ad "Differential Voltage Gain
and AC " Common Mode Voltage Gain
4.87 Option (B) is correct.
The gain of amplifier is
- gm V+ = 8 (3) = 8 kW
Ai = 1+8 3
gb + jwC
Thus the gain of a transistor amplifier falls at high frequencies V+ = V- = 8 V
3
due to the internal capacitance that are diffusion capacitance and Now applying KCL at inverting terminal we get
transition capacitance. V- - 2 + V- - Vo = 0
4.88 Option (A) is correct. 1 5
We have Ri = 1kW, b = 0.2, A = 50 or Vo = 6V- - 10
Thus, Rif = Ri
= 1 kW = 6 # 8 - 10 = 6 V
(1 + Ab) 11 3
4.89 Option (A) is correct. 4.93 Option (C) is correct.
The DC equivalent circuit is shown as below. This is fixed bias The equivalent circuit of 3 cascade stage is as shown in fig.
circuit operating in active region.

In first case V2 = 1k 50V1 = 40V1


1k + 0.25k
VCC - IC1 R2 - VCE1 = 0 1k
Similarly V3 = 50V2 = 40V2
or 6 - 1.5mR2 - 3 = 0 1k + 0.25k
GATE Electronics and Communication Topicwise Solved Paper by RK Kanodia & Ashish Murolia Page 96

or V3 = 40 # 40V1 VO (f)
Now from circuit A= = 1 + R2
Vo = 50V3 = 50 # 40 # 40V1 Vf (f) R1
V (f)
or AV = Vo = 50 # 40 # 40 = 8000 b (f) = 1 +0 = f
V1 6 VO (f)
or 20 log AV = 20 log 8000 = 98 dB Thus from above equation for sustained oscillation
4.94 Option (D) is correct. 6 = 1 + R2
R1
If a constant current is made to flow in a capacitor, the output
or R2 = 5R1
voltage is integration of input current and that is sawtooth waveform
as below : 4.99 Option (C) is correct.
t Let the gain of OPAMP be AV then we have
VC = 1 idt #
C 0 20 log AV = 40 dB
The time period of wave form is or AV = 100
T = 1 = 1 = 2 m sec Let input be Vi = Vm sin wt then we have
f 500
20 # 10 -3 VO = VV Vi = Vm sin wt
1
Thus 3= 6
2 # 10 0
idt # Now dVO = A V w cos wt
V m
dt
or i (2 # 10 - 3 - 0) = 6 # 10 - 6 dVO
Slew Rate c dt m = AV Vm w = AV Vm 2pf
or i = 3 mA max
Thus the charging require 3 mA current source for 2 msec. or Vm = SR
AV V2pf
GATE Electronics & Communication = -6
1
10 # 100 # 2p # 20 # 103
by RK Kanodia
or VM = 79.5 mV
Now in 3 Volume
4.100 Option (A) is correct.
Purchase Online at maximum discount from online store The circuit is shown as below
and get POSTAL and Online Test Series Free
visit www.nodia.co.in
4.95 Option (C) is correct.
In voltage-amplifier or voltage-series amplifier, the Ri increase and
Ro decrease because
Rif = Ri (1 + Ab) I = IZ + IL
Rof = Ro For satisfactory operations
(1 + Ab) Vin - V0 > IZ + IL [IZ + IL = I]
4.96 Option (B) is correct. R
Let x be the gain and it is 20 db, therefore When Vin = 30 V,
20 log x = 20 30 - 10 $ (10 + 1) mA
R
or x = 10
or 20 $ 11 mA
Since Gain band width product is 106 Hz, thus R
So, bandwidth is or R # 1818 W
6 6 when Vin = 50 V
BW = 10 = 10 = 105 Hz = 100 kHz
Gain 10 50 - 10 $ (10 + 1) mA
4.97 Option (A) is correct. R
In multistage amplifier bandwidth decrease and overall gain increase. For more GATE Resources, Mock Test and
From bandwidth point of view only options (A) may be correct Study material join the community
because lower cutoff frequency must be increases and higher must
http://www.facebook.com/gateec2014
be decreases. From following calculation we have
We have fL = 20 Hz and fH = 1 kHz
40 $ 11 # 10 - 3
For n stage amplifier the lower cutoff frequency is R
fL 20
f = Ln 1
= = 39.2 . 401
or R # 3636W
Hz 2n -1 23 -1 Thus R # 1818W
4.101 Option (D) is correct.
The higher cutoff frequency is We have
1
fHn = fH 2 2 - 1 = 0.5 kHz IDSS = 10 mA and VP =- 5 V
4.98 Option (A) is correct. Now VG =0
As per Barkhousen criterion for sustained oscillations Ab $ 1 and and VS = ID RS = 1 # 2.5W = 2.5 V
phase shift must be or 2pn . Thus VGS = VG - VS = 0 - 2.5 =- 2.5 V
GATE Electronics and Communication Topicwise Solved Paper by RK Kanodia & Ashish Murolia Page 97

Now gm = 2IDSS 81 - ` - 2.5 jB = 2 mS f = 1


VP -5 2p LCeq
AV = V0 =- gm RD Ceq = C1 C2 = 2 # 2 = 1 pF
Vi C1 + C2 4
f = 1
So, =- 2ms # 3k =- 6 2p 10 # 10 - 6 # 10 - 12
9
4.102 Option (C) is correct. = 1 # 10 = 50.3 MHz
2p 10
The current gain of a BJT is
4.108 Option (D) is correct.
hfe = gm rp
The circuit is as shown below
4.103 Option (A) is correct.
The ideal op-amp has following characteristic :
Ri " 3
R0 " 0
and A"3
4.104 Option (C) is correct.
Both statements are correct because
Let V- be the voltage of inverting terminal, since non inverting
(1) A stable multivibrator can be used for generating square wave,
terminal a at ground, the output voltage is
because of its characteristic
(2) Bi-stable multivibrator can store binary information, and this SPECIAL EDITION ( STUDY MATERIAL FORM )
multivibrator also give help in all digital kind of storing.
At market Book is available in 3 volume i.e. in 3 book binding
4.105 Option (B) is correct. form. But at NODIA Online Store book is available in 10 book
If fT is the frequency at which the short circuit common emitter gain
binding form. Each unit of Book is in separate binding.
attains unity magnitude then Available Only at NODIA Online Store
gm 38 # 10 - 3
fT =
2p (Cm + Cp)
= Click to Buy
2p # (10 - 14 + 4 # 10 - 13)
or = 1.47 # 1010 Hz www.nodia.co.in
If fB is bandwidth then we have Vo = AOL V- ...(1)
f 10
fB = T = 1.47 # 10 = 1.64 # 108 Hz Now applying KCL at inverting terminal we have
b 90
V- - Vs + V- - V0 = 0 ...(2)
4.106 Option (C) is correct. R1 R2
If we neglect current through RB then it can be open circuit as From (1) and (2) we have
shown in fig. VO = A = - R2
CL
Vs R - R2 + R1
ROL
Substituting the values we have
ACL = - 10k =- 1000 . - 11
1k - 10 k + 1 k 89
100k
4.109 Option (A) is correct.
The first OPAMP stage is the differentiator and second OPAMP
Maximum power will dissipate in Zener diode when current through
stage is integrator. Thus if input is cosine term, output will be also
it is maximum and it will occur at Vin = 30 V
cosine term. Only option (A) is cosine term. Other are sine term.
I = Vin - Vo = 30 - 10 = 1 A However we can calculate as follows. The circuit is shown in fig
20 20
I IC + IZ = bIB + IZ Since IC = bIB
= bIZ + IZ = (b + 1) IZ since IB = IZ
or IZ = I = 1 = 0.01 A
b+1 99 + 1
Power dissipated in zener diode is
PZ = VZ IZ = 9.5 # 0.01 = 95 mW
IC = bIZ = 99 # 0.1 = 0.99 A Applying KCL at inverting terminal of first OP AMP we have
VCE = Vo = 10 V V1 = - wjL = - 100 # 10 # 10 - 3 = - 1
Power dissipated in transistor is VS R 10 10
PT = VC IC = 10 # 0.99 = 9.9 W - jVS
or V1 = = j cos 100t
10
4.107 Option (B) is correct.
Applying KCL at inverting terminal of second OP AMP we have
From the it may be easily seen that the tank circuit is having
2-capacitors and one-inductor, so it is colpits oscillator and frequency VO = - 1/jwC
V1 100
is
=- 1 = j10
j100 # 10 # 10 - 6 # 100
GATE Electronics and Communication Topicwise Solved Paper by RK Kanodia & Ashish Murolia Page 98

or V0 = j10V2 = j10 (- j cos 100t)


V0 = 10 cos 100t
4.110 Option (A) is correct.
Common mode gain is
AC = aRC
REE
Since source resistance of the current source is infinite REE = 3 , R1 V = 5
VT = # 15 = 5 V
common mode gain AC = 0 R1 + R2 C 10 + 5
4.111 Option (D) is correct. Since b is large is large, IC . IE , IB . 0 and
In positive feed back it is working as OP-AMP in saturation region, IE = VT - VBE
RE
and the input applied voltage is +ve.
So, V0 =+ Vsat = 15 V = 5 - 0.7 = 4.3 = 10 mA
0.430kW 0.430KW
4.112 Option (C) is correct. 4.119 Option (C) is correct.
With the addition of RE the DC abis currents and voltages remain The output voltage will be input offset voltage multiplied by open
closer to the point where they were set by the circuit when the by open loop gain. Thus
outside condition such as temperature and transistor parameter b
So V0 = 5mV # 10, 000 = 50 V
change.
But V0 = ! 15 V in saturation condition
GATE Electronics & Communication So, it can never be exceeds ! 15 V
by RK Kanodia So, V0 = ! Vset = ! 15V
Now in 3 Volume 4.120 Option (A) is correct.
Purchase Online at maximum discount from online store 4.121 Option (A) is correct.
and get POSTAL and Online Test Series Free Negative feedback in amplifier reduces the gain of the system.
visit www.nodia.co.in 4.122 Option (A) is correct.
By drawing small signal equivalent circuit
4.113 Option (A) is correct.
At high frequency
gm
Ai =- '
+ jw (C)
gbc
or, Ai \ 1
Capacitance
and Ai a 1
frequency
Thus due to the transistor capacitance current gain of a bipolar
transistor drops.
by applying KCL at E2
4.114 Option (C) is correct.
As OP-AMP is ideal, the inverting terminal at virtual ground due Vp
gm1 Vp - 2
= gm2 Vp
1
rp 2

to ground at non-inverting terminal. Applying KCL at inverting


2

terminal at C2 i 0 =- gm2 Vp 2

sC (v1 sin wt - 0) + sC (V2 sin wt - 0) + sC (Vo - 0) = 0 from eq (1) and (2)


or Vo =- (V1 + V2) sin wt
4.115 Option (D) is correct.
For more GATE Resources, Mock Test and
There is R - C , series connection in parallel with parallel R - C Study material join the community
combination. So, it is a wein bridge oscillator because two resistors http://www.facebook.com/gateec2014
R1 and R2 is also in parallel with them.
gm1 Vp + i 0 =- i
4.116 Option (A) is correct. 1
gm2 rp 0
2
The given circuit is a differentiator, so the output of triangular wave
will be square wave. gm1 Vp =- i 0 :1 + 1 D
1
gm2 rp 2

4.117 Option (B) is correct. gm2 rp = b >> 1


2

In sampling and hold circuit the unity gain non-inverting amplifier so gm1 Vp =- i 0 1
is used. i 0 =- g
m1
4.118 Option (D) is correct. Vp 1

The Thevenin equivalent is shown below i0 = g a Vp = Vi


m1
Vi 1

4.123 Option (B) is correct.


Crossover behavior is characteristic of calss B output stage. Here 2
GATE Electronics and Communication Topicwise Solved Paper by RK Kanodia & Ashish Murolia Page 99

transistor are operated one for amplifying +ve going portion and 4.133 Option (C) is correct.
other for -ve going portion. In series voltage regulator the pass transistor is in common collec-
4.124 Option (C) is correct. tor configuration having voltage gain close to unity.
In Voltage series feedback mode input impedance is given by 4.134 Option (D) is correct.
R in = Ri (1 + bv Av) In bridge rectifier we do not need central tap transformer, so its less
where bv = feedback factor , expensive and smaller in size and its PIV (Peak inverse voltage)
Av = openloop gain is also greater than the two diode circuit, so it is also suitable for
higher voltage application.
and Ri = Input impedance
So, R in = 1 # 103 (1 + 0.99 # 100) = 100 kW
4.135 Option (C) is correct.
Similarly output impedance is given by In the circuit we have

ROUT = R0 R 0 = output impedance V2 = IS # RD


2
(1 + bv Av)
100 and V1 = IS # RD
Thus ROUT = = 1W
(1 + 0.99 # 100) V2 = 1
V1 2
4.125 Option (B) is correct.
V1 = 2V2
Regulation = Vno - load - Vfuel - load
Vfull - load 4.136 Option (C) is correct.
= 30 - 25 # 100 = 20% SPECIAL EDITION ( STUDY MATERIAL FORM )
25
At market Book is available in 3 volume i.e. in 3 book binding
Output resistance = 25 = 25 W
1 form. But at NODIA Online Store book is available in 10 book
4.126 Option (D) is correct. binding form. Each unit of Book is in separate binding.
This is a voltage shunt feedback as the feedback samples a portion Available Only at NODIA Online Store

of output voltage and convert it to current (shunt). Click to Buy


4.127 Option (A) is correct. www.nodia.co.in
In a differential amplifier CMRR is given by
(1 + b) IQ R 0 Option (C) is correct.
CMRR = 1 ;1 + E
4.137

2 VT b The equivalent circuit of given amplifier circuit (when CE is


So where R 0 is the emitter resistance. So CMRR can be improved connected, RE is short-circuited)
by increasing emitter resistance.
4.128 Option (C) is correct.
We know that rise time (tr ) is
tr = 0.35
fH
where fH is upper 3 dB frequency. Thus we can obtain upper 3 dB
Input impedance Ri = RB || r p
frequency it rise time is known.
Voltage gain AV = gm RC
4.129 Option (D) is correct. Now, if CE is disconnected, resistance RE appears in the circuit
In a BJT differential amplifier for a linear response Vid < VT .
4.130 Option (D) is correct.
In a shunt negative feedback amplifier.
Input impedance
R in = Ri
(1 + bA)
where Ri = input impedance of basic amplifier
b = feedback factor
Input impedance R in = RB || [rp + (b + 1)] RE
A = open loop gain
Input impedance increases
So, R in < Ri
gm RC
Similarly Voltage gain AV = Voltage gain decreases.
1 + gm R E
ROUT = R0 4.138 Option (A) is correct.
(1 + bA)
In common emitter stage input impedance is high, so in cascaded
ROUT < R 0
amplifier common emitter stage is followed by common base stage.
Thus input & output impedances decreases.
4.139 Option (C) is correct.
4.131 Option (A) is correct. We know that collect-emitter break down voltage is less than
4.132 Option (D) is correct. compare to collector base breakdown voltage.
Comparator will give an output either equal to + Vsupply or - Vsupply . BVCEO < BVCBO
So output is a square wave. both avalanche and zener break down. Voltage are higher than
GATE Electronics and Communication Topicwise Solved Paper by RK Kanodia & Ashish Murolia Page 100

BVCEO .So BVCEO limits the power supply. 2Va - 4 + Va - V0 = 0


4.140 Option (C) is correct. V0 = 3Va - 4
Va - V0 + Va - 0 = 0
100 10
Va - V0 + 10Va = 0
11Va = V0
Va = V0
11
If we assume consider the diode in reverse bias then Vn should be So V0 = 3V0 - 4
greater than VP . 11
VP < Vn 8V0 =- 4
11
by calculating
V0 =- 5.5 Volts
VP = 10 # 4 = 5 Volt
4+4 4.142 Option (B) is correct.
Vn = 2 # 1 = 2 Volt Circuit with diode forward resistance looks
here VP > Vn (so diode cannot be in reverse bias mode).

GATE Electronics & Communication


by RK Kanodia
So the DC current will
Now in 3 Volume Vm
IDC =
Purchase Online at maximum discount from online store p (R f + RL)
and get POSTAL and Online Test Series Free 4.143 Option (D) is correct.
visit www.nodia.co.in For the positive half cycle of input diode D1 will conduct & D2 will
be off. In negative half cycle of input D1 will be off & D2 conduct so
output voltage wave from across resistor (10 kW) is –

Ammeter will read rms value of current


so I rms = Vm (half wave rectifier)
pR
= 4 = 0.4 mA
apply node equation at node a (10 kW) p p
Va - 10 + Va + Va = 2
4 4 1
4.144 Option (D) is correct.
In given circuit positive feedback is applied in the op-amp., so it
6Va - 10 = 8
works as a Schmitt trigger.
Va = 3 Volt
4.145 Option (D) is correct.
so current Ib = 0 - 3 + 10 - 3 For more GATE Resources, Mock Test and
4 4
10 - 6
Ib =
4
= 1 amp Study material join the community
4.141 Option (D) is correct.
http://www.facebook.com/gateec2014
By applying node equation at terminal (2) and (3) of OP -amp Gain with out feedback factor is given by
V0 = kVi
after connecting feedback impedance Z

given input impedance is very large, so after connecting Z we have


Va - Q Va - V0
5
+
10
=0 Ii = Vi - V0 V0 = kVi
Z
GATE Electronics and Communication Topicwise Solved Paper by RK Kanodia & Ashish Murolia Page 101

Ii = Vi - kVi
Z
input impedance Zin = Vi = Z
Ii (1 - k)
4.146 Option (A) is correct.
4.147 Option (A) is correct.
For the circuit, In balanced condition It will oscillated at a frequency
w
= 1 = 1 = 105 rad/ sec
-3 -6
LC 10 # 10 # .01 # 10
In this condition
R1 = R 3
R2 R4
5 =R
100 1
R = 20 kW = 2 # 10 4 W
4.148 Option (C) is correct.
V0 kept constant at V0 = 6 volt
so current in 50 W resistor SPECIAL EDITION ( STUDY MATERIAL FORM )
I = 9-6 At market Book is available in 3 volume i.e. in 3 book binding
50 W form. But at NODIA Online Store book is available in 10 book
I = 60 m amp binding form. Each unit of Book is in separate binding.
Maximum allowed power dissipation in zener Available Only at NODIA Online Store
PZ = 300 mW Click to Buy
Maximum current allowed in zener
www.nodia.co.in
PZ = VZ (IZ ) max = 300 # 10-3
& = 6 (IZ ) max = 300 # 10-3
& = (IZ ) max = 50 m amp
Given knee current or minimum current in zener
(IZ ) min = 5 m amp
In given circuit I = IZ + I L
I L = I - IZ
(IL) min = I - (IZ ) max
= (60 - 50) m amp = 10 m amp
(IL) max = I - (IZ ) min
= (60 - 5) = 55 m amp
GATE Electronics and Communication Topicwise Solved Paper by RK Kanodia & Ashish Murolia Page 101

UNIT 5
2012 ONE MARK

5.4 Consider the given circuit

DIGITAL CIRCUITS

2013 ONE MARK


In this circuit, the race around
5.1 A bulb in a staircase has two switches, one switch being at the (A) does not occur
ground floor and the other one at the first floor. The bulb can be
(B) occur when CLK = 0
turned ON and also can be turned OFF by any one of the switches
irrespective of the state of the other switch. The logic of switching (C) occur when CLK = 1 and A = B = 1
of the bulb resembles (D) occur when CLK = 1 and A = B = 0
(A) and AND gate (B) an OR gate
5.5 The output Y of a 2-bit comparator is logic 1 whenever the 2-bit input
(C) an XOR gate (D) a NAND gate A is greater than the 2-bit input B . The number of combinations for
5.2 For 8085 microprocessor, the following program is executed. which the output is logic 1, is
MVI A, 05H;
MVI B, 05H; SPECIAL EDITION ( STUDY MATERIAL FORM )
PTR: ADD B; At market Book is available in 3 volume i.e. in 3 book binding
DCR B; form. But at NODIA Online Store book is available in 10 book
JNZ PTR; binding form. Each unit of Book is in separate binding.
ADI 03H; Available Only at NODIA Online Store
HLT; Click to Buy
At the end of program, accumulator contains
(A) 17H (B) 20H www.nodia.co.in
(C) 23H (D) 05H
(A) 4 (B) 6
(C) 8 (D) 10
2013 TWO MARKS
5.6 In the circuit shown
5.3 There are four chips each of 1024 bytes connected to a 16 bit address
bus as shown in the figure below, RAMs 1, 2, 3 and 4 respectively
are mappped to addresses

(A) Y = A B + C (B) Y = (A + B) C
(C) Y = (A + B ) C (D) Y = AB + C

5.7 /
In the sum of products function f (X, Y, Z) = (2, 3, 4, 5), the prime
implicants are
(A) XY, XY (B) XY, X Y Z , XY Z
(C) XY Z , XYZ, XY (D) XY Z , XYZ, XY Z , XY Z

2012 TWO MARKS

5.8 In the CMOS circuit shown, electron and hole mobilities are equal,
(A) 0C00H-0FFFH, 1C00H-1FFFH, 2C00H-2FFFH, 3C00H- and M1 and M2 are equally sized. The device M1 is in the linear
3FFFH region if
(B) 1800H-1FFFH, 2800H-2FFFH, 3800H-3FFFH, 4800H-4FFFH
(C) 0500H-08FFH, 1500H-18FFH, 3500H-38FFH, 5500H-58FFH
(D) 0800H-0BFFH, 1800H-1BFFH, 2800H-2BFFH, 3800H-3BFFH
GATE Electronics and Communication Topicwise Solved Paper by RK Kanodia & Ashish Murolia Page 102

(A) changed from “0” to “1” (B) changed from “1” to “0”
(A) Vin < 1.875 V (B) 1.875 V < Vin < 3.125 V
(C) changed in either direction (D) not changed
(C) Vin > 3.125 V (D) 0 < Vin < 5 V
5.12 The logic function implemented by the circuit below is (ground
5.9 The state transition diagram for the logic circuit shown is implies a logic “0”)

GATE Electronics & Communication (A) F = AND ^P, Q h (B) F = OR ^P, Q h


by RK Kanodia (C) F = XNOR ^P, Q h (D) F = XOR ^P, Q h
Now in 3 Volume
Purchase Online at maximum discount from online store 2011 TWO MARKS
and get POSTAL and Online Test Series Free 5.13 The output of a 3-stage Johnson (twisted ring) counter is fed to
visit www.nodia.co.in a digital-to analog (D/A) converter as shown in the figure below.
Assume all states of the counter to be unset initially. The waveform
which represents the D/A converter output Vo is

2011 ONE MARK

5.10 The output Y in the circuit below is always ‘1’ when

For more GATE Resources, Mock Test and


Study material join the community
http://www.facebook.com/gateec2014

(A) two or more of the inputs P, Q, R are ‘0’


(B) two or more of the inputs P, Q, R are ‘1’
(C) any odd number of the inputs P, Q, R is ‘0’
(D) any odd number of the inputs P, Q, R is ‘1’

5.11 When the output Y in the circuit below is “1”, it implies that data
5.14 Two D flip-flops are connected as a synchronous counter that goes
has
through the following QB QA sequence 00 " 11 " 01 " 10 " 00 " ....
The connections to the inputs DA and DB are
(A) DA = QB, DB = QA
GATE Electronics and Communication Topicwise Solved Paper by RK Kanodia & Ashish Murolia Page 103

(B) DA = Q A, DB = Q B (A) A = 1, B = 1, C = 0 (B) A = 1, B = 0, C = 0


(C) DA = (QA Q B + Q A QB), DB = QA (C) A = 0, B = 1, C = 0 (D) A = 0, B = 0, C = 1
(D) DA = (QA QB + Q A Q B), DB = Q B

5.15 An 8085 assembly language program is given below. Assume that 2010 TWO MARKS
the carry flag is initially unset. The content of the accumulator after 5.19 Assuming that the flip-flop are in reset condition initially, the count
the execution of the program is sequence observed at QA , in the circuit shown is

(A) 8CH (B) 64H


(C) 23H (D) 15H (A) 0010111... (B) 0001011...
(C) 0101111... (D) 0110100....
2010 ONE MARK

5.16 Match the logic gates in Column A with their equivalents in Column SPECIAL EDITION ( STUDY MATERIAL FORM )
B At market Book is available in 3 volume i.e. in 3 book binding
form. But at NODIA Online Store book is available in 10 book
binding form. Each unit of Book is in separate binding.
Available Only at NODIA Online Store

Click to Buy
www.nodia.co.in
5.20 The Boolean function realized by the logic circuit shown is

(A) P-2, Q-4, R-1, S-3 (B) P-4, Q-2, R-1, S-3
(C) P-2, Q-4, R-3, S-1 (D) P-4, Q-2, R-3, S-1
(A) F = Sm (0, 1, 3, 5, 9, 10, 14) (B) F = Sm (2, 3, 5, 7, 8, 12, 13)
5.17 In the circuit shown, the device connected Y5 can have address in
the range (C) F = Sm (1, 2, 4, 5, 11, 14, 15) (D) F = Sm (2, 3, 5, 7, 8, 9, 12)

5.21 For the 8085 assembly language program given below, the content of
the accumulator after the execution of the program is

(A) 00H (B) 45H


(C) 67H (D) E7H

(A) 2000 - 20FF (B) 2D00 - 2DFF


2009 ONE MARK
(C) 2E00 - 2EFF (D) FD00 - FDFF
5.22 The full form of the abbreviations TTL and CMOS in reference to
5.18 For the output F to be 1 in the logic circuit shown, the input logic families are
combination should be (A) Triple Transistor Logic and Chip Metal Oxide Semiconductor
(B) Tristate Transistor Logic and Chip Metal Oxide Semiconduc-
tor
(C) Transistor Transistor Logic and Complementary Metal Oxide
Semiconductor
(D) Tristate Transistor Logic and Complementary Metal Oxide
GATE Electronics and Communication Topicwise Solved Paper by RK Kanodia & Ashish Murolia Page 104

Silicon (1) push buttons pressed/not pressed in equivalent to logic 1/0


respectively.
5.23 In a microprocessor, the service routine for a certain interrupt starts
(2) a segment glowing/not glowing in the display is equivalent to
from a fixed location of memory which cannot be externally set, but
logic 1/0 respectively.
the interrupt can be delayed or rejected Such an interrupt is
(A) non-maskable and non-vectored 5.27 If segments a to g are considered as functions of P1 and P2 , then
(B) maskable and non-vectored which of the following is correct
(C) non-maskable and vectored (A) g = P 1 + P2, d = c + e (B) g = P1 + P2, d = c + e
(D) maskable and vectored (C) g = P1 + P2, e = b + c (D) g = P1 + P2, e = b + c

5.28 What are the minimum numbers of NOT gates and 2 - input OR
2009 TWO MARKS gates required to design the logic of the driver for this 7 - Segment
display
5.24 If X = 1 in logic equation 6X + Z {Y + (Z + XY )}@ {X + X (X + Y)} = 1 (A) 3 NOT and 4 OR
, then
(B) 2 NOT and 4 OR
(A) Y = Z (B) Y = Z
(C) 1 NOT and 3 OR
(C) Z = 1 (D) Z = 0
(D) 2 NOT and 3 OR
5.25 What are the minimum number of 2- to -1 multiplexers required to
generate a 2- input AND gate and a 2- input Ex-OR gate 5.29 Refer to the NAND and NOR latches shown in the figure. The
(A) 1 and 2 (B) 1 and 3 inputs (P1, P2) for both latches are first made (0, 1) and then, after a
few seconds, made (1, 1). The corresponding stable outputs (Q1, Q2)
GATE Electronics & Communication are
by RK Kanodia
Now in 3 Volume
Purchase Online at maximum discount from online store
and get POSTAL and Online Test Series Free
visit www.nodia.co.in
(A) NAND: first (0, 1) then (0, 1) NOR: first (1, 0) then (0, 0)
(C) 1 and 1 (D) 2 and 2
(B) NAND : first (1, 0) then (1, 0) NOR : first (1, 0) then (1, 0)
5.26 What are the counting states (Q1, Q2) for the counter shown in the (C) NAND : first (1, 0) then (1, 0) NOR : first (1, 0) then (0, 0)
figure below
(D) NAND : first (1, 0) then (1, 1) NOR : first (0, 1) then (0, 1)

2008 TWO MARKS

5.30 The logic function implemented by the following circuit at the


terminal OUT is

(A) 11, 10, 00, 11, 10,... (B) 01, 10, 11, 00, 01...
(C) 00, 11, 01, 10, 00... (D) 01, 10, 00, 01, 10...

Statement for Linked Answer Question 5.18 & 5.19 :


Two products are sold from a vending machine, which has two
push buttons P1 and P2 . For more GATE Resources, Mock Test and
When a buttons is pressed, the price of the corresponding product Study material join the community
is displayed in a 7 - segment display. If no buttons are pressed, '0' http://www.facebook.com/gateec2014
is displayed signifying ‘Rs 0’.
If only P1 is pressed, ‘2’ is displayed, signifying ‘Rs. 2’ (A) P NOR Q (B) P NAND Q
If only P2 is pressed ‘5’ is displayed, signifying ‘Rs. 5’ (C) P OR Q (D) P AND Q
If both P1 and P2 are pressed, 'E' is displayed, signifying ‘Error’
5.31 The two numbers represented in signed 2’s complement form are
The names of the segments in the 7 - segment display, and the P + 11101101 and Q = 11100110 . If Q is subtracted from P , the
glow of the display for ‘0’, ‘2’, ‘5’ and ‘E’ are shown below. value obtained in signed 2’s complement is
(A) 1000001111
(B) 00000111
(C) 11111001
(D) 111111001
Consider 5.32 Which of the following Boolean Expressions correctly represents the
GATE Electronics and Communication Topicwise Solved Paper by RK Kanodia & Ashish Murolia Page 105

relation between P, Q, R and M1 3


per the relation VDAC = / 2n - 1 bn Volts, where b3 (MSB), b1, b2 and
b0 (LSB) are the countern =outputs.
0 The counter starts from the
clear state.

(A) M1 = (P OR Q) XOR R
(B) M1 = (P AND Q) X OR R
(C) M1 = (P NOR Q) X OR R
(D) M1 = (P XOR Q) XOR R

5.33 For the circuit shown in the figure, D has a transition from 0 to 1
after CLK changes from 1 to 0. Assume gate delays to be negligible
Which of the following statements is true

5.35 The stable reading of the LED displays is


SPECIAL EDITION ( STUDY MATERIAL FORM )
At market Book is available in 3 volume i.e. in 3 book binding
form. But at NODIA Online Store book is available in 10 book
binding form. Each unit of Book is in separate binding.
(A) Q goes to 1 at the CLK transition and stays at 1 Available Only at NODIA Online Store

(B) Q goes to 0 at the CLK transition and stays 0 Click to Buy


(C) Q goes to 1 at the CLK tradition and goes to 0 when D goes www.nodia.co.in
to 1
(D) Q goes to 0 at the CLK transition and goes to 1 when D goes (A) 06 (B) 07
to 1 (C) 12 (D) 13

5.34 For each of the positive edge-triggered J - K flip flop used in the 5.36 The magnitude of the error between VDAC and Vin at steady state in
following figure, the propagation delay is 3 t . volts is
(A) 0.2 (B) 0.3
(C) 0.5 (D) 1.0

5.37 For the circuit shown in the following, I0 - I3 are inputs to the 4:1
multiplexers, R(MSB) and S are control bits.
The output Z can be represented by

Which of the following wave forms correctly represents the output


at Q1 ?

(A) PQ + PQS + QRS


(B) PQ + PQR + PQS
(C) PQR + PQR + PARS + QRS
(D) PQR + PQRS + PQRS + QRS

5.38 An 8085 executes the following instructions


2710 LXI H, 30A0 H
2713 DAD H
2414 PCHL
All address and constants are in Hex. Let PC be the contents of
the program counter and HL be the contents of the HL register
Statement For Linked Answer Question 5.26 & 5.27 : pair just after executing PCHL. Which of the following statements
In the following circuit, the comparators output is logic “1” if is correct ?
V1 > V2 and is logic "0" otherwise. The D/A conversion is done as PC = 2715H PC = 30A0H
(A) (B)
HL = 30A0H HL = 2715H
GATE Electronics and Communication Topicwise Solved Paper by RK Kanodia & Ashish Murolia Page 106

(C) Q1: normal active; Q2: cut-off; Q3: cut-off; Q4: saturation
PC = 6140H PC = 6140H
(C) (D) (D) Q1: saturation; Q2: saturation; Q3: saturation; Q4: normal active
HL = 6140H HL = 2715H
5.44 The following binary values were applied to the X and Y inputs of
2007 ONE MARK NAND latch shown in the figure in the sequence indicated below :
X = 0,Y = 1; X = 0, Y = 0; X = 1; Y = 1
5.39 X = 01110 and Y = 11001 are two 5-bit binary numbers represented The corresponding stable P, Q output will be.
in two’s complement format. The sum of X and Y represented in
two’s complement format using 6 bits is
(A) 100111 (B) 0010000
(C) 000111 (D) 101001

5.40 The Boolean function Y = AB + CD is to be realized using only 2 -


input NAND gates. The minimum number of gates required is
(A) P = 1, Q = 0; P = 1, Q = 0; P = 1, Q = 0 or P = 0, Q = 1
(A) 2 (B) 3
(B) P = 1, Q = 0; P = 0, Q = 1; or P = 0, Q = 1; P = 0, Q = 1
(C) 4 (D) 5
(C) P = 1, Q = 0; P = 1, Q = 1; P = 1, Q = 0 or P = 0, Q = 1
(D) P = 1, Q = 0; P = 1, Q = 1; P = 1, Q = 1
2007 TWO MARKS
5.45 An 8255 chip is interfaced to an 8085 microprocessor system as an
5.41 The Boolean expression Y = ABC D + ABCD + ABC D + ABC D
I/O mapped I/O as show in the figure. The address lines A0 and A1
can be minimized to
of the 8085 are used by the 8255 chip to decode internally its thee
GATE Electronics & Communication ports and the Control register. The address lines A3 to A7 as well
by RK Kanodia as the IO/M signal are used for address decoding. The range of
addresses for which the 8255 chip would get selected is
Now in 3 Volume
Purchase Online at maximum discount from online store
and get POSTAL and Online Test Series Free
visit www.nodia.co.in
(A) Y = ABC D + ABC + AC D
(A) F8H - FBH (B) F8GH - FCH
(B) Y = ABC D + BCD + ABC D
(C) F8H - FFH (D) F0H - F7H
(C) Y = ABCD + BC D + ABC D
(D) Y = ABCD + BC D + ABC D
Statement for Linked Answer Question 5.37 and 5.38 :
5.42 In the following circuit, X is given by
In the Digital-to-Analog converter circuit shown in the figure below,
VR = 10V and R = 10kW

(A) X = ABC + ABC + ABC + ABC


(B) X = ABC + ABC + ABC + ABC
(C) X = AB + BC + AC
(D) X = AB + BC + AC
For more GATE Resources, Mock Test and
5.43 The circuit diagram of a standard TTL NOT gate is shown in the Study material join the community
figure. Vi = 25 V, the modes of operation of the transistors will be http://www.facebook.com/gateec2014
5.46 The current is
(A) 31.25mA (B) 62.5mA
(C) 125mA (D) 250mA

5.47 The voltage V0 is


(A) - 0.781 V (B) - 1.562 V
(C) - 3.125 V (D) - 6.250 V

Statement for Linked Answer Questions 5.39 & 5.40 :


An 8085 assembly language program is given below.
(A) Q1: revere active; Q2: normal active; Q3: saturation; Q4: cut-off
Line 1: MVI A, B5H
(B) Q1: revere active; Q2: saturation; Q3: saturation; Q4: cut-off
GATE Electronics and Communication Topicwise Solved Paper by RK Kanodia & Ashish Murolia Page 107

2: MVI B, OEH - out shift registers loaded with the data shown are used to feed the
3: XRI 69H data to a full adder. Initially, all the flip - flops are in clear state.
4: ADD B After applying two clock pulse, the output of the full-adder should
5: ANI 9BH be
6: CPI 9FH
7: STA 3010H
8: HLT
5.48 The contents of the accumulator just execution of the ADD
instruction in line 4 will be
(A) C3H (B) EAH
(C) DCH (D) 69H

5.49 After execution of line 7 of the program, the status of the CY and
(A) S = 0, C0 = 0 (B) S = 0, C0 = 1
Z flags will be
(A) CY = 0, Z = 0 (B) CY = 0, Z = 1 (C) S = 1, C0 = 0 (D) S = 1, C0 = 1
(C) CY = 1, Z = 0 (D) CY = 1, Z = 1 5.54 A new Binary Coded Pentary (BCP) number system is proposed
in which every digit of a base-5 number is represented by its
5.50 For the circuit shown, the counter state (Q1 Q0) follows the sequence
corresponding 3-bit binary code. For example, the base-5 number

SPECIAL EDITION ( STUDY MATERIAL FORM )


At market Book is available in 3 volume i.e. in 3 book binding
form. But at NODIA Online Store book is available in 10 book
binding form. Each unit of Book is in separate binding.
Available Only at NODIA Online Store

Click to Buy
(A) 00, 01, 10, 11, 00 (B) 00, 01, 10, 00, 01
www.nodia.co.in
(C) 00, 01, 11, 00, 01 (D) 00, 10, 11, 00, 10
24 will be represented by its BCP code 010100. In this numbering
system, the BCP code 10001001101 corresponds of the following
2006 ONE MARK number is base-5 system
(A) 423 (B) 1324
5.51 The number of product terms in the minimized sum-of-product
expression obtained through the following K - map is (where, "d" (C) 2201 (D) 4231
denotes don’t care states)
5.55 A 4 - bit D/A converter is connected to a free - running 3 - big UP
counter, as shown in the following figure. Which of the following
waveforms will be observed at V0 ?

(A) 2 (B) 3
(C) 4 (D) 5

2006 TWO MARKS


In the figure shown above, the ground has been shown by the sym-
5.52 An I/O peripheral device shown in Fig. (b) below is to be interfaced bol 4
to an 8085 microprocessor. To select the I/O device in the I/O
address range D4 H - D7 H, its chip-select (CS ) should be connected
to the output of the decoder shown in as below :

5.56 Following is the segment of a 8085 assembly language program


LXI SP, EFFF H
CALL 3000 H
:
(A) output 7 (B) output 5 :
(C) output 2 (D) output 0 :
3000 H LXI H, 3CF4
5.53 For the circuit shown in figures below, two 4 - bit parallel - in serial
GATE Electronics and Communication Topicwise Solved Paper by RK Kanodia & Ashish Murolia Page 108

PUSH PSW 2005 TWO MARKS


SPHL 5.61 The transistors used in a portion of the TTL gate show in the figure
POP PSW have b = 100 . The base emitter voltage of is 0.7 V for a transistor
RET in active region and 0.75 V for a transistor in saturation. If the sink
On completion of RET execution, the contents of SP is current I = 1 A and the output is at logic 0, then the current IR will
(A) 3CF0 H (B) 3CF8 H be equal to
(C) EFFD H (D) EFFF H

5.57 Two D - flip - flops, as shown below, are to be connected as a


synchronous counter that goes through the following sequence
00 " 01 " 11 " 10 " 00 " ...
The inputs D0 and D1 respectively should be connected as,

(A) 0.65 mA (B) 0.70 mA


(C) 0.75 mA (D) 1.00 mA
(A) Q 1 and Q0 (B) Q 0 and Q1
(C) Q1 Q0 and Q 1 Q0 (D) Q 1 Q 0 and Q1 Q0 5.62 The Boolean expression for the truth table shown is

GATE Electronics & Communication


by RK Kanodia
Now in 3 Volume
Purchase Online at maximum discount from online store
and get POSTAL and Online Test Series Free
visit www.nodia.co.in

5.58 The point P in the following figure is stuck at 1. The output f will (A) B (A + C)( A + C ) (B) B (A + C )( A + C)
be
(C) B (A + C )( A + C) (D) B (A + C)( A + C )

5.63 The present output Qn of an edge triggered JK flip-flop is logic 0.


If J = 1, then Qn + 1
(A) Cannot be determined (B) Will be logic 0
(C) will be logic 1 (D) will rave around
(A) ABC (B) A
5.64 The given figure shows a ripple counter using positive edge triggered
(C) ABC (D) A flip-flops. If the present state of the counter is Q2 Q1 Q0 = 001 then is
next state Q2 Q1 Q will be
2005 ONE MARK

5.59 Decimal 43 in Hexadecimal and BCD number system is respectively


(A) B2, 0100 011 (B) 2B, 0100 0011
(C) 2B, 0011 0100 (D) B2, 0100 0100

5.60 The Boolean function f implemented in the figure using two input
For more GATE Resources, Mock Test and
multiplexes is Study material join the community
http://www.facebook.com/gateec2014

(A) 010 (B) 111


(A) ABC + ABC (B) ABC + ABC (C) 100 (D) 101
(C) ABC + ABC (D) ABC + ABC
5.65 What memory address range is NOT represents by chip # 1 and
chip # 2 in the figure A0 to A15 in this figure are the address lines
and CS means chip select.
GATE Electronics and Communication Topicwise Solved Paper by RK Kanodia & Ashish Murolia Page 109

(C) 5 (D) 7

5.71 Choose the correct one from among the alternatives A, B, C, D after
matching an item from Group 1 most appropriate item in Group 2.
Group 1 Group 2
P. Shift register 1. Frequency division
Q. Counter 2. Addressing in memory chips
R. Decoder 3. Serial to parallel data conversion
(A) P - 3, Q - 2, R - 1 (B) P - 3, Q - 1, R - 2
(C) P - 2, Q - 1, R - 3 (D) P - 1, Q - 2, R - 2

5.72 The figure the internal schematic of a TTL AND-OR-OR-Invert


(AOI) gate. For the inputs shown in the figure, the output Y is

(A) 0100 - 02FF (B) 1500 - 16FF (A) 0 (B) 1


(C) F900 - FAFF (D) F800 - F9FF (C) AB (D) AB

SPECIAL EDITION ( STUDY MATERIAL FORM )


Statement For Linked Answer Questions 5.57 & 5.58 : At market Book is available in 3 volume i.e. in 3 book binding
Consider an 8085 microprocessor system. form. But at NODIA Online Store book is available in 10 book
5.66 The following program starts at location 0100H. binding form. Each unit of Book is in separate binding.
Available Only at NODIA Online Store
LXI SP, OOFF
LXI H, 0701 Click to Buy
MVI A, 20H www.nodia.co.in
SUB M
2004 TWO MARKS
The content of accumulator when the program counter reaches
0109 H is 5.73 11001, 1001, 111001 correspond to the 2’s complement representation
(A) 20 H (B) 02 H of which one of the following sets of number
(C) 00 H (D) FF H (A) 25,9, and 57 respectively (B) -6, -6, and -6 respectively
(C) -7, -7 and -7 respectively (D) -25, -9 and -57 respectively
5.67 If in addition following code exists from 019H onwards,
ORI 40 H 5.74 In the modulo-6 ripple counter shown in figure, the output of the
ADD M 2- input gate is used to clear the J-K flip-flop
What will be the result in the accumulator after the last instruc- The 2-input gate is
tion is executed ?
(A) 40 H (B) 20 H
(C) 60 H (D) 42 H

2004 ONE MARK

5.68 A master - slave flip flop has the characteristic that


(A) change in the output immediately reflected in the output
(B) change in the output occurs when the state of the master is (A) a NAND gate (B) a NOR gate
affected
(C) an OR gate (D) a AND gare
(C) change in the output occurs when the state of the slave is af-
fected 5.75 The minimum number of 2- to -1 multiplexers required to realize a
(D) both the master and the slave states are affected at the same 4- to -1 multiplexers is
time (A) 1 (B) 2
(C) 3 (D) 4
5.69 The range of signed decimal numbers that can be represented by
6-bits 1’s complement number is 5.76 The Boolean expression AC + BC is equivalent to
(A) -31 to +31 (B) -63 to +63 (A) AC + BC + AC (B) BC + AC + BC + ACB
(C) -64 to +63 (D) -32 to +31 (C) AC + BC + BC + ABC (D) ABC + ABC + ABC + ABC

5.70 A digital system is required to amplify a binary-encoded audio 5.77 A Boolean function f of two variables x and y is defined as follows :
signal. The user should be able to control the gain of the amplifier f (0, 0) = f (0, 1) = f (1, 1) = 1; f (1, 0) = 0
from minimum to a maximum in 100 increments. The minimum Assuming complements of x and y are not available, a minimum
number of bits required to encode, in straight binary, is
cost solution for realizing f using only 2-input NOR gates and 2-
(A) 8 (B) 6
GATE Electronics and Communication Topicwise Solved Paper by RK Kanodia & Ashish Murolia Page 110

input OR gates (each having unit cost) would have a total cost of (A) JNX LOOP, ADD B, DCR C
(A) 1 unit (B) 4 unit (B) ADD B, JNZ LOOP, DCR C
(C) 3 unit (D) 2 unit (C) DCR C, JNZ LOOP, ADD B
5.78 The 8255 Programmable Peripheral Interface is used as described (D) ADD B, DCR C, JNZ LOOP
below.
(i) An A/D converter is interface to a microprocessor through an 2003 ONE MARK
8255.
The conversion is initiated by a signal from the 8255 on Port C. A 5.82 The number of distinct Boolean expressions of 4 variables is
signal on Port C causes data to be stobed into Port A. (A) 16 (B) 256
(ii) Two computers exchange data using a pair of 8255s. Port A (C) 1023 (D) 65536
works as a bidirectional data port supported by appropriate hand-
shaking signals. 5.83 The minimum number of comparators required to build an 8-bits
The appropriate modes of operation of the 8255 for (i) and (ii) flash ADC is
would be (A) 8 (B) 63
(A) Mode 0 for (i) and Mode 1 for (ii) (C) 255 (D) 256
(B) Mode 1 for (i) and Mode 2 for (ii) 5.84 The output of the 74 series of GATE of TTL gates is taken from a
(C) Mode for (i) and Mode 0 for (ii) BJT in
(D) Mode 2 for (i) and Mode 1 for (ii) (A) totem pole and common collector configuration
(B) either totem pole or open collector configuration
GATE Electronics & Communication (C) common base configuration
by RK Kanodia (D) common collector configuration
Now in 3 Volume 5.85 Without any additional circuitry, an 8:1 MUX can be used to obtain
Purchase Online at maximum discount from online store (A) some but not all Boolean functions of 3 variables
and get POSTAL and Online Test Series Free (B) all functions of 3 variables but non of 4 variables
visit www.nodia.co.in (C) all functions of 3 variables and some but not all of 4 variables
(D) all functions of 4 variables
5.79 The number of memory cycles required to execute the following
8085 instructions 5.86 A 0 to 6 counter consists of 3 flip flops and a combination circuit of
(i) LDA 3000 H 2 input gate (s). The common circuit consists of
(ii) LXI D, FOF1H (A) one AND gate
would be (B) one OR gate
(A) 2 for (i) and 2 for (ii) (B) 4 for (i) and 3 for (ii) (C) one AND gate and one OR gate
(C) 3 for (i) and 3 for (ii) (D) 3 for (i) and 4 for (ii) (D) two AND gates
5.80 Consider the sequence of 8085 instructions given below
LXI H, 9258 2003 TWO MARKS
MOV A, M
CMA 5.87 The circuit in the figure has 4 boxes each described by inputs P, Q, R
MOV M, A and outputs Y, Z with Y = P 5 Q 5 R and Z = RQ + PR + QP
Which one of the following is performed by this sequence ? The circuit acts as a
(A) Contents of location 9258 are moved to the accumulator
(B) Contents of location 9258 are compared with the contents of
the accumulator For more GATE Resources, Mock Test and
(C) Contents of location 8529 are complemented and stored in Study material join the community
location 8529
http://www.facebook.com/gateec2014
(D) Contents of location 5892 are complemented and stored in
location 5892

5.81 It is desired to multiply the numbers 0AH by 0BH and store the
result in the accumulator. The numbers are available in registers B
and C respectively. A part of the 8085 program for this purpose is
given below :
MVI A, 00H
LOOP ------
------
-----
HLT
END (A) 4 bit adder giving P + Q
The sequence of instructions to complete the program would be (B) 4 bit subtractor giving P - Q
GATE Electronics and Communication Topicwise Solved Paper by RK Kanodia & Ashish Murolia Page 111

(C) 4 bit subtractor giving Q-P Propagation delay is CMOS ECL TTL TTL
(D) 4 bit adder giving P + Q + R minimum
5.88 If the function W, X, Y and Z are as follows The correct column is
W = R + PQ + RS X = PQRS + PQRS + PQRS (A) P (B) Q
Y = RS + PR + PQ + P .Q (C) R (D) S
Z = R + S + PQ + P .Q .R + PQ .S
Then, 5.92 The circuit shown in figure converts
(A) W = Z, X = Z (B) W = Z, X = Y
(C) W = Y (D) W = Y = Z

5.89 A 4 bit ripple counter and a bit synchronous counter are made using
flip flops having a propagation delay of 10 ns each. If the worst case
delay in the ripple counter and the synchronous counter be R and
S respectively, then
(A) R = 10 ns, S = 40 ns (B) R = 40 ns, S = 10 ns
(C) R = 10 ns S = 30 ns (D) R = 30 ns, S = 10 ns
(A) BCD to binary code (B) Binary to excess - 3 code
5.90 In the circuit shown in the figure, A is parallel-in, parallel-out 4 bit
register, which loads at the rising edge of the clock C . The input (C) Excess -3 to gray code (D) Gray to Binary code
lines are connected to a 4 bit bus, W . Its output acts at input to a SPECIAL EDITION ( STUDY MATERIAL FORM )
16 # 4 ROM whose output is floating when the input to a partial At market Book is available in 3 volume i.e. in 3 book binding
table of the contents of the ROM is as follows form. But at NODIA Online Store book is available in 10 book
binding form. Each unit of Book is in separate binding.
Data 0011 1111 0100 1010 1011 1000 0010 1000 Available Only at NODIA Online Store

Address 0 2 4 6 8 10 11 14 Click to Buy


The clock to the register is shown, and the data on the W bus at
www.nodia.co.in
time t1 is 0110. The data on the bus at time t2 is 5.93 In an 8085 microprocessor, the instruction CMP B has been executed
while the content of the accumulator is less than that of register B
. As a result
(A) Carry flag will be set but Zero flag will be reset
(B) Carry flag will be rest but Zero flag will be set
(C) Both Carry flag and Zero flag will be rest
(D) Both Carry flag and Zero flag will be set

5.94 The circuit shown in the figure is a 4 bit DAC

The input bits 0 and 1 are represented by 0 and 5 V respectively.


The OP AMP is ideal, but all the resistance and the 5 v inputs
have a tolerance of ! 10%. The specification (rounded to nearest
multiple of 5%) for the tolerance of the DAC is
(A) ! 35% (B) ! 20%
(A) 1111 (B) 1011 (C) ! 10% (D) ! 5%
(C) 1000 (D) 0010

5.91 The DTL, TTL, ECL and CMOS famil GATE of digital ICs are 2002 ONE MARK
compared in the following 4 columns 5.95 4 - bit 2’s complement representation of a decimal number is 1000.
The number is
(P) (Q) (R) (S)
(A) +8 (B) 0
Fanout is minimum DTL DTL TTL CMOS
(C) -7 (D) -8
Power consumption is TTL CMOS ECL DTL
minimum 5.96 If the input to the digital circuit (in the figure) consisting of a
GATE Electronics and Communication Topicwise Solved Paper by RK Kanodia & Ashish Murolia Page 112

cascade of 20 XOR - gates is X , then the output Y is equal to

(A) 0 (B) 1
(C) X (D) X

5.97 The number of comparators required in a 3-bit comparators type


ADC
(A) 2 (B) 3
(C) 7 (D) 8 (A) gray code numbers (B) 2 4 2 1 BCD numbers
(C) excess - 3 code numbers (D) none of the above
2002 TWO MARKS 5.101 Consider the following assembly language program
5.98 The circuit in the figure has two CMOS NOR gates. This circuit MVI B, 87H
functions as a: MOV A, B
START : JMP NEXT
MVI B, 00H
XRA B
GATE Electronics & Communication OUT PORT1
HLT
by RK Kanodia NEXT : XRA B
Now in 3 Volume JP START
Purchase Online at maximum discount from online store OUT PORT2
and get POSTAL and Online Test Series Free HTL
The execution of above program in an 8085 microprocessor will
visit www.nodia.co.in result in
(A) an output of 87H at PORT1
(B) an output of 87H at PORT2
(C) infinite looping of the program execution with accumulator
data remaining at 00H
(D) infinite looping of the program execution with accumulator
data alternating between 00H and 87H

2001 ONE MARKS

5.102 The 2’s complement representation of -17 is


(A) flip-flop (B) Schmitt trigger
(A) 101110 (B) 101111
(C) Monostable multivibrator (D) astable multivibrator
(C) 111110 (D) 110001
5.99 The gates G1 and G2 in the figure have propagation delays of 10 ns 5.103 For the ring oscillator shown in the figure, the propagation delay of
and 20 ns respectively. If the input V1, makes an output change from
each inverter is 100 pico sec. What is the fundamental frequency of
logic 0 to 1 at time t = t0 , then the output waveform V0 is
the oscillator output
For more GATE Resources, Mock Test and
Study material join the community
http://www.facebook.com/gateec2014

5.100 If the input X3, X2, X1, X0 to the ROM in the figure are 8 4 2 1 BCD
numbers, then the outputs Y3, Y2, Y1, Y0 are
(A) 10 MHz (B) 100 MHz
(C) 1 GHz (D) 2 GHz

5.104 Ab 8085 microprocessor based system uses a 4K # 8 bit RAM whose


starting address is AA00H. The address of the last byte in this RAM
is
GATE Electronics and Communication Topicwise Solved Paper by RK Kanodia & Ashish Murolia Page 113

(A) OFFFH (B) 1000H


(C) B9FFH (D) BA00H

2001 TWO MARKS

5.105 In the TTL circuit in the figure, S2 and S0 are select lines and X7
(A) 5 V; 3 V; 7 V (B) 4 V; 3 V; 4 V
and X0 are input lines. S0 and X0 are LSBs. The output Y is
(C) 5 V; 5 V; 5 V (D) 4 V; 4 V; 4 V

2000 ONE MARKS

5.109 An 8 bit successive approximation analog to digital communication


has full scale reading of 2.55 V and its conversion time for an analog
input of 1 V is 20 ms. The conversion time for a 2 V input will be
(A) 10 ms (B) 20 ms
(A) indeterminate (B) A 5 B (C) 40 ms (D) 50 ms
(C) A 5 B (D) C (A 5 B ) + C (A 5 B)
5.110 The number of comparator in a 4-bit flash ADC is
5.106 In the figure, the LED (A) 4 (B) 5
SPECIAL EDITION ( STUDY MATERIAL FORM )
At market Book is available in 3 volume i.e. in 3 book binding
form. But at NODIA Online Store book is available in 10 book
binding form. Each unit of Book is in separate binding.
Available Only at NODIA Online Store

Click to Buy
www.nodia.co.in
(A) emits light when both S1 and S2 are closed
(C) 15 (D) 16
(B) emits light when both S1 and S2 are open
(C) emits light when only of S1 and S2 is closed 5.111 For the logic circuit shown in the figure, the required input condition
(D) does not emit light, irrespective of the switch positions. (A, B, C) to make the output (X) = 1 is

5.107 The digital block in the figure is realized using two positive edge
triggered D-flip-flop. Assume that for t < t0, Q1 = Q2 = 0 . The circuit
in the digital block is given by

(A) 1,0,1 (B) 0,0,1


(C) 1,1,1 (D) 0,1,1

5.112 The number of hardware interrupts (which require an external signal


to interrupt) present in an 8085 microprocessor are
(A) 1 (B) 4
(C) 5 (D) 13

5.113 In the microprocessor, the RST6 instruction transfer the program


execution to the following location :
(A)30 H (B) 24 H
(C) 48 H (D) 60 H

2000 TWO MARKS

5.114 The contents of register (B) and accumulator (A) of 8085


microprocessor are 49J are 3AH respectively. The contents of A and
status of carry (CY) and sign (S) after execution SUB B instructions
are
(A) A = F1, CY = 1, S = 1 (B) A = 0F, CY = 1, S = 1
(C) A = F0, CY = 0, S = 0 (D) A = 1F, CY = 1, S = 1

5.115 For the logic circuit shown in the figure, the simplified Boolean
5.108 In the DRAM cell in the figure, the Vt of the NMOSFET is 1 V. For expression for the output Y is
the following three combinations of WL and BL voltages.
GATE Electronics and Communication Topicwise Solved Paper by RK Kanodia & Ashish Murolia Page 114

(A) y = AB (B) y = AB
(C) y = A + B (D) y = A + B

5.120 A Darlington emitter follower circuit is sometimes used in the output


stage of a TTL gate in order to
(A) increase its IOL (B) reduce its IOH
(A) A + B + C (B) A (C) increase its speed of operation (D) reduce power dissipa-
(C) B (D) C tion
5.116 For the 4 bit DAC shown in the figure, the output voltage V0 is 5.121 Commercially available ECL gears use two ground lines and one
negative supply in order to
(A) reduce power dissipation
(B) increase fan-out
(C) reduce loading effect
(D) eliminate the effect of power line glitches or the biasing circuit

5.122 The resolution of a 4-bit counting ADC is 0.5 volts. For an analog
input of 6.6 volts, the digital output of the ADC will be
(A) 1011 (B) 1101
(A) 10 V (B) 5 V
(C) 1100 (D) 1110
(C) 4 V (D) 8 V
GATE Electronics & Communication
by RK Kanodia 1999 TWO MARKS

Now in 3 Volume 5.123 The minimized form of the logical expression


Purchase Online at maximum discount from online store (ABC + ABC + ABC + ABC ) is
and get POSTAL and Online Test Series Free (A) AC + BC + AB (B) AC + BC + AB
visit www.nodia.co.in (C) AC + BC + AB (D) AC + BC + AB
5.117 A sequential circuit using D flip-flop and logic gates is shown in 5.124 For a binary half-subtractor having two inputs A and B, the correct
the figure, where X and Y are the inputs and Z is the inputs. The set of logical expressions for the outputs D (= A minus B) and
circuit is X (= borrow) are
(A) D = AB + AB, X = AB (B)
D = AB + AB + AB , X = AB
(C) D = AB + AB , X = AB (D) D = AB + AB , X = AB

5.125 The ripple counter shown in the given figure is works as a

(A) S - R Flip-Flop with inputs X = R and Y = S


(B) S - R Flip-Flop with inputs X = S and Y = R
(C) J - K Flip-Flop with inputs X = J and Y = K
(D) J - K Flip-Flop with input X = K and Y = J
For more GATE Resources, Mock Test and
5.118 In the figure, the J and K inputs of all the four Flip-Flips are made Study material join the community
high. The frequency of the signal at output Y is
http://www.facebook.com/gateec2014
(A) mod-3 up counter (B) mod-5 up counter
(C) mod-3 down counter (D) mod-5 down counter

5.126 If CS = A15 A14 A13 is used as the chip select logic of a 4 K RAM in
an 8085 system, then its memory range will be
(A) 3000 H - 3 FFF H
(B) 7000 H - 7 FFF H
(A) 0.833 kHz (B) 1.0 kHz (C) 5000 H - 5 FFF H and 6000 H - 6 FFF H
(C) 0.91 kHz (D) 0.77 kHz (D) 6000 H - 6 FFF H and 7000 H - 7 FFF H

1999 ONE MARK

5.119 The logical expression y = A + AB is equivalent to


GATE Electronics and Communication Topicwise Solved Paper by RK Kanodia & Ashish Murolia Page 115

1998 ONE MARK 5.135 For the identity AB + AC + BC = AB + AC , the dual form is
5.127 The minimum number of 2-input NAND gates required to implement (A) (A + B) (A + C) (B + C) = (A + B) (A + C)
of Boolean function Z = ABC , assuming that A, B and C are (B) (A + B ) (A + C ) (B + C ) = (A + B ) (A + C )
available, is (C) (A + B) (A + C) (B + C) = (A + B ) (A + C )
(A) two (B) three (D) AB + AC + BC = AB + AC
(C) five (D) six
5.136 An instruction used to set the carry Flag in a computer can be
5.128 The noise margin of a TTL gate is about classified as
(A) 0.2 V (B) 0.4 V (A) data transfer (B) arithmetic
(C) 0.6 V (D) 0.8 V (C) logical (D) program control
5.129 In the figure is A = 1 and B = 1, the input B is now replaced by a 5.137 The figure is shows a mod-K counter, here K is equal to
sequence 101010....., the output x and y will be

(A) fixed at 0 and 1, respectively SPECIAL EDITION ( STUDY MATERIAL FORM )


(B) x = 1010.....while y = 0101...... At market Book is available in 3 volume i.e. in 3 book binding
(C) x = 1010.....and y = 1010...... form. But at NODIA Online Store book is available in 10 book
(D) fixed at 1 and 0, respectively binding form. Each unit of Book is in separate binding.
Available Only at NODIA Online Store
5.130 An equivalent 2’s complement representation of the 2’s complement Click to Buy
number 1101 is
(A) 110100 (B) 01101
www.nodia.co.in
(C) 110111 (D) 111101
(A) 1 (B) 2
5.131 The threshold voltage for each transistor in the figure is 2 V. For (C) 3 (D) 4
this circuit to work as an inverter, Vi must take the values
5.138 The current I through resistance r in the circuit shown in the figure
is

(A) - 5 V and 0 V (B) - 5 V and 5 V


(C) - 0 V and 3 V (D) 3 V and 5 V (A) - V (B) V
12R 12R
5.132 An I/O processor control the flow of information between (C) V (D) V
6R 3T
(A) cache memory and I/O devices
5.139 The K -map for a Boolean function is shown in the figure is the
(B) main memory and I/O devices
number of essential prime implicates for this function is
(C) two I/O devices
(D) cache and main memories

5.133 Two 2’s complement number having sign bits x and y are added
and the sign bit of the result is z . Then, the occurrence of overflow
is indicated by the Boolean function
(A) xyz (B) x y z
(C) x yz + xyz (D) xy + yz + zx

5.134 The advantage of using a dual slope ADC in a digital voltmeter is


that
(A) its conversion time is small (A) 4 (B) 5
(B) its accuracy is high (C) 6 (D) 8
(C) it gives output in BCD format
(D) it does not require a
GATE Electronics and Communication Topicwise Solved Paper by RK Kanodia & Ashish Murolia Page 116

1997 ONE MARK (C) 30 H to 33 H (D) 70 H to 73 H


5.140 Each cell of a static Random Access Memory contains 5.147 The following instructions have been executed by an 8085 mP
(A) 6 MOS transistors
ADDRESS (HEX) INSTRUCTION
(B) 4 MOS transistors and 2 capacitors
6010 LXI H, 8 A 79 H
(C) 2 MOS transistors and 4 capacitors
6013 MOV A, L
(D) 1 MOS transistors and 1 capacitors
6015 ADDH
5.141 A 2 bit binary multiplier can be implemented using
6016 DAA
(A) 2 inputs ANSs only
6017 MOV H, A
(B) 2 input XORs and 4 input AND gates only
6018 PCHL
(C) Two 2 inputs NORs and one XNO gate
(D) XOR gates and shift registers From which address will the next instruction be fetched ?
(A) 6019 (B) 6379
5.142 In standard TTL, the ‘totem pole’ stage refers to (C) 6979 (D) None of the above
(A) the multi-emitter input stage
(B) the phase splitter 5.148 A signed integer has been stored in a byte using the 2’s complement
(C) the output buffer format. We wish to store the same integer in a 16 bit word. We
should
(D) open collector output stage
(A) copy the original byte to the less significant byte of the word
and fill the more significant with zeros
GATE Electronics & Communication (B) copy the original byte to the more significant byte of the word
by RK Kanodia and fill the less significant byte with zeros
Now in 3 Volume (C) copy the original byte to the less significant byte of the word
Purchase Online at maximum discount from online store and make each fit of the more significant byte equal to the
most significant bit of the original byte
and get POSTAL and Online Test Series Free
(D) copy the original byte to the less significant byte as well as the
visit www.nodia.co.in more significant byte of the word
5.143 The inverter 74 ALSO4 has the following specifications
IOH max =- 0.4 A, IOL max = 8 mA, IIH max = 20 mA, IIL max =- 0.1 mA 1997 TWO MARKS
The fan out based on the above will be
(A) 10 (B) 20
5.149 For the NMOS logic gate shown in the figure is the logic function
implemented is
(C) 60 (D) 100

5.144 The output of the logic gate in the figure is

(A) 0 (B) 1
(C) A (D) F

5.145 In an 8085 mP system, the RST instruction will cause an interrupt


(A) only if an interrupt service routine is not being executed
(B) only if a bit in the interrupt mask is made 0
For more GATE Resources, Mock Test and
(C) only if interrupts have been enabled by an EI instruction
Study material join the community
(D) None of the above
http://www.facebook.com/gateec2014
5.146 The decoding circuit shown in the figure is has been used to generate
the active low chip select signal for a microprocessor peripheral. (A) ABCDE (B) (AB + C ) : (D + E )
(The address lines are designated as AO to A7 for I/O address) (C) A : (B + C) + D : E (D) (A + B ) : C + D : E

5.150 In a J–K flip-flop we have J = Q and K = 1. Assuming the flip flop


was initially cleared and then clocked for 6 pulses, the sequence at
the Q output will be

The peripheral will correspond to I/O address in the range


(A) 60 H to 63 H (B) A4 to A 7H
GATE Electronics and Communication Topicwise Solved Paper by RK Kanodia & Ashish Murolia Page 117

(A) 010000 (B) 011001 (C) 8 (D) 13


(C) 010010 (D) 010101
5.160 The following sequence of instructions are executed by an 8085
5.151 The gate delay of an NMOS inverter is dominated by charge time microprocessor:
rather than discharge time because 1000 LXI SP, 27 FF
(A) the driver transistor has larger threshold voltage than the load 1003 CALL 1006
transistor 1006 POP H
The contents of the stack pointer (SP) and the HL, register pair
(B) the driver transistor has larger leakage currents compared to
on completion of execution of these instruction are
the load transistor
(A) SP = 27 FF, HL = 1003 (B) SP = 27 FD, HL = 1003
(C) the load transistor has a smaller W/L ratio compared to the
(C) SP = 27 FF, HL = 1006 (D) SP = 27 FD, HL = 1006
driver transistor
(D) none of the above

5.152 The boolean function A + BC is a reduced form of


(A) AB + BC (B) (A + B) : (A + C)
(C) AB + ABC (D) (A + C) : B

1996 ONE MARK

5.153 Schottky clamping is resorted in TTl gates SPECIAL EDITION ( STUDY MATERIAL FORM )
(A) to reduce propagation delay (B) to increase noise margins At market Book is available in 3 volume i.e. in 3 book binding
(C) to increase packing density (D) to increase fan-out form. But at NODIA Online Store book is available in 10 book
5.154 A pulse train can be delayed by a finite number of clock periods
binding form. Each unit of Book is in separate binding.
Available Only at NODIA Online Store
using
(A) a serial-in serial-out shift register Click to Buy
(B) a serial-in parallel-out shift register www.nodia.co.in
(C) a parallel-in serial-out shift register
(D) a parallel-in parallel-out shift register

5.155 A 12-bit ADC is operating with a 1 m sec clock period and the total
conversion time is seen to be 14 m sec . The ADC must be of the
(A) flash type (B) counting type
(C) intergrating type (D) successive approximation
type

5.156 The total number of memory accesses involved (inclusive of the op-
code fetch) when an 8085 processor executes the instruction LDA
2003 is
(A) 1 (B) 2
(C) 3 (D) 4

1996 TWO MARKS

5.157 A dynamic RAM cell which hold 5 V has to be refreshed every 20 m


sec, so that the stored voltage does not fall by more than 0.5 V
. If the cell has a constant discharge current of 1 pA, the storage
capacitance of the cell is
(A) 4 # 10-6 F (B) 4 # 10-9 F
(C) 4 # 10-12 F (D) 4 # 10-15 F

5.158 A 10-bit ADC with a full scale output voltage of 10.24 V is designed
to have a ! LSB/2 accuracy. If the ADC is calibrated at 25c C
and the operating temperature ranges from 0c C to 25c C , then the
maximum net temperature coefficient of the ADC should not exceed
(A) ! 200 mV/cC (B) ! 400 mV/cC
(C) ! 600 mV/cC (D) ! 800 mV/cC

5.159 A memory system of size 26 K bytes is required to be designed using


memory chips which have 12 address lines and 4 data lines each.
The number of such chips required to design the memory system is
(A) 2 (B) 4
GATE Electronics and Communication Topicwise Solved Paper by RK Kanodia & Ashish Murolia Page 118

SOLUTIONS For S1 S 0 = 0 0
We have A13 = A12 = 0
and for I/p = 1we obtain
A10 = 1 or A10 = 0
A11 = 1
A14 = 1 or A14 = 0
5.1 Option (C) is correct.
Let A denotes the position of switch at ground floor and B denotes A15 = 1 or A15 = 0
the position of switch at upper floor. The switch can be either in up Since, A 0 - A 9 can have any value 0 or 1
position or down position. Following are the truth table given for Therefore, we have the address range as
different combinations of A and B A15 A14 A13 A12 A11 A10 A 9 A8 A7 A6 A5 A4 A3 A2 A1 A0
A B Y(Bulb) From 0 0 0 0 1 0 0 0 0 0 0 0 0 0 0 0

up(1) up(1) OFF(0) to 0 0 0 0 1 0 1 1 1 1 1 1 1 1 1 1

Down(0) Down(0) OFF(0) In Hexadecimal & 0800 H to 0BFFH


up(1) Down(0) ON(1) Similarly, for chip 2, we obtain the range as follows
Down(0) up(1) ON(1) E = 1 for S1 S 0 = 0 1
When the switches A and B are both up or both down, output so, A13 = 0 and A12 = 1
will be zero (i.e. Bulb will be OFF). Any of the switch changes its and also the I/P = 1 for
A10 = 0 , A11 = 1, A14 = 0 , A15 = 0
GATE Electronics & Communication
so, the fixed I/ps are
by RK Kanodia
Now in 3 Volume A15 A14 A13 A12 A11 A10

Purchase Online at maximum discount from online store 0 0 0 1 1 0

and get POSTAL and Online Test Series Free Therefore, the address range is

visit www.nodia.co.in A15 A14 A13 A12 A11 A10 A 9 A8 A7 A6 A5 A4 A3 A2 A1 A0


From 0 0 0 1 1 0 0 0 0 0 0 0 0 0 0 0

position leads to the ON state of bulb. Hence, from the truth table, to 0 0 0 1 1 0 1 1 1 1 1 1 1 1 1 1

we get In hexadecimal it is from 1800 H to 1BFFH . There is no need to


Y = A5B obtain rest of address ranged as only (D) is matching to two re-
i.e., the XOR gate sults.
5.2 Option (A) is correct. 5.4 Option (A) is correct.
The program is being executed as follows The given circuit is
MVI A, 0.5H; A = 05H
MVI B, 0.5H; B = 05H
At the next instruction, a loop is being introduced in which for the
instruction “DCR B” if the result is zero then it exits from loop so,
the loop is executed five times as follows :
Content in B Output of ADD B (Stored value at
A) Condition for the race-around
05 05 + 05 It occurs when the output of the circuit (Y1, Y2) oscillates between
04 05 + 05 + 04
For more GATE Resources, Mock Test and
03 05 + 05 + 04 + 03
Study material join the community
02 05 + 05 + 04 + 03 + 02
http://www.facebook.com/gateec2014
01 05 + 05 + 04 + 03 + 02 + 01
00 System is out of loop ‘0’ and ‘1’ checking it from the options.
i.e., A = 05 + 05 + 04 + 03 + 02 + 01 = 144 1. Option (A): When CLK = 0
At this stage, the 8085 microprocessor exits from the loop and Output of the NAND gate will be A1 = B1 = 0 = 1. Due to these
reads the next instruction. i.e., the accumulator is being added to input to the next NAND gate, Y2 = Y1 : 1 = Y1 and Y1 = Y2 : 1 = Y2 .
If Y1 = 0 , Y2 = Y1 = 1 and it will remain the same and doesn’t
03 H. Hence, we obtain
oscillate.
A = A + 03 H = 14 + 03 = 17 H If Y2 = 0 , Y1 = Y2 = 1 and it will also remain the same for the
5.3 Option (D) is correct. clock period. So, it won’t oscillate for CLK = 0 .
For chip-1, we have the following conclusions: So, here race around doesn’t occur for the condition CLK = 0 .
it is enable when (i) S1 S 0 = 0 0 2. Option (C): When CLK = 1, A = B = 1
and (ii) Input = 1 A1 = B1 = 0 and so Y1 = Y2 = 1
GATE Electronics and Communication Topicwise Solved Paper by RK Kanodia & Ashish Murolia Page 119

And it will remain same for the clock period. So race around mp COX W
2 b L l8 SD ^ SG
Since, I1 = 2V V - VTp h - V SD 2
B
doesn’t occur for the condition.
m C
3. Option (D): When CLK = 1, A = B = 0 & 0 = p OX bW l [2VSD ^VSG - VTp h - V SD
2
]
2 L
So, A1 = B1 = 1
Solving it we get,
And again as described for Option (B) race around doesn’t occur
2 ^VSG - VTp h = VSD
for the condition.
& 2 ^5 - Vin - 1h = 5 - VD
5.5 Option ( ) is correct.
& Vin = VD + 3
2
For I1 = 0 , VD = 5 V
So, Vin = 5 + 3 = 4 V
2
Y = 1, when A > B So for the NMOS
VGS = Vin - 0 = 4 - 0 = 4 V and VGS > VTn
A = a1 a 0, B = b1 b 0
So it can’t be in cutoff region.
a1 a0 b1 b0 Y Case 2 : M2 must be in saturation region.
0 1 0 0 1 So, I1 = I 2
1 0 0 0 1
1 0 0 1 1 SPECIAL EDITION ( STUDY MATERIAL FORM )
1 1 0 0 1 At market Book is available in 3 volume i.e. in 3 book binding
1 1 0 1 1 form. But at NODIA Online Store book is available in 10 book
1 1 1 0 1 binding form. Each unit of Book is in separate binding.
Available Only at NODIA Online Store

Total combination = 6 Click to Buy


5.6 Option (A) is correct. www.nodia.co.in
Parallel connection of MOS & OR operation
Series connection of MOS & AND operation
mp COX W mn COX W
2 L6 @ = 2 L (VGS - VTn)
2 2
The pull-up network acts as an inverter. From pull down network 2 (VSG - VTp) VSD - V SD
we write Y = (A + B) C = (A + B) + C = A B + C & 2
2 (VSG - VTp) VSD - V SD = (VGS - VTn) 2
5.7 Option (A) is correct. & 2 (5 - Vin - 1) (5 - VD) - (5 - VD) 2 = (Vin - 0 - 1) 2
Prime implicants are the terms that we get by solving K-map & 2 (4 - Vin) (5 - VD) - (5 - VD) 2 = (Vin - 1) 2
Substituting VD = VDS = VGS - VTn and for N -MOS & VD = Vin - 1
& 2 (4 - Vin) (6 - Vin) - (6 - Vin) 2 = (Vin - 1) 2
& 48 - 36 - 8Vin =- 2Vin + 1
& 6Vin = 11
F = XY + XY Vin = 11 = 1.833 V
1prime
44 2 44 3
implicants
&
6
5.8 Option (A) is correct. So for M2 to be in saturation Vin < 1.833 V or Vin < 1.875 V
Given the circuit as below : 5.9 Option (D) is correct.
Let Qn + 1 is next state and Qn is the present state. From the given
below figure.
D = Y = AX 0 + AX1
Qn + 1 = D = AX 0 + AX1
Qn + 1 = A Qn + AQn X 0 = Q , X1 = Q
If A = 0, Qn + 1 = Qn (toggle of previous state)
If A = 1, Qn + 1 = Qn
So state diagram is
Since all the parameters of PMOS and NMOS are equal.
So, mn = mp
COX bW l = COX bW l = COX bW l
L M1 L M2 L
Given that M1 is in linear region. So, we assume that M2 is either in
cutoff or saturation. 5.10 Option (B) is correct.
Case 1 : M2 is in cut off The given circuit is shown below:
So, I 2 = I1 = 0
Where I1 is drain current in M1 and I2 is drain current in M2 .
GATE Electronics and Communication Topicwise Solved Paper by RK Kanodia & Ashish Murolia Page 120

0 1 1 0
1 0 0 0
QB ^t + 1h

(PQ QR ) PR = (PQ + QR PR )
= PQ + QR + PR
QB ^t + 1h = Q A
= PQ + QR + PR
If any two or more inputs are ‘1’ then output y will be 1.
5.11 Option (A) is correct.
For the output to be high, both inputs to AND gate should be high.
The D-Flip Flop output is the same, after a delay.
Let initial input be 0; (Consider Option A) DA = Q A Q B + QA QB
then Q = 1 (For 1 D-Flip Flop). This is given as input to 2nd FF.
st
5.15 Option (C) is correct.
Let the second input be 1. Now, considering after 1 time interval;
Initially Carry Flag, C = 0
The output of 1st Flip Flop is 1 and 2nd FF is also 1. Thus Output
MVI A, 07 H ; A = 0000 0111
RLC ; Rotate left without carry. A = 0000 1110
GATE Electronics & Communication
MVO B, A ; B = A = 0000 1110
by RK Kanodia RLC ; A = 0001 1100
Now in 3 Volume RLC ; A = 0011 1000
Purchase Online at maximum discount from online store ADD B ; A = 0011 1000
; + 0000 1110
and get POSTAL and Online Test Series Free
; 0100 0110
visit www.nodia.co.in
RRC ; Rotate Right with out carry, A = 0010 0011
Thus A = 23 H
= 1.
5.16 Option ( ) is correct.
5.12 Option (D) is correct.
F = S 1 S 0 I 0 + S 1 S 0 I 1 + S 1 S 0 I 2 + S1 S 0 I 3
I0 = I3 = 0
F = PQ + PQ = XOR (P, Q) ( S1 = P, S 0 = Q )
5.13 Option (A) is correct.
All the states of the counter are initially unset.

5.17 Option (B) is correct.


State Initially are shown below in table : Since G2 is active low input, output of NAND gate must be 0
Q2 Q1 Q0 For more GATE Resources, Mock Test and
0 0 0 0 Study material join the community
1 0 0 4 http://www.facebook.com/gateec2014
1 1 0 6
1 1 1 7
G2 = A15 : A14 A13 A12 A11 = 0
0 1 1 3 So, A15 A14 A13 A12 A11 = 00101
0 0 1 1 To select Y5 Decoder input
0 0 0 0 ABC = A 8 A9 A10 = 101
5.14 Option (D) is correct. Address range
The sequence is QB QA A15 A14 A13 A12 A11 A10 A 9 A 8 ...............A 0
0011101........A 0
00 " 11 " 01 " 10 " 00 " ... S2
S D
QB QA QB (t + 1) QA (t + 1) ^2D00 - 2DFF h
0 0 1 1 5.18 Option (A) (B) (C) are correct.
1 1 0 1 In the circuit F = (A 5 B) 9 (A 9 B) 9 C
GATE Electronics and Communication Topicwise Solved Paper by RK Kanodia & Ashish Murolia Page 121

For two variables A5B = A9B A = A 5 B = 00100010 5 01000101 = 01100111 = 674


So, (A 5 B) 9 (A 9 B) = 0 (always) 5.22 Option (C) is correct.
F = 09C = 0$C+1$C = C TTL " Transistor - Transistor logic
So, F = 1 when C = 1 or C = 0 CMOS " Complementary Metal Oxide Semi-conductor
5.19 Option (D) is correct. 5.23 Option (D) is correct.
Let QA (n), QB (n), QC (n) are present states and QA (n + 1), QB (n + 1), Vectored interrupts : Vectored interrupts are those interrupts in
QC (n + 1) are next states of flop-flops. which program control transferred to a fixed memory location.
In the circuit Maskable interrupts : Maskable interrupts are those interrupts which
QA (n + 1) = QB (n) 9 QC (n) can be rejected or delayed by microprocessor if it is performing some
QB (n + 1) QA (n) critical task.
QC (n + 1) QB (n) 5.24 Option (D) is correct.
Initially all flip-flops are reset We have 6X + Z {Y + (Z + XY )}@[X + Z (X + Y)] = 1
Substituting X = 1 and X = 0 we get
1st clock pulse
[1 + Z {Y + (Z + 1Y )}][ 0 + Z (1 + Y)] = 1
QA = 0 9 0 = 1
or [1][ Z (1)] = 1 1 + A = 1 and 0 + A = A
QB = 0
or Z =1)Z=0
QC = 0
5.25 Option (A) is correct.
2 nd clock pulse
QA = 0 9 0 = 1
SPECIAL EDITION ( STUDY MATERIAL FORM )
At market Book is available in 3 volume i.e. in 3 book binding
QB = 1
form. But at NODIA Online Store book is available in 10 book
QC = 0
binding form. Each unit of Book is in separate binding.
3 rd clock pulse Available Only at NODIA Online Store
QA = 1 9 0 = 0
Click to Buy
QB = 1
www.nodia.co.in
QC = 1
4 th clock pulse The AND gate implementation by 2:1 mux is as follows
QA = 1 9 1 = 1
QB = 0
QC = 1
So, sequence QA = 01101.......
Y = AI 0 + AI1 = AB
5.20 Option (D) is correct.
Output of the MUX can be written as The EX - OR gate implementation by 2:1 mux is as follows
F = I 0 S 0 S1 + I1 S 0 S1 + I 2 S 0 S 1 + I 3 S 0 S 1
Here, I 0 = C, I1 = D, I2 = C , I 3 = CD
and S 0 = A, S1 = B
So, F = C A B + D A B + C A B + C DA B
Writing all SOP terms
F = A B C D + A B C D + A BCD + A B C D
1 44 2
m
44 3 1 44 2
m
44 3 S
m
1 44 2
m
4
43
3 2 7 5

+A B C D + A B C D + ABC D Y = BI0 + BI1 = AB + BA


1 44 2
m
4
4 3 1 44 2
m
44 3 S
m
9 8 12
5.26 Option (A) is correct.
F = / m (2, 3, 5, 7, 8, 9, 12)
The given circuit is as follows.
5.21 Option (C) is correct.
By executing instruction one by one
MVI A, 45 H & MOV 45 H into accumulator, A = 45 H
STC & Set carry, C = 1
CMC & Complement carry flag, C = 0
RAR & Rotate accumulator right through carry
The truth table is as shown below. Sequence is 00, 11, 10, 00 ...

CLK J1 K1 Q1 J2 K2 Q2
1 1 1 0 1 1 0
2 1 1 1 1 1 1
A = 00100010 3 0 0 1 0 1 0
XRA B & XOR A and B 4 1 1 0 1 1 0
GATE Electronics and Communication Topicwise Solved Paper by RK Kanodia & Ashish Murolia Page 122

5.27 Option (B) is correct. From the figure shown below it may be easily seen upper MOSFET
The given situation is as follows are shorted and connected to Vdd thus OUT is 1 only when the node
S is 0,

The truth table is as shown below


Since the lower MOSFETs are shorted to ground, node S is 0 only
P1 P2 a b c d e f g when input P and Q are 1. This is the function of AND gate.
0 0 1 1 1 1 1 1 0 5.31 Option (B) is correct.
0 1 1 0 1 1 0 1 1 MSB of both number are 1, thus both are negative number. Now
we get
1 0 1 1 0 1 1 0 1
11101101 = (- 19) 10
1 1 1 0 0 1 1 1 1
and 11100110 = (- 26) 10
From truth table we can write P - Q = (- 19) - (- 26) = 7
a =1 Thus 7 signed two’s complements form is
b = P 1 P 2 + P1 P 2 = P 2 1 NOT Gate (7) 10 = 00000111
GATE Electronics & Communication 5.32 Option (D) is correct.
by RK Kanodia The circuit is as shown below
Now in 3 Volume
Purchase Online at maximum discount from online store
and get POSTAL and Online Test Series Free
visit www.nodia.co.in
c = P1 P2 + P1 P2 = P1 1 NOT Gate X = PQ
d = 1 = c+e Y = (P + Q)
and c = P1 P2 = P1 + P2 1 OR GATE So Z = PQ (P + Q)
f = P1 P2 = P1 + P2 1 OR GATE = (P + Q )( P + Q) = PQ + PQ = P 5 Q
g = P1 P2 = P1 + P2 1 OR GATE and M1 = Z 5 R = (P 5 Q) 5 R
Thus we have g = P1 + P2 and d = 1 = c + e . It may be observed 5.33 Option (A) is correct.
easily from figure that The circuit is as shown below
Led g does not glow only when both P1 and P2 are 0. Thus
g = P1 + P2
LED d is 1 all condition and also it depends on
d = c+e
5.28 Option (D) is correct.
As shown in previous solution 2 NOT gates and 3-OR gates are
required.
5.29 Option (C) is correct. For more GATE Resources, Mock Test and
For the NAND latche the stable states are as follows Study material join the community
http://www.facebook.com/gateec2014
The truth table is shown below. When CLK make transition Q goes
to 1 and when D goes to 1, Q goes to 0
5.34 Option (B) is correct.
Since the input to both JK flip-flop is 11, the output will change
For the NOR latche the stable states are as follows
every time with clock pulse. The input to clock is

The output Q0 of first FF occurs after time 3 T and it is as shown


below
5.30 Option (D) is correct.
GATE Electronics and Communication Topicwise Solved Paper by RK Kanodia & Ashish Murolia Page 123

Thus after execution above instruction contests of PC and HL are


same and that is 6140H
5.39 Option (C) is correct.
MSB of Y is 1, thus it is negative number and X is positive number
The output Q1 of second FF occurs after time 3 T when it gets
input (i.e. after 3 T from t1) and it is as shown below Now we have X = 01110 = (14) 10
and Y = 11001 = (- 7) 10
X + Y = (14) + (- 7) = 7
In signed two’s complements from 7 is (7) 10 = 000111
5.40 Option (B) is correct.
5.35 Option (D) is correct. Y = AB + CD = AB .CD
3
This is SOP form and we require only 3 NAND gate
We have VDAC = / 2n - 1bn = 2- 1b0 + 20 b1 + 21b2 + 22 b3
n=0 5.41 Option (A) is correct.
or VDAC = 0.5b0 + b1 + 2b2 + 4b3 The circuit is as shown below
The counter outputs will increase by 1 from 0000 till Vth > VDAC .
The output of counter and VDAC is as shown below

Clock b3 b3 b2 b0 VDAC
1 0001 0 SPECIAL EDITION ( STUDY MATERIAL FORM )
2 0010 0.5 At market Book is available in 3 volume i.e. in 3 book binding
3 0011 1 form. But at NODIA Online Store book is available in 10 book
4 0100 1.5 binding form. Each unit of Book is in separate binding.
Available Only at NODIA Online Store
5 0101 2
Click to Buy
6 0110 2.5
www.nodia.co.in
7 0111 3
8 1000 3.5
9 1001 4
10 1010 4.5
11 1011 5
12 1100 5.5
13 1101 6
14 1110 6.5
Y = AB + AB
and when VADC = 6.5 V (at 1101), the output of AND is zero and
and X = YC + YC = (AB + AB ) C + (AB + AB ) C
the counter stops. The stable output of LED display is 13.
= (AB + AB) C + (AB + AB ) C
5.36 Option (B) is correct.
= ABC + ABC + ABC + ABC
The VADC - Vin at steady state is
= 6.5 - 6.2 = 0.3V 5.42 Option (D) is correct.
Y = ABCD + ABCD + ABC D + ABC D
5.37 Option (A) is correct.
= ABCD + ABC D + ABC D + ABC D
Z = I0 RS + I1 RS + I2 RS + I3 RS
= ABCD + ABC D + BC D (A + A)
= (P + Q ) RS + PRS + PQRS + PRS
= ABCD + ABC D + BC D A+A = 1
= PRS + QRS + PRS + PQRS + PRS
The k - Map is as shown below 5.43 Option (B) is correct.
In given TTL NOT gate when Vi = 2.5 (HIGH), then
Q1 " Reverse active
Q2 " Saturation
Q3 " Saturation
Q4 " cut - off region
5.44 Option (C) is correct.
For X = 0, Y = 1 P = 1, Q = 0
Z = PQ + PQS + QRS For X = 0, Y = 0 P = 1, Q = 1
5.38 Option (C) is correct. For X = 1, Y = 1 P = 1, Q = 0 or P = 0, Q = 1
2710H LXI H, 30A0H ; Load 16 bit data 30A0 in HL pair 5.45 Option (C) is correct.
2713H DAD H ; 6140H " HL Chip 8255 will be selected if bits A3 to A7 are 1. Bit A0 to A2 can
2714H PCHL ; Copy the contents 6140H of HL in PC
GATE Electronics and Communication Topicwise Solved Paper by RK Kanodia & Ashish Murolia Page 124

be 0 or. ; Since 8 AH < 9BH, CY = 1


1. Thus address range is 7 : STA 3010 H ; Store the contents of A to location 3010
11111000 F8H H
11111111 FFH 8 : HLT ; Stop
5.46 Option (B) is correct. Thus the contents of accumulator after execution of ADD instruction
Since the inverting terminal is at virtual ground the resistor network is EAH.
can be reduced as follows 5.49 Option (C) is correct.
The CY = 1 and Z = 0
5.50 Option (A) is correct.
For this circuit the counter state (Q1, Q0) follows the sequence 00, 01,
10, 00 ... as shown below

Clock D1 D0 Q1 Q0 Q1 NOR Q0
00 1
1st 01 10 0
2nd 10 01 0
3rd 00 00 0
The current from voltage source is
GATE Electronics & Communication
by RK Kanodia
Now in 3 Volume
Purchase Online at maximum discount from online store
and get POSTAL and Online Test Series Free 5.51 Option (A) is correct.
visit www.nodia.co.in As shown below there are 2 terms in the minimized sum of product
expression.
I = VR = 10 = 1 mA 1 0 0 1
R 10k
This current will be divide as shown below 0 d 0 0
0 0 d 1
1 0 0 1

5.52 Option (B) is correct.


The output is taken from the 5th line.
5.53 Option (D) is correct.
After applying two clock poles, the outputs of the full adder is S = 1
, C0 = 1
A B Ci S Co
-3 1st 1 0 0 0 1
Now i = I = 1 # 10 = 62.5 m A
16 16 2nd 1 1 1 1 1
5.47 Option (C) is correct. 5.54 Option (D) is correct.
The net current in inverting terminal of OP - amp is For more GATE Resources, Mock Test and
I- = 1 + 1 = 5I
4 16 16
Study material join the community
http://www.facebook.com/gateec2014
So that V0 =- R # 5I =- 3.125
16
S S S S
100010011001
5.48 Option (B) is correct. 4 2 3 1

Line 5.55 Option (B) is correct.


1 : MVI A, B5H ; Move B5H to A In this the diode D2 is connected to the ground. The following table
2 : MVI B, 0EH ; Move 0EH to B shows the state of counter and D/A converter
3 : XRI 69H ; [A] XOR 69H and store in A
; Contents of A is CDH Q2 Q1 Q0 D3 = Q2 D2 = 0 D1 = Q1 D0 = Q0 Vo
4 : ADDB ; Add the contents of A to contents of B and 000 0 0 0 0 0
; store in A, contents of A is EAH
001 0 0 0 1 1
5 : ANI 9BH ; [a] AND 9BH, and store in A,
; Contents of A is 8 AH 010 0 0 1 0 2
6 : CPI 9FH ; Compare 9FH with the contents of A 011 0 0 1 1 3
GATE Electronics and Communication Topicwise Solved Paper by RK Kanodia & Ashish Murolia Page 125

100 1 0 0 0 8 The circuit is as shown below


101 1 0 0 1 9
110 1 0 1 0 10
111 1 0 1 1 11
000 0 0 0 0 0
001 0 0 0 1 1
Thus option (B) is correct
5.56 Option (B) is correct.
If output is at logic 0, the we have V0 = 0 which signifies BJT Q3 is
LXI, EFFF H ; Load SP with data EFFH
in saturation and applying KVL we have
CALL 3000 H ; Jump to location 3000 H
VBE3 = IR # 1k
:
: or 0.75 = IR # 1k
: or IR = 0.75 mA
3000H LXI H, 3CF4 ; Load HL with data 3CF4H 5.62 Option (A) is correct.
PUSH PSW ; Store contnets of PSW to Stack We have f = ABC + ABC
POP PSW ; Restore contents of PSW from stack
= B (AC + AC ) = B (A + C)( A + C )
PRE ; stop
Before instruction SPHL the contents of SP is 3CF4H. SPECIAL EDITION ( STUDY MATERIAL FORM )
After execution of POP PSW, SP + 2 " SP At market Book is available in 3 volume i.e. in 3 book binding
After execution of RET, SP + 2 " SP form. But at NODIA Online Store book is available in 10 book
Thus the contents of SP will be 3CF4H + 4 = 3CF8H binding form. Each unit of Book is in separate binding.
Available Only at NODIA Online Store
5.57 Option (A) is correct.
The inputs D0 and D1 respectively should be connected as Q1 and Q0 Click to Buy
where Q0 " D1 and Q1 " D0 www.nodia.co.in
5.58 Option (D) is correct. 5.63 Option (C) is correct.
If the point P is stuck at 1, then output f is equal to A Characteristic equation for a jk flip-flop is written as
Qn + 1 = JQ n + K Qn
Where Qn is the present output
Qn + 1 is next output
So, Qn + 1 = 10 + K : 0 Qn = 0
Qn + 1 =1
5.59 Option (B) is correct.
Dividing 43 by 16 we get 5.64 Option (C) is correct.

g
2 Since T2 T1 T0 is at 111, at every clock Q2 Q1 Q0 will be changes. Ir
16 43 present state is 011, the next state will be 100.
32 5.65 Option (D) is correct.
11 5.66 Option (C) is correct.
11 in decimal is equivalent is B in hexamal. 0100H LXI SP, 00FF ; Load SP with 00FFG
Thus 4310 * 2B16 0103H LXI H, 0701 ; Load HL with 0107H
Now 410 * 01002 0106H MVI A, 20H ; Move A with 20 H
310 * 00112 0108 H SUB M ; Subtract the contents of memory
Thus 4310 * 01000011BCD ; location whose address is stored in HL
5.60 Option (A) is correct. ; from the A and store in A
The diagram is as shown in fig 0109H ORI 40H ; 40H OR [A] and store in A
010BH ADD M ; Add the contents of memeory location
; whose address is stored in HL to A
; and store in A
HL contains 0107H and contents of 0107H is 20H
Thus after execution of SUB the data of A is 20H - 20H = 00
5.67 Option (C) is correct.
Before ORI instruction the contents of A is 00H. On execution the
ORI 40H the contents of A will be 40H
f' = BC + BC 00H = 00000000
f = f'A + f'0 = f'A = ABC + ABC 40H = 01000000
ORI 01000000
5.61 Option (C) is correct.
GATE Electronics and Communication Topicwise Solved Paper by RK Kanodia & Ashish Murolia Page 126

After ADD instruction the contents of memory location whose ad- multiplexers is required.
dress is stored in HL will be added to and will be stored in A 5.76 Option (D) is correct.
40H + 20 H = 60 H AC + BC = AC1 + BC 1
5.68 Option (C) is correct. = AC (B + B ) + BC (A + A)
A master slave D-flip flop is shown in the figure. = ACB + ACB + BC A + BC A
5.77 Option (D) is correct.
We have f (x, y) = xy + xy + xy = x (y + y) + xy
= x + xy
or f (x, y) = x + y
Here compliments are not available, so to get x we use NOR gate.
In the circuit we can see that output of flip-flop call be triggered Thus desired circuit require 1 unit OR and 1 unit NOR gate giving
only by transition of clock from 1 to 0 or when state of slave latch total cost 2 unit.
is affected.
5.78 Option (D) is correct.
5.69 Option (A) is correct. For 8255, various modes are described as following.
The range of signed decimal numbers that can be represented by Mode 1 : Input or output with hand shake
n - bits 1’s complement number is - (2n - 1 - 1) to + (2n - 1 - 1). In this mode following actions are executed
Thus for n = 6 we have 1. Two port (A & B) function as 8 - bit input output ports.
2. Each port uses three lines from C as a hand shake signal
GATE Electronics & Communication
3. Input & output data are latched.
by RK Kanodia
Form (ii) the mode is 1.
Now in 3 Volume Mode 2 : Bi-directional data transfer
Purchase Online at maximum discount from online store This mode is used to transfer data between two computer. In this
and get POSTAL and Online Test Series Free mode port A can be configured as bidirectional port. Port A uses
visit www.nodia.co.in five signal from port C as hand shake signal.
For (1), mode is 2
Range =- (26 - 1 - 1) to + (26 - 1 - 1)
5.79 Option (B) is correct.
=- 31 to + 31
LDA 16 bit & Load accumulator directly this instruction copies
5.70 Option (D) is correct. data byte from memory location (specified within the instruction)
The minimum number of bit require to encode 100 increment is the accumulator.
2n $ 100 It takes 4 memory cycle-as following.
or n $7 1. in instruction fetch
5.71 Option (B) is correct. 2. in reading 16 bit address
Shift Register " Serial to parallel data conversion 1. in copying data from memory to accumulator
Counter " Frequency division LXI D, (F0F1) 4 & It copies 16 bit data into register pair D and E.
Decoder " Addressing in memory chips. It takes 3 memory cycles.
5.72 Option (A) is correct. 5.80 Option (A) is correct.
For the TTL family if terminal is floating, then it is at logic 1. LXI H, 9258H ; 9258H " HL
Thus Y = (AB + 1) = AB .0 = 0 MOV A, M ; (9258H) " A
5.73 Option (C) is correct. CMa ; A"A
11001 1001 111001
For more GATE Resources, Mock Test and
00110 0110 000110
+1 +1 +1 Study material join the community
00111 0111 000111 http://www.facebook.com/gateec2014
7 7 7
Thus 2’s complement of 11001, 1001 and 111001 is 7. So the number MOV M, A ; A"M
given in the question are 2’s complement correspond to -7. This program complement the data of memory location 9258H.
5.74 Option (C) is correct. 5.81 Option (D) is correct.
In the modulo - 6 ripple counter at the end of sixth pulse (i.e. after MVI A, 00H ; Clear accumulator
101 or at 110) all states must be cleared. Thus when CB is 11 the LOOP ADD B ; Add the contents of B to A
all states must be cleared. The input to 2-input gate is C and B DCR C ; Decrement C
and the desired output should be low since the CLEAR is active low JNZ LOOP ; If C is not zero jump to loop
Thus when C and B are 0, 0, then output must be 0. In all other HLT
case the output must be 1. OR gate can implement this functions. END
This instruction set add the contents of B to accumulator to contents
5.75 Option (C) is correct.
of C times.
Number of MUX is 4 = 2 and 2 = 1. Thus the total number 3
3 2
GATE Electronics and Communication Topicwise Solved Paper by RK Kanodia & Ashish Murolia Page 127

5.82 Option (D) is correct. = R + S + PQ + PR + PQS + QRS


The number of distinct boolean expression of n variable is 22n . Thus Thus W = Z and X = Z
24 16
2 = 2 = 65536 5.89 Option (B) is correct.
5.83 Option (C) is correct. Propagation delay of flip flop is
In the flash analog to digital converter, the no. of comparators is tpd = 10 nsec
equal to 2n - 1, where n is no. of bit.s Propagation delay of 4 bit ripple counter
So, 2n - 1 = 28 - 1 = 255 R = 4tpd = 40 ns
5.84 Option (B) is correct. and in synchronous counter all flip-flop are given clock simultane-
When output of the 74 series gate of TTL gates is taken from ously, so
BJT then the configuration is either totem pole or open collector
S = tpd = 10 ns
configuration .
5.90 Option (C) is correct.
5.85 Option (D) is correct.
After t = t1, at first rising edge of clock, the output of shift register
A 2n: 1 MUX can implement all logic functions of (n + 1) variable
is 0110, which in input to address line of ROM. At 0110 is applied
without andy additional circuitry. Here n = 3 . Thus a 8 : 1 MUX
to register. So at this time data stroed in ROM at 1010 (10), 1000
can implement all logic functions of 4 variable.
will be on bus.
5.86 Option (D) is correct. When W has the data 0110 and it is 6 in decimal, and it’s data
Counter must be reset when it count 111. This can be implemented value at that add is 1010
by following circuitry SPECIAL EDITION ( STUDY MATERIAL FORM )
At market Book is available in 3 volume i.e. in 3 book binding
form. But at NODIA Online Store book is available in 10 book
binding form. Each unit of Book is in separate binding.
Available Only at NODIA Online Store
5.87 Option (B) is correct.
We have Y = P5Q5R Click to Buy
Z = RQ + PR + QP www.nodia.co.in
Here every block is a full subtractor giving P - Q - R where R is
borrow. Thus circuit acts as a 4 bit subtractor giving P - Q . then 1010 i.e. 10 is acting as odd, at time t2 and data at that
movement is 1000.
5.88 Option (A) is correct.
W = R + PQ + RS 5.91 Option (B) is correct.
The DTL has minimum fan out and CMOS has minimum power
X = PQRS + PQRS + PQRS
consumption. Propagation delay is minimum in ECL.
Y = RS + PR + PQ + PQ
5.92 Option (D) is correct.
= RS + PR $ PQ $ PQ Let input be 1010; output will be 1101
= RS + (P + R )( P + Q)( P + Q) Let input be 0110; output will be 0100
= RS + (P + PQ + PR + QR )( P + Q) Thus it convert gray to Binary code.
= RS + PQ + QR (P + P ) + QR 5.93 Option (A) is correct.
= RS + PQ + QR CMP B & Compare the accumulator content with context of
Z = R + S + PQ + PQR + PQS Register B
If A < R CY is set and zero flag will be reset.
= R + S + PQ $ PQR $ PQS
5.94 Option (A) is correct.
= R + S + (P + Q )( P + Q + R)( P + Q + S) Vo =- V1 :R bo + R b1 + R b2 + R b 3D
R 2R 4R 4R

Exact value when V1 = 5 , for maximum output


= R + S + PQ + PQ + PQS + PR + PQR
+ PRS + PQ + PQS + PQR + QRS VoExact =- 5 :1 + 1 + 1 + 1 D =- 9.375
2 4 8
Maximum Vout due to tolerance
= R + S + PQ + PQS + PR + PQR + PRS
Vo max =- 5.5 :110 + 110 + 110 + 110 D
+ PQS + PQR + QRS 90 2 # 90 4 # 90 8 # 90
=- 12.604
= R + S + PQ (1 + S) + PR (1 + P ) + PRS Tolerance = 34.44% = 35%
+ PQS + PQR + QRS 5.95 Option (D) is correct.
= R + S + PQ + PR + PRS + PQS If the 4- bit 2’s complement representation of a decimal number is
1000, then the number is -8
+ PQR + QRS
5.96 Option (B) is correct.
Output of 1 st XOR = = X $ 1 + X $ 1 = X
= R + S + PQ + PR (1 + Q ) + PQS + QRS
Output of 2 nd XOR = X X + XX = 1
GATE Electronics and Communication Topicwise Solved Paper by RK Kanodia & Ashish Murolia Page 128

So after 4,6,8,...20 XOR output will be 1. Its 1’s complement is 101110


5.97 Option (C) is correct. So 2’s compliment is
In the comparator type ADC, the no. of comparators is equal to 2n - 1 101110
, where n is no. of bit.s + 1
So, 23 - 1 = 7 101111
5.98 Option (C) is correct. 5.103 Option (C) is correct.
The circuit is as shown below The propagation delay of each inverter is tpd then The fundamental
frequency of oscillator output is
f = 1 = 1 = 1 GHz
2ntpd 2 # 5 # 100 # 10 - 12
5.104 Option (C) is correct.
4K # 8 bit means 102410 location of byte are present
The circuit shown is monostable multivibrator as it requires an Now 102410 * 1000H
external triggering and it has one stable and one quasistable state. It starting address is AA00H then address of last byte is
5.99 Option (B) is correct. AA00H + 1000H - 0001H = B9FFH
They have prorogation delay as respectively, 5.105 Option (D) is correct.
G1 " 10 nsec Y = I0 + I3 + I5 + I6
G2 " 20 nsec = C BA + C AB + CBA + CBA
GATE Electronics & Communication or Y = C (A 5 B ) + C (A 5 B)
by RK Kanodia 5.106 Option (D) is correct.
Now in 3 Volume For the LED to glow it must be forward biased. Thus output of
Purchase Online at maximum discount from online store NAND must be LOW for LED to emit light. So both input to
NAND must be HIGH. If any one or both switch are closed, output
and get POSTAL and Online Test Series Free of AND will be LOW. If both switch are open, output of XOR will
visit www.nodia.co.in be LOW. So there can’t be both input HIGH to NAND. So LED
doesn’t emit light.
For abrupt change in Vi from 0 to 1 at time t = t0 we have to as-
5.107 Option (C) is correct.
sume the output of NOR then we can say that option (B) is cor- The output of options (C) satisfy the given conditions
rect waveform.

5.100 Option (B) is correct.


Let X3 X2 X1 X0 be 1001 then Y3 Y2 Y1 Y0 will be 1111.
Let X3 X2 X1 X0 be 1000 then Y3 Y2 Y1 Y0 will be 1110
Let X3 X2 X1 X0 be 0110 then Y3 Y2 Y1 Y0 will be 1100 5.108 Option (B) is correct.
So this converts 2-4-2-1 BCD numbers. 5.109 Option (B) is correct.
5.101 Option (B) is correct. For more GATE Resources, Mock Test and
MVI B, 87H ; B = 87 Study material join the community
MOV A, B ; A = B = 87
START : JMP NEXT ; Jump to next
http://www.facebook.com/gateec2014
XRA B ; A 5 B " A,
; A = 00, B = 87 Conversion time of successive approximate analog to digital
JP START ; Since A = 00 is positive converters is independent of input voltage. It depends upon the
; so jump to START number of bits only. Thus it remains unchanged.
JMP NEXT ;Jump to NEXT ; unconditionally 5.110 Option (C) is correct.
NEXT : XRA ; B ; A 5 B " A, A = 87 , In the flash analog to digital converter, the no. of comparators is
; B = 87 H equal to 2n - 1, where n is no. of bits.
JP START ; will not jump as D7 , of A is 1 So, 2 4 - 1 = 15
OUT PORT2 ; A = 87 " PORT2
5.111 Option (D) is correct.
5.102 Option (B) is correct. As the output of AND is X = 1, the all input of this AND must be
The two’s compliment representation of 17 is 1. Thus
17 = 010001 AB + AB = 1 ...(1)
GATE Electronics and Communication Topicwise Solved Paper by RK Kanodia & Ashish Murolia Page 129

BC + BC = 1 ...(2) Now applying voltage divider rule


C =1 ...(3) V- = 1k V% = 1 Vo ...(2)
1k + 7k 8
From (2) and (3), if C = 1, then B = 1
From (1) and (2) we have
If B = 1, then from (1) A = 0 . Thus A = 0, B = 1 and C = 1
Vo = 8 # 5 = 5V
5.112 Option (C) is correct. 8
Interrupt is a process of data transfer by which an external device 5.117 Option (D) is correct.
can inform the processor that it is ready for communication. 8085 The truth table is shown below
microprocessor have five interrupts namely TRAP, INTR, RST 7.5,
Z = XQ + YQ
RST 6.5 and RST 5.5
Comparing from the truth table of J - K FF
5.113 Option (A) is correct.
Y = J,
For any RST instruction, location of program transfer is obtained
in following way. X =K
RST x & (x ) 8) 10 " convert in hexadecimal X Y Z
So for RST 6 & (6 ) 8) 10 = (48) 10 = (30) H 0 0 Q
5.114 Option (A) is correct. 0 1 0
Accumulator contains A = 49 H 1 0 1
Register B = 3 AH
SUB B = A minus B
SPECIAL EDITION ( STUDY MATERIAL FORM )
A = 49 H = 01001001
At market Book is available in 3 volume i.e. in 3 book binding
form. But at NODIA Online Store book is available in 10 book
B = 3 AH = 00111010
binding form. Each unit of Book is in separate binding.
2’s complement of (- B) = 11000110 Available Only at NODIA Online Store
A - B = A + (- B)
Click to Buy
010 010 01
& +1 1 0 0 0 1 1 0 www.nodia.co.in
0 0 0 0 1111 1 1 Q1
Carry =1
so here outputA =0F 5.118 Option (B) is correct.
Carry CY =1 In the figure the given counter is mod-10 counter, so frequency of
Sign flag S =1 output is 10k = 1k
10
5.115 Option (C) is correct. 5.119 Option (D) is correct.
The circuit is as shown below :
We have y = A + AB
we know from Distributive property
x + yz = (x + y) (x + z)
Thus y = (A + A) (A + B) = A + B
5.120 Option (C) is correct.
Darligton emitter follower provides a low output impedance in both
logical state (1 or 0). Due to this low output impedance, any stray
capacitance is rapidly charged and discharged, so the output state
Y = B + (B + C ) = B (B + C ) = B
changes quickly. It improves speed of operation.
5.116 Option (B) is correct. 5.121 Option (D) is correct.
The circuit is as shown below
5.122 Option (B) is correct.
For ADC we can write
Analog input = (decimal eq of digital output) # resol
6.6 = (decimal eq. of digital output) # 0.5
6.6 = decimal eq of digital. output
0.5
13.2 = decimal equivalent of digital output so out-
put of ADC is = 1101.
5.123 Option (A) is correct.
The voltage at non-inverting terminal is
We use the K -map as below.
V+ = 1 + 1 = 5
8 2 8
V- = V+ = 5 ...(1)
8
GATE Electronics and Communication Topicwise Solved Paper by RK Kanodia & Ashish Murolia Page 130

address range (111)


A15 A14 A13 A12 A11 A10 A 9 A 8 A7 A6 A5 A 4 A 3 A2 A1 A 0
initial 1 1 1 0 0 0 0 0 0 0 0 0 0 0 0 0
address &7000H
final 1 1 1 1 1 1 1 1 1 1 1 1 1 1 11
So given expression equal to address &7FFFH
= AC + BC + AB so address range is (7 0 0 0 H – 7 F F F H)
5.124 Option (C) is correct. 5.127 Option (C) is correct.
For a binary half-subtractor truth table si given below. Given boolean function is
Z = ABC
Now Z = ABC = ACB = AC + B
Thus Z = AC + B
we have Z = X + Y (1 NOR gate)
where X = AC (1 NAND gate)
from truth table we can find expressions of D & X To implement a NOR gate we required 4 NAND gates as shown
D = A 5 B = AB + AB below in figure.
X = AB
GATE Electronics & Communication
by RK Kanodia
Now in 3 Volume
Purchase Online at maximum discount from online store here total no. of NAND gates required
and get POSTAL and Online Test Series Free = 4+1 = 5
visit www.nodia.co.in 5.128 Option (B) is correct.
For TTL worst cases low voltages are
5.125 Option (D) is correct.
VOL (max) = 0.4 V
From the given figure we can write the output
VIL (max) = 0.8 V
Worst case high voltages are
VOH (min) = 2.4 V
VIH (min) = 2 V
The difference between maximum input low voltage and maximum
output low voltage is called noise margin. It is 0.4 V in case of TTL.
5.129 Option (D) is correct.
From the figure we can see
If A =1 B=0
For the state 010 all preset = 1 and output QA QB QC = 111 so here then y =1 x=0
total no. of states = 5 (down counter) If A =1 B=1
5.126 Option (B) is correct. then also y =1 x=0
We have 4 K RAM (12 address lines)
For more GATE Resources, Mock Test and
Study material join the community
http://www.facebook.com/gateec2014
so for sequence B = 101010....output x and y will be fixed at 0 and
1 respectively.
5.130 Option (D) is correct.
Given 2’s complement no. 1101; the no. is 0011
for 6 digit output we can write the no. is – 000011
2’s complement representation of above no. is 111101
5.131 Option (A) is correct.
5.132 Option (B) is correct.
An I/O Microprocessor controls data flow between main memory
and the I/O device which wants to communicate.
so here chip select logic CS = A15 A14 A13
5.133 Option (D) is correct.
GATE Electronics and Communication Topicwise Solved Paper by RK Kanodia & Ashish Murolia Page 131

5.134 Option (B) is correct. The given gate is ex-OR so output


Dual slope ADC is more accurate. F = AB + AB
5.135 Option (A) is correct. Here input B = 0 so,
Dual form of any identity can be find by replacing all AND function F = A1 + A0 = A
to OR and vice-versa. so here dual form will be
5.145 Option (C) is correct.
(A + B) (A + C) (B + C) = (A + B) (A + C)
EI = Enabled Interput flag ,RST will cause an Interrupt
5.136 Option (B) is correct. only it we enable EI .
Carry flag will be affected by arithmetic instructions only.
5.146 Option (A) is correct.
5.137 Option (C) is correct. Here only for the range 60 to 63 H chipselect will be 0, so peripheral
This is a synchronous counter. we can find output as will correspond in this range only chipselect = 1 for rest of the given
QA QB address ranges.
0 0
5.147 Option (B) is correct.
1 0
By executing instructions one by one
0 1
LXI H, 8A79 H (Load HL pair by value 8A79)
0 0
h H = 8AH L = 79 H
So It counts only three states. It is a mod-3 counter. MOV A, L (copy contain of L to accumulator)
K =3
SPECIAL EDITION ( STUDY MATERIAL FORM )
5.138 Option (B) is correct. At market Book is available in 3 volume i.e. in 3 book binding
5.139 Option (A) is correct. form. But at NODIA Online Store book is available in 10 book
Essential prime implicates for a function is no. of terms that we get binding form. Each unit of Book is in separate binding.
by solving K -map. Here we get 4 terms when solve the K -map. Available Only at NODIA Online Store

Click to Buy
www.nodia.co.in
A = 79 H
ADDH (add contain of H to accumulator)
A = 79 H = 0 1111 0 0 1
H = 8AH = add 1 0 0 0 1 0 1 0
=A= 0 0 0 0 0 0 11
Carry = 1
DAA (Carry Flag is set, so DAA adds 6 to high order four bits)
y = B D + A C D + C AB + CA B 0 1111 0 0 1
so no of prime implicates is 4 DAA add 1 0 0 0 1 0 1 0
5.140 Option (A) is correct. A = 0 0 0 0 0 0 1 1 = 63 H
5.141 Option (B) is correct. MOV H, A (copy contain of A to H)
For a 2 bit multiplier H = 63 H
B1 B0 PCHL (Load program counter by HL pair)
# A1 A0 PC = 6379 H
A 0 B1 A0 B0 5.148 Option (C) is correct.
# A1 B1 A1 B 0 5.149 Option (C) is correct.
C3 C2 C1 C0 NMOS In parallel makes OR Gate & in series makes AND so here
This multiplication is identical to AND operation and then addition. we can have
5.142 Option (C) is correct. F = A (B + C) + DE
In totem pole stage output resistance will be small so it acts like a we took complement because there is another NMOS given above
output buffer. (works as an inverter)
5.143 Option (B) is correct. 5.150 Option (D) is correct.
Consider high output state For a J -K flip flop we have characteristic equation as
fan out = IOH max = 400 mA = 20 Q (t + 1) = JQ (t) + KQ (t)
IIH max 20 mA
Q (t) & Q (t + 1) are present & next states.
Consider low output state
In given figure J = Q (t), K = 1 so
fan out = IOL max = 8 mA = 80
IIL max 0.1 mA Q (t + 1) = Q (t) Q (t) + 0Q (t)
Thus fan out is 20 Q (t + 1) = Q (t)[complement of previous state]
5.144 Option (A) is correct. we have initial input Q (t) = 0
GATE Electronics and Communication Topicwise Solved Paper by RK Kanodia & Ashish Murolia Page 132

so for 6 clock pulses sequence at output Q will be 010101


or Tcoff = 10.24
5.151 Option (C) is correct. 2 # 1024 # (50 - 25) cC
5.152 Option (B) is correct. = 200 mV/cC
By distributive property in boolean algebra we have 5.159 Option (D) is correct.
(A + BC) = (A + B) (A + C) 210 # 8 = 13
No. of chips = 26 #
(A + B) (A + C) = AA + AC + AB + BC 12
2 #4
= A (1 + C) + AB + BC 5.160 Option (C) is correct.
= A + AB + BC Given instruction set
= A (1 + B) + BC = A + BC 1000 LXI SP 27FF
1003 CALL 1006
5.153 Option (A) is correct.
1006 POP H
The current in a p n junction diode is controlled by diffusion of
First Instruction will initialize the SP by a value
majority carriers while current in schottky diode dominated by the
flow of majority carrier over the potential barrier at metallurgical 27FF SP ! 27FF
junction. So there is no minority carrier storage in schottky diode, CALL 1006 will “Push PC” and Load PC by value 1006
so switching time from forward bias to reverse bias is very short PUSH PC will store value of PC in stack
compared to p n junction diode. Hence the propagation delay will PC = 1006
reduces.
5.154 Option (B) is correct.
GATE Electronics & Communication
by RK Kanodia
Now in 3 Volume
Purchase Online at maximum discount from online store
and get POSTAL and Online Test Series Free
now POP H will be executed
visit www.nodia.co.in
which load HL pair by stack values
HL = 1006 and
5.155 Option (D) is correct.
SP = SPl + 2
The total conversion time for different type of ADC are given as–
t is clock period SP = SPl + 2 = SP - 2 + 2 = SP
For flash type & 1t SP = 27FF
Counter type & (2n - t) = 4095 m sec
n = no.of bits
Integrating type conver time > 4095 m sec
successive approximation type nt = 12 m sec
here n = 12 so
nt = 12
12t = 12
so this is succ. app. type ADC.
5.156 Option (D) is correct.
LDA 2003 (Load accumulator by a value 2003 H) so here total no.
of memory access will be 4. For more GATE Resources, Mock Test and
1 = Fetching instruction Study material join the community
2 = Read the value from memory http://www.facebook.com/gateec2014
1 = write value to accumulator
5.157 Option (D) is correct.
Storage capacitance
-12
C = i = 1 # 10
dv 5 - 0.5
b dt l b 20 10-3 l
#
-12 -3
= 1 # 10 # 20 # 10 = 4.4 # 10-15 F
4.5
5.158 Option (A) is correct.
Accuracy ! 1 LSB = Tcoff # DT
2
1 10.24 = T
or coff # DT
2 # 210
GATE Electronics and Communication Topicwise Solved Paper by RK Kanodia & Ashish Murolia Page 133

UNIT 6
2013 TWO MARKS

6.8 The impulse response of a continuous time system is given by


h ^ t h = d ^t - 1h + d ^t - 3h. The value of the step response at t = 2 is
SIGNALS & SYSTEMS (A) 0 (B) 1
(C) 2 (D) 3

6.9 A system described by the differential equation


2
dy dy
+ 5 + 6y ^ t h = x ^ t h. Let x ^ t h be a rectangular pulse given by
dt2 dt
2013 ONE MARK 1 0<t<2
x^t h = *
0 otherwise
6.1 Two systems with impulse responses h1 ^ t h and h2 ^ t h are connected
dy
in cascade. Then the overall impulse response of the cascaded system Assuming that y ^0 h = 0 and = 0 at t = 0 , the Laplace trans-
dt
is given by form of y ^ t h is
(A) product of h1 ^ t h and h2 ^ t h (A) e-2s (B) 1 - e-2s
(B) sum of h1 ^ t h and h2 ^ t h s ^s + 2h^s + 3h s ^s + 2h^s + 3h
-2s
e 1 - e-2s
(C) convolution of h1 ^ t h and h2 ^ t h (C) (D)
^s + 2h^s + 3h ^s + 2h^s + 3h
(D) subtraction of h2 ^ t h from h1 ^ t h
6.10 A system described by a linear, constant coefficient, ordinary, first
6.2 The impulse response of a system is h ^ t h = tu ^ t h. For an input
u ^t - 1h, the output is SPECIAL EDITION ( STUDY MATERIAL FORM )
2 t ^t - 1h
(A) t u ^ t h (B) u ^t - 1h At market Book is available in 3 volume i.e. in 3 book binding
2 2
form. But at NODIA Online Store book is available in 10 book
^t - 1h2 2
(C) u ^t - 1h (D) t - 1 u ^t - 1h binding form. Each unit of Book is in separate binding.
2 2
Available Only at NODIA Online Store
6.3 For a periodic signal Click to Buy
v ^ t h = 30 sin 100t + 10 cos 300t + 6 sin ^500t + p/4h, the fundamental
www.nodia.co.in
frequency in rad/s
(A) 100 (B) 300
order differential equation has an exact solution given by y ^ t h for
(C) 500 (D) 1500 t > 0 , when the forcing function is x ^ t h and the initial condition
is y ^0 h. If one wishes to modify the system so that the solution
A band-limited signal with a maximum frequency of 5 kHz is to
becomes - 2y ^ t h for t > 0 , we need to
6.4

be sampled. According to the sampling theorem, the sampling


(A) change the initial condition to - y ^0 h and the forcing function
frequency which is not valid is
to 2x ^ t h
(A) 5 kHz (B) 12 kHz
(B) change the initial condition to 2y ^0 h and the forcing function
(C) 15 kHz (D) 20 kHz
to - x ^ t h
6.5 Which one of the following statements is NOT TRUE for a continuous (C) change the initial condition to j 2 y ^0 h and the forcing func-
time causal and stable LTI system? tion to j 2 x ^ t h
(A) All the poles of the system must lie on the left side of the jw (D) change the initial condition to - 2y ^0 h and the forcing function
axis to - 2x ^ t h
(B) Zeros of the system can lie anywhere in the s-plane
6.11 The DFT of a vector 8a b c dB is the vector 8a b g dB . Consider
(C) All the poles must lie within s = 1
the product
(D) All the roots of the characteristic equation must be located on
R V
the left side of the jw axis. Sa b c d W
Sd a b c W
6.6 Assuming zero initial condition, the response y ^ t h of the system 8p q r sB = 8a b c dBSc d a b W
given below to a unit step input u ^ t h is S W
Sb c d aW
T X
The DFT of the vector 8p q r sB is a scaled version of
(A) 9a2 b2 g2 d2C (B) 9 a b g dC
(C) 8a + b b + d d + g g + aB (D) 8a b g dB

(A) u ^ t h (B) tu ^ t h
2 2012 ONE MARK
(C) t u ^ t h (D) e-t u ^ t h
2
6.12 The unilateral Laplace transform of f (t) is 2 1 . The unilateral
s +s+1
Let g ^ t h = e- pt , and h ^ t h is a filter matched to g ^ t h. If g ^ t h is Laplace transform of tf (t) is
2
6.7

applied as input to h ^ t h, then the Fourier transform of the output is (A) - 2 s (B) - 2 2s + 1 2
2 2 (s + s + 1) 2 (s + s + 1)
(A) e- pf (B) e- pf /2
(C) e- p f (D) e-2pf
2
(C) 2 s 2 (D) 2 2s + 1 2
(s + s + 1) (s + s + 1)
GATE Electronics and Communication Topicwise Solved Paper by RK Kanodia & Ashish Murolia Page 134

6.13 If x [n] = (1/3) n - (1/2) n u [n], then the region of convergence (ROC) 6.19 A system is defined by its impulse response h (n) = 2n u (n - 2). The
of its z -transform in the z -plane will be system is
(A) 1 < z < 3 (B) 1 < z < 1 (A) stable and causal (B) causal but not stable
3 3 2
(C) stable but not causal (D) unstable and non-causal
(C) 1 < z < 3 (D) 1 < z
2 3 6.20 If the unit step response of a network is (1 - e- at), then its unit
impulse response is
2012 TWO MARKS (A) ae- at (B) a-1 e- at
6.14 The input x (t) and output y (t) of a system are related as (C) (1 - a-1)e- at (D) (1 - a) e- at
t
y (t) = # x (t) cos (3t) dt . The system is
-3
2011 TWO MARKS
(A) time-invariant and stable (B) stable and not time-invari-
ant 6.21 An input x (t) = exp (- 2t) u (t) + d (t - 6) is applied to an LTI system
(C) time-invariant and not stable (D) not time-invariant and not with impulse response h (t) = u (t) . The output is
stable (A) [1 - exp (- 2t)] u (t) + u (t + 6)
(B) [1 - exp (- 2t)] u (t) + u (t - 6)
6.15 The Fourier transform of a signal h (t) is H (jw) = (2 cos w) (sin 2w) /w
. The value of h (0) is (C) 0.5 [1 - exp (- 2t)] u (t) + u (t + 6)
(A) 1/4 (B) 1/2 (D) 0.5 [1 - exp (- 2t)] u (t) + u (t - 6)
(C) 1 (D) 2 6.22 Two systems H1 (Z ) and H2 (Z ) are connected in cascade as shown
GATE Electronics & Communication below. The overall output y (n) is the same as the input x (n) with a
one unit delay. The transfer function of the second system H2 (Z ) is
by RK Kanodia
Now in 3 Volume
Purchase Online at maximum discount from online store
and get POSTAL and Online Test Series Free
visit www.nodia.co.in 1 - 0.6z-1 z-1 (1 - 0.6z-1)
(A) (B)
z-1 (1 - 0.4z-1) (1 - 0.4z-1)
6.16 Let y [n] denote the convolution of h [n] and g [n], where
z-1 (1 - 0.4z-1) 1 - 0.4 z-1
h [n] = (1/2) n u [n] and g [n] is a causal sequence. If y [0] = 1 and (C) (D)
(1 - 0.6z-1) z-1 (1 - 0.6z-1)
y [1] = 1/2, then g [1] equals
(A) 0 (B) 1/2 6.23 The first six points of the 8-point DFT of a real valued sequence are
(C) 1 (D) 3/2 5, 1 - j 3, 0, 3 - j 4, 0 and 3 + j 4 . The last two points of the DFT
are respectively
(A) 0, 1 - j 3 (B) 0, 1 + j 3
2011 ONE MARK
(C) 1 + j3, 5 (D) 1 - j 3, 5
2
6.17 The differential equation 100 ddty - 20 dy
2
dt + y = x (t) describes a system
with an input x (t) and an output y (t). The system, which is initially
2010 ONE MARK
relaxed, is excited by a unit step input. The output y ^ t h can be
represented by the waveform 6.24 The trigonometric Fourier series for the waveform f (t) shown below
contains

For more GATE Resources, Mock Test and


Study material join the community
http://www.facebook.com/gateec2014

6.18 The trigonometric Fourier series of an even function does not have
the
(A) dc term (B) cosine terms
(A) only cosine terms and zero values for the dc components
(C) sine terms (D) odd harmonic terms
GATE Electronics and Communication Topicwise Solved Paper by RK Kanodia & Ashish Murolia Page 135

(B) only cosine terms and a positive value for the dc components Which of the above statements are correct ?
(C) only cosine terms and a negative value for the dc components (A) P and S (B) P and R
(D) only sine terms and a negative value for the dc components (C) Q and S (D) Q and R

6.25 Consider the z -transform x (z) = 5z2 + 4z-1 + 3; 0 < z < 3. The 6.32 A function is given by f (t) = sin2 t + cos 2t . Which of the following
inverse z - transform x [n] is is true ?
(A) 5d [n + 2] + 3d [n] + 4d [n - 1] (A) f has frequency components at 0 and 1 Hz
2p
(B) 5d [n - 2] + 3d [n] + 4d [n + 1]
(B) f has frequency components at 0 and 1 Hz
(C) 5u [n + 2] + 3u [n] + 4u [n - 1] p
(D) 5u [n - 2] + 3u [n] + 4u [n + 1] (C) f has frequency components at 1 and 1 Hz
2p p
Two discrete time system with impulse response h1 [n] = d [n - 1]
(D) f has frequency components at 0.1 and 1 Hz
6.26

and h2 [n] = d [n - 2] are connected in cascade. The overall impulse 2p p


response of the cascaded system is
6.33 The ROC of z -transform of the discrete time sequence
(A) d [n - 1] + d [n - 2] (B) d [n - 4]
x (n) = b 1 l u (n) - b 1 l u (- n - 1) is
n n
(C) d [n - 3] (D) d [n - 1] d [n - 2] 3 2
6.27 For a N -point FET algorithm N = 2m which one of the following (A) 1 < z < 1 (B) z > 1
3 2 2
statements is TRUE ?
(A) It is not possible to construct a signal flow graph with both SPECIAL EDITION ( STUDY MATERIAL FORM )
input and output in normal order At market Book is available in 3 volume i.e. in 3 book binding
(B) The number of butterflies in the m th stage in N/m form. But at NODIA Online Store book is available in 10 book
(C) In-place computation requires storage of only 2N data binding form. Each unit of Book is in separate binding.
(D) Computation of a butterfly requires only one complex multipli- Available Only at NODIA Online Store
cation. Click to Buy
www.nodia.co.in
2010 TWO MARKS

3s + 1 (C) z < 1 (D) 2 < z < 3


Given f (t) = L ; 3
s + 4s2 + (k - 3) s E
6.28
-1
. If lim f (t) = 1, then the value 3
t"3
of k is
(A) 1 (B) 2 2009 TWO MARKS
(C) 3 (D) 4 6.34 Given that F (s) is the one-side Laplace transform of f (t), the Laplace
t
6.29 A continuous time LTI system is described by transform of #0 f (t) dt is
d 2 y (t) dy (t) dx (t) (A) sF (s) - f (0) (B) 1 F (s)
+4 + 3y (t) = 2 + 4x (t)
dt 2 dt dt s
Assuming zero initial conditions, the response y (t) of the above s
(D) 1 [F (s) - f (0)]
system for the input x (t) = e-2t u (t) is given by
(C) #0 F (t) dt
s
(A) (et - e3t) u (t) (B) (e-t - e-3t) u (t)
6.35 A system with transfer function H (z) has impulse response h (.)
(C) (e-t + e-3t) u (t) (D) (et + e3t) u (t) defined as h (2) = 1, h (3) =- 1 and h (k) = 0 otherwise. Consider the
6.30 The transfer function of a discrete time LTI system is given by following statements.
S1 : H (z) is a low-pass filter.
2 - 3 z-1 S2 : H (z) is an FIR filter.
H (z) = 4
1 - z + 1 z-2
3 -1 Which of the following is correct?
4 8 (A) Only S2 is true
Consider the following statements:
S1: The system is stable and causal for ROC: z > 1/2 (B) Both S1 and S2 are false
S2: The system is stable but not causal for ROC: z < 1/4 (C) Both S1 and S2 are true, and S2 is a reason for S1
S3: The system is neither stable nor causal for ROC: (D) Both S1 and S2 are true, but S2 is not a reason for S1
1/4 < z < 1/2
Which one of the following statements is valid ? 6.36 Consider a system whose input x and output y are related by the
# x (t - t) g (2t) dt where h (t) is shown in the graph.
3
(A) Both S1 and S2 are true (B) Both S2 and S3 are true equation y (t) =
-3
(C) Both S1 and S3 are true (D) S1, S2 and S3 are all true

2009 ONE MARK

6.31 The Fourier series of a real periodic function has only


(P) cosine terms if it is even Which of the following four properties are possessed by the system
(Q) sine terms if it is even ?
(R) cosine terms if it is odd BIBO : Bounded input gives a bounded output.
(S) sine terms if it is odd
GATE Electronics and Communication Topicwise Solved Paper by RK Kanodia & Ashish Murolia Page 136

Causal : The system is causal, comes zero are


LP : The system is low pass. (A) p, 2p (B) 0.5p, 1.5p
LTI : The system is linear and time-invariant. (C) 0, p (D) 2p, 2.5p
(A) Causal, LP (B) BIBO, LTI
(C) BIBO, Causal, LTI (D) LP, LTI 6.43 A discrete time linear shift - invariant system has an impulse
response h [n] with h [0] = 1, h [1] =- 1, h [2] = 2, and zero otherwise
6.37 The 4-point Discrete Fourier Transform (DFT) of a discrete time The system is given an input sequence x [n] with x [0] = x [2] = 1,
sequence {1,0,2,3} is and zero otherwise. The number of nonzero samples in the output
(A) [0, - 2 + 2j , 2, - 2 - 2j ] (B) [2, 2 + 2j , 6, 2 - 2j ] sequence y [n], and the value of y [2] are respectively
(C) [6, 1 - 3j , 2, 1 + 3j ] (D) [6, - 1 + 3j , 0, - 1 - 3j ] (A) 5, 2 (B) 6, 2
(C) 6, 1 (D) 5, 3
An LTI system having transfer function s +s 2+s 1+ 1 and input
2
6.38 2

x (t) = sin (t + 1) is in steady state. The output is sampled at a rate 6.44 Let x (t) be the input and y (t) be the output of a continuous time
ws rad/s to obtain the final output {x (k)}. Which of the following system. Match the system properties P1, P2 and P3 with system
is true ? relations R1, R2, R3, R4
(A) y (.) is zero for all sampling frequencies ws Properties Relations
(B) y (.) is nonzero for all sampling frequencies ws P1 : Linear but NOT time - invariant R1 : y (t) = t2 x (t)
P2 : Time - invariant but NOT linear R2 : y (t) = t x (t)
(C) y (.) is nonzero for ws > 2 , but zero for ws < 2
P3 : Linear and time - invariant R3 : y (t) = x (t)
(D) y (.) is zero for ws > 2 , but nonzero for w2 < 2 R4 : y (t) = x (t - 5)
(A) (P1, R1), (P2, R3), (P3, R4)
GATE Electronics & Communication (B) (P1, R2), (P2, R3), (P3, R4)
by RK Kanodia (C) (P1, R3), (P2, R1), (P3, R2)
(D) (P1, R1), (P2, R2), (P3, R3)
Now in 3 Volume
Purchase Online at maximum discount from online store 6.45 {x (n)} is a real - valued periodic sequence with a period N . x (n)
and X (k) form N-point Discrete Fourier Transform (DFT) pairs.
and get POSTAL and Online Test Series Free N-1

visit www.nodia.co.in The DFT Y (k) of the sequence y (n) = 1 / x (r) x (n + r) is


N r=0
N-1
2008 ONE MARK (A) X (k) 2 (B) 1 / X (r) X (k + r)
N r=0
The input and output of a continuous time system are respectively N-1
(C) 1 / X (r) X (k + r)
6.39
(D) 0
denoted by x (t) and y (t). Which of the following descriptions N r=0
corresponds to a causal system ?
(A) y (t) = x (t - 2) + x (t + 4) (B) y (t) = (t - 4) x (t + 1)
Statement for Linked Answer Question 6.31 and 6.32:
(C) y (t) = (t + 4) x (t - 1) (D) y (t) = (t + 5) x (t + 5)
In the following network, the switch is closed at t = 0- and the
6.40 The impulse response h (t) of a linear time invariant continuous sampling starts from t = 0 . The sampling frequency is 10 Hz.
time system is described by h (t) = exp (at) u (t) + exp (bt) u (- t)
where u (- t) denotes the unit step function, and a and b are real
constants. This system is stable if
(A) a is positive and b is positive
(B) a is negative and b is negative
(C) a is negative and b is negative
(D) a is negative and b is positive

For more GATE Resources, Mock Test and


2008 TWO MARKS
Study material join the community
6.41 A linear, time - invariant, causal continuous time system has a http://www.facebook.com/gateec2014
rational transfer function with simple poles at s =- 2 and s =- 4
and one simple zero at s =- 1. A unit step u (t) is applied at the 6.46 The samples x (n), n = (0, 1, 2, ...) are given by
input of the system. At steady state, the output has constant value (A) 5 (1 - e-0.05n) (B) 5e-0.05n
of 1. The impulse response of this system is (C) 5 (1 - e-5n) (D) 5e-5n
(A) [exp (- 2t) + exp (- 4t)] u (t)
6.47 The expression and the region of convergence of the z -transform of
(B) [- 4 exp (- 2t) - 12 exp (- 4t) - exp (- t)] u (t)
the sampled signal are
(C) [- 4 exp (- 2t) + 12 exp (- 4t)] u (t) 5z , z < e-0.05
(A) 5z 5 , z < e-5 (B)
(D) [- 0.5 exp (- 2t) + 1.5 exp (- 4t)] u (t) z-e z - e-0.05
(C) 5z , z > e-0.05 (D) 5z -5 , z > e-5
6.42 The signal x (t) is described by z-e -0.05
z-e
1 for - 1 # t # + 1
x (t) = )
0 otherwise Statement for Linked Answer Question 6.33 & 6.34:
Two of the angular frequencies at which its Fourier transform be-
The impulse response h (t) of linear time - invariant continuous
GATE Electronics and Communication Topicwise Solved Paper by RK Kanodia & Ashish Murolia Page 137

time system is given by h (t) = exp (- 2t) u (t), where u (t) denotes 3 t=0
(B) d (t) = )
the unit step function. 0 otherwise
1 t=0
#
3
6.48 The frequency response H (w) of this system in terms of angular (C) d (t) = ) and d (t) dt = 1
frequency w, is given by H (w) 0 otherwise -3

3 t=0
1 (B) sin w #
3
(A) (D) d (t) = ) and d (t) dt = 1
1 + j2w w 0 otherwise -3

1 jw If the region of convergence of x1 [n] + x2 [n] is 1 < z < 2 then the


(C) (D) 6.58
3 3
2 + jw 2 + jw region of convergence of x1 [n] - x2 [n] includes
6.49 The output of this system, to the sinusoidal input x (t) = 2 cos 2t for (A) 1 < z < 3 (B) 2 < z <3
3 3
all time t , is
(A) 0 (B) 2-0.25 cos (2t - 0.125p) (C) 3 < z < 3 (D) 1 < z <2
2 3 3
(C) 2-0.5 cos (2t - 0.125p) (D) 2-0.5 cos (2t - 0.25p)
6.59 In the system shown below, x (t) = (sin t) u (t) In steady-state, the
response y (t) will be
2007 ONE MARK

6.50 If the Laplace transform of a signal Y (s) = 1 , then its final


s (s - 1)
value is
(A) - 1 (B) 0
(C) 1 (D) Unbounded SPECIAL EDITION ( STUDY MATERIAL FORM )
At market Book is available in 3 volume i.e. in 3 book binding
2007 TWO MARKS form. But at NODIA Online Store book is available in 10 book
6.51 The 3-dB bandwidth of the low-pass signal e-t u (t), where u (t) is the binding form. Each unit of Book is in separate binding.
Available Only at NODIA Online Store
unit step function, is given by
(A) 1 Hz (B) 1 2 - 1 Hz Click to Buy
2p 2p
www.nodia.co.in
(C) 3 (D) 1 Hz

` 4j ` 4j
6.52 A 5-point sequence x [n] is given as x [- 3] = 1, x [- 2] = 1, x [- 1] = 0, (A) 1 sin t - p (B) 1 sin t + p
x [0] = 5 and x [1] = 1. Let X (eiw) denoted the discrete-time Fourier 2 2
transform of x [n]. The value of #
p
X (e jw) dw is (C) 1 e-t sin t (D) sin t - cos t
-p 2
(A) 5 (B) 10p
(C) 16p (D) 5 + j10p 2006 TWO MARKS

6.60 Consider the function f (t) having Laplace transform


6.53 The z -transform X (z) of a sequence x [n] is given by X [z] = 1 -0.25z . -1

It is given that the region of convergence of X (z) includes the unit F (s) = 2 w0 2 Re [s] > 0
s + w0
circle. The value of x [0] is
(A) - 0.5 (B) 0 The final value of f (t) would be
(A) 0 (B) 1
(C) 0.25 (D) 05
(C) - 1 # f (3) # 1 (D) 3
6.54 A Hilbert transformer is a
(A) non-linear system (B) non-causal system
6.61 A system with input x [n] and output y [n] is given as y [n] = (sin 56 pn) x [n]
. The system is
(C) time-varying system (D) low-pass system
(A) linear, stable and invertible
6.55 The frequency response of a linear, time-invariant system is given by (B) non-linear, stable and non-invertible
H (f) = 1 + j510pf . The step response of the system is (C) linear, stable and non-invertible
(B) 5 61 - e- 5@ u (t)
t
(A) 5 (1 - e-5t) u (t) (D) linear, unstable and invertible
(C) 1 (1 - e-5t) u (t) (D) 1 ^1 - e- 5 h u (t)
t

2 5 6.62 The unit step response of a system starting from rest is given by
c (t) = 1 - e-2t for t $ 0 . The transfer function of the system is
2006 ONE MARK (A) 1 (B) 2
1 + 2s 2+s
6.56 Let x (t) * X (jw) be Fourier Transform pair. The Fourier Transform (C) 1 (D) 2s
of the signal x (5t - 3) in terms of X (jw) is given as 2+s 1 + 2s
j3w jw j3w jw
(A) 1 e- 5 X b l (B) 1 e 5 X b l The unit impulse response of a system is f (t) = e-t, t $ 0 . For this
5 5 5 5 6.63

jw jw system the steady-state value of the output for unit step input is
(C) 1 e-j3w X b l (D) 1 e j3w X b l equal to
5 5 5 5
(A) - 1 (B) 0
6.57 The Dirac delta function d (t) is defined as (C) 1 (D) 3
1 t=0
(A) d (t) = )
0 otherwise
GATE Electronics and Communication Topicwise Solved Paper by RK Kanodia & Ashish Murolia Page 138

2005 ONE MARK (B) 41


6.64 Choose the function f (t); - 3 < t < 3 for which a Fourier series (C) 42
cannot be defined. (D) 82
(A) 3 sin (25t)
(B) 4 cos (20t + 3) + 2 sin (710t)
2005 TWO MARKS
(C) exp (- t ) sin (25t)
(D) 1 6.70 The output y (t) of a linear time invariant system is related to its
input x (t) by the following equations
6.65 The function x (t) is shown in the figure. Even and odd parts of a y (t)= 0.5x (t - td + T) + x (t - td ) + 0.5x (t - td + T)
unit step function u (t) are respectively,
The filter transfer function H (w) of such a system is given by
(A) (1 + cos wT) e-jwt d

(B) (1 + 0.5 cos wT) e-jwt d

(C) (1 - cos wT) e-jwt d

(D) (1 - 0.5 cos wT) e-jwt d

Match the following and choose the correct combination.


(A) 1 , 1 x (t) (B) - 1 , 1 x (t)
6.71

2 2 2 2 Group 1
(C) 1 , - 1 x (t) (D) - 1 , - 1 x (t) E. Continuous and aperiodic signal
2 2 2 2 F. Continuous and periodic signal
GATE Electronics & Communication G. Discrete and aperiodic signal
by RK Kanodia H. Discrete and periodic signal
Group 2
Now in 3 Volume 1. Fourier representation is continuous and aperiodic
Purchase Online at maximum discount from online store 2. Fourier representation is discrete and aperiodic
and get POSTAL and Online Test Series Free 3. Fourier representation is continuous and periodic
visit www.nodia.co.in 4. Fourier representation is discrete and periodic
(A) E - 3, F - 2, G - 4, H-1
(B) E - 1, F - 3, G - 2, H-4
(C) E - 1, F - 2, G - 3, H-4
6.66 The region of convergence of z - transform of the sequence (D) E - 2, F - 1, G - 4, H-3
5 n 6 n
b 6 l u (n) - b 5 l u (- n - 1) must be 6.72 A signal x (n) = sin (w0 n + f) is the input to a linear time- invariant
system having a frequency response H (e jw). If the output of the
(A) z < 5 (B) z > 5 system Ax (n - n0) then the most general form of +H (e jw) will be
6 6
(A) - n0 w0 + b for any arbitrary real
(C) 5 < z < 6 (D) 6 < z < 3
6 5 5 (B) - n0 w0 + 2pk for any arbitrary integer k
6.67 Which of the following can be impulse response of a causal system ? (C) n0 w0 + 2pk for any arbitrary integer k
(D) - n0 w0 f

Statement of linked answer question 6.59 and 6.60 :


A sequence x (n) has non-zero values as shown in the figure.
For more GATE Resources, Mock Test and
Study material join the community
http://www.facebook.com/gateec2014

6.68 Let x (n) = ( 12 ) n u (n), y (n) = x2 (n) and Y (e jw) be the Fourier
transform of y (n) then Y (e j0)
(A) 1
4
(B) 2

(C) 4
x ( n2 - 1), For n even
The sequence y (n) = *
(D) 4
3 6.73 will be
0, For n odd
6.69 The power in the signal s (t) = 8 cos (20p - p2 ) + 4 sin (15pt) is
(A) 40
GATE Electronics and Communication Topicwise Solved Paper by RK Kanodia & Ashish Murolia Page 139

The system is stable only if


(A) a = 2 , b < 2 (B) a > 2, b > 2
(C) a < 2 , any value of b (D) b < 2 , any value of a

6.81 The impulse response h [n] of a linear time invariant system is given
as
-2 2 n = 1, - 1
h [ n] = * 4 2 n = 2, - 2
0 otherwise
If the input to the above system is the sequence e jpn/4 , then the
output is
(A) 4 2 e jpn/4 (B) 4 2 e-jpn/4
(C) 4e jpn/4 (D) - 4e jpn/4

6.82 Let x (t) and y (t) with Fourier transforms F (f) and Y (f) respectively
be related as shown in Fig. Then Y (f) is

SPECIAL EDITION ( STUDY MATERIAL FORM )


At market Book is available in 3 volume i.e. in 3 book binding
form. But at NODIA Online Store book is available in 10 book
binding form. Each unit of Book is in separate binding.
Available Only at NODIA Online Store

Click to Buy
6.74 The Fourier transform of y (2n) will be www.nodia.co.in
(A) e-j2w [cos 4w + 2 cos 2w + 2] (B) cos 2w + 2 cos w + 2
-jw
(C) e [cos 2w + 2 cos w + 2] (D) e-j2w [cos 2w + 2 cos + 2]

6.75 For a signal x (t) the Fourier transform is X (f). Then the inverse
Fourier transform of X (3f + 2) is given by
j4pt
(A) 1 x` t j e j3pt (B) 1 x` t j e - 3
2 2 3 3
(C) 3x (3t) e-j4pt (D) x (3t + 2)

(A) - 1 X (f/2) e-jpf (B) - 1 X (f/2) e j2pf


2004 ONE MARK 2 2
6.76 The impulse response h [n] of a linear time-invariant system is given (C) - X (f/2) e j2pf (D) - X (f/2) e-j2pf
by h [n] = u [n + 3] + u [n - 2) - 2n [n - 7] where u [n] is the unit step
sequence. The above system is 2003 ONE MARK
(A) stable but not causal (B) stable and causal
6.83 The Laplace transform of i (t) is given by
(C) causal but unstable (D) unstable and not causal
I (s) = 2
6.77 The z -transform of a system is H (z) = z -z0.2 . If the ROC is z < 0.2 s (1 + s)
, then the impulse response of the system is At t " 3, The value of i (t) tends to
(A) (0.2) n u [n] (B) (0.2) n u [- n - 1] (A) 0 (B) 1
(C) - (0.2) n u [n] (D) - (0.2) n u [- n - 1] (C) 2 (D) 3

6.78 The Fourier transform of a conjugate symmetric function is always 6.84 The Fourier series expansion of a real periodic signal with
(A) imaginary (B) conjugate anti-symmetric fundamental frequency f0 is given by gp (t) = /cn e j2pf t . It is given
0

(C) real (D) conjugate symmetric that c3 = 3 + j5 . Then c-3 is n =- 3

(A) 5 + j3 (B) - 3 - j5
(C) - 5 + j3 (D) 3 - j5
2004 TWO MARKS
6.85 Let x (t) be the input to a linear, time-invariant system. The required
6.79 Consider the sequence x [n] = [- 4 - j51 + j25]. The conjugate anti-
- output is 4p (t - 2). The transfer function of the system should be
symmetric part of the sequence is
(A) 4e j4pf (B) 2e-j8pf
(A) [- 4 - j2.5, j2, 4 - j2.5] (B) [- j2.5, 1, j2.5]
(C) 4e-j4pf (D) 2e j8pf
(C) [- j2.5, j2, 0] (D) [- 4, 1, 4]
6.86 A sequence x (n) with the z -transform X (z) = z 4 + z2 - 2z + 2 - 3z-4
6.80 A causal LTI system is described by the difference equation
is applied as an input to a linear, time-invariant system with the
2y [n] = ay [n - 2] - 2x [n] + bx [n - 1] impulse response h (n) = 2d (n - 3) where
GATE Electronics and Communication Topicwise Solved Paper by RK Kanodia & Ashish Murolia Page 140

1, n = 0
d (n) = ) (A) e f u (f) (B) e-f u (f)
0, otherwise
The output at n = 4 is (C) e f u (- f) (D) e-f u (- f)
(A) - 6 (B) zero 6.93 A linear phase channel with phase delay Tp and group delay Tg must
(C) 2 (D) - 4 have
(A) Tp = Tg = constant
2003 TWO MARKS (B) Tp \ f and Tg \ f
(C) Tp = constant and Tg \ f ( f denote frequency)
6.87 Let P be linearity, Q be time-invariance, R be causality and S be
stability. A discrete time system has the input-output relationship, (D) Tp \ f and Tp = constant
x (n) n$1
y (n) = *0, n=0 2002 TWO MARKS
x (n + 1) n # - 1 6.94 The Laplace transform of continuous - time signal x (t) is X (s) = 5-s
s2 - s - 2
where x (n) is the input and y (n) is the output. The above system . If the Fourier transform of this signal exists, the x (t) is
has the properties (A) e2t u (t) - 2e-t u (t) (B) - e2t u (- t) + 2e-t u (t)
(A) P, S but not Q, R (C) - e2t u (- t) - 2e-t u (t) (D) e2t u (- t) - 2e-t u (t)
(B) P, Q, S but not R
6.95 If the impulse response of discrete - time system is
(C) P, Q, R, S h [n] =- 5n u [- n - 1],
then the system function H (z) is equal to
GATE Electronics & Communication
(A) - z and the system is stable
by RK Kanodia z-5
Now in 3 Volume (B) z and the system is stable
z-5
Purchase Online at maximum discount from online store
(C) - z and the system is unstable
and get POSTAL and Online Test Series Free z-5
visit www.nodia.co.in (D) z and the system is unstable
z-5
(D) Q, R, S but not P
2001 ONE MARK

6.96 The transfer function of a system is given by H (s) = 2 1 . The


Common Data For Q. 6.73 & 6.74 : impulse response of the system is s (s - 2)
The system under consideration is an RC low-pass filter (RC-LPF) (A) (t2 * e-2t) u (t) (B) (t * e2t) u (t)
with R = 1 kW and C = 1.0 mF. (C) (te-2 t) u (t) (D) (te-2t) u (t)
6.88 Let H (f) denote the frequency response of the RC-LPF. Let f1 be
H (f1)
6.97 The region of convergence of the z - transform of a unit step function
the highest frequency such that 0 # f # f1 $ 0.95 . Then f1 is
H (0)
(in Hz) is (A) z > 1 (B) z < 1
(A) 324.8 (B) 163.9 (C) (Real part of z ) > 0 (D) (Real part of z ) < 0
(C) 52.2 (D) 104.4
6.98 Let d (t) denote the delta function. The value of the integral
Let tg (f) be the group delay function of the given RC-LPF and
# d (t) cos b 3t l dt is
6.89 3

f2 = 100 Hz. Then tg (f2) in ms, is -3 2


(A) 0.717 (B) 7.17
(C) 71.7 (D) 4.505 For more GATE Resources, Mock Test and
Study material join the community
2002 ONE MARK
http://www.facebook.com/gateec2014
6.90 Convolution of x (t + 5) with impulse function d (t - 7) is equal to
(A) x (t - 12) (B) x (t + 12) (A) 1 (B) - 1
(C) x (t - 2) (D) x (t + 2) (C) 0 (D) p2

6.91 Which of the following cannot be the Fourier series expansion of a


6.99 If a signal f (t) has energy E , the energy of the signal f (2t) is equal
periodic signal? to
(A) x (t) = 2 cos t + 3 cos 3t (A) 1 (B) E/2
(B) x (t) = 2 cos pt + 7 cos t (C) 2E (D) 4E
(C) x (t) = cos t + 0.5
(D) x (t) = 2 cos 1.5pt + sin 3.5pt 2001 TWO MARKS

1 6.100 The impulse response functions of four linear systems S1, S2, S3, S4
6.92 The Fourier transform F {e-1 u (t)} is equal to . Therefore,
1 + j2pf are given respectively by
F'
1 + j2pt 1
1 is
GATE Electronics and Communication Topicwise Solved Paper by RK Kanodia & Ashish Murolia Page 141

h1 (t) = 1, h2 (t) = u (t),


u (t)
h3 (t) = and
t+1
h 4 (t) = e-3t u (t)
where u (t) is the unit step function. Which of these systems is
time invariant, causal, and stable?
(A) S1 (B) S2
(C) S3 (D) S4

2000 ONE MARK

6.101 Given that L [f (t)] = s2+ 2 , L [g (t)] = s2 + 1 and


t s +1 (s + 3) (s + 2)
#
h (t) = f (t) g (t - t) dt .
0
L [h (t)] is
2 A system has a phase response given by f (w), where w is the
(A) s + 1 1 6.107
(B)
s+3 s+3 angular frequency. The phase delay and group delay at w = w0 are
s2 + 1 respectively given by
(C) + s+2 (D) None of the above
(s + 3)( s + 2) s2 + 1

6.102
2
The Fourier Transform of the signal x (t) = e-3t is of the following
SPECIAL EDITION ( STUDY MATERIAL FORM )
form, where A and B are constants : At market Book is available in 3 volume i.e. in 3 book binding
(A) Ae-B f (B) Ae-Bf
2
form. But at NODIA Online Store book is available in 10 book
(C) A + B f 2 (D) Ae-Bf binding form. Each unit of Book is in separate binding.
Available Only at NODIA Online Store
6.103 A system with an input x (t) and output y (t) is described by the Click to Buy
relations : y (t) = tx (t). This system is
(A) linear and time - invariant www.nodia.co.in
(B) linear and time varying
(C) non - linear and time - invariant f (w0) df (w) d2 f (w0)
(A) - , - (B) f (wo), -
(D) non - linear and time - varying w0 dw w = w 0 dw2 w = wo
df (w)
(C) wo , -
wo
6.104 A linear time invariant system has an impulse response e2t, t > 0 . If f (wo) d (w) w = w
(D) wo f (wo), #
-3
f (l)
the initial conditions are zero and the input is e3t , the output for o

t > 0 is
1999 ONE MARK
(A) e3t - e2t
(B) e5t 6.108 The z -transform F (z) of the function f (nT) = anT is
(C) e3t + e2t (A) z T (B) z T
z-a z+a
(D) None of these z z
(C) (D)
z - a-T z + a-T
6.109 If [f (t)] = F (s), then [f (t - T)] is equal to
2000 TWO MARKS
(A) esT F (s) (B) e-sT F (s)
6.105 One period (0, T) each of two periodic waveforms W1 and W2 are shown F (s) F (s)
in the figure. The magnitudes of the nth Fourier series coefficients of (C) (D)
1-e sT
1 - e-sT
W1 and W2 , for n $ 1, n odd, are respectively proportional to
6.110 A signal x (t) has a Fourier transform X (w). If x (t) is a real and odd
function of t , then X (w) is
(A) a real and even function of w
(B) a imaginary and odd function of w
(C) an imaginary and even function of w
(D) a real and odd function of w

(A) n-3 and n-2


(B) n-2 and n-3 1999 TWO MARKS

(C) n-1 and n-2 6.111 The Fourier series representation of an impulse train denoted by
(D) n-4 and n-2 3
s (t) = / d (t - nT0) is given by
n =- 3
Let u (t) be the step function. Which of the waveforms in the figure j2pnt jpnt
(A) 1 / exp - (B) 1 / exp -
6.106 3 3

corresponds to the convolution of u (t) - u (t - 1) with u (t) - u (t - 2) T0 n =- 3 T0 T0 n =- 3 T0


? jpnt j2pnt
(C) 1 / exp (D) 1 / exp
3 3

T0 n =- 3 T0 T0 n =- 3 T0
6.112 The z -transform of a signal is given by
GATE Electronics and Communication Topicwise Solved Paper by RK Kanodia & Ashish Murolia Page 142

1z-1 (1 - z-4) 1997 ONE MARK


C (z) =
4 (1 - z-1) 2 6.120 The function f (t) has the Fourier Transform g (w). The Fourier
Its final value is Transform
(A) 1/4 (B) zero 3

(C) 1.0 (D) infinity ff (t) g (t) e = # g (t) e-jwt dt o is


-3
(A) 1 f (w) (B) 1 f (- w)
2p 2p
1998 ONE MARK
(C) 2pf (- w) (D) None of the above
6.113 If F (s) = 2 w 2 , then the value of Limf (t)
s +w t"3 6.121 The Laplace Transform of eat cos (at) is equal to
(A) cannot be determined (B) is zero (s - a) (s + a)
(A) 2 2 (B)
(C) is unity (D) is infinite (s - a) + a (s - a) 2 + a2
(C) 1 (D) None of the above
6.114 The trigonometric Fourier series of a even time function can have (s - a) 2
only
(A) cosine terms (B) sine terms 1996 ONE MARK
(C) cosine and sine terms (D) d.c and cosine terms 6.122 The trigonometric Fourier series of an even function of time does
not have the
6.115 A periodic signal x (t) of period T0 is given by
(A) dc term (B) cosine terms
(C) sine terms (D) odd harmonic terms
GATE Electronics & Communication
6.123 The Fourier transform of a real valued time signal has
by RK Kanodia (A) odd symmetry (B) even symmetry
Now in 3 Volume (C) conjugate symmetry (D) no symmetry
Purchase Online at maximum discount from online store
and get POSTAL and Online Test Series Free
visit www.nodia.co.in
1, t < T1
x (t) = *
0, T1 < t < T0
2
The dc component of x (t) is
(A) T1 (B) T1
T0 2T0
(C) 2T1 (D) T0
T0 T1
6.116 The unit impulse response of a linear time invariant system is the
unit step function u (t). For t > 0 , the response of the system to an
excitation e-at u (t), a > 0 will be
(A) ae-at (B) (1/a) (1 - e-at)
(C) a (1 - e-at) (D) 1 - e-at
3
6.117 The z-transform of the time function / d (n - k) is
k=0

(A) z - 1 (B) z
z z-1 For more GATE Resources, Mock Test and
(C) z (D)
(z - 1) 2 Study material join the community
(z - 1) 2 z
http://www.facebook.com/gateec2014
6.118 A distorted sinusoid has the amplitudes A1, A2, A 3, .... of the
fundamental, second harmonic, third harmonic,..... respectively. The
total harmonic distortion is
2 2
(A) A2 + A 3 + .... (B) A 2 + A 3 + .....
A1 A1
A 22 + A 32 + ..... A 22 + A 32 + .....
(C) (D) c A1 m
A 12 + A 22 + A 32 + ....
6.119 The Fourier transform of a function x (t) is X (f). The Fourier
dX (t)
transform of will be
df
dX (f)
(A) (B) j2pfX (f)
df
X (f)
(C) jfX (f) (D)
jf
GATE Electronics and Communication Topicwise Solved Paper by RK Kanodia & Ashish Murolia Page 143

SOLUTIONS Given, the maximum frequency of the band-limited signal


fm = 5 kHz
According to the Nyquist sampling theorem, the sampling frequen-
cy must be greater than the Nyquist frequency which is given as
fN = 2fm = 2 # 5 = 10 kHz
6.1 Option (C) is correct. So, the sampling frequency fs must satisfy
If the two systems with impulse response h1 ^ t h and h2 ^ t h are fs $ fN
connected in cascaded configuration as shown in figure, then the fs $ 10 kHz
overall response of the system is the convolution of the individual
only the option (A) doesn’t satisfy the condition therefore, 5 kHz
impulse responses.
is not a valid sampling frequency.
6.5 Option (C) is correct.
For a system to be casual, the R.O.C of system transfer function
H ^s h which is rational should be in the right half plane and to the
right of the right most pole.
For the stability of LTI system. All poles of the system
6.2 Option (C) is correct. should lie in the left half of S -plane and no repeated pole should
Given, the input be on imaginary axis. Hence, options (A), (B), (D) satisfies an LTI
x ^ t h = u ^t - 1h
SPECIAL EDITION ( STUDY MATERIAL FORM )
At market Book is available in 3 volume i.e. in 3 book binding
It’s Laplace transform is
-s form. But at NODIA Online Store book is available in 10 book
X ^s h = e binding form. Each unit of Book is in separate binding.
s
Available Only at NODIA Online Store
The impulse response of system is given
h^t h = t u^t h
Click to Buy
Its Laplace transform is www.nodia.co.in
H ^s h = 12
s system stability and causality both.
Hence, the overall response at the output is But, Option (C) is not true for the stable system as, S = 1
Y ^s h = X ^s h H ^s h have one pole in right hand plane also.
-s
Option (B) is correct.
=e3
6.6

s The Laplace transform of unit step fun n is


its inverse Laplace transform is U ^s h = 1
s
^t - 1h2
y^t h = u ^t - 1h So, the O/P of the system is given as
2
Y ^s h = b 1 lb 1 l
6.3 Option (A) is correct. s s
Given, the signal = 12
s
v ^ t h = 30 sin 100t + 10 cos 300t + 6 sin ^500t + p4 h For zero initial condition, we check
So we have dy ^ t h
u^t h =
w1 = 100 rad/s dt
w2 = 300 rad/s & U ^s h = SY ^s h - y ^0 h
w3 = 500 rad/s & U ^s h = s c 12 m - y ^0 h
s
Therefore, the respective time periods are or, U ^s h = 1
s
T1 = 2p = 2p sec
w1 100 ^y ^0 h = 0h
Hence, the O/P is correct which is
T2 = 2p = 2p sec
w2 300 Y ^s h = 12
s
T3 = 2p sec its inverse Laplace transform is given by
500
y ^ t h = tu ^ t h
So, the fundamental time period of the signal is
LCM ^2p, 2p, 2ph No Option is correct.
L.C.M. ^T1, T2 T3h =
6.7

HCF ^100, 300, 500h The matched filter is characterized by a frequency response that is
or, T0 = 2p given as
100 H ^ f h = G * ^ f h exp ^- j2pfT h
Hence, the fundamental frequency in rad/sec is g^t h G^f h
f
where
w0 = 2p = 100 rad/s Now, consider a filter matched to a known signal g ^ t h. The fourier
10
transform of the resulting matched filter output g 0 ^ t h will be
6.4 Option (A) is correct. G0 ^ f h = H^ f hG^ f h
GATE Electronics and Communication Topicwise Solved Paper by RK Kanodia & Ashish Murolia Page 144

= G * ^ f h G ^ f h exp ^- j2pfT h ordinary, first order differential equation with forcing function x ^ t h
= G ^ f h 2 exp ^- j2pfT h is y ^ t h so, we can define a function relating x ^ t h and y ^ t h as below
T is duration of g ^ t h dy
P + Qy + K = x ^ t h
Assume exp ^- j2pfT h = 1 dt
So, G0 ^ f h = G_ f i 2 where P , Q , K are constant. Taking the Laplace transform both
Since, the given Gaussian function is
the sides, we get
g ^ t h = e- pt
2

Fourier transform of this signal will be P sY ^s h - Py ^0 h + Q Y ^s h = X ^s h ....(1)


g ^ t h = e- pt e- pf = G ^ f h Now, the solutions becomes
2 f 2

Therefore, output of the matched filter is y1 ^ t h =- 2y ^ t h


G 0 ^ f h = e- pf
2 2
or, Y1 ^s h =- 2Y ^s h
So, Eq. (1) changes to
Option (B) is correct.
P sY1 ^s h - P y1 ^0 h + Q Y1 ^s h = X1 ^s h
6.8

Given, the impulse response of continuous time system


h ^ t h = d ^t - 1h + d ^t - 3h or, - 2PSY ^s h - P y1 ^0 h - 2QY1 ^s h = X1 ^s h ....(2)
From the convolution property, we know Comparing Eq. (1) and (2), we conclude that
x ^ t h * d ^t - t 0h = x ^t - t 0h X1 ^s h =- 2X ^s h
So, for the input y1 ^0 h =- 2y ^0 h
x ^ t h = u ^ t h (Unit step fun n ) Which makes the two equations to be same. Hence, we require to
The output of the system is obtained as change the initial condition to - 2y ^0 h and the forcing equation to
- 2x ^ t h
GATE Electronics & Communication 6.11 Option (A) is correct.
by RK Kanodia Given, the DFT of vector 8a b c dB as
Now in 3 Volume D.F.T. %8a b c dB/ = 8a b g dB
Purchase Online at maximum discount from online store Also, we have
R V
and get POSTAL and Online Test Series Free Sa b c d W
Sd a b c W
visit www.nodia.co.in 8p q r sB = 8a b c dBSc d a b W ...(1)
S W
Sb c d aW
y^t h = u^t h * h^t h T X
For matrix circular convolution, we know
= u ^ t h * 6d ^t - 1h + d ^t - 3h@ Rh h h VRx V
S 0 2 1WS 0W
x 6n@ * h 6n@ = Sh1 h 0 h2WSx1W
= u ^t - 1h + u ^t - 3h SSh h h WWSSx WW
2 1 0 1
at t = 2 T XT X
where "x 0, x1, x2, are three point signals for x 6n@ and similarly for
y ^2 h = u ^2 - 1h + u ^2 - 3h
h 6n@, h 0 , h1 and h2 are three point signals. Comparing this trans-
=1
formation to Eq(1), we get
6.9 Option (B) is correct. R VT
Given, the differential equation Sa d c W
Sb a d W
d2y dy 6p q r s@ = Sc b aW 8a b c dB
2 + 5 dt + 6y ^ t h = x ^ t h S W
dt
Sd c b W
Taking its Laplace transform with zero initial conditions, we have T X
= 6a b c d @T * 6a b c d @T
s2 Y ^s h + 5sY ^s h + 6Y ^s h = X ^s h
....(1)
Now, the input signal is For more GATE Resources, Mock Test and
1 0<t<2 Study material join the community
x^t h = *
0 otherwise http://www.facebook.com/gateec2014
i.e., x ^ t h = u ^ t h - u ^t - 2h
R V R V
Taking its Laplace transform, we obtain Sa W Sa W
-2s Sb W Sb W
X ^s h = 1 - e =S W * S W
s s
-2s Sc W Sc W
= 1-e Sd W Sd W
s T X T X
Substituting it in equation (1), we get Now, we know that
X ^s h 2s
x1 6n@ * x2 6n@ = X1DFT 6k @ X2, DFT 6k @
Y ^s h = 2 = 21 - e
s + 5s + 6 s ^s + 5s + 6h So,
R V R V R V R V
= 1 - e-2s Sa W Sa W SaW SaW
s ^s + 2h^s + 3h Sb W Sb W SbW SbW
Sc W * Sc W = Sg W * Sg W
6.10 Option (D) is correct. S W S W S W S W
The solution of a system described by a linear, constant coefficient, Sd W Sd W Sd W Sd W
T X T X T X T X
GATE Electronics and Communication Topicwise Solved Paper by RK Kanodia & Ashish Murolia Page 145

= 9a2 b 2 g2 d 2C
6.12 Option (D) is correct.
Using s -domain differentiation property of Laplace transform.
L
If f (t) F (s)
L dF (s)
tf (t) -
ds
So, inverse Fourier transform of H (jw)
L [tf (t)] = - d ; 2 1 2s + 1
ds s + s + 1E (s2 + s + 1) 2
So, =
h (t) = h1 (t) + h2 (t)
6.13 Option (C) is correct. h (0) = h1 (0) + h2 (0) = 1 + 1 = 1
2 2
x [ n] = b 1 l - b 1 l u [ n ]
n n

3 2 6.16 Option (A) is correct.


x [n] = b 1 l u [n] + b 1 l u [- n - 1] - b 1 l u (n)
n -n n
Convolution sum is defined as
3 3 2 3
Taking z -transform y [n] = h [n] * g [n] = / h [n] g [n - k]
X 6z @
k =- 3
3
3
1 n -n
3
1 -n -n 1 n -n
3 For causal sequence, y [n] = / h [n] g [n - k]
= / b 3 l z u [n ] + / b 3 l z u [- n - 1] - /
b 2 l z u [ n] k=0
n =- 3 n =- 3 n =- 3 y [n] = h [n] g [n] + h [n] g [n - 1] + h [n] g [n - 2] + .....
3 -1 3
1 n -n 1 -n -n 1 n -n
= / b3l z + b3l z - / b2l z / SPECIAL EDITION ( STUDY MATERIAL FORM )
n=0 n =- 3 n=0
3 3 3
At market Book is available in 3 volume i.e. in 3 book binding
/ b 31z l + / b 13 z l / b 21z l
n m n
= - Taking m =- n
n=0 m=1 n=0 form. But at NODIA Online Store book is available in 10 book
14
42 4
43 1 44 2 44 3 14
42 4
43
I II III
binding form. Each unit of Book is in separate binding.
Series I converges if 1 < 1 or z > 1 Available Only at NODIA Online Store
3z 3
Click to Buy
Series II converges if 1 z < 1 or z < 3
3 www.nodia.co.in
Series III converges if 1 < 1 or z > 1
2z 2
For n = 0 , y [0] = h [0] g [0] + h [1] g [- 1] + ...........
Region of convergence of X (z) will be intersection of above three
y [0] = h [0] g [0] g [- 1] = g [- 2] = ....0
So, ROC : 1 < z < 3 y [0] = h [0] g [0] ...(i)
2
6.14 Option (D) is correct. For n = 1, y [1] = h [1] g [1] + h [1] g [0] + h [1] g [- 1] + ....
t
y (t) = # x (t) cos (3t) dt
-3
y [1] = h [1] g [1] + h [1] g [0]
1 = 1 g [1] + 1 g [0] h [1] = b 1 l = 1
1
Time Invariance : 2 2 2 2 2
Let, x (t) = d (t) 1 = g [1] + g [0]
t
y (t) = # d (t) cos (3t) dt
-3
= u (t) cos (0) = u (t) g [1] = 1 - g [0]
y [0] 1
For a delayed input (t - t 0) output is From equation (i), g [0] = = =1
t
h [0] 1
y (t, t 0) = # d (t - t ) cos (3t) dt
-3
0 = u (t) cos (3t 0) So, g [1] = 1 - 1 = 0
Delayed output, 6.17 Option (A) is correct.
y (t - t 0) = u (t - t 0) d2 y dy
y (t, t 0) ! y (t - t 0) System is not time invariant. We have 100 2 - 20 + y = x (t)
dt dt
Stability : Applying Laplace transform we get
Consider a bounded input x (t) = cos 3t
100s2 Y (s) - 20sY (s) + Y (s) = X (s)
1 - cos 6t = 1
# 1dt - 12 # cos 6t dt
t t t t
y (t) =
-3
#cos2 3t =
-3 2 # 2 -3 -3 or H (s) =
Y (s)
= 1
X (s) 100s2 - 20s + 1
As t " 3, y (t) " 3 (unbounded) System is not stable. 1/100
= 2 = A
6.15 Option (C) is correct. s - (1/5) s + 1/100 s2 + 2xwn s + w2
(2 cos w) (sin 2w)
H (jw) = = sin 3w + sin w Here wn = 1/10 and 2xwn =- 1/5 giving x =- 1
w w w
Roots are s = 1/10, 1/10 which lie on Right side of s plane thus
We know that inverse Fourier transform of sin c function is a
unstable.
rectangular function.
6.18 Option (C) is correct.
For an even function Fourier series contains dc term and cosine term
(even and odd harmonics).
6.19 Option (B) is correct.
Function h (n) = an u (n) stable if a < 1 and Unstable if a H 1
We We have h (n) = 2n u (n - 2);
GATE Electronics and Communication Topicwise Solved Paper by RK Kanodia & Ashish Murolia Page 146

Here a = 2 therefore h (n) is unstable and since h (n) = 0 for n < 0


Therefore h (n) will be causal. So h (n) is causal and not stable. Constant term is negative.

6.20 Option (A) is correct.


6.25 Option (A) is correct.
Inverse Z - transform
We know that aZ ! a ad [n ! a]
Impulse response = d (step response)
dt Given that 2
X (z) = 5z + 4z + 3 -1

= d (1 - e- at) Inverse z-transform x [n] = 5d [n + 2] + 4d [n - 1] + 3d [n]


dt
6.26 Option (C) is correct.
= 0 + ae- at = ae- at
We have h1 [n] = d [n - 1] or H1 [Z ] = Z - 1
6.21 Option (D) is correct.
and h 2 [n] = d [n - 2] or H2 (Z ) = Z - 2
We have x (t) = exp (- 2t) m (t) + s (t - 6) and h (t) = u (t)
Response of cascaded system
Taking Laplace Transform we get
H (z ) = H1 (z ) : H2 (z )
X (s) = b 1 + e-6s l and H (s) = 1
s+2 s = z :z-1 -2
=z-3

Now Y (s) = H (s) X (s) or, h [n] = d [n - 3]


-6s
= 1 : 1 + e-6sD = 1 +e 6.27 Option (D) is correct.
s s+2 s (s + 2) s
For an N-point FET algorithm butterfly operates on one pair of
-6s
or Y (s) = 1 - 1 +e samples and involves two complex addition and one complex
2s 2 (s + 2) s
multiplication.
Thus y (t) = 0.5 [1 - exp (- 2t)] u (t) + u (t - 6)
6.28 Option (D) is correct.
GATE Electronics & Communication 3s + 1
f (t) = L - 1 ; 3
s + 4s 2 + (k - 3) s E
We have
by RK Kanodia
Now in 3 Volume and lim f (t) = 1
t"3
Purchase Online at maximum discount from online store By final value theorem
and get POSTAL and Online Test Series Free lim f (t) = lim sF (s) = 1
visit www.nodia.co.in
t"3 s"0

s. (3s + 1)
or lim =1
s"0 s + 4s2 + (k - 3) s
3
6.22 Option (B) is correct.
s (3s + 1)
y (n) = x (n - 1) or lim 2 =1
s " 0 s [s + 4s + (k - 3)]
or Y (z) = z-1 X (z) 1 =1
Y (z) k-3
or = H (z) = z-1
X (z) or k =4
-1
Now H1 (z) H2 (z) = z 6.29 Option (B) is correct.
1 - 0.4z-1
c 1 - 0.6z-1 m H2 (z) = z System is described as
-1

d 2 y (t) dt (t) dx (t)


z-1 (1 - 0.6z-1) +4 + 3y (t) = 2 + 4x (t)
H2 (z) = dt 2 dt dt
(1 - 0.4z-1) Taking Laplace transform on both side of given equation
6.23 Option (B) is correct. s 2 Y (s) + 4sY (s) + 3Y (s) = 2sX (s) + 4X (s)
For 8 point DFT, x* [1] = x [7]; x* [2] = x [6]; x* [3] = x [5] and it is
(s 2 + 4s + 3) Y (s) = 2 (s + 2) X (s) s
conjugate symmetric about x [4], x [6] = 0 ; x [7] = 1 + j3
Transfer function of the system
6.24 Option (C) is correct.
For a function x (t) trigonometric fourier series is
3 For more GATE Resources, Mock Test and
x (t) = Ao + / [An cos nwt + Bn sin nwt] Study material join the community
n=1

Where, Ao 1 # x (t) dt T0 "fundamental period http://www.facebook.com/gateec2014


T0 T
0

Y (s) 2 (s + 2) 2 (s + 2)
and An = 2 # x (t) cos nwt dt H (s) = = =
T0 T X (s) s 2 + 4s + 3 (s + 3) (s + 1)
0

Bn = 2 # x (t) sin nwt dt Input x (t) = e-2t u (t)


T0 T
or, X (s) = 1
0

For an even function x (t), Bn = 0 (s + 2)


Since given function is even function so coefficient Bn = 0 , only cosine Output Y (s) = H (s) : X (s)
and constant terms are present in its fourier series representation
2 (s + 2)
Y (s) = : 1
A0 = 1 #
3T/4
Constant term x (t) dt (s + 3) (s + 1) (s + 2)
T -T/4
By Partial fraction
= 1 : # Adt + # - 2AdtD
T/4 3T/4

T -T/4 T/4 Y (s) = 1 - 1


s+1 s+3
= 1 :TA - 2AT D =- A Taking inverse Laplace transform
T 2 2 2
GATE Electronics and Communication Topicwise Solved Paper by RK Kanodia & Ashish Murolia Page 147

y (t) = (e-t - e-3t) u (t)


6.30 Option (C) is correct.
We have
2 - 34 z - 1
H (z) =
1 - 34 z - 1 + 18 z - 2
By partial fraction H (z ) can be written as
H (z ) = 1 1 It has the finite magnitude values. So it is a finite impulse response
1 -1 +
^1 - 2 z h ^1 - 14 z h
-1
filter. Thus S2 is true but it is not a low pass filter. So S1 is false.
For ROC : z > 1/2 6.36 Option (B) is correct.
Here h (t) ! 0 for t < 0 . Thus system is non causal. Again any
h [n] = b 1 l u [n] + b 1 l u [n], n > 0 1
n n
= an u [n], z > a
2 4 1 - z -1 bounded input x (t) gives bounded output y (t). Thus it is BIBO
Thus system is causal. Since ROC of H (z ) includes unit circle, so stable.
it is stable also. Hence S1 is True Here we can conclude that option (B) is correct.
For ROC : z < 1 6.37 Option (D) is correct.
4
We have x [n] = {1, 0, 2, 3) and N = 4
h [n] =-b 1 l u [- n - 1] + b 1 l u (n), z > 1 , z < 1
n n
N-1
2 4 4 2 X [k ] = / x [n] e -j2pnk/N
k = 0, 1...N - 1
n=0
System is not causal. ROC of H (z ) does not include unity circle,
so it is not stable and S 3 is True SPECIAL EDITION ( STUDY MATERIAL FORM )
6.31 Option (A) is correct.
At market Book is available in 3 volume i.e. in 3 book binding
The Fourier series of a real periodic function has only cosine terms form. But at NODIA Online Store book is available in 10 book
if it is even and sine terms if it is odd. binding form. Each unit of Book is in separate binding.
Available Only at NODIA Online Store
6.32 Option (B) is correct.
Given function is Click to Buy
f (t) = sin2 t + cos 2t = 1 - cos 2t + cos 2t = 1 + 1 cos 2t
www.nodia.co.in
2 2 2
The function has a DC term and a cosine function. The frequency 3

of cosine terms is
For N = 4 , X [k ] = / x [n] e -j2pnk/4
k = 0, 1,... 3
n=0
3
w = 2 = 2pf " f = 1 Hz Now X [0] = / x [ n]
p
n=0
The given function has frequency component at 0 and 1 Hz. = x [0] + x [1] + x [2] + x [3]
p
6.33 Option (A) is correct. = 1+0+2+3 = 6
3
x [n] = b 1 l u (n) - b 1 l u (- n - 1)
n n

3 2
x [1] = / x [n] e -jpn/2

n=0

Taking z transform we have = x [0] + x [1] e-jp/2 + x [2] e-jp + x [3] e-jp3/2
n=3 n =- 1
n
= 1 + 0 - 2 + j3 =- 1 + j3
X (z) = / b 13 l z-n - / 1 n -n
b2l z 3
n=0
n=3
n =- 3
n =- 1 X [2] = / x [n] e -jpn

= / b 1 z-1 l - / b 1 z-1 l
n n
n=0
3 2
n=0 n =- 3 = x [0] + x [1] e-jp + x [2] e-j2p + x [3] e-jp3
First term gives 1 z-1 < 1 " 1< z
3 3 = 1+0+2-3 = 0
3

Second term gives 1 z-1 > 1 "


2
1> z X [3] = / x [n] e -j3pn/2

2 n=0

Thus its ROC is the common ROC of both terms. that is


1< z <1 = x [0] + x [1] e-j3p/2 + x [2] e-j3p + x [3] e-j9p/2
3 2 = 1 + 0 - 2 - j3 =- 1 - j3
6.34 Option (B) is correct. Thus [6, - 1 + j3, 0, - 1 - j3]
By property of unilateral Laplace transform 6.38 Option (A) is correct.
t F (s) 1 0 -

-3
#f (t) dt
L
s
+
s -3 #
f (t) dt 6.39 Option (C) is correct.
The output of causal system depends only on present and past states
Here function is defined for 0 < t < t , Thus
only.
t L F (s)
0
#
f (t)
s In option (A) y (0) depends on x (- 2) and x (4).
In option (B) y (0) depends on x (1).
6.35 Option (A) is correct. In option (C) y (0) depends on x (- 1).
We have h (2) = 1, h (3) =- 1 otherwise h (k) = 0 . The diagram of In option (D) y (0) depends on x (5).
response is as follows : Thus only in option (C) the value of y (t) at t = 0 depends on x (- 1)
past value. In all other option present value depends on future value.
GATE Electronics and Communication Topicwise Solved Paper by RK Kanodia & Ashish Murolia Page 148

6.40 Option (D) is correct. The y (t) = t x (t) is not linear, thus option (B) is wrong and (a) is
We have h (t) = eat u (t) + e bt u (- t) correct. We can see that
This system is stable only when bounded input has bounded out- R1: y (t) = t2 x (t) Linear and time variant.
R2: y (t) = t x (t) Non linear and time variant.
put For stability at < 0 for t > 0 that implies a < 0 and bt > 0
R3: y (t) = x (t) Non linear and time invariant
for t > 0 that implies b > 0 . Thus, a is negative and b is positive.
R4: y (t) = x (t - 5) Linear and time invariant
6.41 Option (C) is correct. 6.45 Option (A) is correct.
K (s + 1)
G (s) = , and R (s) = 1 N-1
(s + 2)( s + 4) s Given : y (n) = 1 /x (r) x (n + r)
N
K (s + 1) r=0
C (s) = G (s) R (s) = It is Auto correlation.
s (s + 2)( s + 4)
DFT
Hence y (n) = rxx (n) X (k) 2
= K + K - 3K
8s 4 (s + 2) 8 (s + 4) 6.46 Option (B) is correct.
Thus c (t) = K :1 + 1 e-2t - 3 e-4tD u (t) Current through resistor (i.e. capacitor) is
8 4 8
I = I (0+) e-t/RC
At steady-state , c (3) = 1
K = 1 or K = 8 Here, I (0+) = V = 5 = 25mA
Thus R 200k
8
8 (s + 1) RC = 200k # 10m = 2 sec
Then, G (s) = = 12 - 4 I = 25e- m A
t

(s + 2)( s + 4) (s + 4) (s + 2) 2

= VR # R = 5e- V
t

GATE Electronics & Communication


2

Here the voltages across the resistor is input to sampler at fre-


by RK Kanodia
quency of 10 Hz. Thus
Now in 3 Volume -n
x (n) = 5e 2 # 10
= 5e-0.05n For t > 0
Purchase Online at maximum discount from online store
6.47 Option (C) is correct.
and get POSTAL and Online Test Series Free
Since x (n) = 5e-0.05n u (n) is a causal signal
visit www.nodia.co.in
Its z transform is
h (t) = L-1 G (s) = (- 4e-2t + 12e-4t) u (t) X (z) = 5 : 1 5z
1 - e-0.05 z-1 D z - e-0.05
=
6.42 Option (A) is correct.
Its ROC is e-0.05 z-1 > 1 " z > e-0.05
1 for - 1 # t # + 1
We have x (t) = )
0 otherwise 6.48 Option (C) is correct.
Fourier transform is h (t) = e-2t u (t)
#- 33e-jwt x (t) dt =
1
#-1 e-jwt 1dt H (jw) = #- 33 h (t) e-jwt dt
#0 3e-2t e-jwt dt = #0 3e-(2 + jw)t dt 1
= 1 [e-jwt]-11 = =
(2 + jw)
- jw
= 1 (e-jw - e jw) = 1 (- 2j sin w) 6.49 Option (D) is correct.
- jw - jw 1
H (jw) =
= 2 sin w (2 + jw)
w
The phase response at w = 2 rad/sec is
This is zero at w = p and w = 2p
+H (jw) =- tan-1 w =- tan-1 2 =- p =- 0.25p
6.43 Option (D) is correct. 2 2 4
Given h (n) = [1, - 1, 2]
For more GATE Resources, Mock Test and
x (n) = [1, 0, 1]
Study material join the community
y (n) = x (n)* h (n)
http://www.facebook.com/gateec2014
The length of y [n] is = L1 + L2 - 1 = 3 + 3 - 1 = 5
3 Magnitude response at w = 2 rad/sec is
y (n) = x (n) * h (n) = / x (k) h (n - k) 1
k =- 3 H (jw) = = 1
3 22 + w2 2 2
y (2) = / x (k) h (2 - k) Input is x (t) = 2 cos (2t)
k =- 3

= x (0) h (2 - 0) + x (1) h (2 - 1) + x (2) h (2 - 2) Output is = 1 # 2 cos (2t - 0.25p)


2 2
= h (2) + 0 + h (0) = 1 + 2 = 3
There are 5 non zero sample in output sequence and the value of = 1 cos [2t - 0.25p]
2
y [2] is 3. 6.50 Option (D) is correct.
6.44 Option (B) is correct. Y (s) = 1
s (s - 1)
Mode function are not linear. Thus y (t) = x (t) is not linear but
Final value theorem is applicable only when all poles of system lies
this functions is time invariant. Option (A) and (B) may be correct.
in left half of S -plane. Here s = 1 is right s -plane pole. Thus it is
GATE Electronics and Communication Topicwise Solved Paper by RK Kanodia & Ashish Murolia Page 149

unbounded. a t = 0 . The area of dirac delta function is unity.


6.51 Option (A) is correct. 6.58 Option (D) is correct.
x (t) = e u (t) -t The ROC of addition or subtraction of two functions x1 (n) and x2 (n)
Taking Fourier transform is R1 + R2 . We have been given ROC of addition of two function and
has been asked ROC of subtraction of two function. It will be same.
X (jw) = 1
1 + jw 6.59 Option (A) is correct.
X (jw) = 1 2 As we have x (t) = sin t , thus w = 1
1+w
1 Now H (s) = 1
Magnitude at 3dB frequency is s+1
2
1 = 1 or H (jw) = 1 = 1
Thus jw + 1 j+1
2 1 + w2
or w = 1 rad or H (jw) = 1 + - 45c
2
or f = 1 Hz Thus y (t) = 1 sin (t - p4 )
2p 2
6.52 Option (B) is correct. 6.60 Option (C) is correct.
For discrete time Fourier transform (DTFT) when N " 3 w0
F (s) =
x [ n] = 1
p
#
X (e jw) e jwn dw s + w2
2
2p - p
SPECIAL EDITION ( STUDY MATERIAL FORM )
Putting n = 0 we get
At market Book is available in 3 volume i.e. in 3 book binding
x [0] = 1
p

2p #-p
X (e jw) e jw0 dw form. But at NODIA Online Store book is available in 10 book
= 1
p

2p #
-p
X (e jw) dw binding form. Each unit of Book is in separate binding.
Available Only at NODIA Online Store
p
or #
-p
jw
X (e ) dw = 2px [0] = 2p # 5 = 10p Click to Buy
6.53 Option (B) is correct. www.nodia.co.in
0.5
X (z) =
1 - 2z-1 L-1 F (s) = sin wo t
Since ROC includes unit circle, it is left handed system f (t) = sin wo t
x (n) =- (0.5) (2) -n u (- n - 1) Thus the final value is - 1 # f (3) # 1
x (0) = 0 6.61 Option (C) is correct.
If we apply initial value theorem y (n) = b sin 5 pn l x (n)
6
x (0) = lim X (z) = lim 0.5 -1 = 0.5
z"3 z " 31 - 2z Let x (n) = d (n)
That is wrong because here initial value theorem is not applicable Now y (n) = sin 0 = 0 (bounded) BIBO stable
because signal x (n) is defined for n < 0 . 6.62 Option (B) is correct.
6.54 Option (A) is correct. c (t) = 1 - e-2t
A Hilbert transformer is a non-linear system. Taking Laplace transform
Option (B) is correct. C (s) 2 #s = 2
6.55
C (s) = =
5 U (s) s (s + 2) s+2
H (f) =
1 + j10pf 6.63 Option (C) is correct.
H (s) = 5 = 5 = 1 1
5^s + 15 h s +
L
1 + 5s 1 h (t) = e-t H (s) =
5 s+1
Step response Y (s) =1 a 1
s ^s + 5 h X (s) = 1
L
x (t) = u (t)
s
or Y (s) =1 11 =5- 51
s ^s + 5 h s s+ 5 Y (s) = H (s) X (s) = 1 # 1 = 1 - 1
s+1 s s s+1
-t/5
or y (t) = 5 (1 - e ) u (t)
y (t) = u (t) - e-t
6.56 Option (A) is correct. In steady state i.e. t " 3, y (3) = 1
F
x (t) X (jw)
6.64 Option (C) is correct.
Using scaling we have
1 X jw Fourier series is defined for periodic function and constant.
5 c 5 m
F
x (5t) 3 sin (25t) is a periodic function.
Using shifting property we get 4 cos (20t + 3) + 2 sin (710t) is sum of two periodic function and also
1 X jw e- j35w a periodic function.
x ;5 bt - 3 lE
5 b5l
F
5 e- t sin (25t) is not a periodic function, so FS can’t be defined for it.
6.57 Option (D) is correct. 1 is constant
Dirac delta function d (t) is defined at t = 0 and it has infinite value 6.65 Option (A) is correct.
GATE Electronics and Communication Topicwise Solved Paper by RK Kanodia & Ashish Murolia Page 150

g (t) + g (- t) 6.70 Option (A) is correct.


Ev{g (t)} =
2 y (t)
g (t) - g (- t)
odd{g (t)} = = 0.5x (t - td + T) + x (t - td ) + 0.5x (t - td - T)
2
Taking Fourier transform we have
Here g (t) = u (t)
Y (w)
u (t) + u (- t) 1
Thus ue (t) = = = 0.5e -jw (- td + T)
X (w) + e-jwt X (w) + 0.5e-jw (- t - T) X (w)
d d

2 2
u (t) - u (- t) x (t) Y (w)
uo (t) = = or = e-jwt [0.5e jwT + 1 + 0.5e-jwT ]
d

2 2 X (w)
= e-jwt [0.5 (e jwT + e-jwT ) + 1]
d

6.66 Option (C) is correct.


= e-jwt [cos wT + 1]
x1 (n) = ` 5 jn u (n)
d

Here
6
Y (w)
1 or H (w) = = e-jwt (cos wT + 1)
ROC : R1 " z > 5
d

X1 (z) = X (w)
1 - ^ 65 z-1h 6
6.71 Option (C) is correct.
x2 (n) =-` 6 jn u (- n - 1) For continuous and aperiodic signal Fourier representation is
5
1 continuous and aperiodic.
X1 (z) = 1 - ROC : R2 " z < 6
1 - ^ 65 z-1h 5 For continuous and periodic signal Fourier representation is discrete
and aperiodic.
Thus ROC of x1 (n) + x2 (n) is R1 + R2 which is 5 < z < 6
6 5 For discrete and aperiodic signal Fourier representation is continuous
GATE Electronics & Communication and periodic.
For discrete and periodic signal Fourier representation is discrete
by RK Kanodia and periodic.
Now in 3 Volume
6.72 Option (B) is correct.
Purchase Online at maximum discount from online store
y (n) = Ax (n - no)
and get POSTAL and Online Test Series Free Taking Fourier transform
visit www.nodia.co.in Y (e jw) = Ae-jw n X (e jw) o o

Option (D) is correct. Y (e jw)


6.67
or H (e jw) = = Ae-jw n o o

For causal system h (t) = 0 for t # 0 . Only (D) satisfy this condition. X (e jw)
Thus +H (e jw) =- wo no
6.68 Option (D) is correct.
For LTI discrete time system phase and frequency of H (e jw) are
x (n) = b 1 l u (n)
n

2 periodic with period 2p. So in general form


y (n) = x2 (n) = b 1 l u2 (n)
2n
q (w) =- no wo + 2pk
2
2 n
6.73 Option (A) is correct.
y (n) = ;b 1 l E u (n) = b 1 l u (n)
n
or ...(1) From x (n) = [ 12 , 1, 2, 1, 1, 12 ]
2 4
n=3 n=3 y (n) = x ^ n2 - 1h, n even
/ b 14 l e-jwn
n
Y (e jw) = / y (n) e-jwn =
= 0 , for n odd
n =- 3 n=0
n
n=3
n =- 2 , y (- 2) = x ( -22 - 1) = x (- 2) = 12
/ ` 14 j = 1 +b1l +b1l+b1l +b1l
1 3 4
or Y (e j0) =
n=0
4 4 4 4 n =- 1, y (- 1) = 0
1 n = 0, y (0) = x ( 20 - 1) = x (- 1) = 1
or Y (e j0) = =4
1- 1
4
3 n = 1, y (1) = 0
Alternative : For more GATE Resources, Mock Test and
Taking z transform of (1) we get Study material join the community
1
Y (z) =
1 - 14 z-1 http://www.facebook.com/gateec2014
Substituting z = e jw we have
n=2 y (2) = x ( 22 - 1) = x (0) = 2
Y (e jw) = 1
1 - 14 e-jw n = 3, y (3) =0
Y (e j0) = 1 1 = 4 n=4 y (4) = x ( 24 - 1) = x (1) = 1
1- 4 3
n = 5, y (5) =0
6.69 Option (A) is correct. n=6 y (6) = x ( 26 - 1) = x (2) = 12
s (t) = 8 cos ` p - 20pt j + 4 sin 15pt Hence y (n)
2
= 8 sin 20pt + 4 sin 15pt = 1 d (n + 2) + d (n) + 2d (n - 2) + d (n - 4) + 1 d (n - 6)
2 2
Here A1 = 8 and A2 = 4 . Thus power is 6.74 Option (C) is correct.
2 2 2 2 Here y (n) is scaled and shifted version of x (n) and again y (2n) is
P = A1 + A2 = 8 + 4 = 40
2 2 2 2 scaled version of y (n) giving
GATE Electronics and Communication Topicwise Solved Paper by RK Kanodia & Ashish Murolia Page 151

z (n) = y (2n) = x (n - 1) x (n) - x* (- n)


xcas (n) =
= 1 d (n + 1) + d (n) + 2d (n - 1) + d (n - 2) + 1 d (n - 3) 2
2 2 = [- 4 - j 25 , 2j 4 - j 25 ]
Taking Fourier transform. -

Option (C) is correct.


Z (e jw) = 1 e jw + 1 + 2e-jw + e-2jw + 1 e-3jw
6.80

2 2 We have 2y (n) = ay (n - 2) - 2x (n) + bx (n - 1)


= e-jw b 1 e2jw + e jw + 2 + e-jw + 1 e-2jw l Taking z transform we get
2 2
2Y (z) = aY (z) z-2 - 2X (z) + bX (z) z-1
e2jw + e-2jw + e jw + 2 + e-jw
b l
-jw
=e Y (z) bz-1 - 2
2 =c
or
X (z) 2 - az-2
m ...(i)
or Z (e jw) = e-jw [cos 2w + 2 cos w + 2]
z ( b - z)
6.75 Option (B) is correct. or H (z) = 22 a
F
(z - 2 )
x (t) X (f)
It has poles at ! a/2 and zero at 0 and b/2 . For a stable system
Using scaling we have
poles must lie inside the unit circle of z plane. Thus
1 X f
a ca m
F
x (at) a <1
2
xb 1 f l
F
Thus 3X (3f) or a <2
3
Using shifting property we get SPECIAL EDITION ( STUDY MATERIAL FORM )
-j2pf0 t
e x (t) = X (f + f0) At market Book is available in 3 volume i.e. in 3 book binding
Thus 1e -j 4 pt
3 xb 1 t l
F
X (3f + 2) form. But at NODIA Online Store book is available in 10 book
3 3
binding form. Each unit of Book is in separate binding.
xb 1 t l
F
e-j2p
2
t
3
3X (3 (f + 23 )) Available Only at NODIA Online Store
3
1 e-jp Click to Buy
xb 1 t l
4
t F
X [3 (f + 23 )]
www.nodia.co.in
3

3 3
6.76 Option (A) is correct.
3 But zero can lie anywhere in plane. Thus, b can be of any value.
A system is stable if / h (n) < 3. The plot of given h (n) is
n =- 3 6.81 Option (D) is correct.
We have x (n) = e jpn/4
and h (n)
= 4 2 d (n + 2) - 2 2 d (n + 1) - 2 2 d (n - 1)
+ 4 2 d (n - 2)
Now y (n) = x (n)* h (n)
3 2

3 6
= / x (n - k) h (k) = / x (n - k) h (k)
Thus / h (n) = /
n =- 3
h (n)
or
k =- 3 k =- 2

y (n) = x (n + 2) h (- 2) + x (n + 1) h (- 1)
n =- 3

= 1+1+1+1+2+2+2+2+2 + x (n - 1) h (1) + x (n - 2) h (2)


= 15 < 3
Hence system is stable but h (n) ! 0 for n < 0 . Thus it is not = 4 2 ej
p
4
(n + 2)
- 2 2 ej
p
4
(n + 1)
- 2 2 ej
p
4
(n - 1)
+ 4 2 ej
p
4
(n - 2)

causal.
= 4 2 6e j @ - 2 2 6e j @
p p p p
(n + 2) (n - 2) (n + 1) (n - 1)
6.77 Option (D) is correct. 4
+ ej 4 4
+ ej 4

H (z) = z z < 0.2


z - 0.2
= 4 2 e j n 6e j + e-j @ - 2 2 e j n 6e j + e-j @
p p p p p p

We know that 4 2 2 2 4 4

p p

- an u [- n - 1] * 1 z <a = 4 2 e j n [0] - 2 2 e j n [2 cos p4 ] 4 4

1 - az-1
y (n) =- 4e j n
r

or 4

Thus h [n] =- (0.2) n u [- n - 1]


6.82 Option (B) is correct.
6.78 Option (C) is correct.
From given graph the relation in x (t) and y (t) is
The Fourier transform of a conjugate symmetrical function is always
y (t) =- x [2 (t + 1)]
real. F
x (t) X (f)
6.79 Option (A) is correct.
Using scaling we have
We have x (n) = [- 4 - j5, 1 + 2j, 4]
1 X f
a ca m
F
- x (at)
x *( n) = [- 4 + j5, 1 - 2j, 4]
1X f
2 c2m
- F
Thus x (2t)
x *( - n) = [4, 1 - 2j, - 4 + j5]
-
Using shifting property we get
GATE Electronics and Communication Topicwise Solved Paper by RK Kanodia & Ashish Murolia Page 152

x (t - t0) = e-j2pft X (f)


0

f j2pf
f Now H (0) = 1
e-j2pf (- 1) 1 X b l = e X b l
F
Thus x [2 (t + 1)] H (f1)
2 2 2 2 1
= $ 0.95
j2p f f H (0) 1 + 4p2 f12 R2 C2
-e Xc m
F
- x [2 (t + 1)]
2 2
or 1 + 4p2 f12 R2 C2 # 1.108
6.83 Option (C) is correct.
or 4p2 f12 R2 C2 # 0.108
From the Final value theorem we have
or 2pf1 RC # 0.329
lim i (t) = lim sI (s) = lim s 2 = lim 2 =2
t"3 s"0 s"0 s (1 + s) s " 0 (1 + s) or f1 # 0.329
2pRC
6.84 Option (D) is correct.
or f1 # 0.329
Here C3 = 3 + j5 2pRC
For real periodic signal or f1 # 0.329
2p1k # 1m
C-k = Ck*
or f1 # 52.2 Hz
Thus C-3 = Ck = 3 - j5
Thus f1 max = 52.2 Hz
6.85 Option (C) is correct.
6.89 Option (A) is correct.
y (t) = 4x (t - 2)
H (w) = 1
Taking Fourier transform we get 1 + jwRC
Y (e j2pf ) = 4e-j2pf2 X (e j2pf ) Time Shifting property q (w) =- tan-1 wRC
GATE Electronics & Communication tg =-
dq (w)
= RC
by RK Kanodia dw 1 + w2 R2 C2
10-3
Now in 3 Volume =
1 + 4p # 10 4 # 10-6
2 = 0.717 ms
Purchase Online at maximum discount from online store 6.90 Option (C) is correct.
and get POSTAL and Online Test Series Free If x (t)* h (t) = g (t)
visit www.nodia.co.in Then x (t - t1)* h (t - t2) = y (t - t1 - t2)
Y (e j2pf ) Thus x (t + 5)* d (t - 7) = x (t + 5 - 7) = x (t - 2)
or = 4e-4jpf
X (e j2pf ) 6.91 Option (B) is correct.
Thus H (e j2pf ) = 4e-4jpf In option (B) the given function is not periodic and does not satisfy
Dirichlet condition. So it cant be expansion in Fourier series.
6.86 Option (B) is correct.
x (t) = 2 cos pt + 7 cos t
We have h (n) = 3d (n - 3)
or H (z) = 2z-3 Taking z transform T1 = 2p = 2
w
4 2 -4
X (z) = z + z - 2z + 2 - 3z
T2 = 2p = 2p
Now Y (z) = H (z) X (z) 1
T1 = 1 = irrational
= 2z-3 (z 4 + z2 - 2z + 2 - 3z-4)
T2 p
= 2 (z + z-1 - 2z-2 + 2z-3 - 3z-7)
6.92 Option (C) is correct.
Taking inverse z transform we have
From the duality property of fourier transform we have
y (n) = 2[ d (n + 1) + d (n - 1) - 2d (n - 2) FT
If x (t) X (f)
+ 2d (n - 3) - 3d (n - 7)] FT
Then X (t) x (- f)
At n = 4 , y (4) = 0
6.87 Option (A) is correct.
For more GATE Resources, Mock Test and
System is non causal because output depends on future value Study material join the community
For n # 1 y (- 1) = x (- 1 + 1) = x (0) http://www.facebook.com/gateec2014
y (n - n0) = x (n - n0 + 1) Time varying
y (n) = x (n + 1) Depends on Future FT 1
Therefore if e-t u (t)
i.e. y (1) = x (2) None causal 1 + j2pf
For bounded input, system has bounded output. So it is stable. Then 1 FT
e f u (- f)
1 + j2pt
y (n) = x (n) for n $ 1
6.93 Option (A) is correct.
= 0 for n = 0
q (w) =- wt0
= x (x + 1) for n #- 1
- q (w)
So system is linear. tp = = t0
w
6.88 Option (C) is correct. dq (w)
and tg =- = t0
The frequency response of RC-LPF is dw
H (f) = 1 Thus tp = tg = t0 = constant
1 + j2pfRC
GATE Electronics and Communication Topicwise Solved Paper by RK Kanodia & Ashish Murolia Page 153

6.94 Option (*) is correct. E = #


3
[f (t)] 2 dt
X (s) = 2 5 - s = 5-s = -2 + 1
-3

s -s-2 (s + 1 )( s - 2 ) s+1 s-2 and E1 = #


3
[f (2t)] 2 dt
Here three ROC may be possible. -3

Re (s) < - 1 Substituting 2t = p we get


dp 1
# # [f (p)] 2 dp = E
3 3
Re (s) > 2 E1 = [f (p)] 2 =
-3 2 2 -3 2
- 1 < Re (s) < 2
6.100 Option (B) is correct.
Since its Fourier transform exits, only - 1 < Re (s) < 2 include
Since h1 (t) ! 0 for t < 0 , thus h1 (t) is not causal
imaginary axis. so this ROC is possible. For this ROC the inverse
h2 (t) = u (t) which is always time invariant, causal and stable.
Laplace transform is u (t)
h3 (t) = is time variant.
x (t) = [- 2e-t u (t) - 2e2t u (- t)] 1+t
6.95 Option (B) is correct. h 4 (t) = e-3t u (t) is time variant.
For left sided sequence we have 6.101 Option (B) is correct.
- an u (- n - 1)
z 1 where z < a h (t) = f (t)* g (t)
1 - az-1 We know that convolution in time domain is multiplication in s -
Thus - 5n u (- n - 1)
z 1 where z < 5 domain.
1 - 5z-1
or - 5n u (- n - 1)
z z where z < 5
z-5 SPECIAL EDITION ( STUDY MATERIAL FORM )
Since ROC is z < 5 and it include unit circle, system is stable. At market Book is available in 3 volume i.e. in 3 book binding
Alternative : form. But at NODIA Online Store book is available in 10 book
h (n) =- 5n u (- n - 1) binding form. Each unit of Book is in separate binding.
3 -1 -1 Available Only at NODIA Online Store
H (z) = / h (n) z -n
= /-5 z n -n
=- / (5z -1 n
)
Click to Buy
n =- 3 n =- 3 n =- 3

Let n =- m, then www.nodia.co.in


-3 3
H (z) =- / (5z -1 -m
) = 1- / (5 -1
z) -m L
n =- 1 m=0 f (t)* g (t) = h (t) H (s) = F (s) # G (s)
1 5-1 z < 1 or z < 5 s2 + 1
= 1-
1 - 5-1 z
, Thus H (s) = s2+ 2 # = 1
s +1 (s + 2 )( s + 3 ) s+3
= 1- 5 = z Option (B) is correct.
5-z z-5 6.102

Since normalized Gaussion function have Gaussion FT


6.96 Option (B) is correct. 2
p e-
FT p2 f2

1 Thus e-at a

= 12 # 1 a
2
s (s - 2) s s-2
6.103 Option (B) is correct.
1 # 1 L
(t * e2t) u (t) Let x (t) = ax1 (t) + bx2 (t)
s 2 s - 2
ay1 (t) = atx1 (t)
Here we have used property that convolution in time domain is
by2 (t) = btx2 (t)
multiplication in s - domain
LT Adding above both equation we have
X1 (s) X2 (s) x1 (t)* x2 (t)
ay1 (t) + by2 (t) = atx1 (t) + btx2 (t)
6.97 Option (A) is correct.
= t [ax1 (t) + bx2 (t)]
We have h (n) = u (n)
3 3 3
= tx (t)
H (z) = / x (n) .z -n
= / 1.z -n
= / (z -1 n
) or ay1 (t) + by2 (t) = y (t) Thus system is linear
n =- 3 n=0 n=0
If input is delayed then we have
H (z) is convergent if
3 yd (d) = tx (t - t0)
/ (z-1) n < 3 If output is delayed then we have
n=0

and this is possible when z-1 < 1. Thus ROC is z-1 < 1 or y (t - t0) = (t - t0) x (t - t0)
z >1 which is not equal. Thus system is time varying.
6.98 Option (A) is correct. 6.104 Option (A) is correct.
LS 1
# d (t) = 1
3
We know that d (t) x (t) = x (0) d (t) and We have h (t) = e2t H (s) =
-3 s-2
X (s) = 1
LS
Let x (t) = cos ( 23 t), then x (0) = 1 and x (t) = e3t
s-3
# d (t) x (t) = # x (0) d (t) dt # d (t) dt = 1
3 3 3
Now = Now output is Y (s) = H (s) X (s)
-3 -3 -3
= 1 # 1 = 1 - 1
6.99 Option (B) is correct. s-2 s-3 s-3 s-2
Let E be the energy of f (t) and E1 be the energy of f (2t), then Thus y (t) = e3t - e2t
GATE Electronics and Communication Topicwise Solved Paper by RK Kanodia & Ashish Murolia Page 154

6.105 Option (C) is correct. z-1 (1 - z-4) (z - 1)


= lim
We know that for a square wave the Fourier series coefficient z"1 4 (1 - z-1) 2
sin nw t z-1 z-4 (z 4 - 1) (z - 1)
Cnsq = At nw t2
0

...(i) = lim
T 2
0 z"1 4z-2 (z - 1) 2
1 -3 (z - 1) (z + 1) (z 2 + 1) (z - 1)
Thus Cnsq \
n = lim z
z"1 4 (z - 1) 2
If we integrate square wave, triangular wave will be obtained, -3
= lim z (z + 1) (z2 + 1) = 1
z"1 4
Hence Cntri \ 12
n 6.113 Option (A) is correct.
6.106 Option (B) is correct. w
We have F (s) =
F (s) = 1 [1 - e-s] s2 + w2
L
u (t) - u (t - 1) = f (t)
s lim f (t) final value theorem states that:
t"3
G (s) = 1 [1 - e-2s]
L
u (t) - u (t - 2) = g (t) lim f (t) = lim sF (s)
s t"3 s"0

f (t)* g (t)
L
F (s) G (s) It must be noted that final value theorem can be applied only if
poles lies in –ve half of s -plane.
= 12 [1 - e-s] [1 - e-2s] Here poles are on imaginary axis (s1, s2 = ! jw) so can not apply
s
final value theorem. so lim f (t) cannot be determined.
= 12 [1 - e-2s - e-s + e-3s] t"3
s 6.114 Option (D) is correct.
Trigonometric Fourier series of a function x (t) is expressed as :
GATE Electronics & Communication 3
x (t) = A 0 + / [An cos nwt + Bn sin nwt]
by RK Kanodia n=1

Now in 3 Volume For even function x (t), Bn = 0


3
Purchase Online at maximum discount from online store So x (t) = A 0 + / An cos nwt
n=1
and get POSTAL and Online Test Series Free Series will contain only DC & cosine terms.
visit www.nodia.co.in 6.115 Option (C) is correct.
-2s -s -3s
Given periodic signal
= 12 - e 2 - e 2 + e 2
L
or f (t)* g (t) 1, t < T1
x (t) = *
s s s s
Taking inverse Laplace transform we have 0, T1 < t < T0
2
f (t)* g (t) The figure is as shown below.
= t - (t - 2) u (t - 2) - (t - 1) u (t - 1) + (t - 3) u (t - 3)
The graph of option (B) satisfy this equation.
6.107 Option (A) is correct.
6.108 Option (A) is correct.
We have f (nT) = anT
Taking z -transform we get
T n
/ b az
3 3 3
F (z) = / anT z-n = / (aT ) n z-n = l
n =- 3 n =- 3 n=0 For x (t) fourier series expression can be written as
= z T
z-a x (t) = A 0 +
3
/ [An cos nwt + Bn sin nwt]
n=1
6.109 Option (B) is correct.
If L [f (t)] = F (s)
For more GATE Resources, Mock Test and
Applying time shifting property we can write
Study material join the community
L [f (t - T)] = e-sT F (s) http://www.facebook.com/gateec2014
6.110 Option (A) is correct. where dc term
A 0 = 1 # x (t) dt = 1 # x (t) dt
T /2 0
6.111 Option (A) is correct. T0 T T0 -T /2
0 0

Option (C) is correct.


= 1 : # x (t) dt + # x (t) dt +
6.112
x (t) dtD
-T T T0 /2

Given z transform T0 -T /2 0
1

-T
1

1
#T
1

C (z) =
z-1 (1 - z-4) = 1 60 + 2T1 + 0@
4 (1 - z-1) 2 T0
Applying final value theorem A 0 = 2T1
T0
lim f (n) = lim (z - 1) f (z)
n"3 z"1 6.116 Option (B) is correct.
-1 -4
z (1 - z ) The unit impulse response of a LTI system is u (t)
lim (z - 1) F (z) = lim (z - 1)
z"1 z"1 4 (1 - z-1) 2 Let h (t) = u (t)
GATE Electronics and Communication Topicwise Solved Paper by RK Kanodia & Ashish Murolia Page 155

Taking LT we have H (s) = 1 6.123 Option (C) is correct.


s The conjugation property allows us to show if x (t) is real, then
If the system excited with an input x (t) = e-at u (t), a > 0 , the
X (jw) has conjugate symmetry, that is
response
X (- jw) = X)(jw) [ x (t) real]
Y (s) = X (s) H (s) Proof :
X (s) = L [x (t)] = 1 3
(s + a) X (jw) = # x (t) e-jwt dt
Y (s) = 1 1 = 1 :1 - 1 D
-3
so replace w by - w then
(s + a) s a s s+a
3
Taking inverse Laplace, the response will be X (- jw) = # x (t) e jwt dt
y (t) = 1 61 - e-at@
-3
a 3 ) 3
X)(jw) = = # x (t) e-jwt dtG = # x)(t) e jwt dt
6.117 Option (B) is correct. -3 -3

3 if x (t) real x)(t) = x (t)


We have x [ n] = / d (n - k) 3
k=0 then X)(jw) = # x (t) e jwt dt = X (- jw)
3 3 3
X (z) = / x [n] z-n = / ; / d (n - k) z-nE -3

k=0 n =- 3 k = 0
Since d (n - k) defined only for n = k so
1 = z
3
X (z) = / z-k =
k=0 (1 - 1/z) (z - 1) SPECIAL EDITION ( STUDY MATERIAL FORM )
6.118 Option (B) is correct. At market Book is available in 3 volume i.e. in 3 book binding
6.119 Option (B) is correct. form. But at NODIA Online Store book is available in 10 book
x (t) X (f)
F binding form. Each unit of Book is in separate binding.
Available Only at NODIA Online Store
by differentiation property;
dx (t) Click to Buy
F;
dt E
= jwX (w)
www.nodia.co.in
dx (t)
F;
dt E
or = j2pfX (f)

6.120 Option (C) is correct.


F
We have f (t) g (w)
by duality property of fourier transform we can write
F
g (t) 2pf (- w)
3
so F [g (t)] = # g (t) e-jwt dt = 2pf (- w)
-3

6.121 Option (B) is correct.


Given function
x (t) = eat cos (at)
Now cos (at)
L s
s2 + a2
L
If x (t) X (s)
s0 t L
then e x (t) X (s - s 0) shifting in s-domain
L (s - a)
so eat cos (at)
(s - a) 2 + a2
6.122 Option (C) is correct.
For a function x (t), trigonometric fourier series is :
3
x (t) = A 0 + / [An cos nwt + Bn sin nwt]
n=1

where A 0 = 1 # x (t) dt T0 =Fundamental period


T0 T 0

An = 2 # x (t) cos nwtdt


T0 T 0

Bn = 2 # x (t) sin nwtdt


T0 T 0

For an even function x (t), coefficient Bn = 0


for an odd function x (t), A0 = 0
An = 0
so if x (t) is even function its fourier series will not contain sine
terms.
GATE Electronics and Communication Topicwise Solved Paper by RK Kanodia & Ashish Murolia Page 156

UNIT 7 approximate value of Ka that will reduce the time constant of the
closed loop system by one hundred times as compared to that of the
open-loop system is
CONTROL SYSTEMS

2013 ONE MARK (A) 1 (B) 5


(C) 10 (D) 100
7.1 The Bode plot of a transfer function G ^s h is shown in the figure
below. 7.3 The signal flow graph for a system is given below. The transfer
Y ^s h
function for this system is
U ^s h

GATE Electronics & Communication (A) s+1 (B) s+1


5s2 + 6s + 2 s 2 + 6s + 2
by RK Kanodia
(C) 2 s + 1 (D) 2 1
Now in 3 Volume s + 4s + 2 5s + 6s + 2
Purchase Online at maximum discount from online store
and get POSTAL and Online Test Series Free Statement for Linked Answer Questions 4 and 5:
visit www.nodia.co.in The state diagram of a system is shown below. A system is
o = AX + Bu ;
described by the state-variable equations X
y = CX + Du

7.4 The state-variable equations of the system shown in the figure above
are
o = >- 1 0 H X + >- 1H u
X Xo = >- 1 0 H X + >- 1H u
(A) 1 -1 1 (B) -1 -1 1
y = 61 - 1@X + u y = 6- 1 - 1@X + u
o = >- 1 0 H X + >- 1H u
X Xo = >- 1 - 1H X + >- 1H u
(C) -1 -1 1 (D) 0 -1 1
y = 6- 1 - 1@X - u y = 61 - 1@ X - u
7.5 The state transition matrix eAt of the system shown in the figure
For more GATE Resources, Mock Test and
Study material join the community
http://www.facebook.com/gateec2014
above is
e-t 0 e-t 0
The gain _20 log G ^s h i is 32 dB and - 8 dB at 1 rad/s and (A) > -t -tH (B) > H
te e - te e-t
-t

10 rad/s respectively. The phase is negative for all w. Then G ^s h is


(A) 39.8 (B) 392.8 e-t 0
(C) > -t -tH (D) >
e-t - te-t
s s e e 0 e-t
H
(C) 32 (D) 322
s s
2012 ONE MARK

2013 TWO MARKS (s2 + 9) (s + 2)


7.6 A system with transfer function G (s) =
(s + 1) (s + 3) (s + 4)
7.2 The open-loop transfer function of a dc motor is given as is excited by sin (wt). The steady-state output of the system is zero
w ^s h
= 10 . When connected in feedback as shown below, the at
Va ^s h 1 + 10s
GATE Electronics and Communication Topicwise Solved Paper by RK Kanodia & Ashish Murolia Page 157

(A) w = 1 rad/s (B) w = 2 rad/s


(C) w = 3 rad/s (D) w = 4 rad/s

2012 TWO MARKS

7.7 The feedback system shown below oscillates at 2 rad/s when

2011 TWO MARKS

7.11 The block diagram of a system with one input u and two outputs y1
(A) K = 2 and a = 0.75 (B) K = 3 and a = 0.75 and y2 is given below.
(C) K = 4 and a = 0.5 (D) K = 2 and a = 0.5

7.8 The state variable description of an LTI system is given by


Jxo1N J 0 a1 0NJx1N J0N
K O K OK O K O
Kxo2O = K 0 0 a2OKx2O + K0O u
Kxo O Ka 0 0OKx 3O K 1O
3 3
L P L PL P L P
Jx1N
K O
y = _1 0 0iKx2O SPECIAL EDITION ( STUDY MATERIAL FORM )
Kx 3O At market Book is available in 3 volume i.e. in 3 book binding
L P
where y is the output and u is the input. The system is control-
form. But at NODIA Online Store book is available in 10 book
lable for binding form. Each unit of Book is in separate binding.
Available Only at NODIA Online Store
(A) a1 ! 0, a2 = 0, a 3 ! 0 (B) a1 = 0, a2 ! 0, a 3 ! 0
(C) a1 = 0, a 3 ! 0, a 3 = 0 (D) a1 ! 0, a2 ! 0, a 3 = 0 Click to Buy
www.nodia.co.in
2011 ONE MARK

7.9 The root locus plot for a system is given below. The open loop
transfer function corresponding to this plot is given by

s (s + 1)
(A) G ^s h H ^s h = k A state space model of the above system in terms of the state vec-
(s + 2) (s + 3)
tor x and the output vector y = [y1 y2]T is
(s + 1)
(B) G ^s h H ^s h = k (A) xo = [2] x + [1] u ; y = [1 2] x
s (s + 2) (s + 3) 2
1
(C) G ^s h H ^s h = k 1 (B) xo = [- 2] x + [1] u; y = > H x
2
s (s - 1) (s + 2) (s + 3)
-2 0 1
(D) G ^s h H ^s h = k
(s + 1) (C) xo = > H x + > H u ; y = 81 2B x
s (s + 2) (s + 3) 0 -2 1
For the transfer function G (jw) = 5 + jw , the corresponding Nyquist 2 0 1 1
7.10
(D) xo = > H x + > H u ; y = > H x
plot for positive frequency has the form 0 2 1 2

Common Data For Q. 7.4 & 7.5


The input-output transfer function of a plant H (s) = 100 .
s (s + 10) 2
The plant is placed in a unity negative feedback configuration as
shown in the figure below.
GATE Electronics and Communication Topicwise Solved Paper by RK Kanodia & Ashish Murolia Page 158

7.12 The gain margin of the system under closed loop unity negative
feedback is
(A) 10s + 1 (B) 100s + 1
(A) 0 dB (B) 20 dB 0.1s + 1 0.1s + 1
(C) 26 dB (D) 46 dB (C) 100s (D) 0.1s + 1
10s + 1 10s + 1
7.13 The signal flow graph that DOES NOT model the plant transfer
function H (s) is
2010 TWO MARKS

7.17 A unity negative feedback closed loop system has a plant with the
transfer function G (s) = 2 1 and a controller Gc (s) in the
s + 2s + 2
feed forward path. For a unit set input, the transfer function of the
controller that gives minimum steady state error is
(A) Gc (s) = s + 1 (B) Gc (s) = s + 2
s+2 s+1
(s + 1) (s + 4)
GATE Electronics & Communication (C) Gc (s) = (D) Gc (s) = 1 + 2 + 3s
(s + 2) (s + 3) s
by RK Kanodia
Now in 3 Volume Common Data For Q. 7.10 & 7.11 :
Purchase Online at maximum discount from online store The signal flow graph of a system is shown below:
and get POSTAL and Online Test Series Free
visit www.nodia.co.in

7.18 The state variable representation of the system can be


1 1 0 -1 1 0
xo= > H x + > H u xo = > H x +> Hu
(A) -1 0 2 (B) -1 0 2
yo = [0 0.5] x yo = 80 0.5B x
1 1 0 -1 1 0
xo = > H x +> Hu xo = > H x +> Hu
(C) -1 0 2 (D) -1 0 2
2010 ONE MARK yo = 80.5 0.5B x yo = 80.5 0.5B x
7.19 The transfer function of the system is
The transfer function Y (s) /R (s) of the system shown is
(A) s2+ 1 s-1
7.14
(B)
s +1 s2+1
(C) 2 s + 1 (D) s-1
s +s+1 s2+s+1

For more GATE Resources, Mock Test and


Study material join the community
1
http://www.facebook.com/gateec2014
(A) 0 (B)
s+1 2009 ONE MARK

(C) 2 (D) 2 7.20 The magnitude plot of a rational transfer function G (s) with real
s+1 s+3
coefficients is shown below. Which of the following compensators
Y (s)
7.15 A system with transfer function = s has an output has such a magnitude plot ?
p X (s) s + p
y (t) = cos a2t - k
3
for the input signal x (t) = p cos a2t - p k. Then, the system param-
2
eter p is
(A) 3 (B) 2/ 3
(C) 1 (D) 3 /2

7.16 For the asymptotic Bode magnitude plot shown below, the system (A) Lead compensator (B) Lag compensator
transfer function can be
GATE Electronics and Communication Topicwise Solved Paper by RK Kanodia & Ashish Murolia Page 159

(C) PID compensator (D) Lead-lag compensator (C) 6 dB and 90c


(D) 3 dB and 90c
7.21 Consider the system
dx = Ax + Bu with A = =1 0G and B = = p G
dt 0 1 q 2008 ONE MARKS
where p and q are arbitrary real numbers. Which of the following 7.26 Step responses of a set of three second-order underdamped systems
statements about the controllability of the system is true ? all have the same percentage overshoot. Which of the following
(A) The system is completely state controllable for any nonzero diagrams represents the poles of the three systems ?
values of p and q
(B) Only p = 0 and q = 0 result in controllability
(C) The system is uncontrollable for all values of p and q
(D) We cannot conclude about controllability from the given data

2009 TWO MARKS

7.22 The feedback configuration and the pole-zero locations of


2
G (s) = s2 - 2s + 2
s + 2s + 2
are shown below. The root locus for negative values of k , i.e. for
SPECIAL EDITION ( STUDY MATERIAL FORM )
- 3 < k < 0 , has breakaway/break-in points and angle of depar-
ture at pole P (with respect to the positive real axis) equal to At market Book is available in 3 volume i.e. in 3 book binding
form. But at NODIA Online Store book is available in 10 book
binding form. Each unit of Book is in separate binding.
Available Only at NODIA Online Store

Click to Buy
www.nodia.co.in

(A) ! 2 and 0c (B) ! 2 and 45c


(C) ! 3 and 0c (D) ! 3 and 45c

7.23 The unit step response of an under-damped second order system


has steady state value of -2. Which one of the following transfer
functions has theses properties ?
(A) 2 - 2.24 (B) 2 - 3.82
s + 2.59s + 1.12 s + 1.91s + 1.91
(C) 2 - 2.24 (D) 2 - 382
s - 2.59s + 1.12 s - 1.91s + 1.91

Common Data For Q. 7.16 and 7.17 :


The Nyquist plot of a stable transfer function G (s) is shown in the
figure are interested in the stability of the closed loop system in
the feedback configuration shown.

7.27 The pole-zero given below correspond to a

7.24 Which of the following statements is true ?


(A) G (s) is an all-pass filter
(B) G (s) has a zero in the right-half plane
(C) G (s) is the impedance of a passive network
(D) G (s) is marginally stable
(A) Law pass filter (B) High pass filter
7.25 The gain and phase margins of G (s) for closed loop stability are
(A) 6 dB and 180c (C) Band filter (D) Notch filter
(B) 3 dB and 180c
GATE Electronics and Communication Topicwise Solved Paper by RK Kanodia & Ashish Murolia Page 160

2008 TWO MARKS

7.28 Group I lists a set of four transfer functions. Group II gives a list
of possible step response y (t). Match the step responses with the
corresponding transfer functions.

(A) P - 3, Q - 1, R - 4, S - 2 (B) P - 3, Q - 2, R - 4, S - 1
GATE Electronics & Communication (C) P - 2, Q - 1, R - 4, S - 2 (D) P - 3, Q - 4, R - 1, S - 2
by RK Kanodia
7.29 A signal flow graph of a system is given below
Now in 3 Volume
Purchase Online at maximum discount from online store
and get POSTAL and Online Test Series Free
visit www.nodia.co.in

The set of equalities that corresponds to this signal flow graph is


Jx1N R b - g 0 VJx1N R0 0 V
K O S WK O S W u1
(A) d K x2O = S g a 0 WK x2O+ S0 1 We o
dt K O S u2
x3 S- a b 0 WWK x3O SS1 0 WW
L P L P
Jx1N RT0 a g XVJx1N TR1 0 XV
S W K O S W u1
(B) d K x2O = S0 - a - g WK x2O+ S0 1 We o
K O
dt K O S u2
x3 S0 b - b WWK x3O SS0 0 WW
L P TR L P
Jx1N - a b 0 VXJx1N RT1 0 VX
K O S WK O S W u1
(C) d K x2O = S- b - g 0 WK x2O+ S0 1 We o
dt K O S u2
x3 S a g 0 WWK x3O SS0 0 WW
L P TR L P
Jx1N - a 0 b XVJx1N TR1 0XV
K O S WK O S W u1
(D) d K x2O = S g 0 a WK x2O+ S0 1 We o
dt K O S u2
x3 S- b 0 - a WWK x3O SS0 0 WW
L P T XL P T X
For more GATE Resources, Mock Test and
Study material join the community
http://www.facebook.com/gateec2014
7.30 A certain system has transfer function
G (s) = 2 s + 8
s + as - 4
where a is a parameter. Consider the standard negative unity
feedback configuration as shown below

Which of the following statements is true?


(A) The closed loop systems is never stable for any value of a
(B) For some positive value of a, the closed loop system is stable,
GATE Electronics and Communication Topicwise Solved Paper by RK Kanodia & Ashish Murolia Page 161

but not for all positive values. T (s) = 5


(C) For all positive values of a, the closed loop system is stable. (s + 5)( s2 + s + 1)
The second-order approximation of T (s) using dominant pole con-
(D) The closed loop system stable for all values of a, both positive cept is
and negative. 1 5
(A) (B)
(s + 5)( s + 1) (s + 5)( s + 1)
7.31 The number of open right half plane of
10 (C) 2 5 (D) 2 1
G (s) = 5 is s +s+1 s +s+1
s + 2s + 3s + 6s2 + 5s + 3
4 3
7.37 The open-loop transfer function of a plant is given as G (s) = s 1- 1 .
(A) 0 (B) 1 2

If the plant is operated in a unity feedback configuration, then the


(C) 2 (D) 3 lead compensator that an stabilize this control system is
10 (s - 1) 10 (s + 4)
7.32 The magnitude of frequency responses of an underdamped second (A) (B)
s+2 s+2
order system is 5 at 0 rad/sec and peaks to 10 at 5 2 rad/sec.
3 10 (s + 2) 2 (s + 2)
The transfer function of the system is (C) (D)
s + 10 s + 10
(A) 2 500 (B) 2 375
s + 10s + 100 s + 5s + 75 7.38 A unity feedback control system has an open-loop transfer function
(C) 2 720 (D) 2 1125 K
G (s) =
s + 12s + 144 s + 25s + 225 2
s (s + 7s + 12)
7.33 Group I gives two possible choices for the impedance Z in the diagram.
The gain K for which s = 1 + j1 will lie on the root locus of this
The circuit elements in Z satisfy the conditions R2 C2 > R1 C1. The
SPECIAL EDITION ( STUDY MATERIAL FORM )
transfer functions V0 represents a kind of controller.
Vi At market Book is available in 3 volume i.e. in 3 book binding
form. But at NODIA Online Store book is available in 10 book
binding form. Each unit of Book is in separate binding.
Available Only at NODIA Online Store

Click to Buy
www.nodia.co.in
Match the impedances in Group I with the type of controllers in
Group II system is
(A) 4 (B) 5.5
(C) 6.5 (D) 10

7.39 The asymptotic Bode plot of a transfer function is as shown in the


figure. The transfer function G (s) corresponding to this Bode plot is

(A) Q - 1, R - 2 (B) Q - 1, R - 3
(C) Q - 2, R - 3 (D) Q - 3, R - 2

2007 ONE MARK

7.34 If the closed-loop transfer function of a control system is given as


s-5 (A) 1 (B) 1
T (s) , then It is (s + 1)( s + 20) s (s + 1)( s + 20)
(s + 2)( s + 3)
(A) an unstable system (B) an uncontrollable system (C) 100 (D) 100
s (s + 1)( s + 20) s (s + 1)( 1 + 0.05s)
(C) a minimum phase system (D) a non-minimum phase sys-
tem
7.40 The state space representation of a separately excited DC servo
motor dynamics is given as
dw
> H = =- 1 - 10G=ia G + =10Gu
dt -1 1 w 0
2007 TWO MARKS dio
dt
7.35 A control system with PD controller is shown in the figure. If the where w is the speed of the motor, ia is the armature current and
velocity error constant KV = 1000 and the damping ratio z = 0.5 , w (s)
then the value of KP and KD are u is the armature voltage. The transfer function of the motor
U (s)
is

(A) 10 (B) 1
s2 + 11s + 11 s2 + 11s + 11
(C) 2 10s + 10 (D) 2 1
(A) KP = 100, KD = 0.09 (B) KP = 100, KD = 0.9 s + 11s + 11 s + s + 11
(C) KP = 10, KD = 0.09 (D) KP = 10, KD = 0.9

7.36 The transfer function of a plant is Statement for linked Answer Question 8.33 & 8.34 :
GATE Electronics and Communication Topicwise Solved Paper by RK Kanodia & Ashish Murolia Page 162

Consider a linear system whose state space representation is 7.47 The transfer function of a phase lead compensator is given by
x (t) = Ax (t). If the initial state vector of the system is x (0) = = G,
1 Gc (s) = 1 + 3Ts where T > 0 The maximum phase shift provide by
- 2 1 + Ts
e-2x such a compensator is
then the system response is x (t) = > H. If the itial state vector (A) p (B) p
- 2e-2t 2 3
1
of the system changes to x (0) = = G, then the system response
-2 (C) p (D) p
e-t 4 6
becomes x (t) = > -tH
-e
7.48 A linear system is described by the following state equation
0 1
Xo (t) = AX (t) + BU (t), A = =
- 1 0G
7.41 The eigenvalue and eigenvector pairs (li vi) for the system are
1 1 1 1
(A) e- 1 = Go and e- 2 = Go (B) e- 1, = Go and e2, = Go
-1 -2 -1 -2 The state transition matrix of the system is
cos t sin t - cos t sin t
(A) = G (B) =
- sin t - cos t G
1 1 1 1
(C) e- 1, = Go and e- 2, = Go (D) e- 2 = Go and e1, = Go - sin t cos t
-1 -2 -1 -2
- cos t - sin t cos t - sin t
(C) = G (D) =
cos t sin t G
7.42 The system matrix A is
0 1 1 1 - sin t cos t
(A) =
- 1 1G
(B) =
- 1 - 2G
2 1 0 1
(C) =
- 1 - 1G
(D) =
- 2 - 3G
Statement for Linked Answer Questions 7.41 & 7.42 :
Consider a unity - gain feedback control system whose open - loop
transfer function is : G (s) = as + 1
GATE Electronics & Communication s2
by RK Kanodia 7.49 The value of a so that the system has a phase - margin equal to p
4
is approximately equal to
Now in 3 Volume (A) 2.40 (B) 1.40
Purchase Online at maximum discount from online store (C) 0.84 (D) 0.74
and get POSTAL and Online Test Series Free
visit www.nodia.co.in 7.50 With the value of a set for a phase - margin of p , the value of unit
4
- impulse response of the open - loop system at t = 1 second is equal
2006 ONE MARK to
(A) 3.40 (B) 2.40
7.43 The open-loop function of a unity-gain feedback control system is
given by (C) 1.84 (D) 1.74

G (s) = K
(s + 1)( s + 2)
2005 ONE MARK
The gain margin of the system in dB is given by
(A) 0 (B) 1 7.51 A linear system is equivalently represented by two sets of state
equations :
(C) 20 (D) 3
Xo = AX + BU and Wo = CW + DU
The eigenvalues of the representations are also computed as [l]
2006 TWO MARKS
and [m]. Which one of the following statements is true ?
7.44 Consider two transfer functions G1 (s) = 2 1 and (A) [l] = [m] and X = W (B) [l] = [m] and X ! W
s s + as + b (C) [l] ! [m] and X = W (D) [l] = [m] and X ! W
G2 (s) = 2 .
s + as + b
The 3-dB bandwidths of their frequency responses are, respectively 7.52 Which one of the following polar diagrams corresponds to a lag
(A) a2 - 4b , a2 + 4b (B) a2 + 4b , a2 - 4b
(C) a2 - 4b , a2 - 4b (D) a2 + 4b , a2 + 4b
For more GATE Resources, Mock Test and
Study material join the community
The Nyquist plot of G (jw) H (jw)for a closed loop control system,
7.45
http://www.facebook.com/gateec2014
passes through (- 1, j0) point in the GH plane. The gain margin of
the system in dB is equal to network ?
(A) infinite (B) greater than zero
(C) less than zero (D) zero

7.46 The positive values of K and a so that the system shown in the
figures below oscillates at a frequency of 2 rad/sec respectively are

(A) 1, 0.75 (B) 2, 0.75


(C) 1, 1 (D) 2, 2
GATE Electronics and Communication Topicwise Solved Paper by RK Kanodia & Ashish Murolia Page 163

7.53 Despite the presence of negative feedback, control systems still have
problems of instability because the
(A) Components used have non- linearities
(B) Dynamic equations of the subsystem are not known exactly.
(C) Mathematical analysis involves approximations.
(D) System has large negative phase angle at high frequencies.

2005 TWO MARKS

7.54 The polar diagram of a conditionally stable system for open loop
gain K = 1 is shown in the figure. The open loop transfer function
of the system is known to be stable. The closed loop system is stable
for
Statement for Linked Answer Question 40 and 41 :
The open loop transfer function of a unity feedback system is given
by
-2s
G (s) = 3e
s (s + 2)
7.59 The gain and phase crossover frequencies in rad/sec are, respectively

SPECIAL EDITION ( STUDY MATERIAL FORM )


At market Book is available in 3 volume i.e. in 3 book binding
form. But at NODIA Online Store book is available in 10 book
binding form. Each unit of Book is in separate binding.
Available Only at NODIA Online Store
(A) K < 5 and 1 < K < 1 (B) K < 1 and 1 < K < 5
2 8 8 2 Click to Buy
(C) K < 1 and 5 < K (D) K > 1 and 5 > K www.nodia.co.in
8 8
7.55 In the derivation of expression for peak percent overshoot (A) 0.632 and 1.26 (B) 0.632 and 0.485
- px (C) 0.485 and 0.632 (D) 1.26 and 0.632
Mp = exp e o # 100%
1 - x2
7.60 Based on the above results, the gain and phase margins of the
Which one of the following conditions is NOT required ?
system will be
(A) System is linear and time invariant
(A) -7.09 dB and 87.5c (B) 7.09 dB and 87.5c
(B) The system transfer function has a pair of complex conjugate
(C) 7.09 dB and - 87.5c (D) - 7.09 and - 87.5c
poles and no zeroes.
(C) There is no transportation delay in the system.
(D) The system has zero initial conditions. 2004 ONE MARK

7.61 The gain margin for the system with open-loop transfer function
7.56 A ramp input applied to an unity feedback system results in 5%
steady state error. The type number and zero frequency gain of the 2 (1 + s)
G (s) H (s) = , is
system are respectively s2
(A) 3 (B) 0
(A) 1 and 20 (B) 0 and 20
(C) 1 (D) - 3
(C) 0 and 1 (D) 1 and 1
20 20
7.62 Given G (s) H (s) = K .The point of intersection of the
7.57 A double integrator plant G (s) = K/s2, H (s) = 1 is to be compensated s (s + 1)( s + 3)
asymptotes of the root loci with the real axis is
to achieve the damping ratio z = 0.5 and an undamped natural
(A) - 4 (B) 1.33
frequency, wn = 5 rad/sec which one of the following compensator
Ge (s) will be suitable ? (C) - 1.33 (D) 4
(A) s + 3 (B) s + 99
s + 99 s+3
2004 TWO MARKS
(C) s - 6 (D) s - 6
s + 8.33 s 7.63 Consider the Bode magnitude plot shown in the fig. The transfer
K (1 - s) function H (s) is
7.58 An unity feedback system is given as G (s) = .
s (s + 3)
Indicate the correct root locus diagram.

(s + 10) 10 (s + 1)
(A) (B)
(s + 1)( s + 100) (s + 10)( s + 100)
GATE Electronics and Communication Topicwise Solved Paper by RK Kanodia & Ashish Murolia Page 164

(D) controllable and observable


102 (s + 1) 103 (s + 100)
(C) (D)
(s + 10)( s + 100) (s + 1)( s + 10) 1 0
Given A = =
0 1G
7.71 , the state transition matrix eAt is given by
7.64 A causal system having the transfer function H (s) = 1/ (s + 2) is
excited with 10u (t). The time at which the output reaches 99% of 0 e-t et 0
its steady state value is (A) > -t H (B) = G
e 0 0 et
(A) 2.7 sec (B) 2.5 sec
e-t 0 0 et
(C) 2.3 sec (D) 2.1 sec (C) > H (D) = t G
0 e-t e 0
7.65 A system has poles at 0.1 Hz, 1 Hz and 80 Hz; zeros at 5 Hz, 100
Hz and 200 Hz. The approximate phase of the system response at 2003 ONE MARK
20 Hz is
7.72 Fig. shows the Nyquist plot of the open-loop transfer function
(A) - 90c (B) 0c
G (s) H (s) of a system. If G (s) H (s) has one right-hand pole, the
(C) 90c (D) - 180c closed-loop system is
7.66 Consider the signal flow graph shown in Fig. The gain x5 is
x1

(A) always stable


GATE Electronics & Communication (B) unstable with one closed-loop right hand pole
by RK Kanodia (C) unstable with two closed-loop right hand poles
Now in 3 Volume (D) unstable with three closed-loop right hand poles
Purchase Online at maximum discount from online store 7.73 A PD controller is used to compensate a system. Compared to the
and get POSTAL and Online Test Series Free uncompensated system, the compensated system has
visit www.nodia.co.in (A) a higher type number (B) reduced damping
(C) higher noise amplification (D) larger transient overshoot
1 - (be + cf + dg) bedg
(A) (B)
abcd 1 - (be + cf + dg)
abcd 1 - (be + cf + dg) + bedg 2003 TWO MARKS
(C) (D)
1 - (be + cf + dg) + bedg abcd 7.74 The signal flow graph of a system is shown in Fig. below. The
-2 2
If A = =
1 - 3G
7.67 , then sin At is transfer function C (s)/ R (s) of the system is

sin (- 4t) + 2 sin (- t) - 2 sin (- 4t) + 2 sin (- t)


(A) 1 = G
3 - sin (- 4t) + sin (- t) 2 sin (- 4t) + sin (- t)
sin (- 2t) sin (2t)
(B) =
sin (t) sin (- 3t)G
(A) 6 (B) 6s
sin (4t) + 2 sin (t) 2 sin (- 4t) - 2 sin (- t)
(C) 1 =
2 sin (4t) + sin (t) G
s2 + 29s + 6 s2 + 29s + 6
3 sin (- 4t) + sin (t)
-
cos (- t) + 2 cos (t) 2 cos (- 4t) + 2 cos (- t) s (s + 2) s (s + 27)
(D) 1 = G
(C) 2
(D) 2
3 - cos (- 4t) + cos (- t) - 2 cos (- 4t) + cos (t) s + 29s + 6 s + 29s + 6
7.75 The root locus of system G (s) H (s) = K has the break-
7.68 The open-loop transfer function of a unity feedback system is s (s + 2)( s + 3)
G (s) = K For more GATE Resources, Mock Test and
2
s (s + s + 2)( s + 3) Study material join the community
The range of K for which the system is stable is
http://www.facebook.com/gateec2014
(A) 21 > K > 0 (B) 13 > K > 0
4
away point located at
(C) 21 < K < 3 (D) - 6 < K < 3
4 (A) (- 0.5, 0) (B) (- 2.548, 0)
7.69 For the polynomial P (s) = s2 + s 4 + 2s3 + 2s2 + 3s + 15 the number (C) (- 4, 0) (D) (- 0.784, 0)
of roots which lie in the right half of the s -plane is
(A) 4 (B) 2 7.76 The approximate Bode magnitude plot of a minimum phase system
is shown in Fig. below. The transfer function of the system is
(C) 3 (D) 1

7.70 The state variable equations of a system are : xo1 =- 3x1 - x2 = u, xo2 = 2x1
and y = x1 + u . The system is
(A) controllable but not observable
(B) observable but not controllable
(C) neither controllable nor observable
GATE Electronics and Communication Topicwise Solved Paper by RK Kanodia & Ashish Murolia Page 165

7.82 The phase margin of a system with the open - loop transfer function
(1 - s)
G (s) H (s) =
(1 + s)( 2 + s)
(A) 0c (B) 63.4c
(C) 90c (D) 3

7.83 The transfer function Y (s)/ U (s) of system described by the state
equation xo (t) =- 2x (t) + 2u (t) and y (t) = 0.5x (t) is
(s + 0.1) 3 (s + 0.1) 3
(A) 108 (B) 10 7
(A) 0.5 (B) 1
(s + 10) 2 (s + 100) (s + 10)( s + 100) (s - 2) (s - 2)
(C) 0.5 1
2
(s + 0.1) (s + 0.1) 3 (D)
(C) (D) (s + 2) (s + 2)
(s + 10) 2 (s + 100) (s + 10)( s + 100) 2
7.77 A second-order system has the transfer function
2002 TWO MARKS
C (s)
= 2 4
R (s) s + 4s + 4 7.84 The system shown in the figure remains stable when
With r (t) as the unit-step function, the response c (t) of the system (A) k < - 1 (B) - 1 < k < 3
is represented by (C) 1 < k < 3 (D) k > 3

7.85 The transfer function of a system is G (s) = 100 . For a


(s + 1)( s + 100)
SPECIAL EDITION ( STUDY MATERIAL FORM )
At market Book is available in 3 volume i.e. in 3 book binding
form. But at NODIA Online Store book is available in 10 book
binding form. Each unit of Book is in separate binding.
Available Only at NODIA Online Store

Click to Buy
www.nodia.co.in
unit - step input to the system the approximate settling time for 2%
criterion is

(A)100 sec (B) 4 sec


7.78 The gain margin and the phase margin of feedback system with
8 (C) 1 sec (D) 0.01 sec
G (s) H (s) = are
(s + 100) 3 7.86 The characteristic polynomial of a system is
(A) dB, 0c (B) 3, 3
q (s) = 2s5 + s 4 + 4s3 + 2s2 + 2s + 1
(C) 3, 0c (D) 88.5 dB, 3
The system is
7.79 The zero-input response of a system given by the state-space equation (A) stable (B) marginally stable
xo1 1 0 x1 x1 (0) 1
=xo G = =1 1G=x G and =x (0)G = = 0 G is (C) unstable (D) oscillatory
2 2 2
7.87 The system with the open loop transfer function
tet et
(A) = G (B) = G G (s) H (s) = 1 has a gain margin of
t t s (s 2
+ s + 1 )
(A) - 6 db (B) 0 db
et t
(C) = t G (D) = t G (C) 35 db (D) 6 db
te te

2002 ONE MARK 2001 ONE MARK

7.80 Consider a system with transfer function G (s) = 2s + 6 . Its 7.88 The Nyquist plot for the open-loop transfer function G (s) of a unity
damping ratio will be 0.5 when the value of k is ks + s + 6 negative feedback system is shown in the figure, if G (s) has no pole
(A) 2 (B) 3 in the right-half of s -plane, the number of roots of the system
6 characteristic equation in the right-half of s -plane is
(C) 1 (D) 6 (A) 0 (B) 1
6
(C) 2 (D) 3
7.81 Which of the following points is NOT on the root locus of a system
with the open-loop transfer function G (s) H (s) = k 7.89 The equivalent of the block diagram in the figure is given is
s (s + 1)( s + 3)
(A) s =- j 3 (B) s =- 1.5
(C) s =- 3 (D) s =- 3
GATE Electronics and Communication Topicwise Solved Paper by RK Kanodia & Ashish Murolia Page 166

7.90 If the characteristic equation of a closed - loop system is s2 + 2s + 2 = 0


, then the system is Z3 (s) - Z3 (s)
(A) ,
(A) overdamped (B) critically damped Z1 (s) + Z3 (s) + Z4 (s) Z1 (s) + Z3 (s)
(C) underdamped (D) undamped - Z3 (s) - Z3 (s)
(B) ,
Z2 (s) - Z3 (s) + Z4 (s) Z1 (s) + Z3 (s)
7.91 The root-locus diagram for a closed-loop feedback system is shown Z3 (s) Z3 (s)
in the figure. The system is overdamped. (C) ,
Z2 (s) + Z3 (s) + Z4 (s) Z1 (s) + Z3 (s)
- Z3 (s) Z3 (s)
(D) ,
Z2 (s) - Z3 (s) + Z4 (s) Z1 (s) + Z3 (s)
7.93 The open-loop DC gain of a unity negative feedback system with
closed-loop transfer function 2 s + 4 is
s + 7s + 13
GATE Electronics & Communication (A) 4 (B) 4
by RK Kanodia 13 9
(C) 4 (D) 13
Now in 3 Volume
Purchase Online at maximum discount from online store 7.94 The feedback control system in the figure is stable
and get POSTAL and Online Test Series Free
visit www.nodia.co.in

(A) for all K $ 0 (B) only if K $ 0


(C) only if 0 # K < 1 (D) only if 0 # K # 1

2000 ONE MARK

7.95 An amplifier with resistive negative feedback has tow left half plane
poles in its open-loop transfer function. The amplifier
(A) will always be unstable at high frequency
(B) will be stable for all frequency
(C) may be unstable, depending on the feedback factor
(D) will oscillate at low frequency.

2000 TWO MARKS

(A) only if 0 # k # 1 (B) only if 1 < k < 5 For more GATE Resources, Mock Test and
(C) only if k > 5 (D) if 0 # k < 1 or k > 5 Study material join the community
http://www.facebook.com/gateec2014
2001 TWO MARK 1
7.96 A system described by the transfer function H (s) = 3 2
is stable. The constraints on a and k are. s + as + ks + 3
7.92 An electrical system and its signal-flow graph representations are
shown the figure (A) and (B) respectively. The values of G2 and H (A) a > 0, ak < 3 (B) a > 0, ak > 3
, respectively are (C) a < 0, ak > 3 (D) a > 0, ak < 3

1999 ONE MARK

7.97 For a second order system with the closed-loop transfer function
T (s) = 2 9
s + 4s + 9
the settling time for 2-percent band, in seconds, is
(A) 1.5 (B) 2.0
GATE Electronics and Communication Topicwise Solved Paper by RK Kanodia & Ashish Murolia Page 167

(C) 3.0 (D) 4.0 the eigen values of the closed-loop system will be
(A) 0, - 1, - 2
7.98 The gain margin (in dB) of a system a having the loop transfer
(B) 0, - 1, - 3
function
(C) - 1, - 1, - 2
G (s) H (s) = 2 is
s (s + 1) (D) 0, - 1, - 1
(A) 0 (B) 3
(C) 6 (D) 3 1998 ONE MARK

7.99 The system modeled described by the state equations is 7.105 The number of roots of s3 + 5s2 + 7s + 3 = 0 in the left half of the s
0 1 0 -plane is
X => H x + > Hu (A) zero (B) one
2 -3 1
Y = 81 1B x (C) two (D) three

(A) controllable and observable 7.106 The transfer function of a tachometer is of the form
(B) controllable, but not observable (A) Ks (B) K
s
(C) observable, but not controllable
(D) neither controllable nor observable (C) K (D) K
(s + 1) s (s + 1)
7.100 The phase margin (in degrees) of a system having the loop transfer
SPECIAL EDITION ( STUDY MATERIAL FORM )
function G (s) H (s) = 2 3 is At market Book is available in 3 volume i.e. in 3 book binding
s (s + 1)
(A) 45c form. But at NODIA Online Store book is available in 10 book
(B) - 30c binding form. Each unit of Book is in separate binding.
Available Only at NODIA Online Store
(C) 60c
(D) 30c Click to Buy
www.nodia.co.in
1999 TWO MARKS
7.107 Consider a unity feedback control system with open-loop transfer
7.101 An amplifier is assumed to have a single-pole high-frequency transfer function G (s) = K .
function. The rise time of its output response to a step function input s (s + 1)
is 35 n sec . The upper 3 dB frequency (in MHz) for the amplifier to The steady state error of the system due to unit step input is
as sinusoidal input is approximately at (A) zero
(A) 4.55 (B) K
(B) 10 (C) 1/K
(C) 20 (D) infinite
(D) 28.6 7.108 The transfer function of a zero-order-hold system is
7.102 If the closed - loop transfer function T (s) of a unity negative feedback (A) (1/s) (1 + e-sT )
system is given by (B) (1/s) (1 - e-sT )
T (s) = n an - 1 s + an (C) 1 - (1/s) e-sT
n-1
s + a1 s + .... + an - 1 s + an (D) 1 + (1/s) e-sT
then the steady state error for a unit ramp input is
(A) an (B) an 7.109 In the Bode-plot of a unity feedback control system, the value of
an - 1 an - 2
phase of G (jw) at the gain cross over frequency is - 125c. The phase
(C) an - 2 (D) zero margin of the system is
an - 2
(A) - 125c
7.103 Consider the points s1 =- 3 + j4 and s2 =- 3 - j2 in the s-plane. (B) - 55c
Then, for a system with the open-loop transfer function
(C) 55c
G (s) H (s) = K 4 (D) 125c
(s + 1)
(A) s1 is on the root locus, but not s2
7.110 Consider a feedback control system with loop transfer function
(B) s2 is on the root locus, but not s1
K (1 + 0.5s)
(C) both s1 and s2 are on the root locus G (s) H (s) =
s (1 + s) (1 + 2s)
(D) neither s1 nor s2 is on the root locus The type of the closed loop system is
(A) zero
7.104 For the system described by the state equation
R 0 1 0V R0V (B) one
S W S W (C) two
xo = S 0 0 1W x + S0W u
SS0.5 1 2WW SS1WW (D) three
T X T X
If the control signal u is given by u = [- 0.5 - 3 - 5] x + v , then
7.111 The transfer function of a phase lead controller is 1 + 3Ts . The
1 + Ts
GATE Electronics and Communication Topicwise Solved Paper by RK Kanodia & Ashish Murolia Page 168

maximum value of phase provided by this controller is (C) 0


(A) 90c (D) None of the above
(B) 60c
(C) 45c ***********

(D) 30c

7.112 The Nyquist plot of a phase transfer function g (jw) H (jw) of a system
encloses the (–1, 0) point. The gain margin of the system is
(A) less than zero
(B) zero
(C) greater than zero
(D) infinity

7.113 The transfer function of a system is 2s2 + 6s + 5


(s + 1) 2 (s + 2)
The characteristic equation of the system is
(A) 2s2 + 6s + 5 = 0
(B) (s + 1) 2 (s + 2) = 0
(C) 2s2 + 6s + 5 + (s + 1) 2 (s + 2) = 0

GATE Electronics & Communication


by RK Kanodia
Now in 3 Volume
Purchase Online at maximum discount from online store
and get POSTAL and Online Test Series Free
visit www.nodia.co.in

(D) 2s2 + 6s + 5 - (s + 1) 2 (s + 2) = 0

7.114 In a synchro error detector, the output voltage is proportional to


[w (t)] n, where w (t) is the rotor velocity and n equals
(A) –2
(B) –1
(C) 1
(D) 2

1997 ONE MARK

7.115 In the signal flow graph of the figure is y/x equals

(A) 3 For more GATE Resources, Mock Test and


(B) 5
Study material join the community
2
(C) 2
http://www.facebook.com/gateec2014
(D) None of the above

7.116 A certain linear time invariant system has the state and the output
equations given below
Xo1 1 - 1 X1 0
> o H = >0 1 H>X H + >1H u
X2 2

y = 81 1B: X1 D
X2
dy
If X1 (0) = 1, X2 (0) =- 1, u (0) = 0, then is
dt t=0
(A) 1
(B) –1
GATE Electronics and Communication Topicwise Solved Paper by RK Kanodia & Ashish Murolia Page 169

SOLUTIONS Pk2 = ^1 h^s-1h^1 h^1 h = s-1


since, all the loops are touching to the paths Pk1 and Pk2 so,
D k 1 = Dk 2 = 1
Now, we have
D = 1 - (sum of individual loops)
7.1 Option (B) is correct. + (sum of product of nontouching
From the given plot, we obtain the slope as loops)
20 log G2 - 20 log G1 Here, the loops are
Slope =
log w2 - log w1
L1 = ^- 4h^1 h =- 4
From the figure
L2 = ^- 4h^s-1h = 4s-1
20 log G2 =- 8 dB
20 log G1 = 32 dB L 3 = ^- 2h^s-1h^s-1h =- 2s-2
and w1 = 1 rad/s L 4 = ^- 2h^s-1h^1 h =- 2s-1
w2 = 10 rad/s As all the loop L1, L2, L 3 and L 4 are touching to each other so,
So, the slope is D = 1 - ^L1 + L2 + L 3 + L 4h
Slope = - 8 - 32 = 1 - ^- 4 - 4s-1 - 2s-2 - 2s-1h
log 10 - log 1
=- 40 dB/decade = 5 + 6s1 + 2s2
Therefore, the transfer function can be given as SPECIAL EDITION ( STUDY MATERIAL FORM )
G ^s h = k2 At market Book is available in 3 volume i.e. in 3 book binding
S
at w = 1
form. But at NODIA Online Store book is available in 10 book
G ^ jwh = k 2 = k
w binding form. Each unit of Book is in separate binding.
In decibel, Available Only at NODIA Online Store
20 log G ^ jwh = 20 log k = 32 Click to Buy
or,
32

k = 10 = 39.8
20
www.nodia.co.in
Hence, the Transfer function is
G ^s h = k2 = 392.8 From Mason’s gain formulae
s s
Y ^s h
7.2 Option (C) is correct. = SPk Dk
Given, open loop transfer function U ^s h D
G ^s h = 10Ka = Ka 1 = s-2 + s-1
1 + 10s s + 10 5 + 6s-1 + 2s-2
By taking inverse Laplace transform, we have
= 2s+1
g ^ t h = e- t 5s + 6s + 2
1
10

Comparing with standard form of transfer function, Ae-t/t , we get 7.4 Option (A) is correct.
the open loop time constant, For the shown state diagram we can denote the states x1 , x2 as below
tol = 10
Now, we obtain the closed loop transfer function for the given
system as
G ^s h
H ^s h = = 10Ka
1 + G ^s h 1 + 10s + 10Ka
= Ka
s + ^Ka + 101 h So, from the state diagram, we obtain
By taking inverse Laplace transform, we get xo1 =- x1 - u
h ^ t h = ka .e-^k + ht
1

xo2 =- x2 + ^1 h^- 1h^1 h^- 1h u + ^- 1h^1 h^- 1h x1


a 10

So, the time constant of closed loop system is obtained as


xo2 =- x2 + x1 + u
tcl = 1 1
ka + 10 and y
or, tcl = 1 = ^- 1h^1 h x2 + ^- 1h^1 h^- 1h x1 + ^1 h^- 1h^1 h^- 1h^1 h u
ka
(approximately) = x1 - x 2 + u
Now, given that ka reduces open loop time constant by a factor of Hence, in matrix form we can write the state variable equations
100. i.e., xo1 - 1 0 x1 -1
> o H = > 1 - 1H >x H + > 1 H u
tcl = tol x2 2
100
x1
or, 1 = 10 and y = 81 - 1B > H + u
ka 100 x2
Hence, ka = 10 which can be written in more general form as
Option (A) is correct. -1 0 -1
Xo = >
1 - 1H
X +> H
7.3

For the given SFG, we have two forward paths 1


Pk1 = ^1 h^s-1h^s-1h^1 h = s-2 y = 81 - 1B X + u
GATE Electronics and Communication Topicwise Solved Paper by RK Kanodia & Ashish Murolia Page 170

7.5 Option (A) is correct. Auxiliary equation A (s) = as2 + (k + 1) = 0


From the obtained state-variable equations
s2 =- k + 1 = - k + 1 (k + 2) =- (k + 2)
We have a (k + 1)
-1 0 s = j k+2
A =>
1 - 1H
jw = j k + 2
S+1 0 w = k + 2 = 2 (Oscillation frequency)
SI - A = >
- 1 S + 1H
So,
k =2

and ^SI - Ah-1 = 1 >S + 1 0 H and a = 2 + 1 = 3 = 0.75


2+2 4
^S + 1h2 1 S + 1
R 1 V 7.8 Option (D) is correct.
S 0 W General form of state equations are given as
S+1
=S 1 1 W
W
S xo = Ax + Bu
S^S + 1h2 S + 1W
T X yo = Cx + Du
Hence, the state transition matrix is obtained as For the given problem
eAt = L-1 ^SI - Ah-1 R 0 a 0V R0V
ZR 1 V_ S 1 W S W
]]S 0 Wbb A = S 0 0 a2W, B = S0W
S+1
= L-1 [S 1 W
1 W` SSa 0 0WW SS1WW
S 3
]S^S + 1h2 S + 1Wb RT 0 a 0VXR0V R 0VT X
\T Xa S 1 WS W S W
AB = S 0 0 a2WS0W = Sa2W
GATE Electronics & Communication SSa
3 0 0WWSS1WW SS 0WW
RT 0 0XT aX1 a2VWTRS0XVW RSa1 a2VW
by RK Kanodia S
Now in 3 Volume A2 B = Sa2 a 3
SS 0 a a
0 0WS0W = S 0W
0WWSS1WW SS 0WW
Purchase Online at maximum discount from online store T
3 1
XT X T X
For controllability it is necessary that following matrix has a tank
and get POSTAL and Online Test Series Free of n = 3 .
visit www.nodia.co.in R0 0 a a V
S 1 2W
U = 6B : AB : A2 B@ = S0 a2 0W
SS1 0 0WW
e-1 0
= > -t -tH So, a2 ! 0 T X
te e
a1 a 2 ! 0 & a1 ! 0 a 3 may be zero or not.
7.6 Option (C) is correct.
(s2 + 9) (s + 2) 7.9 Option (B) is correct.
G (s) = For given plot root locus exists from - 3 to 3, So there must be odd
(s + 1) (s + 3) (s + 4)
(- w2 + 9) (jw + 2) number of poles and zeros. There is a double pole at s =- 3
= Now poles = 0, - 2, - 3, - 3
(jw + 1) (jw + 3) (jw + 4)
The steady state output will be zero if zeros =- 1
G (jw) = 0 k (s + 1)
Thus transfer function G (s) H (s) =
-w 2 + 9 = 0 & w = 3 rad/s s (s + 2) (s + 3) 2
7.10 Option (A) is correct.
7.7 Option (A) is correct.
We have G (jw) = 5 + jw
K (s + 1)
Y (s) = [R (s) - Y (s)] Here s = 5 . Thus G (jw) is a straight line parallel to jw axis.
s + as2 + 2s + 1
3

K (s + 1) K (s + 1) 7.11 Option (B) is correct.


Y (s) ;1 + 3 2 E = 3 R (s)
s + as + s +2 1 s + as2 + 2s + 1
For more GATE Resources, Mock Test and
Y (s) [s3 + as2 + s (2 + k) + (1 + k)] = K (s + 1) R (s)
Y (s)
Study material join the community
Transfer Function, H (s) =
R (s) http://www.facebook.com/gateec2014
K (s + 1)
= 3 dy
s + as2 + s (2 + k) + (1 + k) Here x = y1 and xo = 1
dx
Routh Table :
y1 x 1
y = > H = > H = > Hx
y2 2x 2
Now y1 = 1 u
s+2
y1 (s + 2)
=u
yo1 + 2y1
=u
xo + 2x
=u
a (2 + K) - (1 + K)
For oscillation, =0 =- 2x + u
xo
a
= [- 2] x + [1] u
xo
a = K+1
K+2 Drawing SFG as shown below
GATE Electronics and Communication Topicwise Solved Paper by RK Kanodia & Ashish Murolia Page 171

Y (s)
= 1
R (s) s + 1
7.15 Option (B) is correct.
Transfer function is given as
Y (s)
xo1 = [- 2] x1 + [1] u H (s) = = s
Thus X (s) s + p
y1 = x1 ; y2 = 2x1 jw
H (jw) =
jw + p
y1 1
y = > H = > H x1 Amplitude Response
y2 2
H (jw) = w
Here x1 = x w +p2
2

7.12 Option (C) is correct. Phase Response qh (w) = 90c - tan-1 a w k


100 p
We have G (s) H (s) =
s (s + 10) 2 Input x (t) = p cos a2t - p k
100 2
Now G (jw) H (jw) =
jw (jw + 10) 2 Output y (t) = H (jw) x (t - qh) = cos a2t - p k
3
If wp is phase cross over frequency +G (jw) H (jw) = 180c
H (jw) = p = w
wp
- 180c = 100 tan-1 0 - tan-1 3 - 2 tan-1 a
10 k
Thus w2+ p 2
SPECIAL EDITION ( STUDY MATERIAL FORM )
or - 180c =- 90 - 2 tan-1 (0.1wp)
At market Book is available in 3 volume i.e. in 3 book binding
or 45c = tan-1 (0.1wp)
form. But at NODIA Online Store book is available in 10 book
or tan 45c 0.1wp = 1
binding form. Each unit of Book is in separate binding.
or wp = 10 rad/se Available Only at NODIA Online Store
100
Now G (jw) H (jw) =
w (w2 + 100)
Click to Buy
At w = wp www.nodia.co.in
G (jw) H (jw) = 100 = 1 1 = 2
10 (100 + 100) 20 , (w = 2 rad/ sec)
p 4+p2
Gain Margin =- 20 log 10 G (jw) H (jw) or 4p 2 = 4 + p 2 & 3p 2 = 4
=- 20 log 10 b 1 l or p = 2/ 3
20
= 26 dB Alternative :
7.13 Option (D) is correct. qh = 9- p - a- p kC = p
3 2 6
From option (D) TF = H (s)
p = p - tan-1 w
100 100 So,
6 2 apk
= !
s (s2 + 100) s (s + 10) 2
tan-1 a w k = p - p = p
7.14 Option (B) is correct. p 2 6 3
From the given block diagram w = tan p = 3
p a3k
2 = 3 , (w = 2 rad/ sec)
p
or p = 2/ 3
7.16 Option (A) is correct.
Initial slope is zero, so K = 1
At corner frequency w 1 = 0.5 rad/ sec , slope increases by + 20 dB/
H (s) = Y (s) - E (s) $ 1 decade, so there is a zero in the transfer function at w 1
s+1 At corner frequency w 2 = 10 rad/ sec , slope decreases by - 20 dB/
E (s) = R (s) - H (s) decade and becomes zero, so there is a pole in transfer function at
E (s) w2
= R (s) - Y (s) +
(s + 1)
K a1 + s k
1 w1
E (s) :1 -
s + 1D
= R (s) - Y (s) Transfer function H (s) =
s
a1 + w 2 k
sE (s)
= R (s) - Y (s) ...(1)
(s + 1) 1 a1 + s k (1 + 10s)
0.1
E (s) = =
s (1 + 0.1s)
Y (s) =
s+1
...(2) a1 +
0.1 k
7.17 Option (D) is correct.
From (1) and (2) sY (s) = R (s) - Y (s)
Steady state error is given as
(s + 1) Y (s) = R (s) sR (s)
Transfer function eSS = lim
s"0 1 + G (s) GC (s)
GATE Electronics and Communication Topicwise Solved Paper by RK Kanodia & Ashish Murolia Page 172

R (s) = 1 (unit step unit) D = 1 - [L1 + L2] = 1 - :- 1 - 12 D = 1 + 1 + 12


s s s s s
eSS = lim 1 D1 = 1, D2 = 2
1 + G (s) GC (s)
s"0
Y (s)
1 So, H (s) = = P1 D 1 + P2 D 2
= lim U (s) D
s"0 G (s)
1+ 2 C 1 :1+1:1
s + 2s + 2 2 s (1 + s)
=s = 2
eSS will be minimum if lim GC (s) is maximum 1
1+ + 2 1 (s + s + 1)
s"0
In option (D) s s

lim GC (s) = lim 1 + 2 + 3s = 3 Option (C) is correct.


7.20

s"0 s"0 s This compensator is roughly equivalent to combining lead and lad
So, eSS = lim 1 = 0 (minimum) compensators in the same design and it is referred also as PID
s"0 3
compensator.
7.18 Option (D) is correct. 7.21 Option (C) is correct.
Assign output of each integrator by a state variable 1 0 p
Here A == G and B = = G
0 1 q
1 0 p p
AB = = G= G == G
0 1 q q
p q
S = 8B AB B = =
GATE Electronics & Communication q pG
by RK Kanodia S = pq - pq = 0
Since S is singular, system is completely uncontrollable for all val-
Now in 3 Volume
ues of p and q .
Purchase Online at maximum discount from online store
7.22 Option (B) is correct.
and get POSTAL and Online Test Series Free The characteristic equation is
visit www.nodia.co.in 1 + G (s) H (s) = 0
K (s2 - 2s + 2)
or 1+ =0
s 2 + 2s + 2
or s2 + 2s + 2 + K (s2 - 2s + 2) = 0
2
or K =- s2 + 2s + 2
s - 2s + 2
For break away & break in point differentiating above w.r.t. s we
have
xo1 =- x1 + x2 2 2
xo2 =- x1 + 2u dK =- (s - 2s + 2)( 2s + 2) - (s + 2s + 2)( 2s - 2) = 0
ds (s2 - 2s + 2) 2
y = 0.5x1 + 0.5x2
Thus (s2 - 2s + 2)( 2s + 2) - (s2 + 2s + 2)( 2s - 2) = 0
State variable representation
or s =! 2
-1 1 0
xo = > H x + > Hu Let qd be the angle of departure at pole P , then
-1 0 2
yo = [0.5 0.5] x
7.19 Option (C) is correct.
By masson’s gain formula
For more GATE Resources, Mock Test and
Study material join the community
http://www.facebook.com/gateec2014

Transfer function
H (s) =
Y (s)
=
/ PK DK
U (s) D
Forward path given
P1 (abcdef ) = 2 # 1 # 1 # 0.5 = 12
s s s
P2 (abcdef ) = 2 # 1 # 1 # 0.5
3
Loop gain L1 (cdc) =- 1 - qd - qp1 + qz1 + qz2 = 180c
s
- qd = 180c - (- qp1 + qz1 + q2)
L2 (bcdb) = 1 # 1 # - 1 = -21
s s s
GATE Electronics and Communication Topicwise Solved Paper by RK Kanodia & Ashish Murolia Page 173

= 180c - (90c + 180 - 45c) =- 45c 1 + G (s) H (s) = 0


7.23 Option (B) is correct. 1+ 2 s+8 =0
For under-damped second order response s + as - 4
or s 2 + as - 4 + s + 8 = 0
T (s) = kwn2 where x < 1
s2 + 2xwn s + wn2 or s2 + (a + 1) s + 4 = 0
Thus (A) or (B) may be correct This will be stable if (a + 1) > 0 " a > - 1. Thus system is stable
For option (A) wn = 1.12 and 2xwn = 2.59 " x = 1.12 for all positive value of a.
For option (B) wn = 1.91 and 2xwn = 1.51 " x = 0.69 7.31 Option (C) is correct.
7.24 Option (B) is correct. The characteristic equation is
The plot has one encirclement of origin in clockwise direction. Thus 1 + G (s) = 0
G (s) has a zero is in RHP. or s5 + 2s 4 + 3s3 + 6s2 + 5s + 3 = 0
7.25 Option (C) is correct. Substituting s = z1 we have
The Nyzuist plot intersect the real axis ate - 0.5. Thus 3z5 + 5z 4 + 6z3 + 3z2 + 2z + 1 = 0
G. M. =- 20 log x =- 20 log 0.5 = 6.020 dB The routh table is shown below. As there are tow sign change in
And its phase margin is 90c. first column, there are two RHS poles.
7.26 Option (C) is correct.
Transfer function for the given pole zero plot is:
(s + Z1)( s + Z2) SPECIAL EDITION ( STUDY MATERIAL FORM )
(s + P1)( s + P2) At market Book is available in 3 volume i.e. in 3 book binding
From the plot Re (P1 and P2 )>(Z1 and Z2 ) form. But at NODIA Online Store book is available in 10 book
So, these are two lead compensator. binding form. Each unit of Book is in separate binding.
Hence both high pass filters and the system is high pass filter. Available Only at NODIA Online Store
7.27 Option (C) is correct. Click to Buy
Percent overshoot depends only on damping ratio, x .
2
www.nodia.co.in
Mp = e- xp 1 - x
If Mp is same then x is also same and we get
x = cos q z5 3 6 2

Thus q = constant z4 5 3 1
The option (C) only have same angle. z3 21
5
7
5

7.28 Option (D) is correct. z2 4


3 3
P = 2 25 2xwn = 0, x = 0 " Undamped Graph 3 z1 - 74
s + 25
2
z0 1
Q= 2 6 2xwn = 20, x > 1 " Overdamped Graph 4
s + 20s + 62 7.32 Option (C) is correct.
6 2 For underdamped second order system the transfer function is
R= 2xwn = 12, x = 1 " Critically Graph 1
s + 12s + 62
2
Kwn2
T (s) =
72 s2 + 2xwn s + wn2
S= 2xwn = 7, x < 1 " underdamped Graph 2
s + 7s + 72
2 It peaks at resonant frequency. Therefore
7.29 Option (C) is correct. Resonant frequency wr = wn 1 - 2x2
We labeled the given SFG as below : and peak at this frequency
mr = 5
2x 1 - x2

We have wr = 5 2 , and mr = 10 . Only options (A) satisfy these


3
values.
wn = 10, x = 1
2
From this SFG we have
xo1 =- gx1 + bx3 + m1 where wr = 10 1 - 2` 1 j = 5 2
4
xo2 = gx1 + ax3 and mr = 1 5 1 = 10 Hence satisfied
xo3 =- bx1 - ax3 + u2 22 1- 4 3
R V R VR V R V Option (B) is correct.
Sx1 W S- g 0 b WSx1 W S0 1 W u1
7.33

Thus Sx2 W = S g 0 a WSx2 W+ S0 0 We o The given circuit is a inverting amplifier and transfer function is
SSx WW SS- b 0 - a WWSSx WW SS1 0 WW u2
3 3 Vo = - Z = - Z (sC1 R1 + 1)
7.30
T X T
Option (C) is correct.
XT X T X Vi R 1
sC R + 1
R1
1 1

The characteristic equation of closed lop transfer function is


GATE Electronics and Communication Topicwise Solved Paper by RK Kanodia & Ashish Murolia Page 174

(sC2 R2 + 1)
For Q , Z =
sC2 7.38 Option (D) is correct.
Vo (sC2 R2 + 1) (sC1 R1 + 1) For ufb system the characteristics equation is
=- # PID Controller
Vi sC2 R1 1 + G (s) = 0
For R, Z = R2 K
(sC2 R2 + 1) or 1+ 2
=0
s (s + 7s + 12)
Vo R2 (sC1 R1 + 1) or s (s2 + 7s + 12) + K = 0
=- #
Vi (sC2 R2 + 1) R1
Point s =- 1 + j lie on root locus if it satisfy above equation i.e
Since R2 C2 > R1 C1, it is lag compensator.
(- 1 + j)[( - 1 + j) 2 + 7 (- 1 + j) + 12) + K] = 0
7.34 Option (D) is correct.
or K =+ 10
In a minimum phase system, all the poles as well as zeros are on the
left half of the s -plane. In given system as there is right half zero 7.39 Option (D) is correct.
(s = 5), the system is a non-minimum phase system. At every corner frequency there is change of -20 db/decade in slope
which indicate pole at every corner frequency. Thus
7.35 Option (B) is correct.
G (s) = K
We have Kv = lim sG (s) H (s) s (1 + s)`1 + s j
s"0 20
(Kp + KD s) 100 Bode plot is in (1 + sT) form
or 1000 = lim s = Kp
s (s + 100)
20 log K
s"0
= 60 dB = 1000
Now characteristics equations is w w = 0. 1
Thus K =5
GATE Electronics & Communication 100
Hence G (s) =
by RK Kanodia s (s + 1)( 1 + .05s)
Now in 3 Volume 7.40 Option (A) is correct.
dw
> H = =- 1 - 10G=in G + =10Gu
Purchase Online at maximum discount from online store dt -1 1 w 0
We have dia
and get POSTAL and Online Test Series Free dt
dw =- w + i
visit www.nodia.co.in or
dt n ...(1)

1 + G (s) H (s) = 0 and dia =- w - 10i + 10u ...(2)


a
(K + KD s) 100 dt
1000 = lims " 0 s p = Kp
s (s + 100) Taking Laplace transform (i) we get
Now characteristics equation is sw (s) =- w (s) = Ia (s)
1 + G (s) H (s) = 0 or (s + 1) w (s) = Ia (s) ...(3)
(100 + KD s) 100 Taking Laplace transform (ii) we get
or 1+ =0 Kp = 100
s (s + 10) sIa (s) =- w (s) - 10Ia (s) + 10U (s)
or s2 + (10 + 100KD) s + 10 4 = 0 or w (s) = (- 10 - s) Ia (s) + 10U (s)
Comparing with s2 + 2xwn + wn2 = 0 we get = (- 10 - s)( s + 1) w (s) + 10U (s) From (3)
2xwn = 10 + 100KD or w (s) =- [s2 + 11s + 10] w (s) + 10U (s)
or KD = 0.9 or (s2 + 11s + 11) w (s) = 10U (s)
Option (D) is correct. w (s)
= 2 10
7.36
or
We have T (s) = 5 U (s) (s + 11s + 11)
(s + 5)( s2 + s + 1) 7.41 Option (A) is correct.
= 5 = 2 1
5`1 + s j (s2 + s + 1) s +s+1 For more GATE Resources, Mock Test and
5
In given transfer function denominator is (s + 5)[( s + 0.5) + 2 3
4] Study material join the community
. We can see easily that pole at s =- 0.5 ! j 23 is dominant then http://www.facebook.com/gateec2014
pole at s =- 5 . Thus we have approximated it.
We have xo (t) = Ax (t)
Option (A) is correct. p q
A ==
r sG
7.37

1 = 1 Let
G (s) =
2
s -1 (s + 1)( s - 1)
1
The lead compensator C (s) should first stabilize the plant i.e. For initial state vector x (0) = = G the system response is
-2
1 e-2t
remove
(s - 1)
term. From only options (A), C (s) can remove this x (t) = > H
- 2e-2t
term
d
e-2t p q 1
==
> d (- 2e-2t)H r s G=- 2G
dt
1 10 (s - 1) Thus
Thus G (s) C (s) = # dt
(s + 1)( s - 1) (s + 2) t=0

10 - 2e-2 (0) p q 1
> 4e-2 (0) H = =r s G=- 2G
= Only option (A)
(s + 1)( s + 2) or
satisfies.
GATE Electronics and Communication Topicwise Solved Paper by RK Kanodia & Ashish Murolia Page 175

-2 p - 2q
= 4 G = = r - 2s G
Given system is 2nd order and for 2nd order system G.M. is infinite.
7.44 Option (D) is correct.
We get p - 2q =- 2 and r - 2s = 4 ...(i) 7.45 Option (D) is correct.
1
For initial state vector x (0) = = G the system response is If the Nyquist polt of G (jw) H (jw) for a closed loop system pass
-1
e-t through (- 1, j0) point, the gain margin is 1 and in dB
x (t) = > -tH GM =- 20 log 1
-e
d
e-t p q 1 = 0 dB
==
> d (- e-t)H r s G=- 1G
dt
Thus
dt
7.46 Option (B) is correct.
t=0
The characteristics equation is
- e- (0) p q 1
> e- (0) H = =r s G=- 1G 1 + G (s) H (s) = 0

-1 p-q K (s + 1)
= 1G = = r - s G
1+ =0
s + as2 + 2s + 1
3

We get p - q =- 1 and r - s = 1 ...(2) s3 + as2 + (2 + K) s + K + 1 = 0


Solving (1) and (2) set of equations we get The Routh Table is shown below. For system to be oscillatory
p q 0 1
=r s G = =- 2 - 3G
stable

The characteristic equation SPECIAL EDITION ( STUDY MATERIAL FORM )


lI - A = 0 At market Book is available in 3 volume i.e. in 3 book binding
l -1
form. But at NODIA Online Store book is available in 10 book
=0 binding form. Each unit of Book is in separate binding.
2 l+3
Available Only at NODIA Online Store
or l (l + 3) + 2 = 0
Click to Buy
or l =- 1, - 2
Thus Eigen values are - 1 and - 2
www.nodia.co.in
Eigen vectors for l1 =- 1
(l1 I - A) X1 = 0 a (2 + K) - (K + 1)
=0
a
l1 - 1 x11
or = 2 l + 3G=x G = 0 or a = K+1 ...(1)
1 21 K+2
- 1 - 1 x11
= 2 2 G=x G = 0 Then we have
21
as2 + K + 1 = 0
or - x11 - x21 = 0 At 2 rad/sec we have
or x11 + x21 = 0 s = jw " s2 =- w2 =- 4 ,
We have only one independent equation x11 =- x21 . Thus - 4a + K + 1 = 0 ...(2)
Let x11 = K , then x21 =- K , the Eigen vector will be Solving (i) and (ii) we get K = 2 and a = 0.75 .
x11 K 1
=x G = =- K G = K =- 1G
21 s3 1 2+K
Now Eigen vector for l2 =- 2 s2 a 1+K
(l2 I - A) X2 = 0 s1 (1 + K) a - (1 + K)
a
l2 - 1 x12
or = 2 l + 3G=x G = 0 s0 1+K
2 22
- 2 - 1 x11
or = 2 1 G=x G = 0 7.47 Option (D) is correct.
21 The transfer function of given compensator is
or - x11 - x21 = 0
Gc (s) = 1 + 3Ts T>0
or x11 + x21 = 0 1 + Ts
Comparing with
We have only one independent equation x11 =- x21 .
Gc (s) = 1 + aTs we get a = 3
Let x11 = K, then x21 =- K , the Eigen vector will be 1 + Ts
x12 K 1
=x G = =- 2K G = K =- 2G
The maximum phase sift is
fmax = tan-1 a - 1
22

7.42 Option (D) is correct. 2 a


As shown in previous solution the system matrix is = tan-1 3 - 1 = tan-1 1
0 1 2 3 3
A ==
- 2 - 3G or fmax = p
6
7.43 Option (D) is correct. 7.48 Option (A) is correct.
GATE Electronics and Communication Topicwise Solved Paper by RK Kanodia & Ashish Murolia Page 176

s 0 0 1 s -1 T (s) = 1 + sT
(sI - A) = = G -= G ==
1 sG
b > 1; T > 0
0 s -1 0 1 + sbT
s -1 s 1 1 + w2 T2
= G = > -1
T (jw) =
1 H
2
-1 s +1 s2 + 1
(sI - A) = 2 s 1 + w2 b2 T2
s +1 1 s s +12
s2 + 1

cos t sin t and +T (jw) = tan-1 (wT) - tan-1 (wbT)


==
- sin t cos t G
f (t) = eAt = L-1 [(sI - A)] -1 At w = 0 , T (jw) = 1
7.49 Option (C) is correct. At w = 0 , +T (jw) =- tan-1 0 = 0
We have G (s) = as + 1 At w = 3 , T (jw) = 1
s2 b
+G (jw) = tan-1 (wa) - p At w = 3 , +T (jw) = 0
Since PM is p i.e. 45c, thus 7.53 Option (A) is correct.
4 Despite the presence of negative feedback, control systems still have
p = p + +G (jw ) w " Gain cross over Frequen-
g g
4 problems of instability because components used have nonlinearity.
cy There are always some variation as compared to ideal characteristics.

or p = p + tan-1 (w a) - p 7.54 Option (B) is correct.


g
4
7.55 Option (C) is correct.
or p = tan-1 (w a)
4 g The peak percent overshoot is determined for LTI second order
closed loop system with zero initial condition. It’s transfer function
GATE Electronics & Communication is
wn2
by RK Kanodia T (s) =
s2 + 2xwn s + wn2
Now in 3 Volume Transfer function has a pair of complex conjugate poles and zeroes.
Purchase Online at maximum discount from online store 7.56 Option (A) is correct.
and get POSTAL and Online Test Series Free For ramp input we have R (s) = 12
visit www.nodia.co.in s
Now ess = lim sE (s)
s"0
or awg = 1 R (s) 1
= lim s = lim
At gain crossover frequency G (jwg) = 1 s"0 1 + G (s) s " 0 s + sG (s)
1 + a2 wg2 or ess = lim 1 = 5% = 1 Finite
Thus =1 s " 0 sG (s) 20
wg2
But kv = 1 = lim sG (s) = 20
or 1 + 1 = wg2 (as awg = 1) ess s"0

or wg = (2)
1
4 kv is finite for type 1 system having ramp input.

7.50 Option (C) is correct.


7.57 Option (A) is correct.
For a = 0.84 we have 7.58 Option (C) is correct.
Any point on real axis of s - is part of root locus if number of OL
G (s) = 0.84s2 + 1
s poles and zeros to right of that point is even. Thus (B) and (C) are
Due to ufb system H (s) = 1 and due to unit impulse response possible option.
R (s) = 1, thus The characteristics equation is
C (s) = G (s) R (s) = G (s) 1 + G (s) H (s) = 0

= 0.84s2 + 1 = 12 + 0.84 For more GATE Resources, Mock Test and


s s s
Taking inverse Laplace transform Study material join the community
c (t) = (t + 0.84) u (t) http://www.facebook.com/gateec2014
At t = 1, c (1 sec) = 1 + 0.84 = 1.84 K (1 - s)
or 1+ =0
7.51 Option (C) is correct. s (s + 3)
2
We have Xo = AX + BU where l is set of Eigen values or K = s + 3s
1-s
and Wo = CW + DU where m is set of Eigen values
For break away & break in point
If a liner system is equivalently represented by two sets of state
dK = (1 - s)( 2s + 3) + s2 + 3s = 0
equations, then for both sets, states will be same but their sets of ds
Eigne values will not be same i.e. or - s2 + 2s + 3 = 0
X = W but l ! m which gives s = 3 , - 1
7.52 Option (D) is correct. Here - 1 must be the break away point and 3 must be the break in
The transfer function of a lag network is point.
GATE Electronics and Communication Topicwise Solved Paper by RK Kanodia & Ashish Murolia Page 177

Option (D) is correct. w2 = 3


G (jw) H (jw) = 2 1 +
7.59

-2s
w2
G (s) = 3e
s (s + 2) Thus gain margin is = 1 = 0 and in dB this is - 3 .
-2jw 3
or G (jw) = 3e 7.62 Option (C) is correct.
jw (jw + 2)
3 Centroid is the point where all asymptotes intersects.
G (jw) = SReal of Open Loop Pole - SReal Part of Open Loop Pole
w w2 + 4 s =
SNo.of Open Loop Pole - SNo.of Open Loop zero
Let at frequency wg the gain is 1. Thus
3 = - 1 - 3 =- 1.33
=1 3
wg (wg2 + 4)
7.63 Option (C) is correct.
or wg4 + 4wg2
-9 = 0 The given bode plot is shown below
or wg2 = 1.606
or wg = 1.26 rad/sec
Now +G (jw) =- 2w - p - tan-1 w
2 2
Let at frequency wf we have +GH =- 180c
w
- p =- 2wf - p - tan-1 f
2 2
w SPECIAL EDITION ( STUDY MATERIAL FORM )
or 2wf + tan-1 f = p
2 2 At market Book is available in 3 volume i.e. in 3 book binding
w w 3 form. But at NODIA Online Store book is available in 10 book
or 2wf + c f - 1 ` f j m = p
2 3 2 2 binding form. Each unit of Book is in separate binding.
5wf wf3 Available Only at NODIA Online Store
or - =p
2 24 2 Click to Buy
5wf
2
.p
2
www.nodia.co.in
or wf = 0.63 rad At w = 1 change in slope is +20 dB " 1 zero at w = 1
At w = 10 change in slope is - 20 dB " 1 poles at w = 10
7.60 Option (D) is correct.
At w = 100 change in slope is - 20 dB " 1 poles at w = 100
The gain at phase crossover frequency wf is
K (s + 1)
G (jwg) = 3 = 3 Thus T (s) = s
( 10 + 1)( 100
s
+ 1)
wf (wf2 + 4)
1

0.63 (0.632 + 4) 2

or G (jwg) = 2.27 Now 20 log10 K =- 20 " K = 0.1


0.1 (s + 1) 100 (s + 1)
G.M. =- 20 log G (jwg) Thus T (s) = s =
( 10 + 1)( 100 + 1) (s + 10)( s + 100)
s
- 20 log 2.26 =- 7.08 dB
7.64 Option (C) is correct.
Since G.M. is negative system is unstable.
We have r (t) = 10u (t)
The phase at gain cross over frequency is
w or R (s) = 10
+G (jwg) =- 2wg - p - tan-1 g s
2 2
Now H (s) = 1
=- 2 # 1.26 - p - tan-1 1.26 s+2
2 2
C (s) = H (s) $ R (s) = 1 $ 10 10
or =- 4.65 rad or - 266.5c s + 2 s s (s + 2)
PM = 180c + +G (jwg) = 180c - 266.5c =- 86.5c or C (s) = 5 - 5
s s+2
7.61 Option (D) is correct.
c (t) = 5 [1 - e-2t]
The open loop transfer function is
2 (1 + s) The steady state value of c (t) is 5. It will reach 99% of steady
G (s) H (s) = state value reaches at t , where
s2
Substituting s = jw we have 5 [1 - e-2t] = 0.99 # 5
2 (1 + jw) or 1 - e-2t = 0.99
G (jw) H (jw) = ...(1)
- w2 e-2t = 0.1
+G (jw) H (jw) =- 180c + tan-1 w
or - 2t = ln 0.1
The frequency at which phase becomes - 180c, is called phase
or t = 2.3 sec
crossover frequency.
7.65 Option (A) is correct.
Thus - 180 =- 180c + tan-1 wf Approximate (comparable to 90c) phase shift are
or tan-1 wf = 0 Due to pole at 0.01 Hz " - 90c
or wf = 0 Due to pole at 80 Hz " - 90c
The gain at wf = 0 is Due to pole at 80 Hz " 0
Due to zero at 5 Hz " 90c
GATE Electronics and Communication Topicwise Solved Paper by RK Kanodia & Ashish Murolia Page 178

Due to zero at 100 Hz " 0


1 - 2 x12
Due to zero at 200 Hz " 0 or =- 1 2 G=x G = 0
Thus approximate total - 90c phase shift is provided. 22

We have only one independent equation x12 = 2x22


7.66 Option (C) is correct.
Mason Gain Formula Let x22 = K , then x12 = 2K . Thus Eigen vector will be
x12 2K 2
T (s) =
Spk 3 k =x G = = K G = K = 1 G
3 22

In given SFG there is only one forward path and 3 possible loop. Digonalizing matrix
x11 x12 -1 2
M ==
x21 x22 G = 1 1G
p1 = abcd =
31 = 1
1 -2
3= 1 - (sum of indivudual loops) - (Sum of two non touching Now M-1 = ` - 1 j= G
3 -1 -1
loops)
Now Diagonal matrix of sin At is D where
= 1 - (L1 + L2 + L3) + (L1 L3)
sin (l1 t) 0 sin (- 4t) 0
D == G ==
Non touching loop are L1 and L3 where 0 sin (l2 t) 0 sin (l2 t)G
L1 L2 = bedg
Now matrix B = sin At = MDM-1
C (s) p1 3 1 - 1 2 sin (- 4t) 0 1 -2
=
=-` 1 j= G= sin (- t) - 1 - 1G
G=
Thus
R (s) 1 - (be + cf + dg) + bedg
3 1 1 0

GATE Electronics & Communication


- sin (- 4t) - 2 sin (- t) 2 sin (- 4t) - 2 sin (- t)
=-` 1 j=
sin (- 4t) + 2 sin (t) - 2 sin (- 4t) - sin (- t)G
by RK Kanodia
3
Now in 3 Volume
Purchase Online at maximum discount from online store - sin (- 4t) - 2 sin (- t) 2 sin (- 4t) - 2 sin (- t)
=-` 1 j=
and get POSTAL and Online Test Series Free 3 sin (- 4t) - sin (- t) - 2 sin (- 4t) + 2 sin (- t)G
visit www.nodia.co.in
sin (- 4t) + 2 sin (- t) - 2 sin (- 4t) + 2 sin (- t)
= ` 1 j= Gs
abcd 3 - sin (- 4t + sin (- t) 2 sin (- 4t) + sin (- t)
=
1 - (be + cf + dg) + bedg 7.68 Option (A) is correct.
7.67 Option (A) is correct. For ufb system the characteristic equation is
-2 2
A ==
1 - 3G
1 + G (s) = 0
We have
1 + G (s)
1+ K =0
Characteristic equation is s (s2 + 2s + 2)( s + 3)
[lI - A] = 0 s 4 + 4s3 + 5s2 + 6s + K = 0
The routh table is shown below. For system to be stable,
l + 2 -2
or =0 (21 - 4K)
-1 l + 3 0 < K and 0 <
2/7
or (l + 2)( l + 3) - 2 = 0
This gives 0 < K < 21
or l2 + 5l + 4 = 0 4
Thus l1 =- 4 and l2 =- 1
s4 1 5 K
Eigen values are - 4 and - 1.
Eigen vectors for l1 =- 4
For more GATE Resources, Mock Test and
(l1 I - A) X1 = 0
l1 + 2 - 2 x11
Study material join the community
or = 1 l + 3G=x G = 0 http://www.facebook.com/gateec2014
1 21
- 2 - 2 x11
=- 1 - 1G=x G = 0 s3 4 6 0
21

or - 2x11 - 2x21 = 0 s2 7
2 K
21 - 4K
or x11 + x21 = 0 s1 7/2
0
We have only one independent equation x11 =- x21 . s0 K
Let x21 = K , then x11 =- K , the Eigen vector will be
x11 -K -1 7.69 Option (B) is correct.
=x G = = K G = K = 1 G We have P (s) = s5 + s 4 + 2s3 + 3s + 15
21

Now Eigen vector for l2 =- 1 The routh table is shown below.


If e " 0+ then 2e +e 12 is positive and -15e2-e +2412e - 144 is negative. Thus
2

(l2 I - A) X2 = 0
l2 + 2 - 2 x12 there are two sign change in first column. Hence system has 2 root
or = - 1 l + 3G=x G = 0
2 22
GATE Electronics and Communication Topicwise Solved Paper by RK Kanodia & Ashish Murolia Page 179

on RHS of plane. where L1 and L3 are non-touching


C (s)
s5 1 2 3 This
R (s)
p1 3 1
s4 1 2 15 =
1 - (loop gain) + pair of non - touching loops
s3 e - 12 0 ^ s +s27 h ^ s h
s + 27

= =
s2 2e + 12
e 15 0 1 - ^ -s3 - 24s - s2 h + -s2 . -s3 1 + 29s + s62
s (s + 27)
= 2
s1
2
-15e - 24e - 144
2e + 12 s + 29s + 6
s0 0 7.75 Option (D) is correct.
We have
7.70 Option (D) is correct.
1 + G (s) H (s) = 0
x1 - 3 - 1 x1 1
We have =x G = = 2 0 G=x G + = 0 Gu or 1+ K =0
2 2 s (s + 2)( s + 3)
x1 1
and Y = [1 0]= G + = G u or K =- s (s2 + 5s2 + 6s)
x2 2 dK =- (3s2 + 10s + 6) = 0
-3 -1 1
A == G , B = = G and C = [1 0]
ds
Here
2 0 0
The controllability matrix is which gives s = - 10 ! 100 - 72 =- 0.784, - 2.548
6
1 -3
QC = [B AB ] = =
0 2G SPECIAL EDITION ( STUDY MATERIAL FORM )
det QC ! 0 Thus controllable At market Book is available in 3 volume i.e. in 3 book binding
The observability matrix is form. But at NODIA Online Store book is available in 10 book
Q0 = [CT AT CT ]
binding form. Each unit of Book is in separate binding.
Available Only at NODIA Online Store
1 -3
==
0 - 1G Click to Buy
!0

det Q0 ! 0 Thus observable www.nodia.co.in


7.71 Option (B) is correct.
The location of poles on s - plane is
s 0 1 0 s-1 0
(sI - A) = = G -= G ==
0 s 0 1 0 s - 1G
(s - 1) 0 1
0
1 = 0 (s - 1)G > 0 H
s-1
(sI - A) -1 = = 1
(s - 1) 2 s-1

et 0
eAt = L-1 [(sI - A)] -1 = = G Since breakpoint must lie on root locus so s =- 0.748 is possible.
0 et
7.76 Option (A) is correct.
7.72 Option (A) is correct. The given bode plot is shown below
Z = P-N
N " Net encirclement of (- 1 + j0) by Nyquist plot,
P " Number of open loop poles in right hand side of s - plane
Z " Number of closed loop poles in right hand side of s - plane
Here N = 1 and P = 1
Thus Z =0
Hence there are no roots on RH of s -plane and system is always
stable. At w = 0.1 change in slope is + 60 dB " 3 zeroes at w = 0.1
7.73 Option (C) is correct. At w = 10 change in slope is - 40 dB " 2 poles at w = 10
PD Controller may accentuate noise at higher frequency. It does not At w = 100 change in slope is - 20 dB " 1 poles at w = 100
effect the type of system and it increases the damping. It also reduce K ( 0s.1 + 1) 3
Thus T (s) = s
the maximum overshoot. ( 10 + 1) 2 ( 100
s
+ 1)
7.74 Option (D) is correct. Now 20 log10 K = 20
Mason Gain Formula or K = 10
Spk 3 k 10 ( 0s.1 + 1) 3 108 (s + 0.1) 3
T (s) = Thus T (s) = s =
3 ( 10 + 1) 2 ( 100
s
+ 1) (s + 10) 2 (s + 100)
In given SFG there is only forward path and 3 possible loop. 7.77 Option (B) is correct.
p1 = 1 The characteristics equation is
31 = 1 + 3 + 24 = s + 27 s2 + 4s + 4 = 0
s s s
Comparing with
L1 = - 2 , L2 = - 24 and L3 = - 3 s2 + 2xwn + wn2 = 0
s s s
GATE Electronics and Communication Topicwise Solved Paper by RK Kanodia & Ashish Murolia Page 180

we get 2xwn = 4 and wn2 = 4 Y (s) = 0.5X (s)


Thus x =1 Critically damped 0.5 # 2U (s)
or Y (s) =
s+2
ts = 4 = 4 = 2
xwn 1#2 Y (s) 1
or =
U (s) (s + 2)
7.78 Option (B) is correct.
7.84 Option (D) is correct.
7.79 Option (C) is correct.
From Mason gain formula we can write transfer function as
We have
Y (s) K
K
xo1 1 0 x1 x1 (0) 1 = s
=
=xo G = =1 1G=x2 G
and =
x2 (0)G = 0 G
R (s) 1 - ( s + -sK ) s - 3 (3 - K)
3
=
2
For system to be stable (3 - K) < 0 i.e. K > 3
1 0
A ==
1 1G 7.85 Option (B) is correct.
s 0 1 0 s-1 0 The characteristics equation is
(sI - A) = = G -= G = =
0 s 1 1 - 1 s - 1G (s + 1)( s + 100) = 0
(s - 1) 0 1
0 s2 + 101s + 100 = 0
1 > H = > +1 H
-1 s-1
(sI - A) = Comparing with s2 + 2xwn + wn2 = 0 we get
(s - 1) 2 + 1 (s - 1)
1
(s - 1) 2 s-1
t
2xwn = 101 and wn2 = 100
L-1 [(sI - A) -1] = eAt = = t t G
e 0
te e Thus x = 101 Overdamped
et 0 1 et 20
x (t) = eAt # [x (t0)] = = t t G= G = = t G For overdamped system settling time can be determined by the
te e 0 te
GATE Electronics & Communication dominant pole of the closed loop system. In given system dominant
by RK Kanodia pole consideration is at s =- 1. Thus
Now in 3 Volume 1 =1 and Ts = 4 = 4 sec
Purchase Online at maximum discount from online store T T
and get POSTAL and Online Test Series Free 7.86 Option (B) is correct.
visit www.nodia.co.in Routh table is shown below. Here all element in 3rd row are zero, so
system is marginal stable.

7.80 Option (C) is correct. s5 2 4 2


The characteristics equation is s4 1 2 1
2
ks + s + 6 = 0
s3 0 0 0
or s2 + 1 s + 6 = 0
K K s2
Comparing with s2 + 2xwn s + wn2 = 0 we have s1
we get 2xwn = 1 and wn2 = 6 s0
K K
or 2 # 0.5 # 6 Kw = 1 Given x = 0.5 7.87 Option (B) is correct.
K
The open loop transfer function is
or 6 = 1 & K =1
K 6 G (s) H (s) = 1
K2 2
s (s + s + 1)
7.81 Option (B) is correct. Substituting s = jw we have
Any point on real axis lies on the root locus if total number of poles 1
and zeros to the right of that point is odd. Here s =- 1.5 does not G (jw) H (jw) =
jw (- w2 + jw + 1)
lie on real axis because there are total two poles and zeros (0 and
For more GATE Resources, Mock Test and
- 1) to the right of s =- 1.5 .
Study material join the community
Option (D) is correct.
http://www.facebook.com/gateec2014
7.82

From the expression of OLTF it may be easily see that the maximum
magnitude is 0.5 and does not become 1 at any frequency. Thus gain
+G (jw) H (jw) =- p - tan-1 w 2
cross over frequency does not exist. When gain cross over frequency 2 (1 - w )
does not exist, the phase margin is infinite. The frequency at which phase becomes - 180c, is called phase
7.83 Option (D) is correct. crossover frequency.
We have xo (t) =- 2x (t) + 2u (t) ...(i) wf
Thus - 180 =- 90 - tan-1
Taking Laplace transform we get 1 - wf2
wf
sX (s) =- 2X (s) + 2U (s) or - 90 =- tan-1
1 - wf2
or (s + 2) X (s) = 2U (s)
or 1 - w2f = 0
2U (s)
or X (s) = wf = 1 rad/sec
(s + 2)
The gain margin at this frequency wf = 1 is
Now y (t) = 0.5x (t)
GATE Electronics and Communication Topicwise Solved Paper by RK Kanodia & Ashish Murolia Page 181

GM =- 20 log10 G (jwf) H (jwf)


Comparing (1) and (6) we have
= 20 log10 (wf (1 - w2f) 2 + w2f
Z3 (s)
=- 20 log 1 = 0 H =
Z1 (s) + Z3 (s)
7.88 Option (A) is correct. 7.93 Option (B) is correct.
Z = P-N For unity negative feedback system the closed loop transfer function
N " Net encirclement of (- 1 + j0) by Nyquist plot, is
P " Number of open loop poles in right had side of s - plane G (s) s+4
CLTF = = , G (s) " OL Gain
Z " Number of closed loop poles in right hand side of s - plane 1 + G (s) s2 + 7s + 13
1 + G (s) 2
Here N = 0 (1 encirclement in CW direction and other in CCW) or = s + 7s + 13
G (s) s+4
and P = 0
or 1 = s2 + 7s + 13 - 1 = s2 + 6s + 9
Thus Z = 0 G (s) s+4 s+4
Hence there are no roots on RH of s - plane.
or G (s) = 2 s + 4
s + 6s + 9
7.89 Option (D) is correct. For DC gain s = 0 , thus
Take off point is moved after G2 as shown below
Thus G (0) = 4
9
SPECIAL EDITION ( STUDY MATERIAL FORM )
At market Book is available in 3 volume i.e. in 3 book binding
form. But at NODIA Online Store book is available in 10 book
7.90 Option (C) is correct.
binding form. Each unit of Book is in separate binding.
Available Only at NODIA Online Store
The characteristics equation is
Click to Buy
s2 + 2s + 2 = 0
Comparing with s2 + 2xwn + wn2 = 0 we get
www.nodia.co.in
2xwn = 2 and wn2 = 2
7.94 Option (C) is correct.
From the Block diagram transfer function is
wn =
2
G (s)
and x = 1 T (s) =
1 + G (s) H (s)
2
K (s - 2)
Since x < 1 thus system is under damped Where G (s) =
(s + 2)
7.91 Option (D) is correct. and H (s) = (s - 2)
If roots of characteristics equation lie on negative axis at different
The Characteristic equation is
positions (i.e. unequal), then system response is over damped.
1 + G (s) H (s) = 0
From the root locus diagram we see that for 0 < K < 1, the roots
are on imaginary axis and for 1 < K < 5 roots are on complex plain. K (s - 2)
1+ (s - 2) = 0
For K > 5 roots are again on imaginary axis. (s + 2) 2
Thus system is over damped for 0 # K < 1 and K > 5 . or (s + 2) 2 + K (s - 2) 2 = 0
or (1 + K) s2 + 4 (1 - K) s + 4K + 4 = 0
7.92 Option (C) is correct.
From SFG we have Routh Table is shown below. For System to be stable 1 + k > 0 ,
I1 (s) = G1 Vi (s) + HI2 (s) ...(1) and 4 + 4k > 0 and 4 - 4k > 0 . This gives - 1 < K < 1
I2 (s) = G2 I1 (s) ...(2) As per question for 0 # K < 1
V0 (s) = G3 I2 (s) ...(3) 1+k 4 + 4k
s2
Now applying KVL in given block diagram we have
Vi (s) = I1 (s) Z1 (s) + [I1 (s) - I2 (s)] Z3 (s) ...(4) s1 4 - 4k 0
0 = [I2 (s) - I1 (s)] Z3 (s) + I2 (s) Z2 (s) + I2 (s) Z4 (s) ...(5) s0 4 + 4k
From (4) we have
or Vi (s) = I1 (s)[ Z1 (s) + Z3 (S)] - I2 (s) Z3 (S) 7.95 Option (B) is correct.
1 Z3 (s) It is stable at all frequencies because for resistive network feedback
or I1 (s) = Vi + I2 ...(6)
Z1 (s) + Z3 (s) Z1 (s) + Z3 (s) factor is always less than unity. Hence overall gain decreases.
From (5) we have 7.96 Option (B) is correct.
I1 (s) Z3 (S) = I2 (s)[ Z2 (s) + Z3 (s) + Z4 (s)] ...(7) The characteristics equation is s2 + as2 + ks + 3 = 0
I1 (s) Z3 (s) The Routh Table is shown below
or Is (s) =
Z3 (s) + Z2 (s) + Z4 (s)
For system to be stable a > 0 and aK - 3 > 0
Comparing (2) and (7) we have a
Z3 (s) Thus a > 0 and aK > 3
G2 =
Z3 (s) + Z2 (s) + Z4 (s)
GATE Electronics and Communication Topicwise Solved Paper by RK Kanodia & Ashish Murolia Page 182

s3 1 K The observability matrix is


1 2
Q0 = [CT AT CT ] = =
1 - 2G
s2 a 3 !0
aK - 3
s1 a 0 det Q0 ! 0 Thus observable
s0 3 7.100 Option (D) is correct.
we have G (s) H (s) = 2 3
s (s + 1)
7.97 Option (B) is correct. 2 3
or G (jw) H (jw) =
Closed loop transfer function is given as jw (jw + 1)
Gain cross over frequency
T (s) = 2 9
s + 4s + 9 G (jw) H (jw) at w = w = 1
by comparing with standard form we get natural freq. g

wA2 = 9 or 2 3 =1
w w2 + 1
wn = 3
12 = w2 (w2 + 1)
2xwn = 4
w4 + w2 - 12 = 0
damping factor 4 = 2/3
x =
2#3 (w2 + 4) (w2 - 3) = 0
for second order system the setting time for 2-percent band is given w2 = 3 and w2 =- 4
by which gives w1, w2 = ! 3
GATE Electronics & Communication wg = 3
by RK Kanodia f (w) at w = w =- 90 - tan-1 (wg)
Now in 3 Volume
g

=- 90 - tan-1 3 =- 90 - 60 =- 150
Purchase Online at maximum discount from online store Phase margin = 180 + f (w) at w = w
g

and get POSTAL and Online Test Series Free = 180 - 150 = 30c
visit www.nodia.co.in 7.101 Option (B) is correct.
7.102 Option (C) is correct.
ts = 4 = 4 =4 =2 Closed-loop transfer function is given by
xwn 3 # 2/3 2
T (s) = n an - 1 s + an
7.98 Option (D) is correct. s + a1 sn - 1 + ... + an - 1 s + an
Given loop transfer function is an - 1 s + an
n n-1 2

G (s) H (s) = 2 = s + a1 s + ...an - 2 s


s (s + 1) 1+ n an - 1 s + an
s + a1 sn - 1 + ...an - 2 s2
G (jw) H (jw) = 2
jw (jw + 1) Thus G (s) H (s) = an - 1 s + an
Phase cross over frequency can be calculated as sn + a1 sn - 1 + ....an - 2 s2
For unity feed back H (s) = 1
f (w) at w = w =- 180c
p
Thus G (s) = an - 1 s + an
So here f (w) =- 90c - tan-1 (w) sn + a1 sn - 1 + ....an - 2 s2
- 90c - tan-1 (wp) =- 180c Steady state error is given by
tan-1 (wp) = 90c E (s) = lim R (s) 1
s"0 1 + G (s) H (s)
wp = 3
for unity feed back H (s) = 1
Gain margin
1 For more GATE Resources, Mock Test and
20 log 10 =
G (jw) H (jw) G
at w = wp
Study material join the community
G.M. = 20 log 10 e 1 http://www.facebook.com/gateec2014
G (jw) H (jwp) o

G (jwp) H (jwp) = 2 =0 Here input R (s) = 12 (unit Ramp)


wp w2p + 1 s
so G.M. = 20 log 10 b 1 l = 3 so E (s) = lim 12 1
0 s " 0 s 1 + G (s)
n
= lim 12 s + a1 sn - 1 + .... + an - 2 s2
7.99 Option (A) is correct. s"0 s sn + a1 sn - 1 + .... + an
0 1 0
Here A == G , B = = G and C = [1 1] = an - 2
an
2 -3 1
The controllability matrix is 7.103 Option (B) is correct.
0 1
QC = [B AB ] = =
1 - 3G
7.104

7.105 Option (A) is correct.


det QC ! 0 Thus controllable
7.106 Option (A) is correct.
GATE Electronics and Communication Topicwise Solved Paper by RK Kanodia & Ashish Murolia Page 183

By applying Routh’s criteria Phase is


3 2
s + 5s + 7s + 3 = 0 f (w) = tan-1 (3Tw) - tan-1 (Tw)
f (w) = tan-1 ; 3Tw - T w
s3 1 7 1 + 3T 2 w2 E
s2 5 3 f (w) = tan-1 ; 2Tw2 2 E
7#5-3
1 + 3T w
s1 = 32
0
5 5 For maximum value of phase
s0 3 df (w)
=0
dw
There is no sign change in the first column. Thus there is no root or 1 = 3T 2 w2
lying in the left-half plane.
Tw = 1
7.107 Option (A) is correct. 3
Techometer acts like a differentiator so its transfer function is of the So maximum phase is
form ks . fmax = tan-1 ; 2Tw2 2 E at Tw = 1
1 + 3T w 3
7.108 Option (A) is correct. R V
1
Open loop transfer function is S 2 W
3 W = tan-1 1 = 30c
; 3E
-1
K = tan S
G (s) = SS1 + 3 # 1 WW
s (s + 1) 3
T X
Steady state error
SPECIAL EDITION ( STUDY MATERIAL FORM )
sR (s)
E (s) = lim At market Book is available in 3 volume i.e. in 3 book binding
s " 0 1 + G (s) H (s)

Where R (s) = input H (s) = 1 (unity feedback)


form. But at NODIA Online Store book is available in 10 book
binding form. Each unit of Book is in separate binding.
R (s) = 1 Available Only at NODIA Online Store
s
s1
Click to Buy
s (s + 1)
so E (s) = lim
s"0
s
K
= lim 2
s"0 s + s + K
=0 www.nodia.co.in
1+
s (s + 1)
7.109 Option (B) is correct. 7.113 Option (A) is correct.
Fig given below shows a unit impulse input given to a zero-order hold G (jw) H (jw) enclose the (- 1, 0) point so here G (jwp) H (jwp) > 1
circuit which holds the input signal for a duration T & therefore, wp = Phase cross over frequency
the output is a unit step function till duration T . Gain Margin = 20 log 10 1
G (jwp) H (jwp)
so gain margin will be less than zero.
7.114 Option (B) is correct.
The denominator of Transfer function is called the characteristic
equation of the system. so here characteristic equation is
(s + 1) 2 (s + 2) = 0

h (t) = u (t) - u (t - T)
7.115 Option (C) is correct.
In synchro error detector, output voltage is proportional to [w (t)],
Taking Laplace transform we have
where w (t) is the rotor velocity so here n = 1
H (s) = 1 - 1 e-sT = 1 61 - e-sT @ Option (C) is correct.
s s s 7.116

By masson’s gain formulae


7.110 Option (C) is correct.
y
=
/ Dk Pk
Phase margin = 180c + qg where qg = value of phase at gain x D
crossover frequency.
Forward path gain P1 = 5 # 2 # 1 = 10
Here qg =- 125c
D = 1 - (2 # - 2) = 1 + 4 = 5
so P.M = 180c - 125c = 55c
D1 = 1
7.111 Option (B) is correct. y
so gain = 10 # 1 = 2
Open loop transfer function is given by x 5
K (1 + 0.5s) Option (C) is correct.
G (s) H (s) = 7.117
s (1 + s) (1 + 2s)
By given matrix equations we can have
Close looped system is of type 1.
It must be noted that type of the system is defined as no. of poles Xo1 = dx1 = x1 - x2 + 0
dt
lies at origin in OLTF.
lying Xo2 = dx2 = 0 + x2 + m
dt
7.112 Option (D) is correct.
x1
Transfer function of the phase lead controller is y = [1 1] > H = x1 + x2
1 + (3Tw) j x2
T.F = 1 + 3Ts = dy
1+s 1 + (Tw) j = dx1 + dx2
dt dt dt
GATE Electronics and Communication Topicwise Solved Paper by RK Kanodia & Ashish Murolia Page 184

dy
= x1 + m
dt
dy
= x1 (0) + m (0)
dt t=0

= 1+0 = 0

GATE Electronics & Communication


by RK Kanodia
Now in 3 Volume
Purchase Online at maximum discount from online store
and get POSTAL and Online Test Series Free
visit www.nodia.co.in

For more GATE Resources, Mock Test and


Study material join the community
http://www.facebook.com/gateec2014
GATE Electronics and Communication Topicwise Solved Paper by RK Kanodia & Ashish Murolia Page 184

UNIT 8 The optimum threshold to achieve minimum bit error rate (BER) is
8.6

(A) 1 (B) 4
2 5

COMMUNICATION SYSTEMS (C) 1 (D) 3


2

2012 ONE MARK

8.7 The power spectral density of a real process X (t) for positive
2013 ONE MARK
frequencies is shown below. The values of E [X 2 (t)] and E [X (t)] ,
8.1 The bit rate of a digital communication system is R kbits/s . The respectively, are
modulation used is 32-QAM. The minimum bandwidth required for
ISI free transmission is
(A) R/10 Hz (B) R/10 kHz
(C) R/5 Hz (D) R/5 kHz

2013 TWO MARKS

8.2 Let U and V be two independent zero mean Gaussain random


variables of variances 1 and 1 respectively. The probability
4 9 (A) 6000/p, 0 (B) 6400/p, 0
GATE Electronics & Communication (C) 6400/p, 20/ (p 2 ) (D) 6000/p, 20/ (p 2 )
by RK Kanodia
Now in 3 Volume 8.8 In a baseband communications link, frequencies upto 3500 Hz are
used for signaling. Using a raised cosine pulse with 75% excess
Purchase Online at maximum discount from online store bandwidth and for no inter-symbol interference, the maxi mum
and get POSTAL and Online Test Series Free possible signaling rate in symbols per second is
visit www.nodia.co.in (A) 1750 (B) 2625
P ^3V F 2U h is (C) 4000 (D) 5250

(A) 4/9 (B) 1/2 8.9 A source alphabet consists of N symbols with the probability of the
(C) 2/3 (D) 5/9 first two symbols being the same. A source encoder increases the
probability of the first symbol by a small amount e and decreases
8.3 Consider two identically distributed zero-mean random variables U that of the second by e. After encoding, the entropy of the source
and V . Let the cumulative distribution functions of U and 2V be (A) increases (B) remains the same
F ^x h and G ^x h respectively. Then, for all values of x (C) increases only if N = 2 (D) decreases
(A) F ^x h - G ^x h # 0 (B) F ^x h - G ^x h $ 0
(C) ^F (x) - G (x)h .x # 0 (D) ^F (x) - G (x)h .x $ 0 Two independent random variables X and Y are uniformly
8.10

distributed in the interval 6- 1, 1@. The probability that max 6X, Y @


8.4 Let U and V be two independent and identically distributed random is less than 1/2 is
variables such that P ^U =+ 1h = P ^U =- 1h = 1 . The entropy (A) 3/4 (B) 9/16
2
H ^U + V h in bits is (C) 1/4 (D) 2/3
(A) 3/4 (B) 1
(C) 3/2 (D) log 2 3 2012 TWO MARKS

For more GATE Resources, Mock Test and


Common Data for Questions 5 and 6:
Study material join the community
Bits 1 and 0 are transmitted with equal probability. At the re-
ceiver, the pdf of the respective received signals for both bits are http://www.facebook.com/gateec2014
as shown below.
8.11 A BPSK scheme operating over an AWGN channel with noise
power spectral density of N 0 /2, uses equiprobable signals
s1 (t) = 2E sin (wc t) and s2 (t) =- 2E sin (wc t) over the symbol
T T
interval (0, T). If the local oscillator in a coherent receiver is ahead
in phase by 45c with respect to the received signal, the probability
of error in the resulting system is
(A) Q c 2E m (B) Q c E m
N0 N0
If the detection threshold is 1, the BER will be (C) Q c E E
2N 0 m
(D) Q c
4N 0 m
8.5

(A) 1 (B) 1
2 4 8.12 A binary symmetric channel (BSC) has a transition probability of
(C) 1 (D) 1 1/8. If the binary symbol X is such that P (X = 0) = 9/10, then the
8 16
GATE Electronics and Communication Topicwise Solved Paper by RK Kanodia & Ashish Murolia Page 185

probability of error for an optimum receiver will be (D) P-2, Q-4, R-3, S-1
(A) 7/80 (B) 63/80
(C) 9/10 (D) 1/10
2011 TWO MARKS
8.13 The signal m (t) as shown is applied to both a phase modulator 8.18 X (t) is a stationary random process with auto-correlation function
(with k p as the phase constant) and a frequency modulator (with k f RX (t) = exp (- pt 2). This process is passed through the system
as the frequency constant) having the same carrier frequency. shown below. The power spectral density of the output process Y (t)
is

(A) (4p 2 f 2 + 1) exp (- pf 2) (B) (4p 2 f 2 - 1) exp (- pf 2)


The ratio k p /k f (in rad/Hz) for the same maximum phase devia- (C) (4p 2 f 2 + 1) exp (- pf ) (D) (4p 2 f 2 - 1) exp (- pf )
tion is
(A) 8p (B) 4p 8.19 A message signal m (t) = cos 2000pt + 4 cos 4000pt modulates the
(C) 2p (D) p SPECIAL EDITION ( STUDY MATERIAL FORM )
At market Book is available in 3 volume i.e. in 3 book binding
Statement for Linked Answer Question 14 and 15 : form. But at NODIA Online Store book is available in 10 book
The transfer function of a compensator is given as binding form. Each unit of Book is in separate binding.
Available Only at NODIA Online Store
Gc (s) = s + a
s+b Click to Buy
8.14 Gc (s) is a lead compensator if www.nodia.co.in
(A) a = 1, b = 2 (B) a = 3, b = 2
carrier c (t) = cos 2pfc t where fc = 1 MHz to produce an AM signal.
(C) a =- 3, b =- 1 (D) a = 3, b = 1
For demodulating the generated AM signal using an envelope
8.15 The phase of the above lead compensator is maximum at detector, the time constant RC of the detector circuit should satisfy
(A) 2 rad/s (B) 3 rad/s (A) 0.5 ms < RC < 1 ms (B) 1 μs << RC < 0.5 ms
(C) 6 rad/s (D) 1/ 3 rad/s (C) RC << 1 μs (D) RC >> 0.5 ms

Common Data For Q. 8.5 & 8.6


2011 ONE MARK A four-phase and an eight-phase signal constellation are shown in
8.16 An analog signal is band-limited to 4 kHz, sampled at the Nyquist the figure below.
rate and the samples are quantized into 4 levels. The quantized
levels are assumed to be independent and equally probable. If we
transmit two quantized samples per second, the information rate is
(A) 1 bit/sec (B) 2 bits/sec
(C) 3 bits/sec (D) 4 bits/sec

8.17 The Column -1 lists the attributes and the Column -2 lists the
modulation systems. Match the attribute to the modulation system
that best meets it. 8.20 For the constraint that the minimum distance between pairs of
Column -1 Column -2 signal points be d for both constellations, the radii r 1 , and r 2 of the
P. Power efficient transmission of 1. Conventional circles are
signals AM (A) r 1 = 0.707d, r2 = 2.782d (B) r 1 = 0.707d, r 2 = 1.932d
Q. Most bandwidth efficient 2. FM (C) r 1 = 0.707d, r 2 = 1.545d (D) r 1 = 0.707d, r 2 = 1.307d
transmission of voice signals 8.21 Assuming high SNR and that all signals are equally probable, the
R. Simplest receiver structure 3. VSB additional average transmitted signal energy required by the 8-PSK
S. Bandwidth efficient transmission 4. SSB-SC signal to achieve the same error probability as the 4-PSK signal is
of signals with significant dc (A) 11.90 dB (B) 8.73 dB
component (C) 6.79 dB (D) 5.33 dB
(A) P-4, Q-2, R-1, S-3
(B) P-2, Q-4, R-1, S-3 2010 ONE MARK
(C) P-3, Q-2, R-1, S-4 8.22 Suppose that the modulating signal is m (t) = 2 cos (2pfm t) and the
GATE Electronics and Communication Topicwise Solved Paper by RK Kanodia & Ashish Murolia Page 186

carrier signal is xC (t) = AC cos (2pfC t), which one of the following is gain and cut-off frequency 1 MHz. Let Yk represent the random
a conventional AM signal without over-modulation variable y (tk ).
(A) x (t) = AC m (t) cos (2pfC t) Yk = Nk , if transmitted bit bk = 0
(B) x (t) = AC [1 + m (t)] cos (2pfC t) Yk = a + Nk if transmitted bit bk = 1
Where Nk represents the noise sample value. The noise sample has
(C) x (t) = AC cos (2pfC t) + AC m (t) cos (2pfC t)
4 a probability density function, PNk (n) = 0.5ae- a n (This has mean
(D) x (t) = AC cos (2pfm t) cos (2pfC t) + AC sin (2pfm t) sin (2pfC t) zero and variance 2/a 2 ). Assume transmitted bits to be equiprob-
able and threshold z is set to a/2 = 10-6 V .
8.23 Consider an angle modulated signal
x (t) = 6 cos [2p # 106 t + 2 sin (800pt)] + 4 cos (800pt)
The average power of x (t) is
(A) 10 W (B) 18 W
(C) 20 W (D) 28 W

8.24 Consider the pulse shape s (t) as shown below. The impulse response
h (t) of the filter matched to this pulse is

8.25 The value of the parameter a (in V - 1 ) is


(A) 1010 (B) 107
(C) 1.414 # 10-10 (D) 2 # 10-20
GATE Electronics & Communication
8.26 The probability of bit error is
by RK Kanodia (A) 0.5 # e-3.5 (B) 0.5 # e-5
Now in 3 Volume (C) 0.5 # e-7 (D) 0.5 # e-10
Purchase Online at maximum discount from online store
8.27 The Nyquist sampling rate for the signal
and get POSTAL and Online Test Series Free sin (500pt) sin (700) pt
visit www.nodia.co.in s (t) = # is given by
pt pt
(A) 400 Hz (B) 600 Hz
(C) 1200 Hz (D) 1400 Hz

8.28 X (t) is a stationary process with the power spectral density Sx (f ) > 0
, for all f . The process is passed through a system shown below

Let Sy (f ) be the power spectral density of Y (t). Which one of the


following statements is correct
(A) Sy (f ) > 0 for all f
(B) Sy (f ) = 0 for f > 1 kHz

For more GATE Resources, Mock Test and


Study material join the community
http://www.facebook.com/gateec2014
(C) Sy (f ) = 0 for f = nf0, f0 = 2 kHz kHz, n any integer
(D) Sy (f ) = 0 for f = (2n + 1) f0 = 1 kHz , n any integer

2009 ONE MARK

2010 TWO MARKS 8.29 For a message siganl m (t) = cos (2pfm t) and carrier of frequency fc ,
which of the following represents a single side-band (SSB) signal ?
(A) cos (2pfm t) cos (2pfc t) (B) cos (2pfc t)
Statement for linked Answer Question : 8.10 & 8.11 : (C) cos [2p (fc + fm) t] (D) [1 + cos (2pfm t) cos (2pfc t)
Consider a baseband binary PAM receiver shown below. The
additive channel noise n (t) is with power spectral density
Sn (f ) = N 0 /2 = 10-20 W/Hz . The low-pass filter is ideal with unity
GATE Electronics and Communication Topicwise Solved Paper by RK Kanodia & Ashish Murolia Page 187

2009 TWO MARKS 2008 TWO MARKS

8.30 Consider two independent random variables X and Y with identical 8.37 The probability density function (pdf) of random variable is as
distributions. The variables X and Y take values 0, 1 and 2 with shown below
probabilities 12 , 14 and 1 respectively. What is the conditional
4
probability P (X + Y = 2 X - Y = 0) ?
(A) 0 (B) 1/16
(C) 1/6 (D) 1
The corresponding commutative distribution function CDF has the
8.31 A discrete random variable X takes values from 1 to 5 with form
probabilities as shown in the table. A student calculates the mean
X as 3.5 and her teacher calculates the variance of X as 1.5. Which
of the following statements is true ?

k 1 2 3 4 5
P (X = k) 0.1 0.2 0.3 0.4 0.5
(A) Both the student and the teacher are right
(B) Both the student and the teacher are wrong
(C) The student is wrong but the teacher is right
(D) The student is right but the teacher is wrong SPECIAL EDITION ( STUDY MATERIAL FORM )
8.32 A message signal given by m (t) = ( 12 ) cos w1 t - ( 12 ) sin w2 t At market Book is available in 3 volume i.e. in 3 book binding
amplitude - modulated with a carrier of frequency wC to generator form. But at NODIA Online Store book is available in 10 book
s (t)[ 1 + m (t)] cos wc t . What is the power efficiency achieved by this binding form. Each unit of Book is in separate binding.
modulation scheme ? Available Only at NODIA Online Store
(A) 8.33% (B) 11.11% Click to Buy
(C) 20% (D) 25% www.nodia.co.in
8.33 A communication channel with AWGN operating at a signal to noise
ration SNR >> 1 and bandwidth B has capacity C1. If the SNR is
doubled keeping constant, the resulting capacity C2 is given by
(A) C2 . 2C1 (B) C2 . C1 + B
(C) C2 . C1 + 2B (D) C2 . C1 + 0.3B

Common Data For Q. 8.19 & 8.20 :


The amplitude of a random signal is uniformly distributed between
-5 V and 5 V.
8.34 If the signal to quantization noise ratio required in uniformly
quantizing the signal is 43.5 dB, the step of the quantization is
approximately
(A) 0.033 V (B) 0.05 V
(C) 0.0667 V (D) 0.10 V
8.38 A memory less source emits n symbols each with a probability p.
8.35 If the positive values of the signal are uniformly quantized with a The entropy of the source as a function of n
step size of 0.05 V, and the negative values are uniformly quantized (A) increases as log n (B) decreases as log ( n1 )
with a step size of 0.1 V, the resulting signal to quantization noise (C) increases as n (D) increases as n log n
ration is approximately
(A) 46 dB (B) 43.8 dB 8.39 Noise with double-sided power spectral density on K over all
(C) 42 dB (D) 40 dB frequencies is passed through a RC low pass filter with 3 dB cut-off
frequency of fc . The noise power at the filter output is
(A) K (B) Kfc
2008 ONE MARK (C) kpfc (D) 3
8.36 Consider the amplitude modulated (AM) signal
8.40 Consider a Binary Symmetric Channel (BSC) with probability of
Ac cos wc t + 2 cos wm t cos wc t . For demodulating the signal using
error being p. To transmit a bit, say 1, we transmit a sequence of
envelope detector, the minimum value of Ac should be
three 1s. The receiver will interpret the received sequence to represent
(A) 2 (B) 1
1 if at least two bits are 1. The probability that the transmitted bit
(C) 0.5 (D) 0 will be received in error is
(A) p3 + 3p2 (1 - p) (B) p3
(C) (1 - p3) (D) p3 + p2 (1 - p)
GATE Electronics and Communication Topicwise Solved Paper by RK Kanodia & Ashish Murolia Page 188

8.41 Four messages band limited to W, W, 2W and 3W respectively are (A) E [X2] - E2 [X] (B) E [X2] + E2 [X]
to be multiplexed using Time Division Multiplexing (TDM). The (C) E [X2] (D) E2 [X]
minimum bandwidth required for transmission of this TDM signal is
(A) W (B) 3W
2007 TWO MARKS
(C) 6W (D) 7W
8.50 A Hilbert transformer is a
8.42 Consider the frequency modulated signal
(A) non-linear system (B) non-causal system
10 cos [2p # 105 t + 5 sin (2p # 1500t) + 7.5 sin (2p # 1000t)]
with carrier frequency of 105 Hz. The modulation index is (C) time-varying system (D) low-pass system
(A) 12.5 (B) 10 8.51 In delta modulation, the slope overload distortion can be reduced by
(C) 7.5 (D) 5 (A) decreasing the step size (B) decreasing the granular
noise
8.43 The signal cos wc t + 0.5 cos wm t sin wc t is
(A) FM only (B) AM only (C) decreasing the sampling rate (D) increasing the step size
(C) both AM and FM (D) neither AM nor FM 8.52 The raised cosine pulse p (t) is used for zero ISI in digital
communications. The expression for p (t) with unity roll-off factor
is given by
Common Data For Q. 8.29, 8.30 and 8.31 : sin 4pWt
p (t) =
A speed signal, band limited to 4 kHz and peak voltage varying 4pWt (1 - 16W2 t2)
between +5 V and - 5 V, is sampled at the Nyquist rate. Each
The value of p (t) at t = 1 is
GATE Electronics & Communication 4W
(A) - 0.5 (B) 0
by RK Kanodia
(C) 0.5 (D) 3
Now in 3 Volume
Purchase Online at maximum discount from online store 8.53 In the following scheme, if the spectrum M (f) of m (t) is as shown,
then the spectrum Y (f) of y (t) will be
and get POSTAL and Online Test Series Free
visit www.nodia.co.in

sample is quantized and represented by 8 bits.


8.44 If the bits 0 and 1 are transmitted using bipolar pulses, the minimum
bandwidth required for distortion free transmission is
(A) 64 kHz (B) 32 kHz
(C) 8 kHz (D) 4 kHz

8.45 Assuming the signal to be uniformly distributed between its peak to


peak value, the signal to noise ratio at the quantizer output is
(A) 16 dB (B) 32 dB
(C) 48 dB (D) 4 kHz

8.46 Assuming the signal to be uniformly distributed between its peak to


peak value, the signal to noise ratio at the quantizer output is
(A) 1024 (B) 512
(C) 256 (D) 64 8.54 During transmission over a certain binary communication channel,
For more GATE Resources, Mock Test and
2007 ONE MARK
Study material join the community
8.47 If R (t) is the auto correlation function of a real, wide-sense stationary http://www.facebook.com/gateec2014
random process, then which of the following is NOT true
(A) R (t) = R (- t) bit errors occur independently with probability p. The probability
(B) R (t) # R (0) of AT MOST one bit in error in a block of n bits is given by
(C) R (t) =- R (- t) (A) pn (B) 1 - pn
(D) The mean square value of the process is R (0) (C) np (1 - p) n - 1 + (1 + p) n (D) 1 - (1 - p) n

8.48 If S (f) is the power spectral density of a real, wide-sense stationary 8.55 In a GSM system, 8 channels can co-exist in 200 kHz bandwidth
random process, then which of the following is ALWAYS true? using TDMA. A GSM based cellular operator is allocated 5 MHz
(A) S (0) # S (f) (B) S (f) $ 0 bandwidth. Assuming a frequency reuse factor of 1 , i.e. a five-cell
5
(C) S (- f) =- S (f) (D)
3
#- 3 S (f) df = 0 repeat pattern, the maximum number of simultaneous channels that
can exist in one cell is
8.49 If E denotes expectation, the variance of a random variable X is (A) 200 (B) 40
given by (C) 25 (D) 5
GATE Electronics and Communication Topicwise Solved Paper by RK Kanodia & Ashish Murolia Page 189

8.56 In a Direct Sequence CDMA system the chip rate is 1.2288 # 106 2006 ONE MARK
chips per second. If the processing gain is desired to be AT LEAST 8.61 A low-pass filter having a frequency response H (jw) = A (w) e jf (w)
100, the data rate does not produce any phase distortions if
(A) must be less than or equal to 12.288 # 103 bits per sec (A) A (w) = Cw3, f (w) = kw3 (B) A (w) = Cw2, f (w) = kw
(B) must be greater than 12.288 # 103 bits per sec (C) A (w) = Cw, f (w) = kw2 (D) A (w) = C, f (w) = kw- 1
(C) must be exactly equal to 12.288 # 103 bits per sec
(D) can take any value less than 122.88 # 103 bits per sec
2006 TWO MARKS

Common Data For Q. 8.41 & 8.42 : 8.62 A signal with bandwidth 500 Hz is first multiplied by a signal g (t)
where
Two 4-array signal constellations are shown. It is given that f1 3
and f2 constitute an orthonormal basis for the two constella- g (t) = /(- 1)k d (t - 0.5 # 10- 4 k)
tion. Assume that the four symbols in both the constellations are R =- 3
The resulting signal is then passed through an ideal lowpass filter
equiprobable. Let N0 denote the power spectral density of white
2 with bandwidth 1 kHz. The output of the lowpass filter would be
Gaussian noise.
(A) d (t) (B) m (t)
(C) 0 (D) m (t) d (t)

8.63 The minimum sampling frequency (in samples/sec) required to


SPECIAL EDITION ( STUDY MATERIAL FORM )
At market Book is available in 3 volume i.e. in 3 book binding
form. But at NODIA Online Store book is available in 10 book
binding form. Each unit of Book is in separate binding.
8.57 The if ratio or the average energy of Constellation 1 to the average Available Only at NODIA Online Store
energy of Constellation 2 is Click to Buy
(A) 4a2 (B) 4
(C) 2 (D) 8
www.nodia.co.in
reconstruct the following signal from its samples without distortion
8.58 If these constellations are used for digital communications over an
x (t) = 5` sin 2p100t j + 7` sin 2p100t j would be
3 2
AWGN channel, then which of the following statements is true ? pt pt
(A) Probability of symbol error for Constellation 1 is lower (A) 2 # 10 3
(B) 4 # 10 3

(B) Probability of symbol error for Constellation 1 is higher (C) 6 # 10 3


(D) 8 # 103
(C) Probability of symbol error is equal for both the constellations
8.64 The minimum step-size required for a Delta-Modulator operating
(D) The value of N0 will determine which of the constellations has
at 32k samples/sec to track the signal (here u (t) is the unit-step
a lower probability of symbol error
function)
x (t) = 125[ u (t) - u (t - 1) + (250t)[ u (t - 1) - u (t - 2)]
Statement for Linked Answer Question 8.44 & 8.45 : so that slope-overload is avoided, would be
An input to a 6-level quantizer has the probability density function (A) 2 - 10 (B) 2 - 8
f (x) as shown in the figure. Decision boundaries of the quantizer (C) 2 - 6 (D) 2 - 4
are chosen so as to maximize the entropy of the quantizer output.
It is given that 3 consecutive decision boundaries are’ - 1'.'0' and 8.65 A zero-mean white Gaussian noise is passes through an ideal lowpass
' 1' . filter of bandwidth 10 kHz. The output is then uniformly sampled
with sampling period ts = 0.03 msec. The samples so obtained would
be
(A) correlated (B) statistically independent
(C) uncorrelated (D) orthogonal

8.66 A source generates three symbols with probabilities 0.25, 0.25,


0.50 at a rate of 3000 symbols per second. Assuming independent
generation of symbols, the most efficient source encoder would have
8.59 The values of a and b are
average bit rate is
(A) a = 1 and b = 1 (B) a = 1 and b = 3 (A) 6000 bits/sec (B) 4500 bits/sec
6 12 5 40
(C) a = 1 and b = 1 (D) a = 1 and b = 1 (C) 3000 bits/sec (D) 1500 bits/sec
4 16 3 24
8.60 Assuming that the reconstruction levels of the quantizer are the 8.67 The diagonal clipping in Amplitude Demodulation (using envelop
mid-points of the decision boundaries, the ratio of signal power to detector) can be avoided it RC time-constant of the envelope detector
quantization noise power is satisfies the following condition, (here W is message bandwidth and
w is carrier frequency both in rad/sec)
(A) 152 (B) 64
9 3 (A) RC < 1 (B) RC > 1
W W
(C) 76 (D) 28
3
GATE Electronics and Communication Topicwise Solved Paper by RK Kanodia & Ashish Murolia Page 190

(C) RC < 1 (D) RC > 1 signal and modulation index would be


w w (A) t, 0.5 (B) t, 1.0
8.68 A uniformly distributed random variable X with probability density (C) t, 2.0 (D) t2, 0.5
function
fx (x) = 1 pu (x + 5) - u (x - 5)]
10 Common Data For Q. 8.58 & 8.59 :
where u (.) is the unit step function is passed through a transfor- The following two question refer to wide sense stationary stochas-
mation given in the figure below. The probability density function tic process
of the transformed random variable Y would be 8.73 It is desired to generate a stochastic process (as voltage process)
with power spectral density S (w) = 16/ (16 + w2) by driving a
Linear-Time-Invariant system by zero mean white noise (As voltage
process) with power spectral density being constant equal to 1. The
system which can perform the desired task could be
(A) first order lowpass R-L filter
(A) fy (y) = 1 [u (y + 2.5) - u (y - 2.25)]
5 (B) first order highpass R-C filter
(B) fy (y) = 0.5d (y) + 0.5d (y - 1) (C) tuned L-C filter
(C) fy (y) = 0.25d (y + 2.5) + 0.25d (y - 2.5) + 5d (y) (D) series R-L-C filter
(D) fy (y) = 0.25d (y + 2.5) + 0.25d (y - 2.5) + 1 [u (y + 2.5) - u (y - 2.5)] 8.74 The parameters of the system obtained in previous Q would be
10
(A) first order R-L lowpass filter would have R = 4W L = 1H
GATE Electronics & Communication
(B) first order R-C highpass filter would have R = 4W C = 0.25F
by RK Kanodia
(C) tuned L-C filter would have L = 4H C = 4F
Now in 3 Volume (D) series R-L-C lowpass filter would have R = 1W , L = 4H ,
Purchase Online at maximum discount from online store C = 4F
and get POSTAL and Online Test Series Free
visit www.nodia.co.in
Common Data For Q. 8.60 & 8.61 :
8.69 In the following figure the minimum value of the constant "C" , which Consider the following Amplitude Modulated (AM) signal, where
is to be added to y1 (t) such that y1 (t) and y2 (t) are different , is fm < B
XAM (t) = 10 (1 + 0.5 sin 2pfm t) cos 2pfc t
8.75 The average side-band power for the AM signal given above is
(A) 25 (B) 12.5
(C) 6.25 (D) 3.125

8.76 The AM signal gets added to a noise with Power Spectral Density
(A) 3 (B) 3 Sn (f) given in the figure below. The ratio of average sideband power
2 to mean noise power would be :
2
(C) 3 (D) 3
12 L
8.70 A message signal with bandwidth 10 kHz is Lower-Side Band SSB
modulated with carrier frequency fc1 = 106 Hz. The resulting signal
is then passed through a Narrow-Band Frequency Modulator with
carrier frequency fc2 = 109 Hz.
The bandwidth of the output would be For more GATE Resources, Mock Test and
(A) 4 # 10 4 Hz (B) 2 # 106 Hz Study material join the community
(C) 2 # 109 Hz (D) 2 # 1010 Hz http://www.facebook.com/gateec2014

Common Data For Q. 8.56 & 8.57 :


Let g (t) = p (t)*( pt), where * denotes convolution &
p (t) = u (t) - u (t - 1) lim with u (t) being the unit step function
z"3
8.71 The impulse response of filter matched to the signal
s (t) = g (t) - d (1 - 2)* g (t) is given as :
(A) s (1 - t) (B) - s (1 - t)
(C) - s (t) (D) s (t)

8.72 An Amplitude Modulated signal is given as


xAM (t) = 100 [p (t) + 0.5g (t)] cos wc t 25 25
(A) (B)
in the interval 0 # t # 1. One set of possible values of modulating 8N0 B 4N0 B
GATE Electronics and Communication Topicwise Solved Paper by RK Kanodia & Ashish Murolia Page 191

(C) 25 (D) 25 the probability density function as shown in the figure. The mean
2N0 B N0 B square value of v is

2005 ONE MARK

8.77 Find the correct match between group 1 and group 2.


Group 1 Group 2
P. {1 + km (t) A sin (wc t)} W. Phase modulation
Q. km (t) A sin (wc t) X. Frequency modulation
R. A sin {wc t + km (t)} Y. Amplitude modulation (A) 4 (B) 6
S. A sin ; wc t + k m (t) dt E
t
#- 3 Z. DSB-SC modulation (C) 8 (D) 9

(A) P - Z, Q - Y, R - X, S - W 8.83 A carrier is phase modulated (PM) with frequency deviation of 10


(B) P - W, Q - X, R - Y, S - Z kHz by a single tone frequency of 1 kHz. If the single tone frequency
(C) P - X, Q - W, R - Z, S - Y is increased to 2 kHz, assuming that phase deviation remains
unchanged, the bandwidth of the PM signal is
(D) P - Y, Q - Z, R - W, S - X
(A) 21 kHz (B) 22 kHz
8.78 Which of the following analog modulation scheme requires the (C) 42 kHz (D) 44 kHz
minimum transmitted power and minimum channel bandwidth ?
(A) VSB (B) DSB-SC
(C) SSB (D) AM SPECIAL EDITION ( STUDY MATERIAL FORM )
At market Book is available in 3 volume i.e. in 3 book binding
2005 TWO MARKS
form. But at NODIA Online Store book is available in 10 book
binding form. Each unit of Book is in separate binding.
8.79 A device with input X (t) and output y (t) is characterized by: Available Only at NODIA Online Store
Y (t) = x2 (t). An FM signal with frequency deviation of 90 kHz and
Click to Buy
modulating signal bandwidth of 5 kHz is applied to this device. The
bandwidth of the output signal is www.nodia.co.in
(A) 370 kHz (B) 190 kHz Common Data For Q. 8.69 and 8.70 :
(C) 380 kHz (D) 95 kHz Asymmetric three-level midtread quantizer is to be designed as-
suming equiprobable occurrence of all quantization levels.
8.80 A signal as shown in the figure is applied to a matched filter. Which
of the following does represent the output of this matched filter ?

8.84 If the probability density function is divide into three regions as


shown in the figure, the value of a in the figure is
(A) 1 (B) 2
3 3
(C) 1 (D) 1
2 4
8.85 The quantization noise power for the quantization region between
- a and + a in the figure is
(A) 4 (B) 1
81 9
(C) 5 (D) 2
81 81

2004 ONE MARK

8.86 In a PCM system, if the code word length is increased from 6 to 8


bits, the signal to quantization noise ratio improves by the factor
(A) 8 (B) 12
6
8.81 Noise with uniform power spectral density of N0 W/Hz is passed
though a filter H (w) = 2 exp (- jwtd ) followed by an ideal pass filter (C) 16 (D) 8
of bandwidth B Hz. The output noise power in Watts is 8.87 An AM signal is detected using an envelop detector. The carrier
(A) 2N0 B (B) 4N0 B frequency and modulating signal frequency are 1 MHz and 2 kHz
(C) 8N0 B (D) 16N0 B respectively. An appropriate value for the time constant of the
envelop detector is
8.82 An output of a communication channel is a random variable v with
(A) 500m sec (B) 20m sec
GATE Electronics and Communication Topicwise Solved Paper by RK Kanodia & Ashish Murolia Page 192

(C) 0.2m sec (D) 1m sec symbols are represented as shown in the figure.
The source output is transmitted using two modulation schemes,
8.88 An AM signal and a narrow-band FM signal with identical carriers, namely Binary PSK (BPSK) and Quadrature PSK (QPSK). Let
modulating signals and modulation indices of 0.1 are added together. B1 and B2 be the bandwidth requirements of the above rectangular
The resultant signal can be closely approximated by pulses is 10 kHz, B1 and B2 are
(A) broadband FM (B) SSB with carrier
(C) DSB-SC (D) SSB without carrier

8.89 In the output of a DM speech encoder, the consecutive pulses are of


opposite polarity during time interval t1 # t # t2 . This indicates that
during this interval
(A) the input to the modulator is essentially constant
(B) the modulator is going through slope overload (A) B1 = 20 kHz, B2 = 20 kHz (B) B1 = 10 kHz, B2 = 20 kHz
(C) the accumulator is in saturation (C) B1 = 20 khz, B2 = 10 kHz (D) B1 = 10 kHz, B2 = 10 kHz
(D) the speech signal is being sampled at the Nyquist rate 8.94 A 100 MHz carrier of 1 V amplitude and a 1 MHz modulating signal
of 1 V amplitude are fed to a balanced modulator. The ourput of
8.90 The distribution function Fx (x) of a random variable x is shown in
the modulator is passed through an ideal high-pass filter with cut-
the figure. The probability that X = 1 is
off frequency of 100 MHz. The output of the filter is added with 100
MHz signal of 1 V amplitude and 90c phase shift as shown in the
figure. The envelope of the resultant signal is
GATE Electronics & Communication
by RK Kanodia
Now in 3 Volume
Purchase Online at maximum discount from online store
and get POSTAL and Online Test Series Free (A) constant (B) 1 + sin (2p # 106 t)

visit www.nodia.co.in (C) 5 - sin (2p - 106 t) (D) 5 + cos (2p # 106 t)
4 4
8.95 Two sinusoidal signals of same amplitude and frequencies 10 kHz
and 10.1 kHz are added together. The combined signal is given to an
ideal frequency detector. The output of the detector is
(A) 0.1 kHz sinusoid (B) 20.1 kHz sinusoid
(C) a linear function of time (D) a constant
(A) zero (B) 0.25
(C) 0.55 (D) 0.30 8.96 Consider a binary digital communication system with equally likely
0’s and 1’s. When binary 0 is transmitted the detector input can
lie between the levels - 0.25 V and + 0.25 V with equl probability :
2004 TWO MARKS when binary 1 is transmitted, the voltage at the detector can have
any value between 0 and 1 V with equal probability. If the detector
8.91 A 1 mW video signal having a bandwidth of 100 MHz is transmitted
has a threshold of 0.2 V (i.e., if the received signal is greater than
to a receiver through cable that has 40 dB loss. If the effective one-
0.2 V, the bit is taken as 1), the average bit error probability is
side noise spectral density at the receiver is 10 - 20 Watt/Hz, then the
(A) 0.15 (B) 0.2
signal-to-noise ratio at the receiver is
(A) 50 dB (B) 30 dB (C) 0.05 (D) 0.5
(C) 40 dB (D) 60 dB
For more GATE Resources, Mock Test and
Consider the signal x (t) shown in Fig. Let h (t) denote the impulse
Study material join the community
8.92

response of the filter matched to x (t), with h (t) being non-zero only
in the interval 0 to 4 sec. The slope of h (t) in the interval 3 < t < 4 http://www.facebook.com/gateec2014
sec is
8.97 A random variable X with uniform density in the interval 0 to 1 is
quantized as follows :
If 0 # X # 0.3 , xq = 0
If 0.3 < X # 1, xq = 0.7
where xq is the quantized value of X.
The root-mean square value of the quantization noise is
(A) 0.573 (B) 0.198
(C) 2.205 (D) 0.266
(A) 1 sec - 1 (B) - 1 sec - 1
2 8.98 Choose the current one from among the alternative A, B, C, D after
(C) - 1 sec - 1 (D) 1 sec - 1 matching an item from Group 1 with the most appropriate item in
2 Group 2.
8.93 A source produces binary data at the rate of 10 kbps. The binary Group 1 Group 2
GATE Electronics and Communication Topicwise Solved Paper by RK Kanodia & Ashish Murolia Page 193

1. FM P. Slope overload Common Data For Q. 8.90 & 8.91 :


2. DM Q. m-law X (t) is a random process with a constant mean value of 2 and the
3. PSK R. Envelope detector auto correlation function Rxx (t) = 4 (e - 0.2 t + 1).
4. PCM S. Hilbert transform
T. Hilbert transform 8.105 Let X be the Gaussian random variable obtained by sampling the
U. Matched filter process at t = ti and let
(A) 1 - T, 2 - P, 3 - U, 4 - S (B) 1 - S, 2 - U, 3 - P, 4 - T - 1 e dy
3 x2

Q (a) =
a
# 2p
2

(C) 1 - S, 2 - P, 3 - U, 4 - Q (D) 1 - U, 2 - R, 3 - S, 4 - Q
The probability that 6x # 1@ is
8.99 Three analog signals, having bandwidths 1200 Hz, 600 Hz and 600 (A) 1 - Q (0.5) (B) Q (0.5)
Hz, are sampled at their respective Nyquist rates, encoded with (C) Q c 1 (D) 1 - Q c 1 m
2 2 m 2 2
12 bit words, and time division multiplexed. The bit rate for the
multiplexed. The bit rate for the multiplexed signal is 8.106 Let Y and Z be the random variable obtained by sampling X (t) at
(A) 115.2 kbps (B) 28.8 kbps t = 2 and t = 4 respectively. Let W = Y - Z . The variance of W is
(C) 57.6 kbps (D) 38.4 kbps (A) 13.36 (B) 9.36
(C) 2.64 (D) 8.00
8.100 Consider a system shown in the figure. Let X (f) and Y (f) and denote
the Fourier transforms of x (t) and y (t) respectively. The ideal HPF 8.107 A sinusoidal signal with peak-to-peak amplitude of 1.536 V is
has the cutoff frequency 10 kHz. quantized into 128 levels using a mid-rise uniform quantizer. The

SPECIAL EDITION ( STUDY MATERIAL FORM )


At market Book is available in 3 volume i.e. in 3 book binding
form. But at NODIA Online Store book is available in 10 book
binding form. Each unit of Book is in separate binding.
Available Only at NODIA Online Store

Click to Buy
www.nodia.co.in
quantization-noise power is
(A) 0.768 V (B) 48 # 10 - 6 V2
The positive frequencies where Y (f) has spectral peaks are (B) 12 # 10 - 6 V2 (D) 3.072 V
(A) 1 kHz and 24 kHz (B) 2 kHz and 244 kHz 8.108 Let x (t) = 2 cos (800p) + cos (1400pt). x (t) is sampled with the
(C) 1 kHz and 14 kHz (D) 2 kHz and 14 kHz rectangular pulse train shown in the figure. The only spectral
components (in kHz) present in the sampled signal in the frequency
range 2.5 kHz to 3.5 kHz are
2003 ONE MARK

8.101 The input to a coherent detector is DSB-SC signal plus noise. The
noise at the detector output is
(A) the in-phase component (B) the quadrature - component
(C) zero (D) the envelope

8.102 The noise at the input to an ideal frequency detector is white. The (A) 2.7, 3.4 (B) 3.3, 3.6
detector is operating above threshold. The power spectral density of (C) 2.6, 2.7, 3.3, 3.4, 3.6 (D) 2.7, 3.3
the noise at the output is
(A) raised - cosine (B) flat 8.109 A DSB-SC signal is to be generated with a carrier frequency fc = 1
MHz using a non-linear device with the input-output characteristic
(C) parabolic (D) Gaussian
V0 = a0 vi + a1 vi3 where a0 and a1 are constants. The output of the
8.103 At a given probability of error, binary coherent FSK is inferior to non-linear device can be filtered by an appropriate band-pass filter.
binary coherent PSK by. Let Vi = Aci cos (2pfi ct) + m (t) is the message signal. Then the value
(A) 6 dB (B) 3 dB of fci (in MHz) is
(A) 1.0 (B) 0.333
(C) 2 dB (D) 0 dB
(B) 0.5 (D) 3.0

2003 TWO MARKS

8.104 Let X and Y be two statistically independent random variables


Common Data For Q. 8.95 & 8.96 :
uniformly distributed in the ranges (- 1, 1) and (- 2, 1) respectively. Let m (t) = cos [(4p # 103) t] be the message signal &
Let Z = X + Y . Then the probability that (z #- 1) is c (t) = 5 cos [(2p # 106 t)] be the carrier.
(A) zero (B) 1
6 8.110 c (t) and m (t) are used to generate an AM signal. The modulation
(C) 1 (D) 1 index of the generated AM signal is 0.5. Then the quantity
3 12 Total sideband power
is
Carrier power
GATE Electronics and Communication Topicwise Solved Paper by RK Kanodia & Ashish Murolia Page 194

(A) 1 (B) 1 (A) r = 0.5, S is required (B) r = 1.0, S is required


2 4
(C) r = 0.5, S is not required (D) r = 1.0, S is not required
(C) 1 (D) 1
3 8 8.117 A signal is sampled at 8 kHz and is quantized using 8 - bit uniform
quantizer. Assuming SNRq for a sinusoidal signal, the correct
8.111 c (t) and m (t) are used to generated an FM signal. If the peak
statement for PCM signal with a bit rate of R is
frequency deviation of the generated FM signal is three times the
(A) R = 32 kbps, SNRq = 25.8 dB
transmission bandwidth of the AM signal, then the coefficient of the
term cos [2p (1008 # 103 t)] in the FM signal (in terms of the Bessel (B) R = 64 kbps, SNRq = 49.8 dB
coefficients) is (C) R = 64 kbps, SNRq = 55.8 dB
(A) 5J4 (3) (B) 5 J8 (3) (D) R = 32 kbps, SNRq = 49.8 dB
2
(C) 5 J8 (4) (D) 5J4 (6)
2 2002 ONE MARK
8.112 Choose the correct one from among the alternative A, B, C, D after 8.118 A 2 MHz sinusoidal carrier amplitude modulated by symmetrical
matching an item in Group 1 with most appropriate item in Group
square wave of period 100 m sec . Which of the following frequencies
2.
will NOT be present in the modulated signal ?
Group 1 Group 2
(A) 990 kHz (B) 1010 kHz
P. Ring modulator 1. Clock recovery
Q. VCO 2. Demodulation of FM (C) 1020 kHz (D) 1030 kHz
R. Foster-Seely discriminator 3. Frequency conversion +3
8.119 Consider a sample signal y (t) = 5 # 10 - 6 # (t) / d (t - nTs)
GATE Electronics & Communication n =- 3
where x (t) = 10 cos (8p # 103) t and Ts = 100m sec.
by RK Kanodia When y (t) is passed through an ideal lowpass filter with a cutoff
Now in 3 Volume frequency of 5 KHz, the output of the filter is
Purchase Online at maximum discount from online store (A) 5 # 10 - 6 cos (8p # 103) t (b) 5 # 10 - 5 cos (8p # 103) t
and get POSTAL and Online Test Series Free (C) 5 # 10 - 1 cos (8p # 103) t (D) 10 cos (8p # 103) t
visit www.nodia.co.in 8.120 For a bit-rate of 8 Kbps, the best possible values of the transmitted
frequencies in a coherent binary FSK system are
S. Mixer 4. Summing the two inputs (A) 16 kHz and 20 kHz (C) 20 kHz and 32 kHz
5. Generation of FM
(C) 20 kHz and 40 kHz (D) 32 kHz and 40 kHz
6. Generation of DSB-Sc
(A) P - 1; Q - 3; R - 2; S - 4 (B) P - 6; Q = 5; R - 2; S - 3 8.121 The line-of-sight communication requires the transmit and receive
(C) P - 6; Q - 1; R - 3; S - 2 (D) P - 5; Q - 6; R - 1; S - 3 antennas to face each other. If the transmit antenna is vertically
polarized, for best reception the receiver antenna should be
8.113 A superheterodyne receiver is to operate in the frequency range 550 (A) horizontally polarized
kHz - 1650 kHz, with the intermediate frequency of 450 kHz. Let
(B) vertically polarized
R = Cmax /Cmin denote the required capacitance ratio of the local
oscillator and I denote the image frequency (in kHz) of the incoming (C) at 45c with respect to horizontal polarization
signal. If the receiver is tuned to 700 kHz, then (D) at 45c with respect to vertical polarization
(A) R = 4.41, I = 1600 (B) R = 2.10, I - 1150
(C) R = 3.0, I = 600 (D) R = 9.0, I = 1150 2002 TWO MARKS

8.114 If Eb , the energy per bit of a binary digital signal, is 10 - 5 watt- 8.122 An angle-modulated signal is given by
sec and the one-sided power spectral density of the white noise, s (t) = cos 2p (2 # 106 t + 30 sin 150t + 40 cos 150t).
N0 = 10 - 6 W/Hz, then the output SNR of the matched filter is
For more GATE Resources, Mock Test and
(A) 26 dB (B) 10 dB
(C) 20 dB (D) 13 dB
Study material join the community
http://www.facebook.com/gateec2014
8.115 The input to a linear delta modulator having a step-size 3= 0.628
is a sine wave with frequency fm and peak amplitude Em . If the The maximum frequency and phase deviations of s (t) are
sampling frequency fx = 40 kHz, the combination of the sine-wave (A) 10.5 kHz, 140p rad (B) 6 kHz, 80p rad
frequency and the peak amplitude, where slope overload will take
(C) 10.5 kHz, 100p rad (D) 7.5 kHz, 100p rad
place is
Em fm 8.123 In the figure m (t) = 2 sin 2pt , s (t) = cos 200pt and n (t) = sin 199pt
(A) 0.3 V 8 kHz t t
.
(B) 1.5 V 4 kHz
The output y (t) will be
(C) 1.5 V 2 kHz
(D) 3.0 V 1 kHz
8.116 If S represents the carrier synchronization at the receiver and r
represents the bandwidth efficiency, then the correct statement for
the coherent binary PSK is
GATE Electronics and Communication Topicwise Solved Paper by RK Kanodia & Ashish Murolia Page 195

(A) sin 2pt (B) sin 2pt + sin pt cos 3pt of K0 m (t) is greater than 1. Which of the following could be the
t t t detector output ?
(C) sin 2pt + sin 0.5pt cos 1.5pt (D) sin 2pt + sin pt cos 0.75pt (A) Ac m (t) (B) Ac2 [1 + Ka m (t)] 2
t t t t
3
(C) [Ac (1 + Ka m (t)] (D) Ac [1 + Ka m (t)] 2
8.124 A signal x (t) = 100 cos (24p # 10 ) t is ideally sampled with a sampling
period of 50m sec ana then passed through an ideal lowpass filter 8.133 The frequency range for satellite communication is
with cutoff frequency of 15 kHz. Which of the following frequencies (A) 1 KHz to 100 KHz (B) 100 KHz to 10 KHz
is/are present at the filter output ? (C) 10 MHz to 30 MHz (D) 1 GHz to 30 GHz
(A) 12 kHz only (B) 8 kHz only
(C) 12 kHz and 9 kHz (D) 12 kHz and 8 kHz
2000 TWO MARKS
8.125 If the variance ax2 of d (n) = x (n) - x (n - 1) is one-tenth the 8.134 In a digital communication system employing Frequency Shift
variance ax2 of stationary zero-mean discrete-time signal x (n), then
Keying (FSK), the 0 and 1 bit are represented by sine waves of 10
R (k)
the normalized autocorrelation function xx 2 at k = 1 is KHz and 25 KHz respectively. These waveforms will be orthogonal
(A) 0.95 (B) 0.90 ax for a bit interval of
(C) 0.10 (D) 0.05 (A) 45m sec (B) 200m sec
(C) 50m sec (D) 250m sec
2001 ONE MARK 8.135 A message m (t) bandlimited to the frequency fm has a power of Pm
8.126 A bandlimited signal is sampled at the Nyquist rate. The signal can SPECIAL EDITION ( STUDY MATERIAL FORM )
be recovered by passing the samples through At market Book is available in 3 volume i.e. in 3 book binding
(A) an RC filter form. But at NODIA Online Store book is available in 10 book
(B) an envelope detector binding form. Each unit of Book is in separate binding.
(C) a PLL Available Only at NODIA Online Store
(D) an ideal low-pass filter with the appropriate bandwidth Click to Buy
8.127 The PDF of a Gaussian random variable X is given by www.nodia.co.in
(x - 4) 2

px (x) = 1 e - 18 . The probability of the event {X = 4} is . The power of the output signal in the figure is
3 2p
(A) 1 (B) 1
2 3 2p
(C) 0 (D) 1
4

2001 TWO MARKS (A) Pm cos q (B) Pm


2 4
A video transmission system transmits 625 picture frames per second. 2 2
(C) Pm sin q (D) Pm cos q
8.128

Each frame consists of a 400 # 400 pixel grid with 64 intensity levels 4 4
per pixel. The data rate of the system is 8.136 The Hilbert transform of cos w1 t + sin w2 t is
(A) 16 Mbps (B) 100 Mbps (A) sin w1 t - cos w2 t (B) sin w1 t + cos w2 t
(C) 600 Mbps (D) 6.4 Gbps (C) cos w1 t - sin w2 t (D) sin w1 t + sin w2 t
8.129 The Nyquist sampling interval, for the signal sin c (700t) + sin c (500t) 8.137 In a FM system, a carrier of 100 MHz modulated by a sinusoidal
is signal of 5 KHz. The bandwidth by Carson’s approximation is 1
(A) 1 sec (B) p sec MHz. If y (t) = (modulated waveform) 3 , than by using Carson’s
350 350
approximation, the bandwidth of y (t) around 300 MHz and the and
(C) 1 sec (D) p sec the spacing of spectral components are, respectively.
700 175
(A) 3 MHz, 5 KHz (B) 1 MHz, 15 KHz
8.130 During transmission over a communication channel, bit errors occur
independently with probability p. If a block of n bits is transmitted, (C) 3 MHz, 15 KHz (D) 1 MHz, 5 KHz
the probability of at most one bit error is equal to
(A) 1 - (1 - p) n (B) p + (n - 1)( 1 - p) 1999 ONE MARK
n-1 n n-1
(C) np (1 - p) (D) (1 - p) + np (1 - p) 8.138 The input to a channel is a bandpass signal. It is obtained by linearly
modulating a sinusoidal carrier with a single-tone signal. The output
8.131 The PSD and the power of a signal g (t) are, respectively, Sg (w) and
of the channel due to this input is given by
Pg . The PSD and the power of the signal ag (t) are, respectively,
(A) a2 Sg (w) and a2 Pg (B) a2 Sg (w) and aPg y (t) = (1/100) cos (100t - 10-6) cos (106 t - 1.56)
The group delay (tg) and the phase delay (t p) in seconds, of the
(C) aSg (w) and a2 Pg (D) aSg (w) and aPs
channel are
(A) tg = 10-6, t p = 1.56 (B) tg = 1.56, t p = 10-6
2000 ONE MARK (C) tg = 108, t p = 1.56 # 10-6 (D) tg = 108, t p = 1.56
8.132 The amplitude modulated waveform s (t) = Ac [1 + Ka m (t)] cos wc t 8.139 A modulated signal is given by s (t) = m1 (t) cos (2pfc t) + m2 (t) sin (2pfc t)
is fed to an ideal envelope detector. The maximum magnitude
GATE Electronics and Communication Topicwise Solved Paper by RK Kanodia & Ashish Murolia Page 196

where the baseband signal m1 (t) and m2 (t) have bandwidths of 8.147 The image channel selectivity of superheterodyne receiver depends
10 kHz, and 15 kHz, respectively. The bandwidth of the modulated upon
(A) IF amplifiers only
signal, in kHz, is
(A) 10 (B) 15 (B) RF and IF amplifiers only
(C) 25 (D) 30 (C) Preselector, RF and IF amplifiers
(D) Preselector, and RF amplifiers only
8.140 A modulated signal is given by s (t) = e-at cos [(wc + Dw) t] u (t),
where a wc and Dw are positive constants, and wc >> Dw . The 8.148 In a PCM system with uniform quantisation, increasing the number
complex envelope of s (t) is given by of bits from 8 to 9 will reduce the quantisation noise power by a
(A) exp (- at) exp [j (wc + Dw) t] u (t) factor of
(B) exp (- at) exp (jDwt) u (t) (A) 9 (B) 8
(C) exp (jDwt) u (t) (C) 4 (D) 2
(D) exp [jwc + Dw) t] 8.149 Flat top sampling of low pass signals
(A) gives rise to aperture effect (B) implies oversampling
1999 TWO MARKS (C) leads to aliasing (D) introduces delay dis-
tortion
8.141 The Nyquist sampling frequency (in Hz) of a signal given by
6 # 10 4 sin c2 (400t) * 106 sin c3 (100t) is 8.150 A DSB-SC signal is generated using the carrier cos (we t + q) and
(A) 200 (B) 300 modulating signal x (t). The envelope of the DSB-SC signal is
(A) x (t) (B) x (t)
GATE Electronics & Communication (C) only positive portion of x (t) (D) x (t) cos q
by RK Kanodia
Now in 3 Volume 8.151 Quadrature multiplexing is
(A) the same as FDM
Purchase Online at maximum discount from online store
(B) the same as TDM
and get POSTAL and Online Test Series Free
(C) a combination of FDM and TDM
visit www.nodia.co.in
(D) quite different from FDM and TDM

(C) 500 (D) 1000 8.152 The Fourier transform of a voltage signal x (t) is X (f). The unit of
X (f) is
8.142 The peak-to-peak input to an 8-bit PCM coder is 2 volts. The signal (A) volt (B) volt-sec
power-to-quantization noise power ratio (in dB) for an input of
(C) volt/sec (D) volt 2
0.5 cos (wm t) is
(A) 47.8 (B) 49.8 8.153 Compression in PCM refers to relative compression of
(C) 95.6 (D) 99.6 (A) higher signal amplitudes (B) lower signal amplitudes
(C) lower signal frequencies (D) higher signal frequencies
8.143 The input to a matched filter is given by
"10
6 -4
0 sin (2p # 10 t) 0 < 1 < 10 sec 8.154 For a give data rate, the bandwidth B p of a BPSK signal and the
s (t) =
otherwise bandwidth B 0 of the OOK signal are related as
The peak amplitude of the filter output is (A) B p = B 0 (B) B p = B 0
(A) 10 volts (B) 5 volts 4 2
(C) 10 millivolts (D) 5 millivolts (C) B p = B 0 (D) B p = 2B 0

8.144 Four independent messages have bandwidths of 100 Hz, 8.155 The spectral density of a real valued random process has
200 Hz and 400 Hz , respectively. Each is sampled at the Nyquist
rate, and the samples are time division multiplexed (TDM) and
For more GATE Resources, Mock Test and
transmitted. The transmitted sample rate (in Hz) is Study material join the community
(A) 1600 (B) 800 http://www.facebook.com/gateec2014
(C) 400 (D) 200
(A) an even symmetry (B) an odd symmetry
(C) a conjugate symmetry (D) no symmetry
1998 ONE MARK
8.156 The probability density function of the envelope of narrow band
8.145 The amplitude spectrum of a Gaussian pulse is Gaussian noise is
(A) uniform (B) a sine function (A) Poisson (B) Gaussian
(C) Gaussian (D) an impulse function (C) Rayleigh (D) Rician
8.146 The ACF of a rectangular pulse of duration T is
(A) a rectangular pulse of duration T 1997 ONE MARK
(B) a rectangular pulse of duration 2T
8.157 The line code that has zero dc component for pulse transmission of
(C) a triangular pulse of duration T random binary data is
(D) a triangular pulse of duration 2T
GATE Electronics and Communication Topicwise Solved Paper by RK Kanodia & Ashish Murolia Page 197

(A) Non-return to zero (NRZ) from


(B) Return to zero (RZ) (A) IF stages only (B) RF stages only
(C) Alternate Mark Inversion (AM) (C) detector and RF stages only (D) detector RF and IF stages
(D) None of the above
2 1996 TWO MARKS
8.158 A probability density function is given by p (x) = Ke-x /2 - 3 < x < 3
. The value of K should be 8.164 The number of bits in a binary PCM system is increased from n to
(A) 1 (B) 2 n + 1. As a result, the signal to quantization noise ratio will improve
2p p by a factor
(C) 1 (D) 1 (A) n + 1 (B) 2(n + 1)/n
2 p p 2 n
(C) 22 (n + 1)/n (D) which is independent of n
8.159 A deterministic signal has the power spectrum given in the figure
is, The minimum sampling rate needed to completely represent this 8.165 The auto correlation function of an energy signal has
signal is (A) no symmetry (B) conjugate symmetry
(C) odd symmetry (D) even symmetry

8.166 An FM signal with a modulation index 9 is applied to a frequency


tripler. The modulation index in the output signal will be

SPECIAL EDITION ( STUDY MATERIAL FORM )


At market Book is available in 3 volume i.e. in 3 book binding
form. But at NODIA Online Store book is available in 10 book
binding form. Each unit of Book is in separate binding.
Available Only at NODIA Online Store
(A) 1 kHz (B) 2 kHz
Click to Buy
(C) 3 kHz (D) None of these
www.nodia.co.in
8.160 A communication channel has first order low pass transfer function. (A) 0 (B) 3
The channel is used to transmit pulses at a symbol rate greater than
(C) 9 (D) 27
the half-power frequency of the low pass function. Which of the
network shown in the figure is can be used to equalise the received
pulses?

8.161 The power spectral density of a deterministic signal is given by


[sin (f) /f 2] where f is frequency. The auto correlation function of
this signal in the time domain is
(A) a rectangular pulse (B) a delta function
(C) a sine pulse (D) a triangular pulse

1996 ONE MARK

8.162 A rectangular pulse of duration T is applied to a filter matched to


this input. The out put of the filter is a
(A) rectangular pulse of duration T
(B) rectangular pulse of duration 2T
(C) triangular pulse
(D) sine function

8.163 The image channel rejection in a superheterodyne receiver comes


GATE Electronics and Communication Topicwise Solved Paper by RK Kanodia & Ashish Murolia Page 198

SOLUTIONS for which we have


X = 2U = 0
Y = 2V = 0
and X2 = 4U2 = 1
also, Y2 = 9V2 = 1
Therefore, X - Y is also a normal random variable with
8.1 Option (B) is correct. X-Y = 0
In ideal Nyquist Channel, bandwidth required for ISI (Inter Symbol Hence,
reference) free transmission is P ^X - Y $ 0h = P ^X - Y # 0h = 1
2
W = Rb
2 or, we can say
Here, the used modulation is 32 - QAM (Quantum Amplitude P ^2U - 3V # 0h = 1
2
modulation
i.e., q = 32 Thus, P ^3V $ 2U h = 1
2
or 2v = 32
v = 5 bits 8.3 Option (C) is correct.
So, the signaling rate (sampling rate) is The mean of random variables U and V are both zero
i.e., U =V=0
Rb = R
5 Also, the random variables are identical
(R " given bit rate)
i.e., fU ^u h = fV ^v h
or, FU ^u h = FV ^v h
GATE Electronics & Communication i.e., their cdf are also same. So,
by RK Kanodia FU ^u h = F2V ^2v h
i.e., the cdf of random variable 2V will be also same but for any
Now in 3 Volume instant
Purchase Online at maximum discount from online store 2V $ U
and get POSTAL and Online Test Series Free Therefore,
visit www.nodia.co.in G ^x h = F ^x h
but, x G ^x h $ xF ^x h
Hence, for ISI free transmission, minimum bandwidth is
or, 6F ^x h - G ^x h@x # 0
W = Rb = R kHz
8.4 Option (C) is correct.
2 10
Given, P ^U =+ 1h = P ^U =- 1h = 1
8.2 Option (B) is correct. 2
Given, random variables U and V with mean zero and variances 1 where U is a random variable which is identical to V i.e.,
4
and 1 P ^V =+ 1h = P ^V =- 1h = 1
9 2
i.e., U =V=0 So, random variable U and V can have following values
su2 = 1 U =+ 1, - 1; V =+ 1, - 1
4
2 1 Therefore the random variable U + V can have the following values,
and sv = U+V
9
- 2 When U = V =- 1
P ^U $ 0h = 1
= *0 When U = 1,V = 1 or u =- 1, v = 1
so,
2
and P ^V $ 0h = 1 2 When U = V = 1
2 Hence, we obtain the probabilities for U + V as follows
The distribution is shown in the figure below
U+V P ^U + V h
For more GATE Resources, Mock Test and
Study material join the community
http://www.facebook.com/gateec2014

-2 1 1=1
2#2 4
0 1 1 1 1 1
b2 # 2l+b2 # 2l = 2
fu ^u h = 1 e -u
2s 2
1 1=1
2p su2
u
2
2#2 4
fv ^v h = Therefore, the entropy of the ^U + V h is obtained as
1 e -v 2
2s
2p sv2
v

We can express the distribution in standard form by assuming H ^U + V h = / P^U + V h log 2


1
' P ^U + V h 1
X = u - 0 = u = 2U = 1 log 2 4 + 1 log 2 2 + 1 log 2 4
su Y2 4 2 4
and Y = v - 0 = v = 3V 2 1 2
sv Y3 = + +
4 2 4
GATE Electronics and Communication Topicwise Solved Paper by RK Kanodia & Ashish Murolia Page 199

=3 = 1 ;2 b 1 # 1 # 103 # 6 l + 400E
2 p 2
8.5 Option (D) is correct. = 1 66000 + 400@ = 6400
p p
For the shown received signal, we conclude that if 0 is the transmitted
E [X (t)] is the absolute value of mean of signal X (t) which is also
signal then the received signal will be also zero as the threshold is
equal to value of X (w) at (w = 0).
1 and the pdf of bit 0 is not crossing 1. Again, we can observe that
From given PSD
there is an error when bit 1 is received as it crosses the threshold.
The probability of error is given by the area enclosed by the 1 bit SX (w) w = 0 = 0
pdf (shown by shaded region) SX (w) = X (w) 2 = 0

X (w) 2w = 0 = 0

X (w) w = 0 = 0

8.8 Option (C) is correct.


For raised cosine spectrum transmission bandwidth is given as
BT = W (1 + a) a " Roll of factor
P (error when bit 1 received) = 1 # 1 # 0.25 = 1
2 8
BT = Rb (1 + a) Rb " Maximum signaling rate
2
or P b received 1 l = 1 SPECIAL EDITION ( STUDY MATERIAL FORM )
transmitted 0 8
Since, the 1 and 0 transmission is equiprobable: At market Book is available in 3 volume i.e. in 3 book binding
i.e., P ^ 0 h = P ^1 h = 1 form. But at NODIA Online Store book is available in 10 book
2
binding form. Each unit of Book is in separate binding.
Hence bit error rate (BER) is Available Only at NODIA Online Store
BER
Click to Buy
transmitted 1 ^ h 0l ^ h
= Pb received 0
l P 0 + P b transmitted
received 1 P 1
www.nodia.co.in
= 0+1 #1
8 2
= 1 3500 = Rb (1 + 0.75)
16 2
8.6 Option (B) is correct. Rb = 3500 # 2 = 4000
1.75
The optimum threshold is the threshold value for transmission as
obtained at the intersection of two pdf. From the shown pdf. We
obtain at the intersection 8.9 Option (D) is correct.
Entropy function of a discrete memory less system is given as
(transmitted, received) = b 4 , 1 l
5 5 N-1

we can obtain the intersection by solving the two linear eqs


H = / P log b P1 l
k
k
k=0
x+y = 1 pdf of received bit 0 where Pk is probability of symbol Sk .
y = 0.5 x pdf of received bit 1 For first two symbols probability is same, so
2 N-1
4
Hence for threshold = , we have H = P1 log b 1 l + P2 log b 1 l + Pk log b 1 l
/
5 P1 P2 k=3
Pk
N-1
BER =-e P1 log P1 + P2 log P2 + / P log P o
k k
k=3
= P b received 1 l P ^0 h + P b received 0 l P ^1 h N-1
transmitted 0 transmitted 1 =-e 2P log P + / P log P o
k k (P1 = P2 = P)
= b1 # 1 # 1l# 1 +b1 # 4 # 1l# 1
k=3
2 5 2 2 2 5 5 2 Now, P1 = P + e, P2 = P - e
N-1
= 1 <(BER for threshold = 1) So, Hl =-=(P + e) log (P + e) + (P - e) log (P - e) + / P log P G
20 k k
k=3
Hence, optimum threshold is 4 By comparing, Hl < H , Entropy of source decreases.
5
8.7 Option (A) is correct. 8.10 Option (B) is correct.
The mean square value of a stationary process equals the total area Probability density function of uniformly distributed variables X
under the graph of power spectral density, that is and Y is shown as
#S
3
E [X 2 (t)] = X (f ) df
-3

E [X 2 (t)] = 1 #S
3
or, (w) dw
2p -3
X

E [X 2 (t)] = 2 # 1 SX (w) dw (Since the PSD is even)


#
3
or,
2p 0
= 1 [area under the triangle + integration of delta function]
p P &[max (x, y)] < 1 0
2
GATE Electronics and Communication Topicwise Solved Paper by RK Kanodia & Ashish Murolia Page 200

Since X and Y are independent. where 0 # t # T , E is the transmitted energy per bit.
General function of local oscillator
P &[max (x, y)] < 1 0 = P b X < 1 l P bY < 1 l
2 2 2 2 sin (w t), 0 # t < T
f1 (t) = c
T
PbX < 1 l = shaded area = 3
2 4 But here local oscillator is ahead with 45c. so,
Similarly for Y : P bY < 1 l =3 f1 (t) = 2 sin (w t + 45c)
2 4 T c

So P &[max (x, y)] < 1 0 =3#3= 9 The coordinates of message points are
2 4 4 16 T
s11 = # s (t) f (t) dt
0
1 1

T
2E sin w t 2 sin (w t + 45c) dt
Alternate Method: = #
0 T c
T c

From the given data since random variables X and Y lies in the
2E T
interval [- 1, 1] as from the figure X , Y lies in the region of the =
T # sin (w t) sin (w t + 45c) dt
0
c c

square ABCD . 2E 2 T
1 [sin 45c + sin (2w t + 45c)] dt
=
T T #0 2 c

= 1
T
1 dt + 1 E Tsin (2w t + 45c) dt
T
E #
0 2 T 0
c#
1444444 420
4 44444 3
= E
2
GATE Electronics & Communication
E
by RK Kanodia Similarly, s21 =-
2
Signal space diagram
Now in 3 Volume
Purchase Online at maximum discount from online store
and get POSTAL and Online Test Series Free
visit www.nodia.co.in

Now here the two message points are s11 and s21 .
The error at the receiver will be considered.
When : (i) s11 is transmitted and s21 received
(ii) s21 is transmitted and s11 received
So, probability for the 1st case will be as :
P b s21 received l = P (X < 0) (as shown in diagram)
s11 transmitted
= P _ E/2 + N < 0i
= P _N < - E/2 i
Taking the Gaussian distribution as shown below :

For more GATE Resources, Mock Test and


Study material join the community
http://www.facebook.com/gateec2014
Probability for max 6X, Y @ < 1/2 : The points for max 6X, Y @ < 1/2
will be inside the region of square AEFG .
So, P &max 6X, Y @ < 1 0 = Area of 4AEFG
2 Area of square ABCD
3 3
#2
= 2 = 9
2#2 16
8.11 Option (B) is correct.
In a coherent binary PSK system, the pair of signals s1 (t) and s2 (t)
used to represent binary system 1 and 0 respectively.
s1 (t) = 2E sin wc t
T
s2 (t) =- 2E sin wc t
T
GATE Electronics and Communication Topicwise Solved Paper by RK Kanodia & Ashish Murolia Page 201

Mean of the Gaussian distribution = E/2 Note: For phase lead compensator zero is nearer to the origin as
compared to pole, so option (C) can not be true.
Variance = N 0
2
Putting it in the probability function : 8.15 Option (A) is correct.
`x + E/2 j
f = tan-1 a w k - tan-1 a w k
2

E = 0
1
P bN < -
2l # e- 2N 0 /2 dx a b
-3
2p N 0 df 1/a 1/b
2 = - =0
1 e- `x + NE/2 j dx dw 1 +awk 1 +awk
2
0 2 2
= #-3 pN 0
0 a b
1 + w2 = 1 + 1 w2
x + E/2 a ab2 b b a2
Taking, =t
N 0 /2 1 - 1 = w2 1 - 1
dx = N 0 dt a b ab b a b l
2
w = ab = 1 # 2 = 2 rad/ sec
So, P _N < - E/2 i = 1 e- t2 dt Q E
2

# c N0 m
3

E/N 0 2p
8.16 Option (D) is correct.
where Q is error function. Quantized 4 level require 2 bit representation i.e. for one sample
Since symbols are equiprobable in the 2 nd case 2 bit are required. Since 2 sample per second are transmitted we
So, require 4 bit to be transmitted per second.
P b s11 received l = Q c E m
s21 transmitted N0
SPECIAL EDITION ( STUDY MATERIAL FORM )
So the average probability of error
At market Book is available in 3 volume i.e. in 3 book binding
= 1 ;P b s21 received l + P b s11 received lE form. But at NODIA Online Store book is available in 10 book
2 s11 transmitted s21 transmitted
E +Q E =Q E binding form. Each unit of Book is in separate binding.
= 1 =Q c
2 N0 m c N 0 mG c N0 m Available Only at NODIA Online Store

8.12 Option ( ) is correct. Click to Buy


8.13 Option (B) is correct. www.nodia.co.in
General equation of FM and PM waves are given by 8.17 Option (B) is correct.
t
fFM (t) = Ac cos ;wc t + 2pk f In FM the amplitude is constant and power is efficient transmitted.
# m (t) dtE
0 No variation in power.
fPM (t) = Ac cos [wc t + k p m (t)] There is most bandwidth efficient transmission in SSB- SC. because
For same maximum phase deviation. we transmit only one side band.
t
Simple Diode in Non linear region ( Square law ) is used in
k p [m (t)] max = 2pk f ; m (t) dtE # 0 max conventional AM that is simplest receiver structure.
k p # 2 = 2pk f [x (t)] max In VSB dc. component exists.
t
where, x (t) = # m (t) dt
0
8.18 Option (A) is correct.
We have Sx (f) = F {Rx (t)} = F {exp (- pt2)}
2
= e- pf
The given circuit can be simplified as

Power spectral density of output is


[x (t)] max = 4
Sy (f) = G (f) 2 Sx (f)
So, k p # 2 = 2pk f # 4 2
= j2pf - 1 2 e- pf
kp
= 4p = ( (2pf) 2 + 1) 2 e- pf
2
kf
2
or Sy (f) = (4p2 f 2 + 1) e- pf
8.14 Option (A) is correct. 8.19 Option (B) is correct.
jw + a
GC (s) = s + a = Highest frequency component in m (t) is fm = 4000p/2p = 2000 Hz
s+b jw + b
Carrier frequency fC = 1 MHz
Phase lead angle f = tan-1 a w k - tan-1 a w k For Envelope detector condition
a b
Jw - wN 1/fC << RC << 1/fm
-1 K a b O = tan-1 w (b - a)
f = tan
KK 2
O c ab + w 2 m 1 μs << RC << 0.5 ms
1+ w O
L ab P 8.20 Option (D) is correct.
For phase-lead compensation f > 0
b-a > 0 Four phase signal constellation is shown below
b >a
GATE Electronics and Communication Topicwise Solved Paper by RK Kanodia & Ashish Murolia Page 202

Where m < 1, for no over modulation.


In option (C)
x (t) = AC :1 + 1 m (t)D cos (2pfC t)
4
Thus m = 1 < 1 and this is a conventional AM-signal without over-
4
modulation
8.23 Option (B) is correct.
2 2 2
Now d = r +r 1 1 (6) 2
Power P = = 18 W
d2 = 2r 12 2
r1 = d/ 2 = 0.707d 8.24 Option (C) is correct.
Impulse response of the matched filter is given by
h (t) = S (T - t)

GATE Electronics & Communication


by RK Kanodia
Now in 3 Volume
Purchase Online at maximum discount from online store
and get POSTAL and Online Test Series Free
visit www.nodia.co.in
8.25 Option (B) is correct.
q = 2p = 2p = p Let response of LPF filters
M 8 4 1, f < 1 MHz
Applying Cooine law we have H (f ) = *
0, elsewhere
d2 = r 22 + r 22 - 2r 22 cos p Noise variance (power) is given as
4
H (f ) No df = 22 (given)
fo 2
= 2r 22 - 2r 22 1/ 2 = (2 - 2 ) r 22 P = s2 = #0
a
or r2 = d = 1.3065d
2 # 10-20 df = 22
1 # 106
2- 2 #0
a
8.21 Option (D) is correct.
Here Pe for 4 PSK and 8 PSK is same because Pe depends on d . 2 # 10 # 10 = 22
-20 6
a
Since Pe is same, d is same for 4 PSK and 8 PSK. a = 1014
2

or a = 107
8.26 Option (D) is correct.
Probability of error is given by

For more GATE Resources, Mock Test and


Study material join the community
http://www.facebook.com/gateec2014
Additional Power SNR
= (SNR) 2 - (SNR) 1
Pe = 1 [P (0/1) + P (1/0)]
2
= 10 log b ES2 l - 10 log b ES1 l
No No a/2
P (0/1) = #- 3 0.5e- a n - a dn = 0.5e-10
= 10 log b ES2 l
ES1 where a = 2 # 10-6 V and a = 107 V - 1

P (1/0) = #a/32 0.5e- a n dn = 0.5e-10


= 10 log a r2 k & 20 log a r2 k = 20 log 1.3065d
2
r1 r1 0.707d
Pe = 0.5e-10
Additional SNR = 5.33 dB
8.27 Option (C) is correct.
8.22 Option (C) is correct.
S (t) = sin c (500t) sin c (700t)
Conventional AM signal is given by
S (f ) is convolution of two signals whose spectrum covers f 1 = 250 Hz
x (t) = AC [1 + mm (t)] cos (2pfC t)
GATE Electronics and Communication Topicwise Solved Paper by RK Kanodia & Ashish Murolia Page 203

and f 2 = 350 Hz . So convolution extends X2 = Sxi2 pi (x)


f = 25 + 350 = 600 Hz = 1 # 0.1 + 4 # 0.2 + 9 # 0.4 + 16 # 0.2 + 25 # 0.1
Nyquist sampling rate = 0.1 + 0.8 + 3.6 + 3.2 + 2.5 = 10.2
N = 2f = 2 # 600 = 1200 Hz Variance sx2 = X2 - ^X h2
8.28 Option (D) is correct. = 10.2 - (3) 2 = 1.2
For the given system, output is written as 8.32 Option (C) is correct.
y (t) = d [x (t) + x (t - 0.5)] m (t) = 1 cos w1 t - 1 sin w2 t
dt 2 2
dx (t) dx (t - 0.5) sAM (t) = [1 + m (t)] cos wc t
y (t) = +
dt dt
m (t) max
Taking Laplace on both sides of above equation Modulation index =
Vc
Y (s) = sX (s) + se-0.5s X (s)
m = `1j +`1j = 1
2 2
Y (s) 2 2 2
H (s) = = s (1 + e-0.5s)
X (s) 2
h = m # 100%
H (f ) = jf (1 + e-0.5 # 2pf ) = jf (1 + e- pf ) m2 + 2
` 2j
1 2
Power spectral density of output
= # 100% = 20%
SY (f ) = H (f ) 2 SX (f ) = f 2 (1 + e- pf ) 2 SX (f ) ` 2j
1 2
+ 2
For SY (f ) = 0 , 1 + e- pf = 0
SPECIAL EDITION ( STUDY MATERIAL FORM )
f = (2n + 1) f0
At market Book is available in 3 volume i.e. in 3 book binding
or f0 = 1 KHz
form. But at NODIA Online Store book is available in 10 book
8.29 Option (C) is correct. binding form. Each unit of Book is in separate binding.
cos (2pfm t) cos (2pfc t) $ DSB suppressed carrier Available Only at NODIA Online Store
cos (2pfc t) $ Carrier Only Click to Buy
cos [2p (fc + fm) t] $ USB Only www.nodia.co.in
[1 + cos (2pfm t) cos (2pfc t)] $ USB with carrier
8.33 Option (B) is correct.
8.30 Option (C) is correct.
We have We have C1 = B log2 `1 + S j
N
p (X = 0) = p (Y = 0) = 1 . B log2 ` S j As S >> 1
2 N N
p (X = 1) = p (Y = 1) = 1 If we double the S ratio then
4 N
p (X = 2) = p (Y = 2) = 1 C2 . B log2 ` 2S j
4 N
Let X+Y = 2 $ A
. B log2 2 + B log2 S . B + C1
and X-Y = 0 $ B N
8.34 Option (C) is correct.
Now
We have SNR = 1.76 + 6n
P (A + B)
P (X + Y = 2 X - Y = 0) = or 43.5 = 1.76 + 6n
P (B)
Event P (A + B) happen when X + Y = 2 and X - Y = 0 . It is only 6n = 43.5 + 1.76
the case when X = 1 and Y = 1. 6n = 41.74 $ n . 7
No. of quantization level is
Thus P (A + B) = 1 # 1 = 1
4 4 16 27 = 128
Now event P (B) happen when Step size required is
X - Y = 0 It occurs when X = Y , i.e. 5 - (- 5)
= VH - VL = = 10
X = 0 and Y = 0 or 128 128 128
X = 1 and Y = 1 or = .078125 . .0667
X = 2 and Y = 2 8.35 Option (B) is correct.
Thus P (B) = 1 # 1 + 1 # 1 + 1 # 1 = 6 For positive values step size
2 2 4 4 4 4 16 s+ = 0.05 V
P (A + B) 1/16
Now = =1 For negative value step size
P (B) 6/16 6
s- = 0.1 V
8.31 Option (B) is correct.
No. of quantization in + ive is
The mean is
= 5 = 5 = 100
X = Sxi pi (x) s+ 0.05
= 1 # 0.1 + 2 # 0.2 + 3 # 0.4 + 4 # 0.2 + 5 # 0.1 Thus 2n + = 100 $ n+ = 7
= 0.1 + 0.4 + 1.2 + 0.8 + 0.5 = 3.0 No. of quantization in - ve
GATE Electronics and Communication Topicwise Solved Paper by RK Kanodia & Ashish Murolia Page 204

Q1 = 5 = 5 = 50 = 1 [2W + 2W + 4W + 6W] = 7W
s- 0.1 2
2n = 50 $ n - = 6
-
Thus 8.42 Option (B) is correct.
` N j+ = 1.76 + 6n = 1.76 + 42 = 43.76 dB
S + We have qi = 2p105 t + 5 sin (2p1500t) + 7.5 sin (2p1000t)
wi = dqi = 2p105 + 10p1500 cos (2p1500t) + 15p1000 cos (2p1000t)
` N j- = 1.76 + 6n = 1.76 + 36 = 37.76 dB
S -
dt
Maximum frequency deviation is
` N j0 = 43.76 dB
Best S
3wmax = 2p (5 # 1500 + 7.5 # 1000)
8.36 Option (A) is correct. 3 fmax = 15000
3f
We have xAM (t) = Ac cos wc + 2 cos wm t cos wc t Modulation index is = max = 15000 = 10
fm 1500
= AC c1 + 2 cos wm t m cos wc t
Ac 8.43 Option (C) is correct.
For demodulation by envelope demodulator modulation index 8.44 Option (B) is correct.
must be less than or equal to 1. fm = 4 KHz
Thus 2 #1
Ac fs = 2fm = 8 kHz
Ac $ 2 Bit Rate Rb = nfs = 8 # 8 = 64 kbps
Hence minimum value of Ac = 2 The minimum transmission bandwidth is
BW = Rb = 32 kHz
2
GATE Electronics & Communication
by RK Kanodia 8.45 Option (C) is correct.
S0
Now in 3 Volume c N m = 1.76 + 6n dB
0
Purchase Online at maximum discount from online store = 1.76 + 6 # 8 = 49.76 dB We have n = 8
and get POSTAL and Online Test Series Free
visit www.nodia.co.in 8.46 Option (B) is correct.

8.37 Option (A) is correct. As Noise \ 12


L
CDF is the integration of PDF. Plot in option (A) is the integration To reduce quantization noise by factor 4, quantization level must
of plot given in question.
be two times i.e. 2L .
8.38 Option (A) is correct. Now L = 2n = 28 = 256
The entropy is
m
Thus 2L = 512
H = / pi log2 p1i bits 8.47 Option (C) is correct.
i=1
Autocorrelation is even function.
Since p1 = p2 = ... = pn = 1
n 8.48 Option (B) is correct.
n
Power spectral density is non negative. Thus it is always zero or
H = / n1 log n = log n greater than zero.
i=1
8.39 Option (C) is correct. 8.49 Option (A) is correct.
PSD of noise is N0 = K ...(1) The variance of a random variable x is given by
2
E [X2] - E2 [X]
The 3-dB cut off frequency is
8.50 Option (A) is correct.
fc = 1 ...(2)
2pRC For more GATE Resources, Mock Test and
Output noise power is Study material join the community
= N0 = c N0 m 1 = Kpfc http://www.facebook.com/gateec2014
4RC 2 2RC
A Hilbert transformer is a non-linear system.
8.40 Option (D) is correct.
At receiving end if we get two zero or three zero then its error.
8.51 Option (D) is correct.
Let p be the probability of 1 bit error, the probability that Slope overload distortion can be reduced by increasing the step size
transmitted bit error is 3 $ slope of x (t)
Ts
= Three zero + two zero and single one
8.52 Option (C) is correct.
= 3 C3 p3 + 3C2 p2 (1 - p)
sin (4pWt)
= p3 + p2 (1 - p) We have p (t) =
4pWt (1 - 16W2 t2)
8.41 Option (D) is correct. at t = 1 it is 0 form. Thus applying L' Hospital rule
Bandwidth of TDM is 4W 0
4pW cos (4pWt)
= 1 (sum of Nyquist Rate) p( ) =
1
4W

2 4pW [1 - 48W2 t2]


GATE Electronics and Communication Topicwise Solved Paper by RK Kanodia & Ashish Murolia Page 205

cos (4pWt) 6
= = cos p = 0.5 Rb max = RC = 1.2288 # 10 = 12.288 # 103 bps
1 - 48W2 t2 1-3 Gmin 100
8.53 Option (B) is correct. 8.57 Option (B) is correct.
The block diagram is as shown below Energy of constellation 1 is
Eg1
2 2
= (0) + (- 2 a) + (- 2 a) 2 + ( 2 a) 2 + (- 2 2 a) 2
= 2a2 + 2a2 + 2a2 + 8a2 = 16a2
Energy of constellation 2 is
Eg2 = a2 + a2 + a2 + a2 = 4a2
E 2
Ratio = g1 = 16a2 = 4
Eg2 4a
Here M1 (f) = Mt (f) 8.58 Option (A) is correct.
j2 p B
+ e -j 2 p B Noise Power is same for both which is N0 .
Y1 (f) = M (f) c e m 2
2
j 2p B Thus probability of error will be lower for the constellation 1 as it
Y2 (f) = M1 (f) c e - e -j2pB
2 m has higher signal energy.
Y (f) = Y1 (f) + Y2 (f) 8.59 Option (A) is correct.
All waveform is shown below SPECIAL EDITION ( STUDY MATERIAL FORM )
At market Book is available in 3 volume i.e. in 3 book binding
form. But at NODIA Online Store book is available in 10 book
binding form. Each unit of Book is in separate binding.
Available Only at NODIA Online Store

Click to Buy
www.nodia.co.in
Area under the pdf curve must be unity
Thus 2a + 4a + 4b = 1
2a + 8b = 1 ...(1)
For maximum entropy three region must by equivaprobable thus
2a = 4b = 4b ...(2)
From (1) and (2) we get
b = 1 and a = 1
12 6
8.60 Option (*) is correct.
8.61 Option (B) is correct.
A LPF will not produce phase distortion if phase varies linearly
8.54 Option (C) is correct. with frequency.
By Binomial distribution the probability of error is f (w) \ w
n r n-r
pe = Cr p (1 - p) i.e. f (w) = kw
Probability of at most one error 8.62 Option (B) is correct.
= Probability of no error + Probability of one error Let m (t) is a low pass signal, whose frequency spectra is shown
= n C0 p0 (1 - p) n - 0 + n C1 p1 (1 - p) n - 1 below
= (1 - p) n + np (1 - p) n - 1
8.55 Option (B) is correct.
Bandwidth allocated for 1 Channel = 5 M Hz
Average bandwidth for 1 Channel 5 = 1 MHz
5
Total Number of Simultaneously Channel = 1M # 8 = 40 Channel
200k
8.56 Option (A) is correct. Fourier transform of g (t)
Chip Rate RC = 1.2288 # 106 chips/sec 1
3
G (t) = /d (f - 20 # 103 k)
Data Rate Rb = RC 0.5 # 10-4 k =- 3
G Spectrum of G (f ) is shown below
Since the processing gain G must be at least 100, thus for Gmin we
get
GATE Electronics and Communication Topicwise Solved Paper by RK Kanodia & Ashish Murolia Page 206

= 0.25 log2 4 + 0.25 log2 4 + 0.5 log2 2


= 0.5 + 0.5 + 1 = 3 bits/symbol
2 2
Rb = 3000 symbol/sec
Average bit rate = Rb H
= 3 # 3000 = 4500 bits/sec
2
Now when m (t) is sampled with above signal the spectrum of 8.67 Option (A) is correct.
sampled signal will look like. The diagonal clipping in AM using envelop detector can be avoided
if
1 << RC < 1
wc W
But from 1 $ Wm sin Wt
RC 1 + m cos Wt
We can say that RC depends on W , thus
RC < 1
W
When sampled signal is passed through a LP filter of BW 1 kHz,
only m (t) will remain. 8.68 Option (B) is correct.
8.63 Option (C) is correct. 8.69 Option (B) is correct.
When 3 /2 is added to y (t) then signal will move to next quantization
GATE Electronics & Communication
level. Otherwise if they have step size less than 3 then they will be
by RK Kanodia 2
on the same quantization level.
Now in 3 Volume
8.70 Option (C) is correct.
Purchase Online at maximum discount from online store After the SSB modulation the frequency of signal will be fc - fm i.e.
and get POSTAL and Online Test Series Free 1000 - 10 kHz . 1000 kHz
visit www.nodia.co.in The bandwidth of FM is
The highest frequency signal in x (t) is 1000 # 3 = 3 kHz if expression BW = 2 (b + 1) 3 f
is expanded. Thus minimum frequency requirement is For NBFMb << 1, thus
f = 2 # 3 # 103 = 6 # 103 Hz BWNBFM . 2 3 f = 2 (109 - 106) . 2 # 109
8.64 Option (B) is correct. 8.71 Option (A) is correct.
We have We have p (t) = u (t) - u (t - 1)
x (t) = 125t [u (t) - u (t - 1)] + (250 - 125t) [u (t - 1) - u (t - 2)] g (t) = p (t)* p (t)
The slope of expression x (t) is 125 and sampling frequency fs is s (t) = g (t) - d (t - 2)* g (t) = g (t) - g (t - 2)
32 # 1000 samples/sec. All signal are shown in figure below :
Let 3 be the step size, then to avoid slope overload
3 $ slope x (t)
Ts
3 fc $ slope x (t)
3# 32000 $ 125
3 $ 125
32000
3 = 2- 8 For more GATE Resources, Mock Test and
8.65 Option (A) is correct. Study material join the community
The sampling frequency is http://www.facebook.com/gateec2014
fs = 1 = 33 kHz
0.03m The impulse response of matched filter is
Since fs $ 2fm , the signal can be recovered and are correlated. h (t) = s (T - t) = s (1 - t)
Here T is the time where output SNR is maximum.
8.66 Option (B) is correct.
We have p1 = 0.25 , p2 = 0.25 and p3 = 0.5 8.72 Option (A) is correct.
3 We have xAM (t) = 10 [P (t) + 0.5g (t)] cos wc t
H = / p1 log2 p11 bits/symbol
where p (t) = u (t) - u (t - 1)
i=1

= p1 log2 1 + p2 log2 1 + p3 log2 1 and g (t) = r (t) - 2r (t - 1) + r (t - 2)


p1 p2 p3
For desired interval 0 # t # 1, p (t) = 1 and g (t) = t , Thus we
have,
= 0.25 log2 1 + 0.25 log2 1 + 0.5 log2 1
0.25 0.25 0.5 xAM (t) = 100 (1 - 0.5t) cos wc t
GATE Electronics and Communication Topicwise Solved Paper by RK Kanodia & Ashish Murolia Page 207

Hence modulation index is 0.5 The transfer function of matched filter is


8.73 Option (A) is correct. h (t) = x (t - t) = x (2 - t)
We know that SYY (w) = H (w) 2 .SXX (w) The output of matched filter is the convolution of x (t) and h (t) as
Now SYY (w) = 16 2 and SXX (w) = 1 white noise shown below
16 + w

Thus 16 = H (w) 2
16 + w2
or H (w) = 4
16 + w2
or H (s) = 4
4+s
which is a first order low pass RL filter.
8.74 Option (A) is correct.
We have R = 4
8.81 Option (B) is correct.
R + sL 4+s We have H (f) = 2e - jwt d

R
or = 4 L H (f) = 2
+s R
L
4+s
G0 (f) = H (f) 2 Gi (f)
Comparing we get L = 1 H and R = 4W
8.75 Option (C) is correct. SPECIAL EDITION ( STUDY MATERIAL FORM )
We have xAM (t) = 10 (1 + 0.5 sin 2pfm t) cos 2pfc t At market Book is available in 3 volume i.e. in 3 book binding
The modulation index is 0.5 form. But at NODIA Online Store book is available in 10 book
(10) 2 binding form. Each unit of Book is in separate binding.
Carrier power Pc = = 50 Available Only at NODIA Online Store
2
(10) 2 Click to Buy
Side band power Ps = = 50
2 www.nodia.co.in
2 (0.5) 2 (50)
Side band power Ps = m Pc = = 6.25
2 2 = 4No W/Hz
8.76 Option (B) is correct. The noise power is = 4No # B
Mean noise power = Area under the PSD curve 8.82 Option (C) is correct.
= 4 ; 1 # B # No E = BNo As the area under pdf curve must be unity
2 2 1 (4 # k) = 1 $ k = 1
The ratio of average sideband power to mean noise power is 2 2
Side Band Power = 6.25 = 25 Now mean square value is
Noise Power N0 B 4No B +3
sv2 = #- 3 v2 p (v) dv
8.77 Option (D) is correct.
v2 ` v j dv
4
{1 + km (t)} A sin (wc t) $ Amplitude modulation = #0 as p (v) = 1 v
8 8
dm (t) Asin (wc t) $ DSB-SC modulation 3
c 8 m dv = 8
4
v
A sin {cos t + km (t)} $ Phase Modulation =
0
#
A sin [wct + k] t- 3 m (t) dt $ Frequency Modulation 8.83 Option (D) is correct.
8.78 Option (C) is correct. The phase deviation is
VSB $ fm + fc 3f
b = = 10 = 10
DSB - SC $ 2fm fm 1
SSB $ fm If phase deviation remain same and modulating frequency is
AM $ 2fm changed
Thus SSB has minimum bandwidth and it require minimum pow- BW = 2 (b + 1) fm' = 2 (10 + 1) 2 = 44 kHz
er. 8.84 Option (B) is correct.
As the area under pdf curve must be unity and all three region are
8.79 Option (A) is correct.
equivaprobable. Thus are under each region must be 13 .
Let x (t) be the input signal where
x (t) = cos (cos t + b1 cos wm t) 2a # 1 = 1 $ a = 2
4 3 3
cos (2wc t + 2b1 cos wm t)
y (t) = x2 (t) = 1 + 8.85 Option (A) is correct.
2 2 3 a 3
x $ 1 dx = 1 ; x E = a
+a a 2
3f Nq = #- a x2 p (x) dx = 2 #0
Here b = 2b1 and b1 = = 90 = 18 4 2 3 0 6
fm 5
BW = 2 (b + 1) fm = 2 (2 # 18 + 1) # 5 = 370 kHz Substituting a = 2 we have
3
8.80 Option (C) is correct. Nq = 4
81
GATE Electronics and Communication Topicwise Solved Paper by RK Kanodia & Ashish Murolia Page 208

8.86 Option (C) is correct. 8.90 Option (D) is correct.


When word length is 6 F (x1 # X < x2) = p (X = x2) - P (X = x1)
` N jN = 6 = 2 = 2
S 2#6 12
or P (X = 1) = P (X = 1+) - P (X = 1 -)
= 0.55 - 0.25 = 0.30
When word length is 8
Option (A) is correct.
` N jN = 8 = 2 = 2
8.91
S 2#8 16
The SNR at transmitter is
^ N hN = 8 SNRtr = Ptr
S 16
Now = 212 = 2 4 = 16
^ N hN = 6
S
2 NB
Thus it improves by a factor of 16. 10 - 3 = 109
10 - 20 # 100 # 106
8.87 Option (B) is correct.
In dB SNRtr = 10 log 109 = 90 dB
Carrier frequency fc = 1 # 106 Hz
Cable Loss = 40 db
Modulating frequency
At receiver after cable loss we have
fm = 2 # 103 Hz
SNRRc = 90 - 40 = 50 dB
For an envelope detector
8.92 Option (B) is correct.
2pfc > 1 > 2pfm The impulse response of matched filter is
Rc
1 < RC < 1 h (t) = x (T - t)
2pfc 2pfm Since here T = 4 , thus
GATE Electronics & Communication h (t) = x (4 - t)
by RK Kanodia The graph of h (t) is as shown below.
Now in 3 Volume
Purchase Online at maximum discount from online store
and get POSTAL and Online Test Series Free
visit www.nodia.co.in
1 < RC < 1
2pfc 2pfm
1 < RC < 1 From graph it may be easily seen that slope between 3 < t < 4 is
6
2p10 2 # 103 - 1.
1.59 # 10 - 7 < RC < 7.96 # 10 - 5
8.93 Option (C) is correct.
so, 20 msec sec best lies in this interval.
The required bandwidth of M array PSK is
Option (B) is correct.
BW = 2Rb
8.88

SAM (t) = Ac [1 + 0.1 cos wm t] cos wm t n


sNBFM (t) = Ac cos [wc t + 0.1 sin wm t] where 2n = M and Rb is bit rate
s (t) = SAM (t) + SNB fm (t) For BPSK, M = 2 = 2n $ n = 1
Thus B1 = 2Rb = 2 # 10 = 20 kHz
1
= Ac [1 + 0.1 cos wm t] cos wc t + Ac cos (wc t + 0.1 sin wm t)
For QPSK, M = 4 = 2n $ n = 2
= Ac cos wc t + Ac 0.1 cos wm t cos wc t
Thus B2 = 2Rb = 10 kHz
2
+ Ac cos wc t cos (0.1 sin wm t) - Ac sin wc t. sin (0.1 sin wm t)
For more GATE Resources, Mock Test and
As 0.1 sin wm t ,+ 0.1 to - 0.1
Study material join the community
so, cos (0.1 sin wm t) . 1
http://www.facebook.com/gateec2014
As when q is small cos q . 1 and sin q , q, thus
sin (0.1 sin wm t) 8.94 Option (C) is correct.
= 0.1 sincos wc t cos wm t + Ac cos wc t We have fc = 100 MHz = 100 # 106 and fm = 1
- Ac 0.1 sin wm t sin wc t MHz
= 2Ac cos wc t + 0.1Ac cos (wc + wm) t = 1 # 106
1 44 2 44 3 1 4444 4 2 4444 43 The output of balanced modulator is
cosec USB
VBM (t) = [cos wc t][ cos wc t]
Thus it is SSB with carrier.
= 1 [cos (wc + wm) t + cos (wc - wm) t]
8.89 Option (A) is correct. 2
Consecutive pulses are of same polarity when modulator is in slope If VBM (t) is passed through HPF of cut off frequency fH = 100 # 106
overload. , then only (wc + wm) passes and output of HPF is
Consecutive pulses are of opposite polarity when the input is
VHP (t) = 1 cos (wc + wm) t
constant. 2
GATE Electronics and Communication Topicwise Solved Paper by RK Kanodia & Ashish Murolia Page 209

Now V0 (t) = VHP (t) + sin (2p # 100 # 106) t Probability of error of 1
P (0 # X # 0.2) = 0.2
= 1 cos [2p100 # 106 + 2p # 1 # 106 t] + sin (2p # 100 # 106) t Probability of error of 0 :
2
P (0.2 # X # 0.25) = 0.05 # 2 = 0.1
= 1 cos [2p108 + 2p106 t] + sin (2p108) t
2 P (0 # X # 0.2) + P (0.2 # X # 0.25)
Average error =
2
= 1 [cos (2p108 t) t cos (2p106 t)] - sin [2p108 t sin (2p106 t) + sin 2p108 t] = 0.2 + 0.1 = 0.15
2 0
8.97 Option (B) is correct.
= 1 cos (2p106 t) cos 2p108 t + `1 - 1 sin 2p106 t j sin 2p108 t The square mean value is
2 2 3
This signal is in form s2 = #- 3 (x - xq) 2 f (x) dx
= A cos 2p108 t + B sin 2p108 t 1

Now
= #0 (x - xq) 2 f (x) dx

GATE = #0
0. 3
(x - 0) 2 f (x) dx +
0.1
#0.3 (x - 0.7) 2 f (x) dx
ELECTRONICS & COMMUNICATION
by
RK Kanodia & Ashish Murolia SPECIAL EDITION ( STUDY MATERIAL FORM )
in 3 Volumes At market Book is available in 3 volume i.e. in 3 book binding
form. But at NODIA Online Store book is available in 10 book
Fully Revised binding form. Each unit of Book is in separate binding.
Available Only at NODIA Online Store

Every Unit Contain All Questions of Last 15 Year Papers


Click to Buy
Purchase from Nodia Online Store at Maximum Discount with free www.nodia.co.in
Shipping
3 0.3 3 2 1
The envelope of this signal is = ; x E + ; x + 0.49x - 14 x E
= A2 + B2 3 0 3 2 0.3
or s2 = 0.039
RMS = s2 = 0.039 = 0.198
` 2 cos (2p10 t)j + `1 - 2 sin (2p10 t j
1 6 2 1 6 2
=
8.98 Option (C) is correct.
FM $ Capture effect
= 1 cos2 (2p106 t) + 1 + 1 sin2 (2p106 t) - sin (2p106 t) DM $ Slope over load
4 4
PSK $ Matched filter
= 1 + 1 - sin (2p106 t)
4 PCM $ m - law
= 5 - sin (2p106 t) 8.99 Option (C) is correct.
4
Since fs = 2fm , the signal frequency and sampling frequency are as
8.95 Option (A) is correct. follows
s (t) fm1 = 1200 Hz $ 2400 samples per sec
= A cos [2p10 # 10 t] + A cos [2p10.1 # 103 t]
3
fm2 = 600 Hz $ 1200 samples per sec
Here T1 = 1 = 100m sec fm3 = 600 Hz $ 1200 samples per sec
10 # 103
Thus by time division multiplexing total 4800 samples per second
and T2 = 1 = 99m sec
10.1 # 103 will be sent. Since each sample require 12 bit, total 4800 # 12 bits
Period of added signal will be LCM [T1, T2] per second will be sent
Thus T = LCM [100, 99] = 9900m sec Thus bit rate Rb = 4800 # 12 = 57.6 kbps
Thus frequency f = 1 = 0.1 kHz 8.100 Option (B) is correct.
9900m
The input signal X (f) has the peak at 1 kHz and - 1 kHz. After
8.96 Option (A) is correct. balanced modulator the output will have peak at fc ! 1 kHz i.e. :
The pdf of transmission of 0 and 1 will be as shown below : 10 ! 1 $ 11 and 9 kHz
10 ! (- 1) $ 9 and 11 kHz
9 kHz will be filtered out by HPF of 10 kHz. Thus 11 kHz will re-
main. After passing through 13 kHz balanced modulator signal will
have 13 ! 11 kHz signal i.e. 2 and 24 kHz.
Thus peak of Y (f) are at 2 kHz and 24 kHz.
GATE Electronics and Communication Topicwise Solved Paper by RK Kanodia & Ashish Murolia Page 210

8.101 Option (A) is correct. 8.106 Option (C) is correct.


The input is a coherent detector is DSB - SC signal plus noise. W = Y-Z
The noise at the detector output is the in-phase component as the E [W2] = E [Y - Z] 2 = E [Y2] + E [Z2] - 2E [YZ]
quadrature component nq (t) of the noise n (t) is completely rejected
= sw2
by the detector.
We have E [X2 (t)] = Rx (10)
8.102 Option (C) is correct.
= 4 [e - 0.2 0 + 1] = 4 [1 + 1] = 8
The noise at the input to an ideal frequency detector is white. The
PSD of noise at the output is parabolic E [Y2] = E [X2 (2)] = 8
E [Z2] = E [X2 (4)] = 8
8.103 Option (B) is correct.
Ed E [YZ] = RXX (2) = 4 [e-0.2 (4 - 2) + 1] = 6.68
Pe = 1 erfc c
2h m
We have
2 E [W2] = sw2 = 8 + 8 - 2 # 6.68 = 2.64
Since Pe of Binary FSK is 3 dB inferior to binary PSK 8.107 Option (C) is correct.
Option (D) is correct. 2mp
= 1.536 = 0.012 V
8.104
Step size d =
The pdf of Z will be convolution of pdf of X and pdf of Y as shown L 128
below. 2 (0.012) 2
Quantization Noise power = d =
z 12 12
Now p [Z # z] = #- 3 fZ (z) dz = 12 # 10-6 V2
-2
p [Z #- 2] = #- 3fZ (z) dz 8.108 Option (D) is correct.
The frequency of pulse train is
GATE Electronics & Communication
f 1- 3 = 1 k Hz
by RK Kanodia 10
Now in 3 Volume The Fourier Series coefficient of given pulse train is
-T /2
Purchase Online at maximum discount from online store Cn = 1 #
o

Ae-jnw t dt o

To -T /2
and get POSTAL and Online Test Series Free o

-To /6

visit www.nodia.co.in = 1 # Ae-jhw t dt


o

To -To /6

= A [e-jw t] --TT //66o o

= Area [z #- 2] To (- jhwo) o

= 1 # 1 #1 = 1 = A (e-jw t - e jhw T /6)


o o o
2 6 12 (- j2pn)
= A (e jhp/3 - e-jhp/3)
j2pn
or Cn = A sin ` np j
pn 3
From Cn it may be easily seen that 1, 2, 4, 5, 7, harmonics are present
and 0, 3, 6, 9,.. are absent. Thus p (t) has 1 kHz, 2 kHz, 4 kHz, 5 kHz,
7 kHz,... frequency component and 3 kHz, 6 kHz.. are absent.
The signal x (t) has the frequency components 0.4 kHz and 0.7
kHz. The sampled signal of x (t) i.e. x (t)* p (t) will have
1 ! 0.4 and 1 ! 0.7 kHz
2 ! 0.4 and 2 ! 0.7 kHz
4 ! 0.4 and 4 ! 0.7 kHz
For more GATE Resources, Mock Test and
Study material join the community
http://www.facebook.com/gateec2014

Thus in range of 2.5 kHz to 3.5 kHz the frequency present is


8.105 Option (D) is correct.
2 + 0.7 = 2.7 kHz
We have RXX (t) = 4 (e - 0.2 t + 1)
4 - 0.7 = 3.3 kHz
RXX (0) = 4 (e - 0.2 0 + 1) = 8 = s2
or s =2 2 Given 8.109 Option (C) is correct.
mean m =0 vi = Ac1 cos (2pfc t) + m (t)
Now P (x # 1) = Fx (1) v0 = ao vi + avi3
X-m v0
= 1 - Qc
s m
at x = 1
= a0 [Ac' cos (2pfc' t) + m (t)] + a1 [Ac' cos (2pfc' t) + m (t)] 3
= 1 - Qc 1 - 0 m = 1 - Qc 1 m
2 2 2 2
= a0 Ac' cos (2pfc' t) + a0 m (t) + a1 [(Ac' cos 2pfc' t) 3
GATE Electronics and Communication Topicwise Solved Paper by RK Kanodia & Ashish Murolia Page 211

then for coherent binary PSK r = 0.5 and s is required.


+ (Ac' cos (2pfc') t) 2 m (t) + 3Ac' cos (2pfc' t) m2 (t) 3
+ m (t)] 8.117 Option (B) is correct.
Bit Rate = 8k # 8 = 64 kbps
= a0 Ac' cos (2pfc' t) + a0 m (t) + a1 (Ac' cos 2fc' t) 3 (SNR)q = 1.76 + 6.02n dB
1 + cos (4pfc' t)
+ 3a1 Ac'2 ; E m (t) = 1.76 + 6.02 # 8 = 49.8 dB
2
8.118 Option (C) is correct.
= 3a1 Ac' cos (2pfc' t) m2 (t) + m3 (t)
The frequency of message signal is
The term 3a1 Ac' ( cos 42pf t ) m (t) is a DSB-SC signal having carrier
'
c

frequency 1. MHz. Thus 2fc' = 1 MHz or fc' = 0.5 MHz fc = 1000 kHz
1 The frequency of message signal is
8.110 Option (D) is correct.
2 fm = 1 = 10 kHz
PT = Pc c1 + a m 100 # 10 - 6
2
Here message signal is symmetrical square wave whose FS has only
2 P (0.5) 2
Psb = Pc a = c odd harmonics i.e. 10 kHz, 30 kHz 50 kHz. Modulated signal con-
2 2
Psb = 1 tain fc ! fm frequency component. Thus modulated signal has
or
Pc 8 fc ! fm = (1000 ! 10) kH = 1010 kHz, 990 kHz
8.111 Option (D) is correct.
fc ! 3fm = (1000 ! 10) kH = 1030 kHz, 970 kHz
AM Band width = 2fm
Peak frequency deviation = 3 (2fm) = 6fm SPECIAL EDITION ( STUDY MATERIAL FORM )
6f
Modulation index b = m = 6 At market Book is available in 3 volume i.e. in 3 book binding
fm form. But at NODIA Online Store book is available in 10 book
The FM signal is represented in terms of Bessel function as binding form. Each unit of Book is in separate binding.
3
Available Only at NODIA Online Store
xFM (t) = Ac /Jn (b) cos (wc - nwn) t
n =- 3 Click to Buy
3
wc + nwm = 2p (1008 # 10 )
www.nodia.co.in
2p10 + n4p # 103 = 2p (1008 # 103), n = 4
6

Thus coefficient = 5J4 (6) Thus, there is no 1020 kHz component in modulated signal.
8.112 Option (B) is correct. 8.119 Option (C) is correct.
+3
Ring modulation $ Generation of DSB - SC
We have y (t) = 5 # 10 - 6 x (t) / d (t - nTs)
VCO $ Generation of FM n =- 3

Foster seely discriminator $ Demodulation of fm x (t) = 10 cos (8p # 103) t


mixer $ frequency conversion Ts = 100m sec
8.113 Option (A) is correct. The cut off fc of LPF is 5 kHz
fmax = 1650 + 450 = 2100 kHz We know that for the output of filter
x (t) y (t)
fmin = 550 + 450 = 1000 kHz =
Ts
or f = 1
2p LC 10 cos (8p # 103) t # 5 # 10 - 6
=
frequency is minimum, capacitance will be maximum 100 # 10 - 6
f2 = 5 # 10 - 1 cos (8p # 103) t
R = Cmax = max = (2.1) 2
Cmin 2
fmin 8.120 Option (C) is correct.
or R = 4.41 Transmitted frequencies in coherent BFSK should be integral of bit
fi = fc + 2fIF = 700 + 2 (455) = 1600 kHz rate 8 kHz.
8.114 Option (D) is correct. 8.121 Option (B) is correct.
For best reception, if transmitting waves are vertically polarized,
Eb = 10 - 6 watt-sec
then receiver should also be vertically polarized i.e. transmitter and
No = 10 - 5 W/Hz receiver must be in same polarization.
(SNR) matched filler = Eo
= 106 = .05
N 8.122 Option (D) is correct.
2 2 # 10 - 5
o

(SNR)dB = 10 log 10 (0.05) = 13 dB s (t) = cos 2p (2 # 106 t + 30 sin 150t + 40 cos 150t)
= cos {4p106 t + 100p sin (150t + q)}
8.115 Option (B) is correct.
3 fs Angle modulated signal is
For slopeoverload to take place Em $
2pfm s (t) = A cos {wc t + b sin (wm t + q)}
This is satisfied with Em = 1.5 V and fm = 4 kHz Comparing with angle modulated signal we get
8.116 Option (A) is correct. Phase deviations b = 100p
If s " carrier synchronization at receiver Frequency deviations
r " represents bandwidth efficiency 3 f = bfm = 100p # 150 = 7.5 kHz
2p
GATE Electronics and Communication Topicwise Solved Paper by RK Kanodia & Ashish Murolia Page 212

8.126 Option (A) is correct.


8.123 Option (*) is correct. An ideal low - pass filter with appropriate bandwidth fm is used to
We have m (t) s (t) = y1 (t) recover the signal which is sampled at nyquist rate 2fm .
2 sin (2pt) cos (200pt) 8.127 Option (A) is correct.
=
t For any PDF the probability at mean is 1 . Here given PDF is
sin (202pt) - sin (198pt) 2
= Gaussian random variable and X = 4 is mean.
t
8.128 Option (C) is correct.
y1 (t) + n (t)
We require 6 bit for 64 intensity levels because 64 = 26
= y2 (t) = sin 202pt - sin 198pt + sin 199pt Data Rate = Frames per second # pixels per frame # bits per pixel
t t
y2 (t) s (t) = u (t) = 625 # 400 # 400 # 6 = 600 Mbps sec
8.129 Option (C) is correct.
We have
[sin 202pt - sin 198pt + sin 199pt] cos 200pt sin (700pt) sin (500pt)
= sin c (700t) + sin c (500t) = +
t 700pt 500pt
Here the maximum frequency component is 2pfm = 700p i.e.
= 1 [sin (402pt) + sin (2pt) - {sin (398pt) - sin (2pt)} fm = 350 Hz
2
+ sin (399pt) - sin (pt)] Thus Nyquist rate fs = 2fm
= 2 (350) = 700 Hz
GATE Electronics & Communication Thus sampling interval = 1 sec
700
by RK Kanodia
Option (D) is correct.
Now in 3 Volume
8.130

Probability of error = p
Purchase Online at maximum discount from online store
Probability of no error = q = (1 - p)
and get POSTAL and Online Test Series Free
Probability for at most one bit error
visit www.nodia.co.in
= Probability of no bit error
After filtering + probability of 1 bit error
sin (2pt) + sin (2pt) - sin (pt) = (1 - p) n + np (1 - p) n - 1
y (t) =
2t
sin (2pt) + 2 sin (0.5t) cos (1.5pt) 8.131 Option (A) is correct.
= FT
2t If g (t) G (w)
= sin 2 p t + sin 0 .5pt cos 1.5pt
2t t then PSD of g (t) is
8.124 Option (B) is correct. Sg (w) = G (w) 2
The signal frequency is and power is
Pg = 1
3
3
fm = 24p10 = 12 kHz 2p
#- 3Sg (w) dw
2p FT
Now ag (t) aG (w)
Ts = 50m sec " fs = 1 = 1 # 106 = 20
Ts 50 PSD of ag (t) is
kHz
Sag (w) = a (G (w)) 2
After sampling signal will have fs ! fm frequency component i.e. 32 = a2 G (w) 2
and 12 kHz or Sag (w) = a2 Sg (w)
At filter output only 8 kHz will be present as cutoff frequency is For more GATE Resources, Mock Test and
15 kHz.
Study material join the community
8.125 Option (A) is correct. http://www.facebook.com/gateec2014
d (n) = x (n) - x (n - 1)
E [d (n)] 2 = E [x (n) - x (n - 1)] 2 Similarly Pag = a2 Pg
or E [d (n)] 2 8.132 Option (C) is correct.
= E [x (n)] 2 + E [x (n - 1)] 2 - 2E [x (n) x (n - 1)] The envelope of the input signal is [1 + ka m (t)] that will be output
or sd2 = sx2 + sx2 - 2Rxx (1) as k = 1 of envelope detector.
2
As we have been given sd2 = sx , therefore 8.133 Option (D) is correct.
10 Frequency Range for satellite communication is 1 GHz to 30 GHz,
sx2 = s2 + s2 - 2R (1) 8.134 Option (B) is correct.
x x xx
10
Waveform will be orthogonal when each bit contains integer number
or 2Rxx (1) = 19 sx2 of cycles of carrier.
10
Rxx = 19 = 0.95 Bit rate Rb = HCF (f1, f2)
or
sx2 20 = HCF (10k, 25k)
GATE Electronics and Communication Topicwise Solved Paper by RK Kanodia & Ashish Murolia Page 213

= 5 kHz 8.141 Option (B) is correct.


Thus bit interval is Tb = 1 = 1 = 0.2 msec = 200 msec Given function
Rb 5k g (t)
8.135 Option (D) is correct. = 6 # 10 sin c (400t) ) 10 sin c3 (100t)
4 2 6

We have Pm = m2 (t) Let g1 (t) = 6 # 10 4 sin c2 (400t)


The input to LPF is g2 (t) = (106) sin c3 (100t)
x (t) = m (t) cos wo t cos (wo t + q) We know that g1 (t) ) g2 (t) ? G1 (w) G2 (w) occupies minimum of
Bandwidth of G1 (w) or G2 (w)
m (t)
= [cos (2wo t + q) + cos q] Band width of G1 (w)
2
m (t) cos (2wo t + q) m (t) cos q = 2 # 400 = 800 rad/ sec or = 400 Hz
= +
2 2 Band width of G2 (w)
The output of filter will be = 3 # 100 = 300 rad/ sec or 150 Hz
m (t) cos q Sampling frequency = 2 # 150 = 300 Hz
y (t) =
2
8.142 Option (B) is correct.
Power of output signal is
For a sinusoidal input SNR (dB) is PCM is obtained by following
2
Py = y2 (t) = 1 m2 (t) cos2 q = Pm cos q formulae.
4 4
8.136 Option (A) is correct. SPECIAL EDITION ( STUDY MATERIAL FORM )
Hilbert transformer always adds - 90c to the positive frequency At market Book is available in 3 volume i.e. in 3 book binding
component and 90c to the negative frequency component. form. But at NODIA Online Store book is available in 10 book
Hilbert Trans form
binding form. Each unit of Book is in separate binding.
cos wt " sin wt Available Only at NODIA Online Store
sin wt " cos wt Click to Buy
Thus cos w1 t + sin w2 t " sin w1 t - cos w2 t
www.nodia.co.in
8.137 Option (A) is correct.
SNR (dB) = 1.8 + 6n n is no. of bits
We have x (t) = Ac cos {wc t + b sin wm t}
Here n =8
y (t) = {x (t)} 3
So, SNR (dB) = 1.8 + 6 # 8 = 49.8

= Ac2 cos (3wc t + 3b sin wm t) + 3 cos (wc t + b sin wm t) 8.143 Option (D) is correct.
We know that matched filter output is given by
Thus the fundamental frequency doesn’t change but BW is three
# g (l) g (T - t + l) dl at
3
times. g 0 (t) = 0
-3
BW = 2 (3 f') = 2 (3 f # 3) = 3 MHz t = T0
Option (C) is correct.
6g 0 (t)@max = # g (l) g (l) dl = #
8.138 3 3
g 2 (t) dt
-3 -3
8.139 Option (C) is correct. 1 # 10-4
This is Quadrature modulated signal. In QAM, two signals having = #
0
[10 sin (2p # 106) 2] dt
bandwidth. B 1 & B 2 can be transmitted simultaneous over a
bandwidth of (B 1 + B 2) Hz [g 0 (t)] max = 1 # 100 # 10-4 = 5 mV
2
so B.W. = (15 + 10) = 25 kHz 8.144 Option (B) is correct.
8.140 Option (B) is correct. Sampling rate must be equal to twice of maximum frequency.
A modulated signal can be expressed in terms of its in-phase and f s = 2 # 400 = 800 Hz
quadrature component as 8.145 Option (C) is correct.
S (t) The amplitude spectrum of a gaussian pulse is also gaussian as
= S1 (t) cos (2pfc t) - SQ (t) sin (2pfc t) shown in the fig.
-y 2
fY (y) = 1 exp c
2 m
Here S (t)
-at at
= [e cpsDwt cos wc t - e sin Dwt sin wc t] m (t) 2p

= [e-at cos Dwt] cos 2pfc t - [e-at sin Dwt] sin 2pfc t
= S1 (t) cos 2pfc t - SQ (t) sin 2pfc t
Complex envelope of s (t) is
S (t) = S1 (t) + jSQ (t)
= e-at cos Dwt + je-at sin Dwt
= e-at [cos Dwt + j sin Dwt]
= exp (- at) exp (jDwt) m (t)
8.146 Option (C) is correct.
GATE Electronics and Communication Topicwise Solved Paper by RK Kanodia & Ashish Murolia Page 214

Let the rectangular pulse is given as (SNR) 1 2#8


= 22 # 9 = 22 = 1
(SNR) 2 2 4
so SNR will increased by a factor of 4
8.149 Option (A) is correct.
In flat top sampling an amplitude distortion is produced while
reconstructing original signal x (t) from sampled signal s (t). High
frequency of x (t) are mostly attenuated. This effect is known as
aperture effect.
8.150 Option (A) is correct.
Auto correlation function is given by
T/2 Carrier C (t) = cos (we t + q)
Rxx (t) = 1 x (t) x (t - t) dt
# Modulating signal = x (t)
T -T/2
When x (t) is shifted to right (t > 0), x (t - t) will be shown as dotted DSB - SC modulated signal = x (t) c (t) = x (t) cos (we t + q)
line. envelope = x (t)

8.151 Option (D) is correct.


In Quadrature multiplexing two baseband signals can transmitted
or modulated using I 4 phase & Quadrature carriers and its quite
different form FDM & TDM.
GATE Electronics & Communication 8.152 Option (A) is correct.
by RK Kanodia Fourier transform perform a conversion from time domain to
Now in 3 Volume frequency domain for analysis purposes. Units remain same.

Purchase Online at maximum discount from online store 8.153 Option (A) is correct.
In PCM, SNR is depends an step size (i.e. signal amplitude) SNR
and get POSTAL and Online Test Series Free
can be improved by using smaller steps for smaller amplitude. This
visit www.nodia.co.in is obtained by compressing the signal.
8.154 Option (C) is correct.
Band width is same for BPSK and APSK(OOK) which is equal to
twice of signal Bandwidth.
8.155 Option (A) is correct.
The spectral density of a real value random process symmetric about
vertical axis so it has an even symmetry.
8.156 Option (A) is correct.

T
8.157 Option (C) is correct.
Rxx (t) = 1
+t

# It is one of the advantage of bipolar signalling (AMI) that its


2

A2 dt
T T
- +t
spectrum has a dc null for binary data transmission PSD of bipolar
2
2 2
= A :T + T - tD = A :T - tD signalling is
T 2 2 T 2
(t) can be negative or positive, so generalizing above equations
2
Rxx (t) = A :T - t D
T 2
Rxx (t) is a regular pulse of duration T .
For more GATE Resources, Mock Test and
Study material join the community
http://www.facebook.com/gateec2014

8.147 Option (B) is correct.


Selectivity refers to select a desired frequency while rejecting all
others. In super heterodyne receiver selective is obtained partially
by RF amplifier and mainly by IF amplifier.
8.148 Option (C) is correct.
8.158 Option (A) is correct.
In PCM, SNR a 22n
Probability Density function (PDF) of a random variable x defined
so if bit increased from 8 to 9
as
GATE Electronics and Communication Topicwise Solved Paper by RK Kanodia & Ashish Murolia Page 215

Px (x) = 1 e-x /2
2
factor of 22 (n + 1)/n
2p 8.165 Option (D) is correct.
so here K = 1
The auto correlation of energy signal is an even function.
2p
auto correlation function is gives as
8.159 Option (C) is correct. 3
Here the highest frequency component in the spectrum is 1.5 kHz R (t) = # x (t) x (t + t) dt
-3
[at 2 kHz is not included in the spectrum] 3

Minimum sampling freq. = 1.5 # 2 = 3 kHz put R (- t) = # x (t) x (t - t) dt


-3
8.160 Option (B) is correct. Let t-t = a
We need a high pass filter for receiving the pulses. dt = da
3
8.161 Option (D) is correct. R (- t) = # x (a + t) x (a) da
Power spectral density function of a signal g (t) is fourier transform -3

of its auto correlation function change variable a " t


F 3
Rg (t) Sg (w) R (- t) = # x (t) x (t + t) dt = R (t)
here Sg (w) = sin c2 (f) -3

so Rg (t) is a triangular pulse. R (- t) = R (t) even function


f [triang.] = sin c2 (f) 8.166 Option (D) is correct.
8.162 Option (C) is correct. SPECIAL EDITION ( STUDY MATERIAL FORM )
For a signal g (t), its matched filter response given as At market Book is available in 3 volume i.e. in 3 book binding
h (t) = g (T - t) form. But at NODIA Online Store book is available in 10 book
so here g (t) is a rectangular pulse of duration T .
binding form. Each unit of Book is in separate binding.
Available Only at NODIA Online Store

Click to Buy
www.nodia.co.in

output of matched filter


y (t) = g (t) ) h (t)

if we shift g (- t) for convolution y (t) increases first linearly then


decreases to zero.

8.163 Option (C) is correct.


The difference between incoming signal frequency (fc) and its image
frequency (fc) is 2I f (which is large enough). The RF filter may
provide poor selectivity against adjacent channels separated by a
small frequency differences but it can provide reasonable selectivity
against a station separated by 2I f . So it provides adequate suppression
of image channel.
8.164 Option (C) is correct.
In PCM SNR is given by
SNR = 3 22n
2
if no. of bits is increased from n to (n + 1) SNR will increase by a
GATE Electronics and Communication Topicwise Solved Paper by RK Kanodia & Ashish Murolia Page 216

UNIT 9 p # 10 ^x + z h
(C) 45c and E 0 ^atx - atz h e-j 3 2
2
4

V/m

ELECTROMAGNETICS
(D) 60c and E 0 ^atx - atz h e-j 3 V/m
4
p # 10 z

2
9.5 The expression for Evr is
p # 10 ^x - z h
4

(A) 0.23 E 0 ^atx + atz h e-j 3 2 V/m


2013 ONE MARK
2

v ^rvh. The closed loop line integral A (B) - E 0 ^atx + atz h e j 3 V/m
4
p # 10 z
9.1 Consider a vector field A v : dlv# 2
can be expressed as
(A) ^d # A
## vh : dsv over the closed surface bounded by the loop p # 10 ^x - z h
4

(C) 0.44 E 0 ^atx + atz h e-j 3 2 V/m


(B) ^d : A
### vh dv over the closed volume bounded by the loop 2
(C) ### ^d : Avhdv over the open volume bounded by the loop (D) E 0 ^atx + atz h e-j
p # 10 ^x + z h
4

V/m
## ^d # Avh : dsv over the open surface bounded by the loop
3
(D) 2

The divergence of the vector field Av = xatx + yaty + zatz is


9.2
2012 ONE MARK
(A) 0 (B) 1/3
9.6 A plane wave propagating in air with
GATE Electronics & Communication E = (8ax + 6ay + 5az ) e j (wt + 3x - 4y) V/m is incident on a perfectly
by RK Kanodia conducting slab positioned at x # 0 . The E field of the reflected
Now in 3 Volume wave is
(A) (- 8ax - 6ay - 5az ) e j (wt + 3x + 4y) V/m
Purchase Online at maximum discount from online store -
(B) (- 8ax + 6ay - 5az ) e j (wt + 3x + 4y) V/m
and get POSTAL and Online Test Series Free
(C) (- 8ax - 6ay - 5az ) e j (wt - 3x - 4y) V/m
visit www.nodia.co.in
(D) (- 8ax + 6ay - 5az ) e j (wt - 3x - 4y) V/m
(C) 1 (D) 3
9.7 The electric field of a uniform plane electromagnetic wave in free
9.3 The return loss of a device is found to be 20 dB. The voltage space, along the positive x direction is given by E = 10 (ay + jaz ) e-j 25x
standing wave ratio (VSWR) and magnitude of reflection coefficient . The frequency and polarization of the wave, respectively, are
are respectively (A) 1.2 GHz and left circular (B) 4 Hz and left circular
(A) 1.22 and 0.1 (B) 0.81 and 0.1 (C) 1.2 GHz and right circular (D) 4 Hz and right circular
(C) – 1.22 and 0.1 (D) 2.44 and 0.2
9.8 A coaxial-cable with an inner diameter of 1 mm and outer diameter
of 2.4 mm is filled with a dielectric of relative permittivity 10.89.
2013 TWO MARKS -9
Given m0 = 4p # 10-7 H/m, e0 = 10 F/m , the characteristic
36p
impedance of the cable is
Statement for Linked Answer Questions 52 and 53: (A) 330 W (B) 100 W
A monochromatic plane wave of wavelength l = 600 mm is propa- (C) 143.3 W (D) 43.4 W
gating in the direction as shown in the figure below. Evi , Evr and
Evt denote incident, reflected, and transmitted electric field vectors 9.9 The radiation pattern of an antenna in spherical co-ordinates is given
associated with the wave. by F (q) = cos 4 q ; 0 # q # p/2 . The directivity of the antenna is

For more GATE Resources, Mock Test and


Study material join the community
http://www.facebook.com/gateec2014
(A) 10 dB (B) 12.6 dB
(C) 11.5 dB (D) 18 dB

2012 TWO MARKS

9.10 A transmission line with a characteristic impedance of 100 W is used


to match a 50 W section to a 200 W section. If the matching is to be
9.4 The angle of incidence qi and the expression for Evi are
p # 10 ^x + 2h
4 done both at 429 MHz and 1 GHz, the length of the transmission
(A) 60c and E 0 ^atx - atz h e-j 3 2 V/m line can be approximately
2
(A) 82.5 cm (b) 1.05 m
(B) 45c and E 0 ^atx + atz h e-j 3 V/m (C) 1.58 cm (D) 1.75 m
4
p # 10 z

2
GATE Electronics and Communication Topicwise Solved Paper by RK Kanodia & Ashish Murolia Page 217

9.11 The magnetic field among the propagation direction inside a (A) 0.8 # 108 m/s (B) 1.2 # 108 m/s
rectangular waveguide with the cross-section shown in the figure is (C) 1.6 # 108 m/s (D) 3 # 108 m/s
Hz = 3 cos (2.094 # 102 x) cos (2.618 # 102 y) cos (6.283 # 1010 t - bz)
9.16 The modes in a rectangular waveguide are denoted by TE mn where
TM mn
m and n are the eigen numbers along the larger and smaller di-
mensions of the waveguide respectively. Which one of the following
statements is TRUE?
(A) The TM 10 mode of the waveguide does not exist
(B) The TE 10 mode of the waveguide does not exist
The phase velocity v p of the wave inside the waveguide satisfies (C) The TM 10 and the TE 10 modes both exist and have the same
(A) v p > c (B) v p = c cut-off frequencies
(C) 0 < v p < c (D) v p = 0 (D) The TM 10 and the TM 01 modes both exist and have the same
cut-off frequencies

Statement for Linked Answer Question 7 and 8 :


2011 TWO MARKS
An infinitely long uniform solid wire of radius a carries a uniform
dc current of density J 9.17 A current sheet Jv = 10uty A/m lies on the dielectric interface x = 0
between two dielectric media with er 1 = 5, mr 1 = 1 in Region-1
9.12 The magnetic field at a distance r from the center of the wire is
proportional to SPECIAL EDITION ( STUDY MATERIAL FORM )
(A) r for r < a and 1/r 2 for r > a (B) 0 for r < a and 1/r for r > a At market Book is available in 3 volume i.e. in 3 book binding
(C) r for r < a and 1/r for r > a (D) 0 for r < a and 1/r 2 for r > a form. But at NODIA Online Store book is available in 10 book
9.13 A hole of radius b (b < a) is now drilled along the length of the wire binding form. Each unit of Book is in separate binding.
Available Only at NODIA Online Store
at a distance d from the center of the wire as shown below.
Click to Buy
www.nodia.co.in
(x < 0) and er2 = 2, mr2 = 2 in Region-2 (x 2 0). If the magnetic field
in Region-1 at x = 0- is Hv1 = 3utx + 30uty A/m the magnetic field in
Region-2 at x = 0+ is

The magnetic field inside the hole is


(A) uniform and depends only on d
(B) uniform and depends only on b (A) Hv2 = 1.5utx + 30uty - 10utz A/m
(C) uniform and depends on both b and d (B) Hv2 = 3utx + 30uty - 10utz A/m
(D) non uniform
(C) Hv2 = 1.5utx + 40uty A/m
(D) Hv2 = 3utx + 30uty + 10utz A/m
2011 ONE MARK
9.18 A transmission line of characteristic impedance 50 W is terminated
9.14 Consider the following statements regarding the complex Poynting in a load impedance ZL . The VSWR of the line is measured as 5 and
vector Pv for the power radiated by a point source in an infinite the first of the voltage maxima in the line is observed at a distance
homogeneous and lossless medium. Re(Pv ) denotes the real part of of l/4 from the load. The value of ZL is
Pv, S denotes a spherical surface whose centre is at the point source, (A) 10 W (B) 250 W
and nt denotes the unit surface normal on S . Which of the following (C) (19.23 + j 46.15) W (D) (19.23 - j 46.15) W
statements is TRUE?
(A) Re(Pv ) remains constant at any radial distance from the source 9.19 The electric and magnetic fields for a TEM wave of frequency 14
(B) Re(Pv ) increases with increasing radial distance from the GHz in a homogeneous medium of relative permittivity er and
source relative permeability mr = 1 are given by Ev = E p e j (wt - 280py) utz V/m
and Hv = 3e j (wt - 280py) utx A/m . Assuming the speed of light in free space
(C) ##s Re (Pv) : nt dS remains constant at any radial distance from
to be 3 # 108 m/s , the intrinsic impedance of free space to be 120p
the source
, the relative permittivity er of the medium and the electric field
(D) ##s Re (Pv) : nt dS decreases with increasing radial distance from amplitude E p are
the source (A) er = 3, E p = 120p (B) er = 3, E p = 360p
(C) er = 9, E p = 360p (D) er = 9, E p = 120p
9.15 A transmission line of characteristic impedance 50 W is terminated
by a 50 W load. When excited by a sinusoidal voltage source at 10
GHz, the phase difference between two points spaced 2 mm apart on 2010 ONE MARK
the line is found to be p/4 radians. The phase velocity of the wave
9.20 If the scattering matrix [S ] of a two port network is
along the line is
GATE Electronics and Communication Topicwise Solved Paper by RK Kanodia & Ashish Murolia Page 218

0.2+0c 0.9+90c
[S ] = >
0.9+90c 0.1+90cH
, then the network is
(A) lossless and reciprocal (B) lossless but not reciprocal
(C) not lossless but reciprocal (D) neither lossless nor reciprocal

9.21 A transmission line has a characteristic impedance of 50 W and a


resistance of 0.1 W/m . If the line is distortion less, the attenuation
constant(in Np/m) is
(A) 500 (B) 5 (A) 1.00 (B) 1.64
(C) 0.014 (D) 0.002 (C) 2.50 (D) 3.00

9.22 The electric field component of a time harmonic plane EM wave


traveling in a nonmagnetic lossless dielectric medium has an 2009 ONE MARK
amplitude of 1 V/m. If the relative permittivity of the medium is 4, 9.26 Two infinitely long wires carrying current are as shown in the figure
the magnitude of the time-average power density vector (in W/m2 below. One wire is in the y - z plane and parallel to the y - axis.
) is The other wire is in the x - y plane and parallel to the x - axis.
(A) 1 (B) 1 Which components of the resulting magnetic field are non-zero at
30p 60p
the origin ?
(C) 1 (D) 1
120p 240p

GATE Electronics & Communication


by RK Kanodia
Now in 3 Volume
Purchase Online at maximum discount from online store
and get POSTAL and Online Test Series Free
visit www.nodia.co.in
2010 TWO MARKS
v = xyatx + x 2 aty , then (A) x, y, z components (B) x, y components
9.23 If A # Av $ dlv over the path shown in the figure
o
is
C
(C) y, z components (D) x, z components

9.27 Which of the following statements is true regarding the fundamental


mode of the metallic waveguides shown ?

(A) Only P has no cutoff-frequency


(B) Only Q has no cutoff-frequency
(C) Only R has no cutoff-frequency
(A) 0 (B) 2
3 (D) All three have cutoff-frequencies
(C) 1 (D) 2 3

A plane wave having the electric field components


For more GATE Resources, Mock Test and
9.24

Evi = 24 cos ^3 # 108 - by h atx V/m and traveling in free space is


incident normally on a lossless medium with m = m0 and e = 9e0 Study material join the community
which occupies the region y $ 0 . The reflected magnetic field http://www.facebook.com/gateec2014
component is given by
2009 TWO MARKS
(A) 1 cos (3 # 108 t + y) atx A/m
10p
9.28 If a vector field V is related to another vector field A through
(B) 1 cos (3 # 108 t + y) atx A/m V = 4# A , which of the following is true? (Note : C and SC refer to
20p
any closed contour and any surface whose boundary is C . )
(C) - 1 cos (3 # 108 t + y) atx A/m
20p #C
(A) V $ dl = #S #C
A $ dS #C
(B) A $ dl = #S #C
V $ dS
(D) - 1 cos (3 # 108 t + y) atx A/m
10p (C) #C D # V $ dl = #S #C D # A $ d S (D) #C D # V $ dl = #S #CV $ d S
In the circuit shown, all the transmission line sections are lossless.
A transmission line terminates in two branches, each of length l ,
9.25
9.29
The Voltage Standing Wave Ration(VSWR) on the 60 W line is 4
as shown. The branches are terminated by 50W loads. The lines are
lossless and have the characteristic impedances shown. Determine
the impedance Zi as seen by the source.
GATE Electronics and Communication Topicwise Solved Paper by RK Kanodia & Ashish Murolia Page 219

(C) 35 dB (D) 45 dB

2007 ONE MARK

9.38 A plane wave of wavelength l is traveling in a direction making an


angle 30c with positive x - axis and 90c with positive y - axis. The
E field of the plane wave can be represented as (E0 is constant)
(A) 200W (B) 100W 3p p p 3p
(A) E = yEt 0 e j c wt - l x - l z m t 0 e jc wt - l x - l z m
(B) E = yE
(C) 50W (D) 25W 3 p x+ p z p 3pz
t 0 e jc wt +
(C) E = yE l l m t 0 e jc wt - l x +
(D) E = yE l m
9.30 A magnetic field in air is measured to be
y If C is code curve enclosing a surface S , then magnetic field intensity
B = B0 c 2 x 2 yt - 2 xt m
9.39

x +y x + y2 H , the current density j and the electric flux density D are related
What current distribution leads to this field ? by
c j + 2t m $ d t
[Hint : The algebra is trivial in cylindrical coordinates.] 2D
t t (A) ## H $ ds = ##
(A) J = B0 z c 2 1 2 m, r ! 0 (B) J =- B0 z c 2 2 2 m, r ! 0 S c
m0 x + y m0 x + y
t (B) #S H $ d l = ##S c j + 22Dt m $ dS
(C) J = 0, r ! 0 (D) J = B0 z c 2 1 2 m, r ! 0
m0 x + y
(C) ##S H $ dS = #C c j + 22Dt m $ d t
2008 ONE MARK SPECIAL EDITION ( STUDY MATERIAL FORM )
9.31 For a Hertz dipole antenna, the half power beam width (HPBW) in At market Book is available in 3 volume i.e. in 3 book binding
the E -plane is form. But at NODIA Online Store book is available in 10 book
(A) 360c (B) 180c binding form. Each unit of Book is in separate binding.
Available Only at NODIA Online Store
(C) 90c (D) 45c
Click to Buy
For static electric and magnetic fields in an inhomogeneous source-
www.nodia.co.in
9.32

free medium, which of the following represents the correct form of


Maxwell’s equations ?
(A) 4$ E = 0 , 4# B = 0 (B) 4$ E = 0 , 4$ B = 0 (D) #C H $ d l # = ##S c j + 22Dt m $ ds
c
(C) 4# E = 0 , 4# B = 0 (D) 4# E = 0 , 4$ B = 0
2007 TWO MARKS

2008 TWO MARKS 9.40 The E field in a rectangular waveguide of inner dimension a # b is
9.33 A rectangular waveguide of internal dimensions (a = 4 cm and given by
wm
E = 2 ` l j H0 sin ` 2px j sin (wt - bz) yt
2
b = 3 cm) is to be operated in TE11 mode. The minimum operating
frequency is h 2 a
(A) 6.25 GHz (B) 6.0 GHz Where H0 is a constant, and a and b are the dimensions along the
(C) 5.0 GHz (D) 3.75 GHz x - axis and the y - axis respectively. The mode of propagation in
the waveguide is
9.34 One end of a loss-less transmission line having the characteristic
(A) TE20 (B) TM11
impedance of 75W and length of 1 cm is short-circuited. At 3 GHz,
the input impedance at the other end of transmission line is (C) TM20 (D) TE10
(A) 0 (B) Resistive 9.41 A load of 50 W is connected in shunt in a 2-wire transmission line of
(C) Capacitive (D) Inductive Z0 = 50W as shown in the figure. The 2-port scattering parameter
matrix (s-matrix) of the shunt element is
9.35 A uniform plane wave in the free space is normally incident on
an infinitely thick dielectric slab (dielectric constant e = 9 ). The
magnitude of the reflection coefficient is
(A) 0 (B) 0.3
(C) 0.5 (D) 0.8

(A) > H
In the design of a single mode step index optical fibre close to upper - 12 1
0 1
(B) =
1 0G
9.36
2
cut-off, the single-mode operation is not preserved if 1
2 - 1
2
(A) radius as well as operating wavelength are halved
(C) > H (D) > H
(B) radius as well as operating wavelength are doubled - 13 2
3
1
4 - 43
2
- 1
- 3 1
(C) radius is halved and operating wavelength is doubled 3 3 4 4

(D) radius is doubled and operating wavelength is halved 9.42 The parallel branches of a 2-wirw transmission line re terminated in
100W and 200W resistors as shown in the figure. The characteristic
9.37 At 20 GHz, the gain of a parabolic dish antenna of 1 meter and 70%
impedance of the line is Z0 = 50W and each section has a length of
efficiency is l . The voltage reflection coefficient G at the input is
(A) 15 dB (B) 25 dB 4
GATE Electronics and Communication Topicwise Solved Paper by RK Kanodia & Ashish Murolia Page 220

(C) 2 (D) 3

2006 ONE MARK

9.47 The electric field of an electromagnetic wave propagation in the positive


direction is given by E = atx sin (wt - bz) + aty sin (wt - bz + p/2). The
wave is
(A) Linearly polarized in the z -direction
(B) Elliptically polarized
(C) Left-hand circularly polarized
(D) Right-hand circularly polarized
(A) - j 7 (B) - 5
5 7
9.48 A transmission line is feeding 1 watt of power to a horn antenna
(C) j 5 (D) 5 having a gain of 10 dB. The antenna is matched to the transmission
7 7
line. The total power radiated by the horn antenna into the free
9.43 The H field (in A/m) of a plane wave propagating in free space is
space is
given by H = xt 5 3 cos (wt - bz) + yt`wt - bz + p j . (A) 10 Watts (B) 1 Watts
h0 2
The time average power flow density in Watts is (C) 0.1 Watts (D) 0.01 Watt
h
(A) 0 (B) 100
100 h0
2006 TWO MARKS
GATE Electronics & Communication
by RK Kanodia 9.49 When a planes wave traveling in free-space is incident normally on a
medium having the fraction of power transmitted into the medium
Now in 3 Volume is given by
Purchase Online at maximum discount from online store (A) 8 (B) 1
and get POSTAL and Online Test Series Free 9 2

visit www.nodia.co.in (C) 1 (D) 5


3 6
9.50 A medium of relative permittivity er2 = 2 forms an interface with
(C) 50h20 (D) 50 free - space. A point source of electromagnetic energy is located in
h0
the medium at a depth of 1 meter from the interface. Due to the
9.44 An air-filled rectangular waveguide has inner dimensions of 3 cm total internal reflection, the transmitted beam has a circular cross-
# 2 cm. The wave impedance of the TE20 mode of propagation in section over the interface. The area of the beam cross-section at the
the waveguide at a frequency of 30 GHz is (free space impedance interface is given by
h0 = 377 W ) (A) 2p m 2 (B) p2 m 2
(A) 308 W (B) 355 W (C) p m 2 (D) p m 2
2
(C) 400 W (D) 461 W
9.51 A rectangular wave guide having TE10 mode as dominant mode is
9.45 A l dipole is kept horizontally at a height of l0 above a perfectly having a cut off frequency 18 GHz for the mode TE30 . The inner
2 2
conducting infinite ground plane. The radiation pattern in the lane broad - wall dimension of the rectangular wave guide is
of the dipole (E plane) looks approximately as
(A) 5 cm (B) 5 cm
3
(C) 5 cm (D) 10 cm
2

For more GATE Resources, Mock Test and


Study material join the community
http://www.facebook.com/gateec2014
9.52 A medium is divide into regions I and II about x = 0 plane, as
shown in the figure below.
9.46 A right circularly polarized (RCP) plane wave is incident at an angle
60c to the normal, on an air-dielectric interface. If the reflected wave
is linearly polarized, the relative dielectric constant xr2 is.

An electromagnetic wave with electric field E1 = 4atx + 3aty + 5atz is


incident normally on the interface from region I . The electric file
E2 in region II at the interface is
(A) 2 (B) 3 (A) E2 = E1 (B) 4atx + 0.75aty - 1.25atz
GATE Electronics and Communication Topicwise Solved Paper by RK Kanodia & Ashish Murolia Page 221

(C) 3atx + 3aty + 5atz (D) - 3atx + 3aty + 5atz Statement of Linked Answer Questions 9.46 & 9.47 :
9.53 A mast antenna consisting of a 50 meter long vertical conductor Voltage standing wave pattern in a lossless transmission line with
operates over a perfectly conducting ground plane. It is base-fed at characteristic impedance 50 and a resistive load is shown in the
a frequency of 600 kHz. The radiation resistance of the antenna is figure.
Ohms is
2 2
(A) 2p (B) p
5 5
2
(C) 4p (D) 20p2
5

2005 ONE MARK


9.59 The value of the load resistance is
9.54 The magnetic field intensity vector of a plane wave is given by
(A) 50 W (B) 200 W
H (x, y, z, t) = 10 sin (50000t + 0.004x + 30) aty
(C) 12.5 W (D) 0
where aty , denotes the unit vector in y direction. The wave is
propagating with a phase velocity. 9.60 The reflection coefficient is given by
(A) 5 # 10 4 m/s (B) - 3 # 108 m/s (A) - 0.6 (B) - 1
(C) - 1.25 # 107 m/s (D) 3 # 108 m/s (C) 0.6 (D) 0

9.55 Refractive index of glass is 1.5. Find the wavelength of a beam of SPECIAL EDITION ( STUDY MATERIAL FORM )
light with frequency of 1014 Hz in glass. Assume velocity of light is At market Book is available in 3 volume i.e. in 3 book binding
3 # 108 m/s in vacuum form. But at NODIA Online Store book is available in 10 book
(A) 3 mm (B) 3 mm binding form. Each unit of Book is in separate binding.
(C) 2 mm (D) 1 mm Available Only at NODIA Online Store

Click to Buy
2005 TWO MARKS www.nodia.co.in
9.56 Which one of the following does represent the electric field lines 9.61 Many circles are drawn in a Smith Chart used for transmission line
for the mode in the cross-section of a hollow rectangular metallic
calculations. The circles shown in the figure represent
waveguide ?

(A) Unit circles


(B) Constant resistance circles
(C) Constant reactance circles
9.57 Characteristic impedance of a transmission line is 50 W. Input
impedance of the open-circuited line when the transmission line a (D) Constant reflection coefficient circles.
short circuited, then value of the input impedance will be.
(A) 50 W (B) 100 + j150W 2004 ONE MARK
(C) 7.69 + j11.54W (D) 7.69 - j11.54W
9.62 The phase velocity of an electromagnetic wave propagating in a
9.58 Two identical and parallel dipole antennas are kept apart by a hollow metallic rectangular waveguide in the TE10 mode is
distance of l in the H - plane. They are fed with equal currents (A) equal to its group velocity
4
but the right most antenna has a phase shift of + 90c. The radiation (B) less than the velocity of light in free space
pattern is given as. (C) equal to the velocity of light in free space
(D) greater than the velocity of light in free space

9.63 Consider a lossless antenna with a directive gain of + 6 dB. If 1 mW


of power is fed to it the total power radiated by the antenna will be
(A) 4 mW (B) 1 mW
(C) 7 mW (D) 1/4 mW

2004 TWO MARKS

9.64 A parallel plate air-filled capacitor has plate area of 10 - 4 m 2 and


plate separation of 10 - 3 m. It is connect - ed to a 0.5 V, 3.6 GHz
GATE Electronics and Communication Topicwise Solved Paper by RK Kanodia & Ashish Murolia Page 222

source. The magnitude of the displacement current is ( e = 1


36p 10 - 9 the medium should be
F/m) (A) 120p W (B) 60p W
(A) 10 mA (B) 100 mA (C) 600p W (D) 24p W
(C) 10 A (D) 1.59 mA
9.70 A lossless transmission line is terminated in a load which reflects a
9.65 Consider a 300 W, quarter - wave long (at 1 GHz) transmission line part of the incident power. The measured VSWR is 2. The percentage
as shown in Fig. It is connected to a 10 V, 50 W source at one end of the power that is reflected back is
and is left open circuited at the other end. The magnitude of the (A) 57.73 (B) 33.33
voltage at the open circuit end of the line is (C) 0.11 (D) 11.11

2003 ONE MARK

9.71 The unit of 4# H is


(A) Ampere (B) Ampere/meter
(C) Ampere/meter 2 (D) Ampere-meter
(A) 10 V (B) 5 V
(C) 60 V (D) 60/7 V 9.72 The depth of penetration of electromagnetic wave in a medium
having conductivity s at a frequency of 1 MHz is 25 cm. The depth
9.66 In a microwave test bench, why is the microwave signal amplitude of penetration at a frequency of 4 MHz will be
modulated at 1 kHz (A) 6.25 dm (B) 12.50 cm
(C) 50.00 cm (D) 100.00 cm
GATE Electronics & Communication
by RK Kanodia
Now in 3 Volume 2003 TWO MARKS

Purchase Online at maximum discount from online store 9.73 Medium 1 has the electrical permittivity e1 = 1.5e0 farad/m and
and get POSTAL and Online Test Series Free occupies the region to the left of x = 0 plane. Medium 2 has the
electrical permittivity e2 = 2.5e0 farad/m and occupies the region to
visit www.nodia.co.in the right of x = 0 plane. If E1 in medium 1 is E1 = (2ux - 3uy + 1uz )
volt/m, then E2 in medium 2 is
(A) To increase the sensitivity of measurement
(A) (2.0ux - 7.5uy + 2.5uz ) volt/m
(B) To transmit the signal to a far-off place
(B) (2.0ux - 2.0uy + 0.6uz ) volt/m
(C) To study amplitude modulations
(C) (2.0ux - 3.0uy + 1.0uz ) volt/m
(D) Because crystal detector fails at microwave frequencies
(D) (2.0ux - 2.0uy + 0.6uz ) volt/m
9.67 If E = (atx + jaty) e jkz - kwt and H = (k/wm) (aty + katx ) e jkz - jwt , the time- 9.74 If the electric field intensity is given by E = (xux + yuy + zuz ) volt/m,
averaged Poynting vector is the potential difference between X (2, 0, 0) and Y (1, 2, 3) is
(A) null vector (B) (k/wm) atz (A) + 1 volt (B) - 1 volt
(C) (2k/wm) atz (D) (k/2wm) atz (C) + 5 volt (D) + 6 volt
9.68 Consider an impedance Z = R + jX marked with point P in an 9.75 A uniform plane wave traveling in air is incident on the plane
impedance Smith chart as shown in Fig. The movement from point boundary between air and another dielectric medium with er = 4 .
P along a constant resistance circle in the clockwise direction by an The reflection coefficient for the normal incidence, is
angle 45c is equivalent to (A) zero (B) 0.5+180c
(B) 0.333+0c (D) 0.333+180c

For more GATE Resources, Mock Test and


Study material join the community
http://www.facebook.com/gateec2014
9.76 If the electric field intensity associated with a uniform plane
electromagnetic wave traveling in a perfect dielectric medium is
given by E (z, t) = 10 cos (2p107 t - 0.1pz) V/m, then the velocity of
(A) adding an inductance in series with Z the traveling wave is
(B) adding a capacitance in series with Z (A) 3.00 # 108 m/sec (B) 2.00 # 108 m/sec
(C) adding an inductance in shunt across Z (C) 6.28 # 107 m/sec (D) 2.00 # 107 m/sec
(D) adding a capacitance in shunt across Z
9.77 A short - circuited stub is shunt connected to a transmission line as
9.69 A plane electromagnetic wave propagating in free space is incident shown in fig. If Z0 = 50 ohm, the admittance Y seen at the junction
normally on a large slab of loss-less, non-magnetic, dielectric material of the stub and the transmission line is
with e > e0 . Maxima and minima are observed when the electric
field is measured in front of the slab. The maximum electric field is
found to be 5 times the minimum field. The intrinsic impedance of
GATE Electronics and Communication Topicwise Solved Paper by RK Kanodia & Ashish Murolia Page 223

(C) elliptically polarized (D) unpolarized

9.84 Distilled water at 25c C is characterized by s = 1.7 # 10 - 4 mho/m


and e = 78eo at a frequency of 3 GHz. Its loss tangent tan d is
( e = 10
-9

36p F/m)

(A) 1.3 # 10-5 (B) 1.3 # 10-3


(C) 1.3 # 10-4 /78 (D) 1.3 # 10-5 /78e0

9.85 The electric field on the surface of a perfect conductor is 2 V/m.


The conductor is immersed in water with e = 80eo . The surface
charge density on the conductor is ( e = 10
-9

36p F/m)
(A) 0 C/m 2 (B) 2 C/m 2
(A) (0.01 - j0.02) mho (B) (0.02 - j0.01) mho (C) 1.8 # 10 - 11 C/m 2 (D) 1.41 # 10 - 9 C/m 2
(C) (0.04 - j0.02) mho (D) (0.02 + j0) mho 9.86 A person with receiver is 5 Km away from the transmitter. What
is the distance that this person must move further to detect a 3-dB
9.78 A rectangular metal wave guide filled with a dielectric material of
decrease in signal strength
relative permittivity er = 4 has the inside dimensions 3.0 cm # 1.2
(A) 942 m (B) 2070 m
cm. The cut-off frequency for the dominant mode is
(A) 2.5 GHz (B) 5.0 GHz (C) 4978 m (D) 5320 m
(C) 10.0 GHz (D) 12.5 GHz SPECIAL EDITION ( STUDY MATERIAL FORM )
At market Book is available in 3 volume i.e. in 3 book binding
9.79 Two identical antennas are placed in the q = p/2 plane as shown form. But at NODIA Online Store book is available in 10 book
in Fig. The elements have equal amplitude excitation with 180c
polarity difference, operating at wavelength l. The correct value
binding form. Each unit of Book is in separate binding.
Available Only at NODIA Online Store
of the magnitude of the far-zone resultant electric field strength
normalized with that of a single element, both computed for f = 0 Click to Buy
, is www.nodia.co.in
2001 ONE MARK

9.87 A transmission line is distortonless if


(A) RL = 1 (B) RL = GC
GC
(C) LG = RC (D) RG = LC
(A) 2 cos b 2ps l (B) 2 sin b 2ps l 2 2
l l 9.88 If a plane electromagnetic wave satisfies the equal d E2x = c2 d E2x ,
dZ dt
(C) 2 cos a ps k (D) 2 sin a ps k the wave propagates in the
l l (A) x - direction
(B) z - direction
2002 ONE MARK
(C) y - direction
9.80 The VSWR can have any value between (D) xy plane at an angle of 45c between the x and z direction
(A) 0 and 1 (B) - 1 and + 1
(C) 0 and 3 (D) 1 and 3 9.89 The plane velocity of wave propagating in a hollow metal waveguide
is
9.81 In in impedance Smith movement along a constant resistance circle (A) grater than the velocity of light in free space
gives rise to (B) less than the velocity of light in free space
(A) a decrease in the value of reactance
(C) equal to the velocity of light free space
(B) an increase in the value of reactance
(D) equal to the velocity of light in free
(C) no change in the reactance value
(D) no change in the impedance 9.90 The dominant mode in a rectangular waveguide is TE10 , because this
mode has
9.82 The phase velocity for the TE10 -mode in an air-filled rectangular (A) the highest cut-off wavelength
waveguide is (c is the velocity of plane waves in free space) (B) no cut-off
(A) less than c (B) equal to c
(C) no magnetic field component
(C) greater than c (D) none of these
(D) no attenuation

2002 TWO MARKS


2001 TWO MARKS
jp/2 jwt - jkz
9.83 A plane wave is characterized by E = (0.5xt + ye
t )e . This 9.91 A material has conductivity of 10 - 2 mho/m and a relative permittivity
wave is of 4. The frequency at which the conduction current in the medium
(A) linearly polarized (B) circularly polarized is equal to the displacement current is
GATE Electronics and Communication Topicwise Solved Paper by RK Kanodia & Ashish Murolia Page 224

(A) 45 MHz (B) 90 MHz (l1 /l2) is


(C) 450 MHz (D) 900 MHz (A) er1 /er2 (B) er2 /er1
(C) er1 /er2 (D) er2 /er1
9.92 A uniform plane electromagnetic wave incident on a plane surface
of a dielectric material is reflected with a VSWR of 3. What is the 9.101 For an 8 feet (2.4m) parabolic dish antenna operating at 4 GHz, the
percentage of incident power that is reflected ? minimum distance required for far field measurement is closest to
(A) 10% (B) 25% (A) 7.5 cm (B) 15 cm
(C) 50% (D) 75% (C) 15 m (D) 150 m

9.93 A medium wave radio transmitter operating at a wavelength of


492 m has a tower antenna of height 124. What is the radiation 1999 ONE MARK
resistance of the antenna? 9.102 An electric field on a place is described by its potential
(A) 25 W (B) 36.5 W
V = 20 (r-1 + r-2)
(C) 50 W (D) 73 W
where r is the distance from the source. The field is due to
9.94 In uniform linear array, four isotropic radiating elements are spaced (A) a monopole (B) a dipole
l apart. The progressive phase shift between required for forming (C) both a monopole and a dipole (D) a quadruple
4
the main beam at 60c off the end - fire is : 9.103 Assuming perfect conductors of a transmission line, pure TEM
(A) - p (B) - p2 radians
propagation is NOT possible in
(C) - p4 radians (D) - p8 radians (A) coaxial cable
GATE Electronics & Communication (B) air-filled cylindrical waveguide
by RK Kanodia (C) parallel twin-wire line in air
Now in 3 Volume (D) semi-infinite parallel plate wave guide

Purchase Online at maximum discount from online store 9.104 Indicate which one of the following will NOT exist in a rectangular
and get POSTAL and Online Test Series Free resonant cavity.
(A) TE110 (B) TE 011
visit www.nodia.co.in
(C) TM110 (D) TM111
2000 ONE MARK

9.95 The magnitudes of the open-circuit and short-circuit input


9.105 Identify which one of the following will NOT satisfy the wave
impedances of a transmission line are 100 W and 25 W respectively. equation.
The characteristic impedance of the line is, (A) 50e j (wt - 3z) (B) sin [w (10z + 5t)]
2
(A) 25 W (B) 50 W (C) cos (y + 5t) (D) sin (x) cos (t)
(C) 75 W (D) 100 W
1999 TWO MARKS
9.96 A TEM wave is incident normally upon a perfect conductor. The E
and H field at the boundary will be respectively, 9.106 In a twin-wire transmission line in air, the adjacent voltage maxima
(A) minimum and minimum (B) maximum and maximum are at 12.5 cm and 27.5 cm . The operating frequency is
(C) minimum and maximum (D) maximum and minimum (A) 300 MHz (B) 1 GHz
(C) 2 GHz (D) 6.28 GHz
9.97 If the diameter of a l dipole antenna is increased from l to l
2 100 50 A transmitting antenna radiates 251 W isotropically. A receiving
, then its 9.107

antenna, located 100 m away from the transmitting antenna, has an


(A) bandwidth increases (B) bandwidth decrease effective aperture of 500 cm2 . The total received by the antenna is
(C) gain increases (D) gain decreases
For more GATE Resources, Mock Test and
2000 TWO MARKS
Study material join the community
http://www.facebook.com/gateec2014
9.98 A uniform plane wave in air impings at 45c angle on a lossless
dielectric material with dielectric constant dr . The transmitted (A) 10 mW (B) 1 mW
wave propagates is a 30c direction with respect to the normal. The (C) 20 mW (D) 100 mW
value of dr is
(A) 1.5 (B) 1.5 9.108 In air, a lossless transmission line of length 50 cm with L = 10 mH/m
(C) 2 (D) 2 , C = 40 pF/m is operated at 25 MHz . Its electrical path length is
(A) 0.5 meters (B) l meters
9.99 A rectangular waveguide has dimensions 1 cm # 0.5 cm. Its cut-off (C) p/2 radians (D) 180 deg rees
frequency is
(A) 5 GHz (B) 10 GHz 9.109 A plane wave propagating through a medium [er = 8, vr = 2, and s = 0]
(C) 15 GHz (D) 12 GHz has its electric field given by Ev = 0.5Xe
t - (z/3) sin (108 t - bz) V/m . The
wave impedance, in ohms is
9.100 Two coaxial cable 1 and 2 are filled with different dielectric constants (A) 377 (B) 198.5+180c
er1 and er2 respectively. The ratio of the wavelength in the cables (C) 182.9+14c (D) 133.3
GATE Electronics and Communication Topicwise Solved Paper by RK Kanodia & Ashish Murolia Page 225

1998 ONE MARK 9.119 An antenna in free space receives 2 mW of power when the incident
9.110 The intrinsic impedance of copper at high frequencies is electric field is 20 mV/m rms. The effective aperture of the antenna
(A) purely resistive is
(A) 0.005 m2
(B) purely inductive
(B) 0.05 m2
(C) complex with a capacitive component
(C) 1.885 m2
(D) complex with an inductive component
(D) 3.77 m2
9.111 The Maxwell equation V # H = J + 2D is based on
2t 9.120 The maximum usable frequency of an ionospheric layer at 60c
(A) Ampere’s law (B) Gauss’ law incidence and with 8 MHz critical frequency is
(C) Faraday’s law (D) Coulomb’s law (A) 16 MHz
(B) 16 MHz
9.112 All transmission line sections shown in the figure is have a 3
characteristic impedance R 0 + j 0 . The input impedance Zin equals (C) 8 MHz
(D) 6.93 MHz

9.121 A loop is rotating about they y -axis in a magnetic field


Bv = B 0 cos (wt + f) avx T. The voltage in the loop is
(A) zero
SPECIAL EDITION ( STUDY MATERIAL FORM )
At market Book is available in 3 volume i.e. in 3 book binding
form. But at NODIA Online Store book is available in 10 book
binding form. Each unit of Book is in separate binding.
Available Only at NODIA Online Store

Click to Buy
(A) 2 R 0
3
(B) R 0 www.nodia.co.in
(C) 3 R 0 (D) 2R 0 (B) due to rotation only
2
(C) due to transformer action only
1998 TWO MARKS (D) due to both rotation and transformer action
9.113 The time averages Poynting vector, in W/m2 , for a wave with 9.122 The far field of an antenna varies with distance r as
Ev = 24e j (wt + bz) avy V/m in free space is (A) 1 (B) 12
r
(A) - 2.4 avz (B) 2.4 av r
p p z
(C) 13 (D) 1
(C) 4.8 avz (D) - 4.8 avz r r
p p
9.114 The wavelength of a wave with propagation constant (0.1p + j0.2p) m-1 1997 ONE MARK
is
2 m 9.123 A transmission line of 50 W characteristic impedance is terminated
(A) (B) 10 m with a 100 W resistance. The minimum impedance measured on the
0.05
(C) 20 m (D) 30 m line is equal to
(A) 0 W
9.115 The depth of penetration of wave in a lossy dielectric increases with (B) 25 W
increasing (C) 50 W
(A) conductivity (B) permeability
(D) 100 W
(C) wavelength (D) permittivity
9.124 A rectangular air filled waveguide has cross section of 4 cm #10 cm
9.116 The polarization of wave with electric field vector . The minimum frequency which can propagate in the waveguide is
Ev = E 0 e j^wt + bz h ^avx + avy h is (A) 0.75 GHz
(A) linear (B) elliptical
(B) 2.0 GHz
(C) left hand circular (D) right hand circular
(C) 2.5 GHz
9.117 The vector H in the far field of an antenna satisfies (D) 3.0 GHz
(A) d $ Hv = 0 and d # Hv = 0 (B) d $ Hv ! 0 and d # Hv ! 0
9.125 A parabolic dish antenna has a conical beam 2c wide, the directivity
(C) d $ Hv = 0 and d # Hv ! 0 (D) d $ Hv ! 0 and d # Hv = 0
of the antenna is approximately
9.118 The radiation resistance of a circular loop of one turn is 0.01 W. The (A) 20 dB
radiation resistance of five turns of such a loop will be (B) 30 dB
(A) 0.002 W (B) 0.01 W (C) 40 dB
(C) 0.05 W (D) 0.25 W (D) 50 dB
GATE Electronics and Communication Topicwise Solved Paper by RK Kanodia & Ashish Murolia Page 226

1997 TWO MARKS (A) 0%


9.126 A very lossy, l/4 long, 50 W transmission line is open circuited at (B) 4%
the load end. The input impedance measured at the other end of the (C) 20%
line is approximately (D) 100%
(A) 0 (B) 50 W
(C) 3 (D) None of the above 9.133 The critical frequency of an ionospheric layer is 10 MHz. What is
the maximum launching angle from the horizon for which 20 MHz
9.127 The skin depth at 10 MHz for a conductor is 1 cm. The phase velocity wave will be reflected by the layer ?
of an electromagnetic wave in the conductor at 1, 000 MHz is about (A) 0c
(A) 6 # 106 m/ sec (B) 30c
(B) 6 # 107 m/ sec (C) 45c
(C) 3 # 108 m/ sec (D) 90c
(D) 6 # 108 m/ sec
9.134 A 1 km long microwave link uses two antennas each having 30 dB
gain. If the power transmitted by one antenna is 1 W at 3 GHz, the
1996 ONE MARK power received by the other antenna is approximately
9.128 A lossless transmission line having 50 W characteristic impedance (A) 98.6 mW
and length l/4 is short circuited at one end and connected to an (B) 76.8 mW
ideal voltage source of 1 V at the other end. The current drawn from (C) 63.4 mW
(D) 55.2 mW
GATE Electronics & Communication
by RK Kanodia 9.135 Some unknown material has a conductivity of 106 mho/m and a
permeability of 4p # 10-7 H/m . The skin depth for the material at
Now in 3 Volume
1 GHz is
Purchase Online at maximum discount from online store (A) 15.9 mm
and get POSTAL and Online Test Series Free (B) 20.9 mm
visit www.nodia.co.in (C) 25.9 mm
the voltage source is (D) 30.9 mm
(A) 0 (B) 0.02 A
(C) 3 (D) none of these

9.129 The capacitance per unit length and the characteristic impedance of
a lossless transmission line are C and Z 0 respectively. The velocity
of a travelling wave on the transmission line is
(A) Z 0 C (B) 1
Z0 C
(C) Z 0 (D) C
C Z0
9.130 A transverse electromagnetic wave with circular polarization is
received by a dipole antenna. Due to polarization mismatch, the
power transfer efficiency from the wave to the antenna is reduced
to about
(A) 50%
(B) 35.3%
For more GATE Resources, Mock Test and
(C) 25%
Study material join the community
(D) 0%
http://www.facebook.com/gateec2014
9.131 A metal sphere with 1 m radius and a surface charge density of
10 Coulombs/m2 is enclosed in a cube of 10 m side. The total
outward electric displacement normal to the surface of the cube is
(A) 40p Coulombs
(B) 10p Coulombs
(C) 5p Coulombs
(D) None of these

1996 TWO MARKS

9.132 A uniform plane wave in air is normally incident on infinitely thick


slab. If the refractive index of the glass slab is 1.5, then the percentage
of incident power that is reflected from the air-glass interface is
GATE Electronics and Communication Topicwise Solved Paper by RK Kanodia & Ashish Murolia Page 227

SOLUTION Also, direction of propagation is


v v
avk = ax + az
2
So, k = x + z
2
Substituting it in equation (1), we get
p # 10 ^x + z h
4

9.1 Option (D) is correct. Evi = Eo _avx - avz i e-j 3 2


Stoke’s theorem states that the circulation a vector field A v around 2
v over
a closed path l is equal to the surface integral of the curl of A 9.5 Option (A) is correct.
the open surface S bounded by l . We obtain the reflection coefficient for parallel polarized wave (since,
i.e., A#
v : dlv = ^d # A
## vh : dsv electric field is in the plane of wave propagation) as
h cos qt - h1 cos qi
Gz = 2
Here, line integral is taken across a closed path which is denoted h2 cos qt + h1 cos qi
...(1)
by a small circle on the integral notation where as, the surface in-
tegral of ^d # Avh is taken over open surface bounded by the loop. As we have already obtained
9.2 Option (D) is correct. qi = 45c, qt = 19.2c
Given, the vector field m 1 = h0
Also, h2 = = h0
v = xavx + yavy + zavz e 4.5 4.5
A
so,
SPECIAL EDITION ( STUDY MATERIAL FORM )
v ) = 2Ax + 2Ay + 2Az
v (Divergence of A
d$A
2x 2y 2z At market Book is available in 3 volume i.e. in 3 book binding
= 1+1+1 = 3 form. But at NODIA Online Store book is available in 10 book
binding form. Each unit of Book is in separate binding.
9.3 Option (A) is correct. Available Only at NODIA Online Store
Given, the return loss of device as 20 dB
i.e., G in dB =- 20 dB (loss) Click to Buy
^ h
www.nodia.co.in
or, 20 log G =- 20
G = 10-1 = 0.1 m
= h0 1 = h0
&
and h1 =
Therefore, the standing wave ration is given by e 1
1+ G Substituting these in eq. (1) we get
VSWR =
1- G
G z = cos 19.2c - 4.5 cos 45c
cos 19.2c + 4.5 cos 45c
= 1 + 0.1 = 1.1 = 1.22 =- 0.227
1 - 0.1 0.9
9.4 Option (C) is correct. .- 0.23
For the given incidence of plane wave, we have the transmitting Therefore, the reflected field has the magnitude given by
angle Ero
= T 11'
qt = 19.2c Eio
From Snell’s law, we know or Ero = G z Eio =- 0.23 Eio
n1 sin qi = n 2 sin qt Hence, the expression of reflected electric field is
Evr =- 0.23 Eo _- avx - avz i e-j 3 k
4
p # 10

c m1 e1 sin qi = c m2 e2 sin qt 2
(2)
...(1)
For the given interfaces, we have Again, we have the propagation vector of reflected wave as
m1 = m2 = 1 v v
avk = ax - az
e1 = 1, e2 = 4.5 2
So, from Eq. (1) or, k = x - z
sin qi = 4.5 sin 19.2 2
Substituting it in Eq. (2), we get
or, qi . 45c
Evr =- 0.23 Eo _- avx - avz i e-j 3 b 2 l
4
p # 10 x - z
v
Now, the component of Ei can be obtained as 2
Evi = _Eox avx - Eoz avz i e-jbk jp # 10 ^x - z h V
4

(observed from the shown figure) Evr = 0.23 o _avx + avz i e- 3 2


E m
2
Since, the angle qi = 45c so,
9.6 Option (C) is correct.
Eox = Eoz = Eo Electric field of the propagating wave in free space is given as
2
_ax - avz i e-jbk
Therefore, v
Ei = E o v Ei = (8ax + 6ay + 5az ) e j (wt + 3x - 4y) V/m
...(1) 2 So, it is clear that wave is propagating in the direction (- 3ax + 4ay)
.
Now, the wavelength of EM wave is Since, the wave is incident on a perfectly conducting slab at x = 0 .
l = 600 mm So, the reflection coefficient will be equal to - 1.
So, b = 2p = p # 10 4 i.e. Er = (- 1) Ei =- 8ax - 6ay - 5az
l 3
0 0

Again, the reflected wave will be as shown in figure.


GATE Electronics and Communication Topicwise Solved Paper by RK Kanodia & Ashish Murolia Page 228

l = (2m + 1) l
4

f1 = 429 MHz, l1 =c = 3 # 108 = 0.174 m


f1 # 4 429 # 106 # 4
8
f2 = 1 GHz , l2 = c = 3 # 10 = 0.075 m
f2 # 4 1 # 10 # 49

Only option (C) is odd multiple of both l1 and l2 .


i.e. the reflected wave will be in direction 3ax + 4ay . Thus, the (2m + 1) = 1.58 = 9
l1
electric field of the reflected wave will be.
Ex = (- 8ax - 6ay - 5az ) e j (wt - 3x - 4y) V/m (2m + 1) = 1.58 - 21
l2
9.7 Option (A) is correct. 9.11 Option (D) is correct.
The field in circular polarization is found to be
Hz = 3 cos (2.094 # 102 x) cos (2.618 # 102 y) cos (6.283 # 1010 t - bz)
Es = E 0 (ay ! jaz ) e-jbx propagating in + ve x -direction.
bx = 2.094 # 102
where, plus sign is used for left circular polarization and minus
by = 2.618 # 102
sign for right circular polarization. So, the given problem has left
w = 6.283 # 1010 rad/s
circular polarization. For the wave propagation,
b = 25 = w w 2 - (b 2 + b 2)
c b = x y
c2
Substituting above values,
GATE Electronics & Communication
6.283 # 1010 2 - (2.0942 + 2.6182) 10 4
by RK Kanodia b = c 3 108 m #
#
Now in 3 Volume - j261
Purchase Online at maximum discount from online store b is imaginary so mode of operation is non-propagating.
and get POSTAL and Online Test Series Free vp = 0
visit www.nodia.co.in 9.12 Option ( ) is correct.
For r > a , Ienclosed = (pa2) J
2pf 8
25 = & f = 25 # c = 25 # 3 # 10 = 1.2 GHz # H : dl = Ienclosed
c 2p 2 # 3.14
9.8 Option (B) is correct. H # 2pr = (pa2) J
Let b " outer diameter
H = Io Io = (pa2) J
a " inner diameter 2pr
Characteristic impedance, H \ 1 , for r > a
r
m0 4p # 10-7 # 36p ln 2.4 = 100 W
Z0 = ln b b l = b 1 l J (pr 2) Jr 2
e0 er a 10-9 # 10.89 For r < a , Ienclosed = = 2
pa2 a
9.9 Option (A) is correct.
The directivity is defined as
So, # H : dl= Ienclosed
2

D = Fmax H # 2pr = Jr2


Favg a
Fmax = 1 H = Jr 2
2pa
Favg = 1 F (q, f) dW
# H \ r , for r < a
4p
= 1 ;
2p 2p
F (q, f) sin qdq dfE
For more GATE Resources, Mock Test and
4p 0 0# #
Study material join the community
p/2
http://www.facebook.com/gateec2014
2p
= 1 ; # # cos 4 q sin qdqdfE
4p 0 0
5 p/2 Option (B) is correct.
= 1 ;2p b- cos q lE = 1 # 2p :- 0 + 1 D
9.13

4p 5 0
4p 5 Assuming the cross section of the wire on x -y plane as shown in
= 1 # 2p = 1 figure.
4p 5 10
D = 1 = 10
10
or, D (in dB) = 10 log 10 = 10 dB
9.10 Option (C) is correct.
Since Z0 = Z1 Z 2
100 = 50 # 200
This is quarter wave matching. The length would be odd multiple
of l/4 .
GATE Electronics and Communication Topicwise Solved Paper by RK Kanodia & Ashish Murolia Page 229

Further if H z = 1.5utx + Auty + Buz


Then from Boundary condition
10ut
(3utx + 30uty) utx = (1.5utx + Auty + Butz ) xt + v y
J
=- 30utz =- Autz + Buty + 10uty
Comparing we get A = 30 and B =- 10
So H z = 1.5utx + 30uty - 10utz A/m
9.18 Option (A) is correct.
Since voltage maxima is observed at a distance of l/4 from the
load and we know that the separation between one maxima and
minima equals to l/4 so voltage minima will be observed at the
load, Therefore load can not be complex it must be pure resistive.
Now G = s-1
s+1

Since, the hole is drilled along the length of wire. So, it can be also RL = R 0 (since voltage maxima is formed at the load)
s
assumed that the drilled portion carriers current density of - J . RL = 50 = 10 W
5
Now, for the wire without hole, magnetic field intensity at point P
will be given as 9.19 Option (D) is correct.
Hf1 (2pR) = J (pR2) SPECIAL EDITION ( STUDY MATERIAL FORM )
At market Book is available in 3 volume i.e. in 3 book binding
Hf1 (2pR) = JR
2 form. But at NODIA Online Store book is available in 10 book
Since, point o is at origin. So, in vector form
binding form. Each unit of Book is in separate binding.
H1 = J (xax + yay) Available Only at NODIA Online Store
2
Again only due to the hole magnetic field intensity will be given as. Click to Buy
(Hf2) (2pr) =- J (pr 2) www.nodia.co.in
Hf2 = - Jr From the expressions of Ev & Hv , we can write,
2
Again, if we take Ol at origin then in vector form b = 280 p
2p = 280 p & l = 1
H2 = - J (xlax + ylay) or
l 140
2
v E
where xl and yl denotes point ‘P ’ in the new co-ordinate system. Wave impedance, Zw = E = p = 120 p
Now the relation between two co-ordinate system will be. Hv 3 er
x = xl + d again, f = 14 GHz
y = yl 8
Now l = C = 3 # 10 9 = 3
So, H2 = - J [(x - d) ax + yay] er f er 14 # 10 140 er
2 3 1
or =
So, total magnetic field intensity = H1 + H2 = J dax 140 er 140
2
or er = 9
So, magnetic field inside the hole will depend only on ‘d ’.
Ep
Option (C) is correct. Now = 120p = E p = 120p
9.14 3 9
Power radiated from any source is constant.
9.20 Option (C) is correct.
9.15 Option (C) is correct. For a lossless network
We have d = 2 mm and f = 10 GHz S11 2 + S21 2 = 1
Phase difference = 2p d = p ; For the given scattering matrix
l 4
S11 = 0.2 0c , S12 = 0.9 90c
or = l = 8d = 8 # 2 mm = 16 mm
S21 = 0.9 90c , S22 = 0.1 90c
v = fl = 10 # 109 # 16 # 10-3
Here, (0.2) 2 + (0.9) 2 ! 1 (not lossless)
= 1.6 # 108 m/ sec Reciprocity :
9.16 Option (A) is correct. S12 = S21 = 0.9 90c (Reciprocal)
TM11 is the lowest order mode of all the TMmn modes.
9.21 Option (D) is correct.
9.17 Option (A) is correct. For distortion less transmission line characteristics impedance
From boundary condition
Z0 = R
Bn1 = Bn2 G
Attenuation constant
m1 Hx1 = m2 Hx2
a = RG
or Hx2 = Hx1 = 1.5
2 So, a = R = 0.1 = 0.002
Z0 50
or Hx2 = 1.5utx 9.22 Option (C) is correct.
GATE Electronics and Communication Topicwise Solved Paper by RK Kanodia & Ashish Murolia Page 230

Intrinsic impedance of EM wave So, tan bl = tan b 2p : l l = 3


m m0 l 4
h = = = 120p = 60p R ZL V
e 4e0 2
S tan bl + jZo W 2
Time average power density Z in = Zo S W = Z 0 = 60 W
2 S Zo + jZL W ZL
Pav = 1 EH = 1 E = 1 = 1 S tan bl W
2 2 h 2 # 60p 120p T X
For length of l/8 transmission line
9.23 Option (C) is correct. Z + jZo tan bl
Z in = Zo ; L
Av = xyatx + x 2 aty Zo + jZL tan bl E
v = dxatx + dyaty
dl Zo = 30 W, ZL = 0 (short)
# Av : dl tan bl = tan b 2p : l l = 1
v = # (xyatx + x 2 aty) : (dxatx + dyaty)
C C l 8
= # (xydx + x 2 dy)
C Z in = jZo tan bl = 30j
Circuit is shown below.
= #1/
2/ 3
xdx +
1/ 3
#2/ 3xdx + #1
3 4 dy + #3
1 1 dy
3 3 3 3

= 1 : 4 - 1 D + 3 :1 - 4 D + 4 [3 - 1] + 1 [1 - 3] = 1
2 3 3 2 3 3 3 3
9.24 Option (A) is correct.
GATE Electronics & Communication
by RK Kanodia
Now in 3 Volume Reflection coefficient
60 + 3j - 60
Purchase Online at maximum discount from online store t = ZL - Zo = = 1
ZL + Zo 60 + 3j + 60 17
and get POSTAL and Online Test Series Free 1+ t
visit www.nodia.co.in VSWR = = 1 + 17 = 1.64
1- t 1 - 17
In the given problem 9.26 Option (D) is correct.
Due to 1 A current wire in x - y plane, magnetic field be at origin
will be in x direction.
Due to 1 A current wire in y - z plane, magnetic field be at origin
will be in z direction.
Thus x and z component is non-zero at origin.
9.27 Option (A) is correct.
Rectangular and cylindrical waveguide doesn’t support TEM modes
Reflection coefficient and have cut off frequency.
h - h1 Coaxial cable support TEM wave and doesn’t have cut off frequency.
t = 2 = 400p - 120p =- 1
h2 + h 1 40p + 120p 2 9.28 Option (B) is correct.
t is negative So magnetic field component does not change its We have V = 4# A ...(1)
direction Direction of incident magnetic field By Stokes theorem
atE # atH = atK
atZ # atH = aty
# A $ dl = ## (4 # A) $ ds ...(2)

atH = atx ( + x direction) For more GATE Resources, Mock Test and
So, reflection magnetic field component Study material join the community
Hr = t # 24 cos (3 # 108 + by) atx , y $ 0 http://www.facebook.com/gateec2014
h
= 1 # 24 cos (3 # 108 + by) atx , y $ 0 From (1) and (2) we get
2 # 120p
8 # A $ dl = ##V $ ds
b = w = 3 # 108 = 1
vC 3 # 10 9.29 Option (D) is correct.
So, Hr = 1 cos (3 # 108 + y) atx , y $ 0 The transmission line are as shown below. Length of all line is l
10p 4
9.25 Option (B) is correct.
For length of l/4 transmission line
Z + jZo tan bl
Z in = Zo ; L
Zo + jZL tan bl E
ZL = 30 W , Zo = 30 W, b = 2p , l = l
l 4
GATE Electronics and Communication Topicwise Solved Paper by RK Kanodia & Ashish Murolia Page 231

2 2
Zi1 = Z01 = 100 = 200W Reflection coefficient
ZL1 50
2 h2 - h1
2
G=
Zi2 = Z02 = 100 = 200W h2 + h1
ZL2 50
Substituting values for h1 and h2 we have
ZL3 = Zi1 Zi2 = 200W 200W = 100W
mo mo
-
t = eo er eo
= 1- er = 1 - 9 since
Zi = Z02 2
= 50 = 25W
mo
eo er + mo
eo 1+ er 1+ 9
ZL3 100 er = 9
9.30 Option (C) is correct. =- 0.5
x a - y a
We have Bv = B0 c 2 y xm ...(1) Option (C) is correct.
x + y2 x2 + y2 9.36

To convert in cylindrical substituting In single mode optical fibre, the frequency of limiting mode increases
as radius decreases
x = r cos f and y = r sin f
ax = cos far - sin faf Hence r \ 1
f
and ay = sin far + cos faf So. if radius is doubled, the frequency of propagating mode gets
In (1) we have halved, while in option (D) it is increased by two times.
Bv = Bv0 af 9.37 Option (D) is correct.
v Bv a
Now Hv = B = 0 f constant
m0 m0 SPECIAL EDITION ( STUDY MATERIAL FORM )
Jv = 4# Hv = 0 since H is constant At market Book is available in 3 volume i.e. in 3 book binding
9.31 Option (C) is correct. form. But at NODIA Online Store book is available in 10 book
The beam-width of Hertizian dipole is 180c and its half power beam- binding form. Each unit of Book is in separate binding.
width is 90c. Available Only at NODIA Online Store

9.32 Option (D) is correct. Click to Buy


Maxwell equations www.nodia.co.in
4- B = 0
4$ E = r/E 8

4# E =- B l = c = 3 # 10 9 = 3
f 20 # 10 200
4# Ht = D + J Gain Gp = hp2 ` D j = 0.7 # p2 c 13 m = 30705.4
2 2

For static electric magnetic fields l 100

4$ B = 0 = 44.87 dB

4$ E = r/E 9.38 Option (A) is correct.


4# E = 0 g = b cos 30cx ! b sin 30cy
S = 2p 3 x ! 2p 1 y
4# H = J l 2 l 2
9.33 Option (A) is correct. = p 3 x! py
Cut-off Frequency is l l
E = ay E0 e j (wt - g) = ay E0 e j;wt - c l mE
p 3 x! p y
fc = c ` a j +`b j
m 2 n 2 l
2 9.39 Option (D) is correct.
For TE11 mode,
4# H = J + 2D Maxwell Equations
3 # 1010 2t
` 4 j + ` 3 j = 6.25 GHz
fc = 1 2 1 2
2
## 4# H $ ds = ## `J + 22Dt j .ds Integral form
9.34 Option (D) is correct. s s
ZL + iZo tan (bl)
Zin = Zo
Zo + iZL tan (bl)
# H $ dl = ## `J + 22Dt j .ds Stokes Theorem
s
For ZL = 0 , Zin = iZo tan (bl) 9.40 Option (A) is correct.
The wavelength is wm p
2 `2j 0
H sin ` 2px j sin (wt - bz) yt
2
E =
8 h a
l = c = 3 # 109 = 0.1 m or 10 cm
f 3 # 10 This is TE mode and we know that
mpy
Ey \ sin ` mpx j cos `
b j
bl = 2p l = 2p # 1 = p
l 10 5 a
Thus Zin = iZo tan p Thus m = 2 and n = 0 and mode is TE20
5
9.41 Option (C) is correct.
Thus Zin is inductive because Zo tan p is positive The 2-port scattering parameter matrix is
5
S11 S12
S ==
S21 S22 G
9.35 Option (C) is correct.
m
We have h =
e
GATE Electronics and Communication Topicwise Solved Paper by RK Kanodia & Ashish Murolia Page 232

(ZL Z0) - Zo (50 50) - 50 The Brewster angle is


S11 = = =- 1
(ZL Z0) + Zo (50 50) + 50 3 er2
tan qn =
2 (ZL Zo) 2 (50 50) er1
S12 = S21 = = =2
(ZL Zo) + Zo (50 50) + 50 3 er2
tan 60c =
(Z Z ) - Zo (50 50) - 50 1
S22 = L o = =- 1
(ZL Zo) + Zo (50 50) + 50 3 or er2 = 3
9.42 Option (D) is correct. 9.47 Option (C) is correct.
The input impedance is We have E = atxx sin (wt - bz) + aty sin (wt - bz + p/2)
2
Zin = Zo ; if l = l Here Ex = Ey and fx = 0, fy = p2
ZL 4 p
2 2
Phase difference is 2 , thus wave is left hand circularly polarized.
Zin1 = Zo1 = 50 = 25 Option (A) is correct.
ZL1 100 9.48

2 2 We have 10 log G = 10 dB
Zin2 = Zo2 = 50 = 12.5
ZL2 200 or G = 10
Now ZL = Zin1 Zin2 Now gain G = Prad
Pin
25 12.5 = 25
3
or 10 = Prad
(50) 2 1W
Zs = = 300
25/3 or Prad = 10 Watts
Option (A) is correct.
GATE Electronics & Communication
9.49

mo mo
h2 - h1 -
by RK Kanodia G=
h2 + h1
= eo er
mo
eo
mo
+
Now in 3 Volume
eo er eo
= 1 + er = 1 - 4 =- 1
1 + er 1+ 4 3
Purchase Online at maximum discount from online store
The transmitted power is
and get POSTAL and Online Test Series Free
visit www.nodia.co.in Pt = (1 - G2) Pi = 1 - 1 = 8
9 9
or Pt = 8
G = ZS - Zo = 300 - 50 = 5 Pi 9
ZS + Zo 300 + 50 7
9.50 Option (D) is correct.
9.43 Option (D) is correct.
sin q = 1 = 1
= c 5 3 m + c 5 m = c 10 m
2 2 2
We have H =
2 Hx2 + Hy2 er 2
ho ho ho
or q = 45c = p
E 2 ho H 2
h 4
= o c 10 m = 50
2
For free space P = =
2ho 2 2 ho ho The configuration is shown below. Here A is point source.
watts

9.44 Option (C) is correct.


The cut-off frequency is
fc = c ` a j +`b j
m 2 n 2
2
Since the mode is TE20, m = 2 and n = 0
8 Now AO = 1 m
fc = c m = 3 # 10 # 2 = 10 GHz
2 2 2 # 0.03 From geometry BO = 1 m
ho 377
h' = fc 2
= = 400W For more GATE Resources, Mock Test and
1-c m
10
1 - c 10 10 m
2
f 3 # 10 Study material join the community
9.45 Option (B) is correct. http://www.facebook.com/gateec2014
Using the method of images, the configuration is as shown below
Thus area = pr2 = p # OB = p m 2
9.51 Option (C) is correct.
The cut-off frequency is
fc = c ` a j +` b j
m 2 m 2
2
Since the mode is TE30 , m = 3 and n = 0
Here d = l, a = p, thus bd = 2p
fc = c m
bd cos y + a 2 a
Array factor is = cos ; E 8
2 or 18 # 109 = 3 # 10 3
2p cos y + p 2 a
= cos ; E = sin (p cos y)
2
or a = 1 m = 5 cm
9.46 Option (D) is correct. 40 2
GATE Electronics and Communication Topicwise Solved Paper by RK Kanodia & Ashish Murolia Page 233

9.52 Option (C) is correct.


We have E1 = 4ux + 3uy + 5uz 9.61 Option (C) is correct.
Since for dielectric material at the boundary, tangential component The given figure represent constant reactance circle.
of electric field are equal 9.62 Option (D) is correct.
We know that vp > c > vg .
E21 = E1t = 3aty + 5atz
at the boundary, normal component of displacement vector are 9.63 Option (A) is correct.
equal 4pU (q, f)
We have GD (q, f) =
Prad
i.e. Dn2 = Dn1
For lossless antenna
or e2 E2n = e1 E1n
Prad = Pin
or 4eo E2n = 3eo 4atz
Here we have Prad = Pin = 1 mW
or E2n = 3atx
and 10 log GD (q, f) = 6 dB
Thus E2 = E2t + E2a = 3atx + 3aty + 5atz
or GD (q, f) = 3.98
9.53 Option (C) is correct.
Thus the total power radiated by antenna is
Since antenna is installed at conducting ground,
2 4pU (q, f) = Prad GD (q, f) = 1 m # 3.98 = 3.98 mW
Rrad = 80p2 ` dl j = 80p2 c = 4p W
2
50
m
2
l 0.5 # 10 3 5 9.64 Option (D) is correct.
9.54 Option (C) is correct.
SPECIAL EDITION ( STUDY MATERIAL FORM )
w = 50, 000 and b =- 0.004
4
At market Book is available in 3 volume i.e. in 3 book binding
Phase Velocity is vP = w = 5 # 10 - 3 = 1.25 # 107 m/s form. But at NODIA Online Store book is available in 10 book
b - 4 # 10
binding form. Each unit of Book is in separate binding.
9.55 Option (C) is correct. Available Only at NODIA Online Store
Refractive index of glass m = 1.5
Click to Buy
Frequency f = 1014 Hz
c = 3 # 108 m/sec
www.nodia.co.in
8 The capacitance is
l = c = 3 # 10 = 3 # 10 - 6
f 14 - 12 -4
10 C = eo A = 8.85 # 10 - 3 # 10 = 8.85 # 10 - 13
wavelength in glass is d 10
-6 The charge on capacitor is
lg = a = 3 # 10 = 2 # 10 - 6 m
m 1.5 Q = CV = 8.85 # 10 - 13 = 4.427 # 10 - 13
9.56 Option (D) is correct. Displacement current in one cycle
Option (D) is correct. Q
9.57
I = = fQ = 4.427 # 10 - 13 # 3.6 # 109 = 1.59 mA
T
Zo2 = ZOC .ZSC
2 9.65 Option (C) is correct.
ZZC = Zo = 50 # 50 = 50 VL = ZO
ZOC 100 + j150 2 + 3j
50 (2 - 3j) Vin Zin
= = 7.69 - 11.54j
13 or VL = ZO Vin = 10 # 300 = 60 V
Zin 50
9.58 Option (A) is correct.
9.66 Option (D) is correct.
The array factor is
bd sin q + a 9.67 Option (A) is correct.
A = cos b l
2 Ravg = 1 Re [E # H*]
2
Here b = 2p , d = l and a = 90c
l 4 E # H* = (atx + jaty) e jkz - jwt # k (- jatx + aty) e-jkz + jwt
2p l p wm
sin q +
Thus A = cos c l 4 2
m = cos ` sin q + j
p p
2 4 2 = atz ; k - (- j) (j) k E = 0
wm wm
The option (A) satisfy this equation.
Thus Ravg = 1 Re [E # H*] = 0
9.59 Option (C) is correct. 2
From the diagram, VSWR is 9.68 Option (A) is correct.
s = Vmax = 4 = 4 Suppose at point P impedance is
Vmin 1
Z = r + j (- 1)
When minima is at load ZO = s.ZL
If we move in constant resistance circle from point P in clockwise
or ZL = Zo = 50 = 12.5W direction by an angle 45c, the reactance magnitude increase. Let us
s 4
9.60 Option (A) is correct. consider a point Q at 45c from point P in clockwise direction. It’s
The reflection coefficient is impedance is
Z1 = r - 0.5j
G = ZL - ZO = 12.5 - 50 =- 0.6
ZL + ZO 125. + 50
GATE Electronics and Communication Topicwise Solved Paper by RK Kanodia & Ashish Murolia Page 234

or Z1 = Z + 0.5j or e1 E1n = e2 E2n


Thus movement on constant r - circle by an +45c in CW direction or 1.5eo 2ux = 2.5eo E2n
is the addition of inductance in series with Z . or E2n = 3 ux = 1.2ux
2.5
9.69 Option (D) is correct.
Thus E2 = E2t + E2n =- 3uy + uz + 1.2ux
1- G
We have VSWR = Emax = 5 = Option (C) is correct.
Emin 1+ G 9.74

We have E = xux + yuy + zuz


or G =2
3 dl = utx dx + uty dy + utz dz
Thus G =- 2 Y
3 VXY =- #XE.dl
h - h1
Now G = 2 2 0 0
h2 + h1 = #1 xdxutx + #2 ydyutz + #3 zdzuzt
h - 120p
or -2 = 2 y2
0
=-= x +z G
2 2 2 0
3 h2 + 120p +
2 1 2 2 2 3
or h2 = 24p
9.70 Option (D) is correct. =- 1 [22 - 12 + 02 - 22 + 02 - 32] = 5
2
1- G
The VSWR 2= 9.75 Option (D) is correct.
1+ G
m
h =
e
GATE Electronics & Communication Reflection coefficient
by RK Kanodia h2 - h1
t =
Now in 3 Volume h2 + h1
Purchase Online at maximum discount from online store Substituting values for h1 and h2 we have
mo m0
and get POSTAL and Online Test Series Free t = eo er - eo
= 1- er = 1 - 4 since
mo mo
visit www.nodia.co.in er = 4
eo er + eo 1+ er 1+ 4

or G =1 = - 1 = 0.333+180c
3 3
Pref
Thus = G2= 1 9.76 Option (B) is correct.
Pinc 9
We have E (z, t) = 10 cos (2p # 107 t - 0.1pz)
or Pref = Pinc where w = 2p # 107 t
9
i.e. 11.11% of incident power is reflected. b = 0.1p
7
9.71 Option (C) is correct. Phase Velocity u = w = 2p # 10 = 2 # 108 m/s
b 0.1p
By Maxwells equations
9.77 Option (A) is correct.
4# H = 2D + J The fig of transmission line is as shown below.
2t
Thus 4# H has unit of current density J that is A/m2 [Z + jZo tan bl]
We know that Zin = Zo L
[Zo + jZL tan bl]
9.72 Option (B) is correct.
For line 1, l = l and b = 2p , ZL1 = 100W
We know that d \ 1 2 l
f
d2 = f1 For more GATE Resources, Mock Test and
Thus
d1 f2 Study material join the community
d2 = 1 http://www.facebook.com/gateec2014
25 4
or d2 = 1 # 25 = 12.5 cm
4 [ZL + jZo tan p]
Thus Zin1 = Zo = ZL = 100W
9.73 Option (C) is correct. [Zo + jZL tan p]
We have E1 = 2ux - 3uy + 1uz For line 2, l = l and b = 2p , ZL2 = 0 (short circuit)
8 l
E1t = - 3uy + uy and E1n = 2ux [0 + jZo tan p4 ]
Since for dielectric material at the boundary, tangential component Thus Zin2 = Zo = jZo = j50W
[Zo + 0]
of electric field are equal
Y = 1 + 1 = 1 + 1 = 0.01 - j0.02
E1t =- 3uy + uy = E2t (x = 0 plane) Zin1 Zin2 100 j50
E1n = 2ux
At the boundary the for normal component of electric field are
D1n = D2n
GATE Electronics and Communication Topicwise Solved Paper by RK Kanodia & Ashish Murolia Page 235

9.84 Option (A) is correct.


Loss tangent tan a = s = 1.7 # 10 - 4
we 2p # 3 # 109 # 78eo
-4 9
= 1.7 # 10 9# 9 # 10 = 1.3 # 10 - 5
3 # 10 # 39
9.85 Option (D) is correct.
The flux density is
s = eE = e0 er E = 80 # 8.854 # 10 - 12 # 2
or s = 1.41 # 10 - 9 C/m 2
9.86 Option (B) is correct.
P \ 12
9.78 Option (A) is correct. r
8 P1 = r22
u = c = 3 # 10 = 1.5 # 108 Thus
P2 r12
e0 2
In rectangular waveguide the dominant mode is TE10 and 3 dB decrease $ Strength is halved
P1 = 2
fC = v ` m j2 + ` n j2 Thus
P2
2 a b
8
= 1.5 # 10 1 2 + 0 2 = 1.5 # 108 = 2.5 GHz
` 0.03 j ` b j
SPECIAL EDITION ( STUDY MATERIAL FORM )
2 0.06
At market Book is available in 3 volume i.e. in 3 book binding
9.79 Option (D) is correct. form. But at NODIA Online Store book is available in 10 book
y
Normalized array factor = 2 cos binding form. Each unit of Book is in separate binding.
2
Available Only at NODIA Online Store
y = bd sin q cos f + d
Click to Buy
q = 90c,
d = 2 s,
www.nodia.co.in
f = 45c,
Substituting values we have
d = 180c 2
y bd sin q cos f + d 2 = r22
Now 2 cos = 2 cos ; E 5
2 2
or r2 = 5 2 kM = 7071 m
= 2 cos 8 2p 2 s cos 45c + 180 B Distance to move = 7071 - 5000 = 2071 m
l. 2 2
= 2 cos 8 ps + 90cB = 2 sin ` ps j 9.87 Option (C) is correct.
l l
A transmission line is distortion less if LG = RC
9.80 Option (D) is correct. 9.88 Option (B) is correct.
VSWR s = 1+G where G varies from 0 to 1 d2 Ex = c2 d2 Ex
1-G We have
dz2 dt2
Thus s varies from 1 to 3.
This equation shows that x component of electric fields Ex is
9.81 Option (B) is correct.
traveling in z direction because there is change in z direction.
Reactance increases if we move along clockwise direction in the
constant resistance circle. 9.89 Option (A) is correct.
In wave guide vp > c > vg and in vacuum vp = c = vg
9.82 Option (C) is correct.
where vp $ Phase velocity
Phase velocity
VC c $ Velocity of light
VP =
f 2 vg $ Group velocity
1-c c m
f 9.90 Option (A) is correct.
When wave propagate in waveguide fc < f $ VP > VC In a wave guide dominant gives lowest cut-off frequency and hence
9.83 Option (C) is correct. the highest cut-off wavelength.
p
We have t j ) e j (wt - kz)
E = (0.5xt + ye 2
9.91 Option (A) is correct.
Ic = Id
Ex = 0.5e j (wt - kz)
or sE = jw d E
jp j (wt - kz)
Ey = e e 2 or s = 2pfeo er w = 2pf and e = er e0
9 -2
Ex p or f = s = 2s = 9 # 10 # 2 # 10
= 0.5e- 2 2p # eo er 4peo er 4
Ey
or f = 45 # 106 = 45 MHz
Ex
Since ! 1, it is elliptically polarized. Option (B) is correct.
Ey 9.92
GATE Electronics and Communication Topicwise Solved Paper by RK Kanodia & Ashish Murolia Page 236

VSWR = 1 + G or er = 2
1-G
9.99 Option (C) is correct.
or 3 = 1+G vp
` a j +`b j
1-G Cutoff frequency fc = m 2 n 2
2
or G = 0.5
Pr = G2 = 0.25 For rectangular waveguide dominant mode is TE01
Now v 8
Pi Thus fc = p = 3 # 10- 2 = 15 # 109 For air
2a 2 # 10
Thus 25% of incident power is reflected. vp = 3 # 108
9.93 Option (A) is correct.
= 15 GHz
We have l = 492 m
9.100 Option (B) is correct.
and height of antenna = 124 m . l
4 Phase Velocity b = 2p = w me
It is a quarter wave monopole antenna and radiation resistance is l
25 W. or l = 2p
w me
9.94 Option (C) is correct.
The array factor is Thus l \ 1
y = bd cos q + d e

where d =l Distance between elements we get l1 = e2


4 l2 e1
Option (D) is correct.
GATE Electronics & Communication 9.101

` 2 jd = l
l
by RK Kanodia
2

Now in 3 Volume 8
l = c = 3 # 10 9 = 3 m
Purchase Online at maximum discount from online store f 4 # 10 40

` 40 # 2 j d = (2.4)
and get POSTAL and Online Test Series Free 3 2

visit www.nodia.co.in 80 # (2.4) 2


or d = . 150 m
3
y =0 Because of end fire 9.102 Option (C) is correct.
q = 60c We know that for a monopole its electric field varies inversely with
Thus 0 = 2p # l cos 60c + d = p # 1 + d r 2 while its potential varies inversely with r . Similarly for a dipole
l 4 2 2 its electric field varies inversely as r 3 and potential varies inversely
or d =- p as r 2 .
4 In the given expression both the terms a _ r1 + r1 i are present, so
-1 -2

9.95 Option (B) is correct. this potential is due to both monopole & dipole.
Zo = ZOC .ZSC = 100 # 25 = 10 # 5 = 50W 9.103 Option (D) is correct.
9.96 Option (C) is correct. In TE mode Ez = 0 , at all points within the wave guide. It implies
As the impedance of perfect conductor is zero, electric field is that electric field vector is always perpendicular to the waveguide
minimum and magnetic field is maximum at the boundary. axis. This is not possible in semi-infine parallel plate wave guide.
9.97 Option (B) is correct. 9.104 Option (A) is correct.
BW \ 1 9.105 Option (C) is correct.
(Diameter) A scalar wave equation must satisfy following relation
As diameter increases Bandwidth decreases. For more GATE Resources, Mock Test and
9.98 Option (C) is correct. Study material join the community
The fig is as shown below :
http://www.facebook.com/gateec2014

2 2 E - m 22 2 E = 0 ...(1)
2t 2 2z 2
Where m = w (Velocity)
b
Basically w is the multiply factor of t and b is multiply factor of
As per snell law z or x or y .
sin qt = 1 In option (A) E = 50e j (wt - 3z)
sin qi er m =w=w
sin 30c = 1 b 3
or
sin 45c er We can see that equations in option (C) does not satisfy equation
1
2
1
= 1 (1)
2
er
GATE Electronics and Communication Topicwise Solved Paper by RK Kanodia & Ashish Murolia Page 237

9.106 Option (B) is correct. l = 2 = 10 m


We know that distance between two adjacent voltage maxima is 0.2
equal to l/2 , where l is wavelength.
9.115 Option (C) is correct.
l = 27.5 - 12.5
2 The depth of penetration or skin depth is defined as –
d= 1
l = 2 # 15 = 30 cm
10
pfms
Frequency u = C = 3 # 10 = 1 GHz
l 30
d\ 1 \ l
9.107 Option (D) is correct. f
Power received by antenna so depth increases with increasing in wavelength.
-4 Option (A) is correct.
PR = PT 2 # (apeture) = 251 # 500 # 102 = 100 mW
9.116

4pr 4 # p # (100) Given


j (wt + bz) v
9.108 Option (C) is correct. E (z, t) = Eo e a x + e0 e j (wt + bz) avy ...(1)
Electrical path length = bl Generalizing
Where b = 2p , l = 50 cm E (z) = avx E1 (z) + avy E2 (z) ...(2)
l
Comparing (1) and (2) we can see that E1 (z) and E2 (z) are in space
We know that
quadrature but in time phase, their sum E will be linearly polarized
l =u =1# 1 au= 1
f f LC LC SPECIAL EDITION ( STUDY MATERIAL FORM )
= 1 1
25 # 10 6 # At market Book is available in 3 volume i.e. in 3 book binding
10 # 10 # 40 # 10-12
-6

form. But at NODIA Online Store book is available in 10 book


7
= 5 # 10 6 = 2 m binding form. Each unit of Book is in separate binding.
25 # 10 Available Only at NODIA Online Store
Electric path length = 2p # 50 # 10-2 = p radian Click to Buy
5 2
9.109 Option (D) is correct. www.nodia.co.in
In a lossless dielectric (s = 0) median, impedance is given by along a line that makes an angle f with x -axis as shown below.
m m0 mr mr Option (C) is correct.
h = 0c = = 120p # 9.117
e e0 er er
Hv = 1 4v v
A
= 120p # 2 = 188.4 W m #
8 v is auxiliary potential function.
where A
9.110 Option (D) is correct. So 4: H = 4: (4 # A) = 0
Impedance is written as
4# H = 4# (4 # A) =
Y 0
jwm
h = Option (D) is correct.
s + jwe 9.118

Copper is good conductor i.e. s >> we Radiation resistance of a circular loop is given as

So h =
jwm
=
wm
45c Rr = 8 hp3 :ND2 S D
s s 3 l
2
Impedance will be complex with an inductive component. Rx \ N N " no. of turns
9.111 Option (A) is correct. So, Rr 2 = N 2 # Rr 1
This equation is based on ampere’s law as we can see = (5) 2 # 0.01 = 0.25 W
#l H $ dl = I enclosed (ampere's law) 9.119 Option (C) is correct.
We have
or #l H $ dl = #s Jds Power Re ceived
Aperture Area =
Applying curl theorem Polynting vector of incident wave

#s (4 # H) $ ds = #s Jds A =W
P
4# H = J 2
P = E h0 = 120p is intrinsic impedance
then it is modified to h0
of space
4# H = J + 2D Based on continuity equation
2t
So A
9.112 Option (A) is correct. -6 -6
= 2 # 10 = 2 # 10
# 120 # 3.14
E2 (20 # 10-3) 2
Option (B) is correct. c h0 m
9.113

9.114 Option (B) is correct.


-6
Propagation constant = 2 # 10 # 12 -# 3.14 = 1.884 m2
400 # 10 6
r = a + ib = 0.1p + j0.2p
Option (B) is correct.
b = 2p = 0.2p
9.120
here
l Maximum usable frequency
GATE Electronics and Communication Topicwise Solved Paper by RK Kanodia & Ashish Murolia Page 238

fo
fm = 1
sin Ae or = 10-2 f 1 = 10 MHz
6
p # 10 # 10 # m s
fm = 8MHz = 8 = 16 MHz -3
sin 60c 3
c 2 m
3 or ms = 10
p
9.121 Option (D) is correct. Now phase velocity at another frequency
When a moving circuit is put in a time varying magnetic field educed f 2 = 1000 MHz is
emf have two components. One for time variation of B and other
4pf 2
turn motion of circuit in B . V =
ms
Option (A) is correct. -3
ms = 10 in above equation
9.122
Put
p
Far field \ 1
r V
9.123 Option (B) is correct. 6
4 # p # 1000 # 10 # p - 6 106 m/ sec
= #
Z in min = Z 0 10-3
S
9.128 Option (A) is correct.
where S = standing wave ratio
Input impedance of a lossless transmission line is given by
1 + GL
Z + jZ 0 tan bl
Zin = Z 0 ; L
Z 0 + jZL tan bl E
S =
1 - GL
GL = reflection coefficient where Z 0 = Charateristic impedance of line
ZL = Load impedance
GATE Electronics & Communication
by RK Kanodia b = 2p l = length
l
Now in 3 Volume so here bl = 2p l = p
l 4 2
Purchase Online at maximum discount from online store
ZL = 0 (Short circuited)
and get POSTAL and Online Test Series Free
and Z 0 = 50 W
visit www.nodia.co.in
0 + j50 tan p/2
Zin = 50 =
50 + j0 tan p/2G
so =3
GL = ZL - Z 0 = 100 - 50 = 50 = 1
ZL + Z 0 100 + 50 150 3 Thus infinite impedance, and current will be zero.

1+1 9.129 Option (B) is correct.


S = 3 =2 For lossless transmission line, we have
1- 1
3 Velocity V =w= 1 ...(1)
50 b LC
Z in min = = 25 W
2 Characteristics impedance for a lossless transmission line
9.124 Option (A) is correct. Z0 = L ...(2)
The cutoff frequency is given by C
ml From eqn. (1) and (2)
m 2+ n 2
2 a a k a2k
fc = 1
V = = 1
Here a < b , so minimum cut off frequency will be for mode TE 01 C (Z 0 C ) Z 0 C
m = 0, n = 1 9.130 Option (C) is correct.

a ml = c Option (A) is correct.


9.131
8
fc = 3 # 10 1
* 2
2#2 (10 # 10-12) 9.132 Option (C) is correct.
c = 3 # 108
= 3 # 108 = 0.75 GHz For more GATE Resources, Mock Test and
2 # 2 # 10 # 10-2
Study material join the community
9.125 Option (B) is correct. http://www.facebook.com/gateec2014
9.126 Option (A) is correct.
Reflected power
For any transmission line we can write input impedance
Er = GEi Ei " Incident power
Z + jZ 0 tanh lg
Zin = Z 0 ; L
Z 0 + jZL tanh lg E G = Reflection coefficient
h - h1
Here given ZL = 3 (open circuited at load end) G = 2 = 1.5 - 1 = 1
h 2 + h1 1.5 + 1 5
R jZ tanh lg V
S1 + 0 W
Z0 So Er = 1 # Ei
so Zin = Z 0 lim S ZL W= 5
Z " 3S Z 0 W j tanh lg
S ZL + j tanh lg W Er = 20%
L

T X Ei
9.127 Option (A) is correct. 9.133 Option (B) is correct.
We know that skin depth is given by We have maximum usable frequency formulae as
s = 1 = 1 # 10-2 m f0
pf1 ms fm =
sin Ae
GATE Electronics and Communication Topicwise Solved Paper by RK Kanodia & Ashish Murolia Page 239

6
20 # 106 = 10 # 10
sin Ae
sin Ae = 1
2
Ae = 30c
9.134 Option (C) is correct.
9.135 Option (A) is correct.
Skin depth d= 1
pfms
Putting the given value
d= 1
3.14 # 1 # 109 # 4p # 10-7 # 106
= 15.9 mm

SPECIAL EDITION ( STUDY MATERIAL FORM )


At market Book is available in 3 volume i.e. in 3 book binding
form. But at NODIA Online Store book is available in 10 book
binding form. Each unit of Book is in separate binding.
Available Only at NODIA Online Store

Click to Buy
www.nodia.co.in
GATE Electronics and Communication Topicwise Solved Paper by RK Kanodia & Ashish Murolia Page 240

UNIT 10 10.8
(C)
9 ^9n - 1h
8
+n (D)
9 ^9n - 1h
8
+ n2

Statement: There were different streams of freedom movements


GENERAL APTITUDE in colonial India carried out by the moderates, liberals, radicals,
socialists, and so on.
Which one of the following is the best inference from the above
statement?
(A) The emergence of nationalism in colonial India led to our Inde-
YEAR 2013 ONE MARK
pendence
10.1 Choose the grammatically CORRECT sentence: (B) Nationalism in India emerged in the context of colonialism
(A) Two and two add four (B) Two and two become four (C) Nationalism in India is homogeneous
(C) Two and two are four (D) Two and two make four (D) Nationalism in India is heterogeneous
10.2 Statement: You can always give me a ring whenever you need. 10.9 The set of values of p for which the roots of the equation
Which one of the following is the best inference from the above 3x2 + 2x + p ^p - 1h = 0 are of opposite sign is
statement? (A) ^- 3, 0h (B) ^0, 1h
(A) Because I have a nice caller tune.
(C) ^1, 3h (D) ^0, 3h
(B) Because I have a better telephone facility
(C) Because a friend in need is a friend indeed 10.10 What is the chance that a leap year, selected at random, will contain
53 Sundays?
GATE Electronics & Communication (A) 2/7 (B) 3/7
by RK Kanodia (C) 1/7 (D) 5/7
Now in 3 Volume
Purchase Online at maximum discount from online store
2012 ONE MARK
and get POSTAL and Online Test Series Free
If (1.001) 1259 = 3.52 and (1.001) 2062 = 7.85, then (1.001) 3321
visit www.nodia.co.in
10.11

(A) 2.23 (B) 4.33


(D) Because you need not pay towards the telephone bills when (C) 11.37 (D) 27.64
you give me a ring
10.12 Choose the most appropriate alternate from the options given below
10.3 In the summer of 2012, in New Delhi, the mean temperature of to complete the following sentence :
Monday to Wednesday was 41°C and of Tuesday to Thursday was If the tired soldier wanted to lie down, he..................the mattress
43cC . If the temperature on Thursday was 15% higher than that of out on the balcony.
Monday, then the temperature in cC on Thursday was (A) should take (B) shall take
(A) 40 (B) 43
(C) should have taken (D) will have taken
(C) 46 (D) 49
10.13 Choose the most appropriate word from the options given below to
10.4 Complete the sentence: Dare .................. mistakes. complete the following sentence :
(A) commit (B) to commit Give the seriousness of the situation that he had to face, his........
(C) committed (D) committing was impressive.
(A) beggary (B) nomenclature
10.5 They were requested not to quarrel with others.
(C) jealousy (D) nonchalance
Which one of the following options is the closest in meaning to the
word quarrel? 10.14 Which one of the following options is the closest in meaning to the
(A) make out (B) call out
For more GATE Resources, Mock Test and
(C) dig out (D) fall out
Study material join the community
10.1 Option (D) is correct. http://www.facebook.com/gateec2014
They were requested not to quarrel with others.
Quarrel has a similar meaning to ‘fall out’ word given below ?
Latitude
YEAR 2013 TWO MARKS
(A) Eligibility (B) Freedom
(C) Coercion (D) Meticulousness
10.6 A car travels 8 km in the first quarter of an hour, 6 km in the
second quarter and 16 km in the third quarter. The average speed 10.15 One of the parts (A, B, C, D) in the sentence given below contains
of the car in km per hour over the entire journey is an ERROR. Which one of the following is INCORRECT ?
(A) 30 (B) 36 I requested that he should be given the driving test today instead
(C) 40 (D) 24 of tomorrow.
(A) requested that (B) should be given
10.7 Find the sum to n terms of the series 10 + 84 + 734 + ... (C) the driving test (D) instead of tomorrow
9 ^9n + 1h 9 ^9n - 1h
(A) +1 (B) +1
10 8
GATE Electronics and Communication Topicwise Solved Paper by RK Kanodia & Ashish Murolia Page 241

2012 TWO MARKS 10.22 The question below consists of a pair of related words followed by
10.16 One of the legacies of the Roman legions was discipline. In the four pairs of words. Select the pair that best expresses the relations
legious, military law prevailed and discipline was brutal. Discipline in the original pair :
on the battlefield kept units obedient, intact and fighting, even when Gladiator : Arena
the odds and conditions were against them. (A) dancer : stage (B) commuter : train
Which one of the following statements best sums up the meaning (C) teacher : classroom (D) lawyer : courtroom
of the above passage ?
(A) Through regimentation was the main reason for the efficiency
10.23 Choose the most appropriate word from the options given below to
of the Roman legions even in adverse circumstances. complete the following sentence :
Under ethical guidelines recently adopted by the Indian Medical
(B) The legions were treated inhumanly as if the men were animals
Association, human genes are to be manipulated only to correct
(C) Disciplines was the armies inheritance from their seniors diseases for which...................treatments are unsatisfactory.
(D) The harsh discipline to which the legions were subjected to led (A) similar (B) most
to the odds and conditions being against them. (C) uncommon (D) available
10.17 Raju has 14 currency notes in his pocket consisting of only Rs. 20 10.24 Choose the word from the from the options given below that is most
notes and Rs. 10 notes. The total money values of the notes is Rs. opposite in meaning to the given word :
230. The number of Rs. 10 notes that Raju has is Frequency
(A) 5 (B) 6 (A) periodicity (B) rarity
(C) 9 (D) 10
SPECIAL EDITION ( STUDY MATERIAL FORM )
10.18 There are eight bags of rice looking alike, seven of which have At market Book is available in 3 volume i.e. in 3 book binding
equal weight and one is slightly heavier. The weighing balance is form. But at NODIA Online Store book is available in 10 book
of unlimited capacity. Using this balance, the minimum number of
weighings required to identify the heavier bag is
binding form. Each unit of Book is in separate binding.
Available Only at NODIA Online Store
(A) 2 (B) 3
(C) 4 (D) 8
Click to Buy
www.nodia.co.in
10.19 The data given in the following table summarizes the monthly
budget of an average household. (C) gradualness (D) persistency

Category Amount (Rs.) 10.25 Choose the most appropriate word from the options given below to
complete the following sentence :
Food 4000
It was her view that the country’s had been ............. by foreign
Clothing 1200 techno-crafts, so that to invite them to come back would be coun-
Rent 2000 ter-productive.
Savings 1500 (A) identified (B) ascertained
Other Expenses 1800 (C) exacerbated (D) analysed

The approximate percentages of the monthly budget NOT spent


2011 TWO MARKS
on savings is
(A) 10% (B) 14% 10.26 The fuel consumed by a motor cycle during a journey while travelling
(C) 81% (D) 86% at various speed is indicated in the graph below.

10.20 A and B are friends. They decide to meet between 1 PM and 2 PM


on a given day. There is a conditions that whoever arrives first will
not wait for the other for more than 15 minutes. The probability
that they will meet on that days is
(A) 1/4 (B) 1/16
(C) 7/16 (D) 9/16

2011 ONE MARK


The distance covered during four laps of the journey are listed in
10.21 There are two candidates P and Q in an election. During the the table below
campaign, 40% of voter promised to vote for P , and rest for Q
Lap Distance (km) Average speed (km/hour)
. However, on the day of election 15% of the voters went back on
their promise to vote for P and instead voted for Q . 25% of the P 15 15
voter went back on their promise to vote for Q and instead voted Q 75 45
for P . Suppose, P lost by 2 votes, then what was the total number R 40 75
of voters ?
S 10 10
(A) 100 (B) 110
From the given data, we can conclude that the fuel consumed per
(C) 90 (D) 95 kilometre was least during the lap
GATE Electronics and Communication Topicwise Solved Paper by RK Kanodia & Ashish Murolia Page 242

(A) P (B) Q 10.34 Choose the most appropriate word from the options given below to
(C) R (D) S complete the following sentence :
His rather casual remarks on politics..................his lack of serious-
10.27 The horse has played a little known but very important role in the ness about the subject.
field of medicine. Horses were injected with toxins of disease until (A) masked (B) belied
their blood build up immunities. Then a serum was made from their (C) betrayed (D) suppressed
blood. Serums to fight with diphteria and tetanus were developed
this way. 10.35 25 persons are in a room 15 of them play hockey, 17 of them play
It can be inferred from the passage, that horses were football and 10 of them play hockey and football. Then the number
(A) given immunity to diseases of persons playing neither hockey nor football is
(B) generally quite immune to diseases (A) 2 (B) 17
(C) given medicines to fight toxins (C) 13 (D) 3
(D) given diphtheria and tetanus serums
2010 TWO MARKS
10.28 The sum of n terms of the series 4 + 44 + 444 + ........
(A) (4/81) [10n + 1 - 9n - 1] (B) (4/81) [10n - 1 - 9n - 1] 10.36 Modern warfare has changed from large scale clashes of armies to
(C) (4/81) [10n + 1 - 9n - 10] (D) (4/81) [10n - 9n - 10] suppression of civilian populations. Chemical agents that do their
work silently appear to be suited to such warfare ; and regretfully,
10.29 Given that f (y) = y /y, and q is any non-zero real number, the their exist people in military establishments who think that chemical
value of f (q) - f (- q) is agents are useful fools for their cause.
Which of the following statements best sums up the meaning of
GATE Electronics & Communication the above passage ?
by RK Kanodia (A) Modern warfare has resulted in civil strife.
Now in 3 Volume (B) Chemical agents are useful in modern warfare.
Purchase Online at maximum discount from online store (C) Use of chemical agents in ware fare would be undesirable.
and get POSTAL and Online Test Series Free (D) People in military establishments like to use chemical agents in
visit www.nodia.co.in war.

10.37 If 137 + 276 = 435 how much is 731 + 672 ?


(A) 0 (B) - 1
(A) 534 (B) 1403
(C) 1 (D) 2
(C) 1623 (D) 1531
10.30 Three friends R, S and T shared toffee from a bowl. R took 1/3 rd 10.38 5 skilled workers can build a wall in 20 days; 8 semi-skilled workers
of the toffees, but returned four to the bowl. S took 1/4 th of what
can build a wall in 25 days; 10 unskilled workers can build a wall
was left but returned three toffees to the bowl. T took half of the
in 30 days. If a team has 2 skilled, 6 semi-skilled and 5 unskilled
remainder but returned two back into the bowl. If the bowl had 17
workers, how long will it take to build the wall ?
toffees left, how many toffees were originally there in the bowl ?
(A) 20 days (B) 18 days
(A) 38 (B) 31
(C) 16 days (D) 15 days
(C) 48 (D) 41
10.39 Given digits 2, 2, 3, 3, 3, 4, 4, 4, 4 how much distinct 4 digit numbers
greater than 3000 can be formed ?
2010 ONE MARK (A) 50 (B) 51
10.31 Which of the following options is the closest in meaning to the word (C) 52 (D) 54
below ?
Circuitous 10.40 Hari (H), Gita (G), Irfan (I) and Saira (S) are siblings (i.e. brothers
(A) Cyclic (B) Indirect For more GATE Resources, Mock Test and
(C) Confusing (D) Crooked Study material join the community
10.32 The question below consist of a pair of related words followed by http://www.facebook.com/gateec2014
four pairs of words. Select the pair that best expresses the relation
in the original pair. and sisters.) All were born on 1st January. The age difference between
Unemployed : Worker any two successive siblings (that is born one after another) is less
(A) Fallow : Land (B) Unaware : Sleeper than 3 years. Given the following facts :
(C) Wit : Jester (D) Renovated : House 1. Hari’s age + Gita’s age > Irfan’s age + Saira’s age.
2. The age difference between Gita and Saira is 1 year. However,
10.33 Choose the most appropriate word from the options given below to Gita is not the oldest and Saira is not the youngest.
complete the following sentence : 3. There are no twins.
If we manage to ........ our natural resources, we would leave a bet-
In what order were they born (oldest first) ?
ter planet for our children.
(A) HSIG (B) SGHI
(A) unhold (B) restrain
(C) IGSH (D) IHSG
(C) cherish (D) conserve
***********
GATE Electronics and Communication Topicwise Solved Paper by RK Kanodia & Ashish Murolia Page 243

SOLUTIONS S2 =
9 ^9 - 1h
8
+ 22 = 94
S3
9 ^93 - 1h
= + 32 = 828
8
10.8 Option (D) is correct.
10.1 Option (D) is correct Nationalism in India is heterogeneous
Two and two make four
10.9 Option (B) is correct.
10.2 Option (C) is correct. Given, the quadratic equation
You can always given me a ring whenever you need. Because a friend
is need is a friend indeed 3x2 + 2x + P ^P - 1h = 0
It will have the roots with opposite sign if
10.3 Option (C) is correct.
Let the temperature on Monday, Tuesday, Wednesday and Thursday P ^P - 1h < 0
be respectively as TM , TTU , TW , TTH So it can be possible only when
So, from the given data we have P < 0 and P - 1 > 0
TH + TTU + TW = 41 ....(1) or P > 0 and P - 1 < 0
3
The 1 st condition tends to no solution for P .
and TTU + TW + TTH = 43 ....(2)
3
SPECIAL EDITION ( STUDY MATERIAL FORM )
also, as the temperature on Thursday was 15% higher than that of At market Book is available in 3 volume i.e. in 3 book binding
Monday form. But at NODIA Online Store book is available in 10 book
i.e. TTH = 1.15 TM ....(3) binding form. Each unit of Book is in separate binding.
solving eq (1), (2) and (3), we obtain Available Only at NODIA Online Store
TTH = 46cC Click to Buy
10.4 Option (B) is correct. www.nodia.co.in
Dare to commit mistakes
Hence, from the second condition, we obtain
10.5 Option (D) is correct.
They were requested not to quarrel with others. 0 <P<1
Quarrel has a similar meaning to ‘fall out’ i.e., P is in the range ^0, 1h
10.10 Option (A) is correct.
10.6 Option (C) is correct.
In a leap year, there are 366 days So, 52 weeks will have 52 saturdays
Given, the distance travelled by the car in each quarter intervals
and for remaining two days ^366 - 52 # 7 = 2h. We can have the
as
following combinations
Distance Time Duration Saturday, Sunday
8 km 1
hr Sunday, Monday
4
Monday, Tuesday
6 km 1
4 hr
Tuesday, Wednesday
16 km 1
4 hr Wednesday, Thursday
Therefore, the total time taken = 1 + 1 + 1 + 3 hr Thursday, Friday
4 4 4 4
Friday, Saturday
Total distance travelled = 8 + 6 + 16 = 30 km Out of these seven possibilities, only two consist a saturday.
Hence, average speed = Total distance travelled Therefore, the probability of saturday is given as
Total time taken
P =2
= 30 = 40 km/hr 7
3/4 10.11 Option (D) is correct.
10.7 Option (D) is correct. Let 1.001 = x
It will be easy to check the options for given series. From the given So in given data :
series. x1259 = 3.52
10 + 84 + 734 + ...... x2062 = 7.85
We get Again x3321 = x1259 + 2062
Sum of 1 term = S1 = 10
= x1259 x2062
Sum of 2 terms = S2 = 10 + 84 = 94
and sum of 3 terms = S 3 = 10 + 84 + 734 = 828 = 3.52 # 7.85
Checking all the options one by one, we observe that only (D) op- = 27.64
tion satisfies as 10.12 Option (C) is correct.
9 ^9n - 1h
Sn = + n2 10.13 Option (D) is correct.
8
9 ^92 - 1h 10.14 Option (B) is correct.
so, S1 + 22 = 10
8
GATE Electronics and Communication Topicwise Solved Paper by RK Kanodia & Ashish Murolia Page 244

10.15 Option (B) is correct. = 9000 # 100% = 86%


10500
10.16 Option (A) is correct.
10.20 Option (S) is correct.
10.17 Option (A) is correct.
The graphical representation of their arriving time so that they
Let no. of notes of Rs.20 be x and no. of notes of Rs. 10 be y .
met is given as below in the figure by shaded region.
Then from the given data.
x + y = 14
20x + 10y = 230
Solving the above two equations we get
x = 9, y = 5
So, the no. of notes of Rs. 10 is 5.
10.18 Option (A) is correct.
We will categorize the 8 bags in three groups as :
(i) A1 A2 A 3 , (ii) B1 B2 B 3 , (iii) C1 C2
Weighting will be done as bellow :
1st weighting " A1 A2 A 3 will be on one side of balance and B1 B2 B 3
on the other. It may have three results as described in the following
cases.
Case 1 : A1 A 2 A 3 = B1 B 2 B 3
So, the area of shaded region is given by
GATE Electronics & Communication Area of 4PQRS - (Area of TEFQ +
by RK Kanodia Area of TGSH )
Now in 3 Volume = 60 # 60 - 2 b 1 # 45 # 45 l
Purchase Online at maximum discount from online store 2

and get POSTAL and Online Test Series Free = 1575


visit www.nodia.co.in So, the required probability = 1575 = 7
3600 16
This results out that either C1 or C2 will heavier for which we will
10.21 Option (A) is correct.
have to perform weighting again. Let us assume total voters are 100. Thus 40 voter (i.e. 40 %) promised
2 nd weighting " C1 is kept on the one side and C2 on the other. to vote for P and 60 (rest 60 % ) promised to vote fore Q.
if C1 > C 2 then C1 is heavier. Now, 15% changed from P to Q (15 % out of 40)
Changed voter from P to Q 15 40 = 6
C1 < C 2 then C2 is heavier. 100 #
Case 2 : A1 A 2 A 3 > B1 B 2 B 3 Now Voter for P 40 - 6 = 34
it means one of the A1 A2 A 3 will be heavier So we will perform next
Also, 25% changed form Q to P (out of 60%)
weighting as:
Changed voter from Q to P 25 60 = 15
2 nd weighting " A1 is kept on one side of the balance and A2 on the 100 #
other.
Now Voter for P 34 + 15 = 49
if A1 = A 2 it means A 3 will be heavier
Thus P P got 49 votes and Q got 51 votes, and P lost by 2 votes,
A1 > A 2 then A1 will be heavier
which is given. Therefore 100 voter is true value.
A1 < A 2 then A2 will be heavier
10.22 Option (A) is correct.
Case 3 : A1 A 2 A 3 < B 1 B 2 B 3
A gladiator performs in an arena. Commutators use trains. Lawyers
This time one of the B1 B2 B 3 will be heavier, So again as the above
case weighting will be done. For more GATE Resources, Mock Test and
2 nd weighting " B1 is kept one side and B2 on the other Study material join the community
if B1 = B 2 B 3 will be heavier http://www.facebook.com/gateec2014
B1 > B 2 B1 will be heavier
B1 < B 2 B2 will be heavier performs, but do not entertain like a gladiator. Similarly, teachers
So, as described above, in all the three cases weighting is done educate. Only dancers performs on a stage.
only two times to give out the result so minimum no. of weighting 10.23 Option (D) is correct.
required = 2. Available is appropriate because manipulation of genes will be done
10.19 Option (D) is correct. when other treatments are not useful.
Total budget = 4000 + 1200 + 2000 + 1500 + 1800 10.24 Option (B) is correct.
= 10, 500 Periodicity is almost similar to frequency. Gradualness means
The amount spent on saving = 1500 something happening with time. Persistency is endurance. Rarity is
So, the amount not spent on saving opposite to frequency.
= 10, 500 - 1500 = 9000 10.25 Option (C) is correct.
So, percentage of the amount The sentence implies that technocrats are counterproductive
GATE Electronics and Communication Topicwise Solved Paper by RK Kanodia & Ashish Murolia Page 245

(negative). Only (C) can bring the same meaning. 10.33 Option (D) is correct.
10.26 Option (B) is correct. Here conserve is most appropriate word.
Since fuel consumption/litre is asked and not total fuel consumed, 10.34 Option (C) is correct.
only average speed is relevant. Maximum efficiency comes at 45 km/ Betrayed means reveal unintentionally that is most appropriate.
hr, So least fuel consumer per litre in lap Q 10.35 Option (D) is correct.
10.27 Option (B) is correct. Number of people who play hockey n (A) = 15
Option B fits the sentence, as they built up immunities which helped Number of people who play football n (B) = 17
humans create serums from their blood. Persons who play both hockey and football n (A + B) = 10
10.28 Option (C) is correct. Persons who play either hockey or football or both :
4 + 44 + 444 + .............. 4 (1 + 11 + 111 + .......) n (A , B) = n (A) + n (B) - n (A + B)
= 4 (9 + 99 + 999 + ............) = 15 + 17 - 10 = 22
9
Thus people who play neither hockey nor football = 25 - 22 = 3
10.36 Option (D) is correct.
= 4 [(10 - 1) + (100 - 1) + ........] Option (C) is correct.
9 10.37

Since 7 + 6 = 13 but unit digit is 5 so base may be 8 as 5 is the


= 4 [10 (1 + 10 + 102 + 103) - n]
9 remainder when 13 is divided by 8. Let us check.
n
= 4 :10 # 10 - 1 - nD
9 10 - 1 SPECIAL EDITION ( STUDY MATERIAL FORM )
= 4 610n + 1 - 10 - 9n@ At market Book is available in 3 volume i.e. in 3 book binding
81
form. But at NODIA Online Store book is available in 10 book
10.29 Option (D) is correct. binding form. Each unit of Book is in separate binding.
y Available Only at NODIA Online Store
f (y) =
y Click to Buy
Now f (- y) =
-y
=- f (y) www.nodia.co.in
y
or f (q) - f (- q) = 2f (q) = 2 137 8 731 8
10.30 Option (C) is correct. 276 8 672 8
Let total no of toffees be x . The following table shows the all cal- 435 Thus here base is 8. Now 1623
culations.
10.38 Option (D) is correct.
Friend Bowl Status Let W be the total work.
R = x -4 = 2x + 4 Per day work of 5 skilled workers =W
3 3 20
Per day work of one skill worker = W =W
S = 1 :2x + 4D - 3 = 2x + 4 - x + 2 5 # 20 100
4 3 3 6
Similarly per day work of 1 semi-skilled workers = W = W
= x +1-3 = x -2 = x +6 8 # 25 200
6 6 2
Similarly per day work of one semi-skill worker = W = W
10 # 30 300
T = 1 a x + 6k - 2 = x +6-x -1 Thus total per day work of 2 skilled, 6 semi-skilled and 5 unskilled
2 2 2 4
workers is = 2W + 6W + 5W = 12W + 18W + 10W = W
= x +1 = x +5 100 200 300 600 15
4 4
Therefore time to complete the work is 15 days.
Now, x + 5 = 17
4 10.39 Option (B) is correct.
x = 17 - 5 = 12 As the number must be greater than 3000, it must be start with 3
or or 4. Thus we have two case:
4
x = 12 # 4 = 48 Case (1) If left most digit is 3 an other three digits are any of 2, 2,
3, 3, 4, 4, 4, 4.
10.31 Option (B) is correct.
Circuitous means round about or not direct. Indirect is closest in (1) Using 2, 2, 3 we have 3223, 3232, 3322 i.e. 3! = 3 no.
2!
meaning to this circuitous
(A) Cyclic : Recurring in nature (2) Using 2, 2, 4 we have 3224, 3242, 3422 i.e. 3! = 3 no.
2!
(B) Indirect : Not direct
(3) Using 2, 3, 3 we have 3233, 3323, 3332 i.e. 3! = 3 no.
(C) Confusing : lacking clarity of meaning 2!
(D) Crooked : set at an angle; not straight (4) Using 2, 3, 4 we have 3! = 6 no.

Option (B) is correct. (5) Using 2, 4, 4 we have 3244, 3424, 3442 i.e. 3! = 3 no.
10.32 2!
A worker may by unemployed. Like in same relation a sleeper may (6) Using 3, 3, 4 we have 3334, 3343, 3433 i.e. 3! = 3 no.
be unaware. 2!
GATE Electronics and Communication Topicwise Solved Paper by RK Kanodia & Ashish Murolia Page 246

(7) Using 3, 4, 4 we have 3344, 3434, 3443 i.e. 3! = 3 no.


2!
(8) Using 4, 4, 4 we have 3444 i.e. 3! = 1 no.
3!
Total 4 digit numbers in this case is
1 + 3 + 3 + 3 + 6 + 3 + 3 + 3 + 1 = 25
Case 2 : If left most is 4 and other three digits are any of 2, 2, 3, 3,
3, 4, 4, 4.
(1) Using 2, 2, 3 we have 4223, 4232, 4322 i.e. . 3! = 3 no
2!
(2) Using 2, 2, 4 we have 4224, 4242, 4422 i.e. . 3! = 3 no
2!
(3) Using 2, 3, 3 we have 4233, 4323, 4332 i.e. . 3! = 3 no
2!
(4) Using 2, 3, 4 we have i.e. . 3! = 6 no
(5) Using 2, 4, 4 we have 4244, 4424, 4442 i.e. . 3! = 3 no
2!
(6) Using 3, 3, 3 we have 4333 i.e 3! = 1. no.
3!
(7) Using 3, 3, 4 we have 4334, 4343, 4433 i.e. . 3! = 3 no
2!
GATE Electronics & Communication
by RK Kanodia
Now in 3 Volume
Purchase Online at maximum discount from online store
and get POSTAL and Online Test Series Free
visit www.nodia.co.in

(8) Using 3, 4, 4 we have 4344, 4434, 4443 i.e. . 3! = 3 no


2!
(9) Using 4, 4, 4 we have 4444 i.e. 3! = 1. no
3!
Total 4 digit numbers in 2nd case
= 3 + 3 + 3 + 6 + 3 + 3 + 1 + 3 + 1 = 26
Thus total 4 digit numbers using case (1) and case (2) is = 25 + 26 = 51
10.40 Option (B) is correct.
Let H , G , S and I be ages of Hari, Gita, Saira and Irfan respectively.
Now from statement (1) we have H + G > I + S
Form statement (2) we get that G - S = 1 or S - G = 1
As G can’t be oldest and S can’t be youngest thus either GS or
SG possible.
From statement (3) we get that there are no twins
(A) HSIG : There is I between S and G which is not possible
(B) SGHI : SG order is also here and For more GATE Resources, Mock Test and
S > G > H > I and G + H > S + I which is possible. Study material join the community
(C) IGSH : This gives I > G and S > H and adding these both http://www.facebook.com/gateec2014
inequalities we have I + S > H + G which is not possible.
(D) IHSG : This gives I > H and S > G and adding these both
inequalities we have I + S > H + G which is not possible.

You might also like